You are on page 1of 284

37

Blood Physiology
1. A fall in plasma albumin concentration would produce increased:
A. Fluid exit from capillaries C. Blood volume
B. Colloidal osmotic pressure D. Fluid entry into the capillaries

2. Hematocrit is increased in:


A. Dehydration C. Chronic renal failure
B. Pregnancy D. Thalassemia

3. Which of the WBCs rise in number in parasitic infections?


A. Monocyte B. Basophil C. Eosinophil D. Neutrophil

4. Which of the WBCs is similar to mast cells in its function?


A. Neutrophil B. Basophil C. Eosinophil D. Monocyte

5. Plasma erythropoietin level is increased in all of the following conditions EXCEPT:


A. High altitude C. Venous arterial shunt
B. Bone marrow tumor D. Pulmonary disease

6. Reticulocytosis occurs in:


A. Bone marrow stimulation C. Iron deficiency
B. Vitamin B12 deficiency D. X ray radiation

7. Hemoglobin structure is abnormal in:


A. Thalassemia C. Iron deficiency anemia
B. Pernicious anemia D. Polycythemia

8. Mean corpuscular volume is increased in:


A. Iron deficiency C. Hereditary spherocytosis
B. Acute blood loss D. Vitamin B12 deficiency

9. The immediate precursor of RBC in its development is:


A. Stem cell C. Reticulocyte
B. Proerythroblast D. Late normoblast

10. Iron is present in all of the following EXCEPT:


A. Heme B. Bilirubin C. Myoglobin D. Ferritin

11. Tissue macrophage is formed from:


A. Lymphocyte B. Monocyte C. Neutrophil D. Eosinophil

12. The bleeding disorder due to platelet deficiency is:


A. Hemophilia C. Von Willebrand disease
B. Christmas disease D. Purpura

13. B lymphocytes are preprocessed in:


A. Liver B. Kidney C. Bone marrow D. A and C

14. Chronic blood loss in a person can result in which type of anemia?
A. Hemolytic anemia B. Hypochromic anemia
38
Blood Physiology
C. Megaloblastic anemia D. Aplastic anemia

15. The protein present in the vascular endothelium that combines with thrombin and prevents
intravascular clotting?
A. Plasminogen B. Thrombomodulin C. Thrombosthenin D. Heparin

16. The aggregation of platelets is inhibited by:


A. ADP B. Prostacyclin C. Thrombosthenin D. Thromboxane A 2

17. The activation of which clotting factor is crucial for the extrinsic mechanism of blood coagulation?
A. V B. VII C. VIII D. X
18. The rejection of foreign transplanted tissue is caused by:
A. B lymphocyte B. T lymphocyte C. Macrophage D. Natural killer cells

19. Antigen presenting cells include:


A. Dendritic cells of lymph node C. B lymphocyte
B. Macrophage D. All of the above

20. Which of the following regulates immune system by the release of cytokines?
A. Plasma cell B. Cytotoxic T cell C. Helper T cell D. B lymphocyte

21. What does a hematocrit reveal?


A. The ability of the body to fight infection C. How much oxygen the blood can carry
B. The ability of the blood to clot D. The amount of iron in the blood

22. Which substance allows RBCs to transport oxygen?


A. Erythropoietin (EPO) C. Fibrin
B. Thrombin D. Hemoglobin

23. A move to high altitude would trigger which change in the blood?
A. An increased number of reticulocytes C. An increased number of neutrophils
B. A decreased number of reticulocytes D. A decreased number of neutrophils

24. Hemolysis may produce which physical sign?


A. Jaundice B. Pallor C. Cyanosis D. Ruddiness

25. When an infecting organism pierces the skin, which of the following WBCs would quickly migrate out
of the blood vessels and into the tissues to ingest the foreign invader?
A. Basophils B. Eosinophils C. Lymphocytes D. Neutrophils

26. Which of the following WBCs have the longest life span?
A. Neutrophils B. Eosinophils C. Basophils D. Monocytes

27. Which of the following is an unwanted part of hemostasis?


A. Vascular spasm C. Formation of a thrombus
B. Formation of fibrin D. Formation of thrombin

28. How would someone experiencing a heart attack caused by a blood clot benefit from a drug that
stimulates the conversion of plasminogen into plasmin?
39
Blood Physiology
A. Plasmin encourages the growth of new blood vessels that can bypass the clot
B. Plasmin increases the oxygen-carrying capacity of RBCs.
C. Plasmin inhibits the formation of new blood clots.
D. Plasmin dissolves the fibrin meshwork around blood clots.

29. What substance, carried by each red blood cell, determines blood type?
A. Antibody B. Antigen C. Hemoglobin D. Globin

30. Which of the following will occur if someone with type A blood receives a transfusion with type B
blood?
A. Leukocytosis B. Polycythemia C. Agglutination D. Fibrinolysis

31. The plasma proteins are not involved in:


A. Blood clotting D. Transport of oxygen
B. Blood viscosity E. Transport of hormones.
C. Plasma colloid osmotic pressure

32. During intrauterine life, formation of the red blood cells begins in:
A. Bone marrow C. Lymph nodes. E. Spleen
B. Liver D. Mesoderm of the yolk sac

33. One molecule of hemoglobin contains:


A. One heme + one polypeptide chain D. Four heme + four polypeptide chains.
B. One heme + two polypeptide chains. E. Two heme + four polypeptide chains.
C. Four heme + two polypeptide chains.

34. In a woman with iron deficiency anemia, there is an increase in:


A. MCV (mean corpuscle volume
B. MCH (mean corpuscle hemoglobin)
C. MCHC (mean corpuscle hemoglobin concentration)
D. Serum ferritin concentration
E. Serum total iron binding capacity (TIBC)

35. In a patient having vitamin K deficiency, there is:


A. Anemia D. Prolonged clotting time
B. Normal clotting time E. Thrombocytopenia
C. Prolonged bleeding time

36. The white blood cells having receptors for IgE on their membrane are:
A. Basophils. C. Lymphocytes. E. Monocytes.
B. Eosinophils. D. Neutrophils.

37. The white blood cells actively involved in phagocytosis are:


A. Basophils and eosinophils. D. Neutrophils and monocytes.
B. Basophils and neutrophils. E. Monocytes and lymphocytes.
C. Lymphocytes and eosinophils.

38. A blood transfusion reaction is likely to occur if a patient with blood group A+ is transfused with
blood of group:
40
Blood Physiology
A. A+ B. A- C. O+ D. O- E. AB-

39. The hormonal stimulus that prompts red blood cell formation is
A. Serotonin, B. Heparin, C. Erythropoietin, D. Thrombopoietin

40. All of the following are true of RBCs EXCEPT


A. Biconcave disc shape, C. Contain hemoglobin,
B. Life span of approximately 120 days, D. Contain nuclei

41. The most numerous WBC is the


A. Eosinophil, B. Neutrophil, C. Monocyte, D. Lymphocyte

42. The white blood cell that releases histamine and other inflammatory chemicals is the
A. Basophil, B. Neutrophil, C. Monocyte, D. Eosinophil

43. Suppose your blood was found to be AB positive. This means that
A. Agglutinogens A and B are present on your red blood cells,
B. There are no anti-A or anti-B agglutinins in your plasma,
C. Your blood is Rh+,
D. All of the above

44. Regarding blood group antigens (agglutinogens), all are false EXCEPT:
A. Carried on the hemoglobin molecule C. Present in fetal blood
B. Beta globulins. D. Inherited as recessive Mendelian characteristics.

45. Regarding breakdown of erythrocytes in the body, all are true EXCEPT:
A. Occurs when they are 17 weeks old
B. Takes place in the reticulo-endothelial system
C. Yields iron, most of which is retained for further use
D. Yields bilirubin which is carried by plasma protein to the liver
E. Is required for the synthesis of bile salts.

46. Blood platelets assist in arresting bleeding by the following EXCEPT:


A. Releasing factors promoting blood clotting
B. Adhering together to form plugs when exposed to collagen
C. Liberating high concentrations of calcium
D. Releasing factors causing vasoconstriction

47. Monocytes, all are true EXCEPT:


A. They originate from stem cells in bone marrow
B. Like granulocytes, monocytes migrate out to become tissue macrophages.
C. Can transform into large multinucleated cells in certain infections.
D. Manufacture immunoglobulin M.

48. Erythrocytes, all are true EXCEPT:


A. Are responsible for the major part of blood viscosity
B. Contain the enzyme carbonic anhydrase
C. Metabolize glucose to produce CO2 and H2O
D. Have rigid walls to protect them against hemolysis.
41
Blood Physiology

49. Human plasma albumin, all are true EXCEPT:


A. Contributes more to plasma colloid osmotic pressure than globulin
B. Filters freely at the renal capillaries.
C. Is negatively charged at the normal pH of blood
D. It contains essential and non-essential aminoacids

50. Neutrophil granulocytes, all are true EXCEPT:


A. Are the most common leukocyte in normal blood
B. Contain proteolytic enzymes
C. Have a lifespan in the circulation of 3–4 weeks
D. Contain actin and myosin microfilaments
E. Are present in high concentration in pus

51. Antibodies, all are true EXCEPT:


A. Are protein molecules
B. Are absent from the blood in early fetal life
C. Are produced at a greater rate after a first, than after a second, exposure to an antigen six weeks later
D. Circulating as free immunoglobulins that are produced by B lymphocytes

52. Circulating red blood cells, all are true EXCEPT:


A. Are about 1 per cent nucleated
B. May show an intracellular network pattern if appropriately stained
C. Deform as they pass through the capillaries
D. Have excess surface area for their volumes

53. Lymphocyte, all are true EXCEPT:


A. Are motile
B. Can transform into plasma cells
C. Decrease in number following removal of the adult thymus gland
D. Decrease in number during immunosuppressive drug therapy

54. Blood, all are true EXCEPT:


A. Makes up about 7 per cent of body weight
B. Forms a higher percentage of body weight in fat than in thin people
D. Volume rises after water is drunk
E. Expresses serum when it clots

55. Antigens, all are true EXCEPT:


A. Are usually proteins or polypeptide molecules
B. Can only be recognized by immune system cells of new and also previously exposed to that antigen
C. Are normally absorbed from the gut via lymphatics and carried to mesenteric lymph nodes
D. Induce a smaller immune response when protein synthesis is suppressed
E. Are taken up by antigen-presenting macrophages which activate the immune system

56. Blood eosinophils


A. Have agranular cytoplasm
B. Are about a quarter of all leukocytes
C. Are relatively abundant in the mucosa of the respiratory, urinary and alimentary tracts
42
Blood Physiology
D. Increase in number in viral infections

57. Normal blood clotting requires


A. Inactivation of heparin D. An adequate intake of vitamin A.
B. Inactivation of plasmin (fibrinolysin) E. An adequate intake of vitamin C.
C. Calcium ions

58. Antibodies (agglutinins) of the A and B red cell antigens (agglutinogens)


A. Are present in fetal plasma
B. Cause hemolysis of RBCs containing the A and B antigens when added to a suspension of red cells in
saline
C. Do not normally cross the placental barrier
D. Have one binding site (monovalent)

59. Blood platelets, all are true EXCEPT:


A. Are formed in the bone marrow
B. Are normally more numerous than white cells
C. Have a small single-lobed nucleus
D. Increase in number after injury and surgery
E. Alter shape when in contact with collagen

60. The conversion of fibrinogen to fibrin, all are true EXCEPT:


A. Is effected by prothrombin
B. Involves the disruption of certain peptide linkages by a proteolytic enzyme
C. Is followed by polymerization of fibrin monomers
D. Is inhibited by heparin
E. Is not reversed by plasmin (fibrinolysin)

61. Hemolytic disease of the newborn


A. Affects mainly babies of Rh-positive mothers
B. Occurs mainly in babies who lack D agglutinogen
C. Causes jaundice which clears rapidly after birth
D. Can be treated by transfusing the affected baby with Rh-positive blood
E. Can be prevented by injecting the mother with anti-D agglutinins just after delivery

62. The appearance of centrifuged blood may suggest the following EXCEPT
A. Anemia is present if there is more plasma than packed cells
B. The plasma lipid level is high
C. The patient has jaundice
D. Hemolysis has occurred
E. The patient has leukemia

63. Iron deficiency


A. Frequently follows persistent loss of blood from the body
B. Is more common in men than in women
C. May cause anemia by inhibiting the rate of multiplication of RBC stem cells
D. May cause large pale erythrocytes to appear in peripheral blood
E. Anemia should normally be treated by injections of iron
43
Blood Physiology
64. Severe reactions are likely after transfusion of blood group
A. A to a group B person C. A to a group AB person
B. O to a group AB person D. O Rh- negative to a group AB Rh-positive person

65. The hematocrit (packed cell volume), all are true EXCEPT:
A. May be obtained by centrifugation of blood
B. May be calculated by multiplying the mean cell volume by the red cell count
C. Rises in a patient who sustains widespread burns
D. Rises in macrocytic megaloblastic anemia such as pernicious (B12 deficiency) anemia

66. Red cell formation is increased, all are true EXCEPT:


A. By giving vitamin B12 injections to healthy people on a normal diet
B. In blood donors one week after a blood donation
C. In patients with hemolytic anemia
D. By giving injections of erythropoietin to patients with removed kidneys
E. In patients who have a raised blood reticulocyte count

67. Deficiency of factor VIII (antihemophilic globulin)


A. Increases the clotting time
B. Is due to an abnormal gene on the Y chromosome
C. Causes small (petechial) hemorrhages into the skin to cause purpura
D. Affects the extrinsic, rather than the intrinsic, pathway for blood coagulation

68. A 40-year-old woman visits the clinic complaining of fatigue. She had recently been treated for an
infection. Her laboratory values are as follows: red blood cell (RBC. count, 1.8 × 10 6 /μl; hemoglobin
(HB., 5.2 g/dL; hematocrit (Hct), 15; white blood cell (WBC. count, 7.6 × 10 3 /μl; platelet count,
320,000/μl; mean corpuscular volume (MCV), 92 fL; and reticulocyte count, 24%. What is the most likely
explanation for this presentation?
A. Aplastic anemia C. Hereditary spherocytosis
B. Hemolytic anemia D. B12 deficiency

69. What RBC enzyme facilitates transport of carbon dioxide (CO2)?


A. Myeloperoxidase C. Superoxide dismutase
B. Carbonic anhydrase D. Globin reductase

70. How many oxygen atoms can be transported by each hemoglobin molecule?
A. 2 B. 4 C. 8 D. 16

71. During the second trimester of pregnancy, where is the predominant site of RBC production?
A. Yolk sac B. Bone marrow C. Lymph nodes D. Liver

72. What function do vitamin B12 and folic acid perform that is critical to hematopoiesis?
A. Support porphyrin production
B. Serve as cofactors for iron uptake
C. Support terminal differentiation of erythroid and myeloid cells
D. Support production of thymidine triphosphate

73. Which phagocytes can extrude digestion products and continue to survive and function for many
months?
44
Blood Physiology
A. Neutrophils B. Basophils C. Macrophages D. Eosinophils

74. During an inflammatory response, what is the correct order of cellular events?
A. Filtration of monocytes from blood  increased production of neutrophils  activation of tissue
macrophages  infiltration of neutrophils from the blood
B. Activation of tissue macrophages infiltration of neutrophils from the blood infiltration of monocytes
from blood increased production of neutrophils
C. Increased production of neutrophils activation of tissue macrophages infiltration of neutrophils
from the blood infiltration of monocytes from blood
D. Infiltration of neutrophils from the blood activation of tissue macrophages infiltration of monocytes
from blood increased production of neutrophils

75. A 45-year-old man presents to the emergency department with a 2-week history of diarrhea that has
gotten progressively worse during the past several days. He has minimal urine output and is admitted to
the hospital for dehydration. His stool specimen is positive for parasitic eggs. Which type of WBC would
have an elevated number?
A. Eosinophils C. T lymphocytes E. Monocytes
B. Neutrophils D. B lymphocytes

76. A 24-year-old man came to the emergency department with a broken leg. A blood test revealed his
WBC count to be 22 × 103 /μl. Five hours later, a second blood test revealed values of 7 × 103 /μl. What is
the cause of the increased WBC count in the first test?
A. Increased production of WBCs by the bone marrow
B. Release of pre-formed, mature WBCs into the circulation
C. Decreased destruction of WBCs
D. Increased production of selectins

77. Where does the transmigration of WBCs occur in response to infectious agents?
A. Arterioles C. Capillaries and venules
B. Lymphatic ducts D. Inflamed arteries

78. An 8-year-old boy frequently comes to the clinic for persistent skin infections that do not heal within
a normal time frame. He had a normal recovery from the measles. A check of his antibodies after
immunizations yielded normal antibody responses. A defect in which of the following cells would most
likely be the cause of the continual infections?
A. B lymphocytes C. Neutrophils E. T lymphocytes
B. Plasma cells D. Macrophages

79. Which cell type in the inflammatory sites can clean up necrotic tissue and direct tissue remodeling?
A. Neutrophil B. Macrophage C. Dendritic cell D. Eosinophil

80. Fluid exudation into the tissue in an acute inflammatory reaction is due to which of the following?
A. Decreased blood pressure D. Increased clotting factors
B. Decreased protein in the interstitium E. Increased vascular permeability
C. Obstruction of the lymph vessels

81. Which statement is true concerning erythroblastosis fetalis (hemolytic disease of the newborn)?
A. HDN occurs when an Rh-positive mother has an Rh-negative child
B. HDN is prevented by giving the mother a blood transfusion
45
Blood Physiology
C. A complete blood transfusion after the first birth will prevent HDN
D. The father of the child must be Rh positive

82. Which statement is true?


A. In a transfusion reaction, agglutination of the recipient blood occurs
B. Shutdown of the kidneys after a transfusion reaction occurs slowly
C. Blood transfusion of Rh-positive blood into any Rh-negative recipient will result in an immediate
transfusion reaction
D. A person with type AB Rh-positive blood is considered a universal recipient

83. A woman whose blood type is A, Rh positive, and a man whose blood type is B, Rh positive, come to
the clinic with a 3-year-old girl whose blood type is O, Rh negative. What can be said about the
relationship of these two adults to this child?
A. The woman can be the child’s natural mother, but the man cannot be the natural father
B. The man can be the child’s natural father, but the woman cannot be the natural mother
C. Neither adult can be the natural parent of this child
D. This couple can be the natural parents of this child

84. What is the appropriate treatment for an infant born with severe erythroblastosis fetalis?
A. Passive immunization with anti-Rh(D) immunoglobulin
B. Immunization with Rh(D) antigen
C. Exchange transfusion with Rh(D)-positive blood
D. Exchange transfusion with Rh(D)-negative blood

85. Which of the following transfusions will result in an immediate transfusion reaction?
A. O Rh-negative whole blood to an O Rh-positive patient
B. A Rh-negative whole blood to a B Rh-negative patient
C. AB Rh-negative whole blood to an AB Rh-positive patient
D. B Rh-negative whole blood to a B Rh-negative patient

86. Which blood unit carries the least risks for inducing an immediate transfusion reaction into a B-
positive (B, rhesus positive) recipient?
A. Whole blood A positive D. Packed red blood cells O positive
B. Whole blood O positive E. Packed red blood cells AB negative
C. Whole blood AB positive

87. What condition leads to a deficiency in factor IX that can be corrected by an intravenous injection of
vitamin K?
A. Classic hemophilia C. Bile duct obstruction
B. Hepatitis B D. Genetic deficiency in antithrombin III

88. Which transfusion will result in a transfusion reaction? Assume that the patient has never had a
transfusion.
A. Type O Rh-negative packed cells to an AB Rh-positive patient
B. Type A Rh-positive packed cells to an A Rh-negative patient
C. Type AB Rh-positive packed cells to an AB Rh-positive patient
D. Type A Rh-positive packed cells to an O Rh-positive patient
46
Blood Physiology
89. A 55-year-old man who has been undergoing stable and successful anticoagulation with warfarin for
recurrent deep vein thrombosis is treated for pneumonia, and 8 days later he presents with lower
intestinal bleeding. His prothrombin time is quite prolonged. What is the appropriate therapy?
A. Treatment with tissue plasminogen activator
B. Infusion of calcium citrate
C. Treatment with fresh frozen plasma and vitamin K
D. Rapid infusion of protamine (antidote for heparin overdose)

90. A woman whose blood type is A positive and who has always been healthy just delivered her second
child. The father’s blood type is O negative. Because the child’s blood type is O negative (O, Rh negative),
what would you expect to find in this child?
A. Erythroblastosis fetalis due to rhesus incompatibility
B. Erythroblastosis fetalis due to ABO blood group incompatibility
C. Both A and B
D. The child would not be expected to have erythroblastosis fetalis

91. A 2-year-old boy bleeds excessively from minor injuries and has previously had bleeding gums. The
maternal grandfather has a bleeding disorder. The child’s physical examination shows slight tenderness
of his knee with fluid accumulation in the knee joint. You suspect this patient is deficient in which
coagulation factor?
A. Prothrombin activator C. Factor VIII
B. Factor II D. Factor X

92. A patient has a congenital deficiency in factor XIII (fibrin-stabilizing factor). What would analysis of
his blood reveal?
A. Prolonged prothrombin time C. Prolonged partial thromboplastin time
B. Prolonged whole blood clotting time D. Easily breakable clot

93. Which agent is not effective as an in vitro anticoagulant?


A. Heparin
B. Warfarin (Coumadin)
C. Ethylenediamine tetraacetic acid (EDTA) (calcium chelator)
D. Sodium citrate (calcium chelator)

94. What would most likely be used for prophylaxis of an ischemic heart attack?
A. Heparin B. Warfarin C. Aspirin D. Streptokinase

95. A 63-year-old woman returned to work after a vacation in New Zealand. Several days after returning
home, she awoke with swelling and pain in her right leg, which was blue. She immediately went to the
emergency department, where examination showed an extensive deep vein thrombosis involving the
femoral and iliac veins on the right side. After resolution of the clot, this patient will require which
treatment in the future?
A. Continual heparin infusion C. Aspirin
B. Warfarin D. Vitamin K

96. Which coagulation pathway begins with tissue thromboplastin?


A. Extrinsic pathway C. Common pathway
B. Intrinsic pathway D. Fibrin stabilization
47
Blood Physiology
97. Which of the following would best explain a prolonged bleeding time test?
A. Hemophilia A C. Thrombocytopenia
B. Hemophilia B D. Warfarin use

98. Which of the following is appropriate therapy for a massive pulmonary embolism?
A. Heparin C. Aspirin
B. Warfarin D. Tissue plasminogen activator

99. What is the primary mechanism by which heparin prevents blood coagulation?
A. Antithrombin III activation C. Binding available calcium
B. Binding and inhibition of tissue factor D. Inhibition of platelet-activating factor

100. Macrocytes have an MCV > than:


A. 80 fl C. 90 fl
B. 95 fl D. 85 fl

101. Bleeding from a small cut in the skin, all are true EXCEPT:
A. Is normally diminished by local vascular spasm
B. Ceases within about five minutes in normal people
C. Is prolonged in severe factor VIII (antihemophilic globulin) deficiency
D. Is greater from warm skin than from cold skin
E. Is reduced if the affected limb is elevated

102. Each red blood cell contains Hb approximately:


A. 20 pg B. 25 pg C. 30 pg D. 35 pg

103. The most reliable RBC index is:


A. MCV B. MCH C. MCHC D. RBC diameter

104. The amount of hemoglobin present in 100 ml of red blood cells is called:
A. MCH C. Hb index
B. MCHC D. Price Jones index

105. MCHC does not usually exceed:


A. 30 g/dL B. 33 g/dL C. 37 g/dL D. 25 g/dL

106. Primary hemostasis refers to cessation of bleeding due to:


A. Formation of a definitive clot
B. Clot retraction
C. Formation of a temporary platelet plug

107. Platelet aggregation is stimulated by:


A. Thromboxane A2 and fibrinogen C. Epinephrine
B. ADP and thrombin D. All of the above

108. Platelet aggregation is inhibited by:


A. ADP C. PGI 2
B. 5-HT D. Thromboxane A2
48
Blood Physiology
109. Which of the following clotting factors is not vitamin K dependent?
A. Factor II C. Factor VII E. Factor X
B. Factor III D. Factor IX

110. The extrinsic pathway is triggered by the release of:


A. Factor VII C. Tissue factor pathway inhibitor
B. Tissue factor D. Contact factor

111. The enzyme that ultimately lyses fibrin is:


A. Plasminogen B. TPA C. Urokinase D. Plasmin

112. The extrinsic pathway is inhibited by:


A. Tissue factor C. Tissue factor pathway inhibitor (TFPI)
B. Thromboplastin D. Contact factor

113. The test that screens the extrinsic pathway is:


A. Prothrombin time (PT)
B. Activated partial thromboplastin time (APTT)
C. Thrombin time
D. Urea solubility test
E. Clot lysis time

114. Red cell antigens A and B are chemically:


A. Phospholipids B. Glycolipids C. Glycopeptides D. Polypeptides

115. Red blood cell antigens A and B are also present in:
A. Saliva, and semen B. Amniotic fluid C. Pancreas D. All of the above

116. CO is formed as an end product of:


A. Heme metabolism B. Arginine metabolism C. Oxidation of acetoacetate

117. Heme is converted to bilirubin mainly in the:


A. Kidneys B. Liver C. Spleen D. Bone marrow

118. Which of the following is not synthesized in the liver?


A. Igg B. Α2 macroglobulin C. Albumin D. Angiotensinogen

119. ESR is increased in:


A. Anemia C. Spherocytosis
B. Hypofibrinogenemia D. All of the above

120. The protein content of lymph draining from the _______ is highest.
A. Choroid plexus C. Liver
B. Skeletal muscle D. Gastrointestinal tract

121. Osmotic fragility of red blood cells is decreased in:


A. Sickle cell anemia D. Macrocytic anemia
B. Hereditary spherocytosis E. (A) & (C)
C. Microcytic hypochromic anemia F. (B) & (D)
49
Blood Physiology

122. When a serum sample is electrophoresed, which one of the following bands is absent?
A. Albumin
B. Α1 globulin C. Α2 globulin D. Fibrinogen E. λ-globulin

123. Erythropoietin is produced mainly by:


A. Interstitial cells surrounding peritubular capillaries in the kidneys
B. Perivenous hepatocytes
C. Kupffer cells of liver

124. The average half-life of neutrophils in the circulation is:


A. 6 hours B. 5 days C. 2 weeks D. 1 month

125. Platelets, all true EXCEPT:


A. Are produced in the bone marrow
B. Increase in number after tissue damage
C. Have a small nucleus
D. Alter their shape when they make contact with collagen
E. Are activated by ADP and thrombin

126. The group A antigen, all true EXCEPT:


A. Is present on the red cells of a group A patient
B. May occur in the saliva of a group A patient
C. Is transmitted as an autosomal dominant characteristic
D. Is more common than the group B antigen
E. Is the most common cause of hemolytic disease of the newborn

127. The first cell that can be identified as belongings to RBC series is:
A. Basophil erythroblast C Reticulocyte E. CFU-S.
B. Proerythroblast D. Megaloblast

128. Mast cells and basophils play important role in some types of:
A. Allergic reactions D. Chronic inflammatory diseases
B. Parasitic infections E. Polycythemia
C. Acute infections

129. Vitamin K is required by liver for normal formation of:


A. Fibrinogen C. Platelets E. Fibrin
B. Globulins D. Prothrombin

130. Two antigens – type A and type B- are present:


A. On surface of RBCs D. Inside the lymphocytes
B. In plasma E. As a part of agglutinins
C. Along with plasma albumin

131. Extrinsic pathway for initiating clotting begins with:


A. Activation of factor XII D. Traumatized vascular wall
B. Formation of fibrin threads E. Conversion of prothrombin to thrombin
C. Blood trauma
50
Blood Physiology

132. A 52-year-old man is brought to the Emergency Department with severe chest pain. Angiography
demonstrates a severe coronary occlusion. A thrombolytic agent is administered to reestablish perfusion.
Which of the following does the thrombolytic agent activate?
A. Heparin C. Thrombin E. Prothrombin
B. Plasminogen D. Kininogen

133. A 42-year-old patient with a rare blood type is scheduled for surgery that will likely require a
transfusion. Because the patient has a rare blood type, an autologous blood transfusion is planneD. Prior
to surgery, 1500 mL of blood is collected. The collection tubes contain calcium citrate, which prevents
coagulation by which of the following actions?
A. Blocking thrombin C. Binding vitamin K E. Activating plasminogen
B. Binding factor XII D. Chelating calcium

134. Prior to having his first colonoscopy, a 50-year-old male undergoes a bleeding time test to rule out
any clotting disorders. Bleeding time is determined by nicking the skin superficially with a scalpel blade
and measuring the time required for hemostasis. It will be markedly abnormal (prolonged) in a person
who has which of the following?
A. Anemia C. Thrombocytopenia E. Hemophilia
B. Vitamin K deficiency D. Leukopenia

135. A 67-year-old woman with a history of venous thromboembolism is placed on warfarin (Coumadin)
prophylactically. The blood concentration of Coumadin becomes too high and bleeding occurs. The
bleeding can best be treated by the administration of which of the following?
A. Fibrinogen
B. Thrombin C. Platelets D. Protein C E. Vitamin K

136. A 61-year-old male presents to his family physician with the chief complaint of frequent diarrhea
accompanied by weight loss. He reports a tendency to bruise easily and laboratory data reveal a
prothrombin time of 19 seconds (normal = 11–14 seconds). The bruising and prolonged prothrombin time
can be explained by a decrease in which of the following vitamins?
A. Vitamin A C. Vitamin D E. Vitamin K
B. Vitamin C D. Vitamin E

137. Majority of clotting factors are produced in


A. Liver B. Kidney C. Heart D. Brain

138. Endothelial cells synthesize


A. Factor I (Fibrinogen) C. Factor 10 (Thrombokinase)
B. Factor 8 (antihemophilic factor) D. Factor 12 (Hageman factor)

139. Cellular immunity is due to


A. B lymphocytes B. T lymphocytes C. Neutrophils D. Eosinophils

140. Action of plasmin is


A. To remove calcium C. To stimulate heparin
B. Antithrombin action D. To degenerate fibrin

141. Which is the most rare blood group


51
Blood Physiology
A. A Rh+ B. AB Rh+ C. AB Rh- D. B Rh-

142. Hematocrit of 45% means that in the sample of blood analyzed


A. 45% Hb is in the plasma
B. 45% of total blood volume is made up of plasma
C. 45% of Hb is in the RBC
D. 45% of the total blood volume is made up of RBC's and WBC's

143. The normal Albumin/Globulin ratio in blood is


A. 1:2 B. 2:1 C. 1:3 D. 3:1

144. Which of the following statements concerning the monocyte is incorrect?


A. More common in blood than eosinophils and basophils
B. Produced in the adult by the bone marrow and lymph nodes
C. Unlike neutrophil does not accumulate outside circulation in area of inflammation
D. Not classified as a granulocyte

145. The antibodies that circulate in the plasma are made by


A. The liver, which makes all of the plasma proteins
B. The kidney, which monitors plasma composition
C. B lymphocytes, which are part of the immune system
D. The spleen, which uses them to tag red blood cells for destruction

146. If you have some blood from which the formed elements have been removed, how can you tell if it
is plasma or serum?
A. Plasma contains hemoglobin; serum does not
B. Serum is yellow; plasma has no color
C. Serum contains antibodies; plasma does not
D. Plasma contains clotting proteins; serum does not

147. Blood colloid osmotic pressure


A. Is the component of total blood osmotic pressure due to the presence of proteins in the plasma
B. Would increase if blood sodium levels increase
C. Would increase if the number of RBCs increases
D. Would increase in a person who has liver damage

148. In which choice are the major groups of plasma proteins listed in order from highest to lowest
percent?
A. Fibrinogen, globulins, albumins C. Albumins, globulins, fibrinogen
B. Globulins, fibrinogen, albumins D. Albumins, fibrinogen, globulins

149. In which choice is the plasma protein correctly matched with one of its functions?
A. Albumin - antibodies involved in immune response
B. Fibrinogen - attacks and destroys pathogens
C. Globulins - transport minerals, lipids, hormones and vitamins
D. Globulins - colloid osmotic pressure

150. The benefit of the biconcave shape of red blood cells is that it
A. Decreases the distance between the cell membrane and hemoglobin molecules, facilitating oxygen
52
Blood Physiology
Uptake
B. Allows the cells to hold more hemoglobin
C. Gives more room for the nucleus and organelles
D. Keeps them from leaking out through the walls of capillaries

151. Blood cells all develop from a common stem cell in bone marrow, but early in development they
split into separate lineages. Which three of these cells develop from the myeloblast lineage?
A. Basophil, eosinophil, neutrophil C. Thrombocyte, erythrocyte, lymphocyte
B. Lymphocyte, monocyte, erythrocyte D. Monocyte, erythrocyte, neutrophil

152. As red blood cells get older, they


A. Are less likely to become damaged
B. Become more fragile
C. Are less likely to rupture
D. Are less likely to be phagocytized in the spleen or liver

153. Damaged red blood cells are phagocytized by macrophages in the spleen or liver and their
components are Recycled, EXCEPT for the
A. Iron atom from the heme group, which is excreted in bile
B. Globin peptide chains, which are sent to the kidneys for excretion
C. Globin peptide chains, which are converted into bilirubin
D. Heme group (minus the iron atom), which is converted into a green pigment called biliverdin

154. What would you suspect to find if someone has high blood erythropoietin levels?
A. Blood oxygen deficiency C. A high red blood cell count
B. High blood oxygen levels D. A high hemoglobin level

155. Which hormone stimulates red blood cell formation?


A. Thrombin B. Transferrin C. Bilirubin D. Erythropoietin

156. Which condition would stimulate erythropoietin secretion?


A. High red blood cell count C. High hemoglobin levels
B. Low blood oxygen level D. Increased blood flow through kidneys

157. The lower the total amount of iron stored in the body,
A. The higher the rate of red blood cell formation
B. The higher the absorption of ingested iron in the intestine
C. The lower the level of transferrin in the blood
D. The higher the level of ferritin in the liver

158. Which blood component is NOT correctly matched with its function?
A. Erythrocytes - transport oxygen
B. Leukocytes - protect against disease
C. Platelets - phagocytize bacteria
D. Plasma proteins - maintain blood osmotic pressure; involved in clotting

159. In which choice are the formed elements arranged in order of increasing size?
A. Red blood cell, neutrophil, platelet C. Neutrophil, platelet, red blood cell
B. Platelet, red blood cell, neutrophil D. Platelet, neutrophil, red blood cell
53
Blood Physiology

160. In adults, red blood cells are made in one place, spend most of their lifespan in another, and most
are finally destroyed in yet another place. Which choice lists these locations in chronological order?
A. Bone marrow; blood vessels; spleen C. Spleen; blood vessels; spleen
B. Bone marrow; spleen; blood vessels D. Blood vessels; spleen; bone marrow

161. Which type of leukocyte is correctly matched with its function?


A. Basophils - release histamine in areas of damaged tissue
B. Neutrophils - leave the blood vessels and become macrophages
C. Lymphocytes - phagocytize bacteria
D. Neutrophils - defend against parasitic worm infections

162. Joe had to have both of his kidneys removed and is now kept alive only by dialysis machines that
remove waste molecules from his blood and adjust electrolyte levels. Because Joe has no remaining
kidney tissue, he may also need
A. Clotting proteins C. Erythropoietin to stimulate RBC production
B. Vitamin B12 to stimulate RBC production D. A bone marrow transplant

163. The term "hematopoiesis" refers to


A. The mechanisms that prevent blood loss
B. A disease in which there is insufficient blood cell production
C. The rupture of red blood cells
D. The process of blood cell formation

164. Without the RBC enzyme carbonic anhydrase


A. The blood would not be able to carry oxygen
B. The blood would not be able to coagulate
C. The blood would not be able to convert carbon dioxide into bicarbonate
D. We would not be able to remove and break down heme

165. What is the role of the nitric oxide (NO) carried by hemoglobin?
A. NO stimulates the formation of more red blood cells
B. NO relaxes blood vessels and decreases blood pressure
C. NO helps the hemoglobin binds to oxygen
D. NO prevents the hemoglobin from picking up carbon monoxide

166. Which process attracts leukocytes to damaged tissue?


A. Chemotaxis B. Hemostasis C. Hemopoiesis D. Agglutination

167. If someone has no megakaryocytes, he/she will


A. Not be able to make white blood cells
B. Not be able to make red blood cells
C. Have a reduced ability to prevent blood loss
D. Not be able to phagocytize bacteria

168. An overdose of erythropoietin would cause


A. Anemia C. Low blood viscosity
B. Polycythemia D. Low blood pressure
54
Blood Physiology
169. All of these describe normal erythrocytes EXCEPT
A. Biconcave disk C. Contain mitochondria
B. Contain no nucleus D. Contain hemoglobin

170. Which choice represents an abnormal value?


A. Total RBC count - 5.1 million/ml C. Total platelet count - 250,000/ml
B. Total WBC count - 1500/ml D. Hemoglobin - 15 g/dL

171. Immature erythrocytes that contain a network of endoplasmic reticulum and normally make up
about 1 - 3% of circulating blood are called
A. Granulocytes C. Proerythroblasts
B. Basophilic erythroblasts D. Reticulocytes

172. Which statement is correct?


A. Thrombopoiesis occurs mainly in the lungs
B. Thrombopoietin is a hormone that stimulates platelet production
C. Platelets have a large nucleus containing many sets of chromosomes
D. Megakaryocytes divide by mitosis to produce platelets

173. In which choice are the leukocytes correctly arranged in order from most common to least
common?
A. Monocytes, lymphocytes, neutrophils, eosinophils, basophils
B. Lymphocytes, neutrophils, monocytes, eosinophils, basophils
C. Neutrophils, monocytes, lymphocytes, basophils, eosinophils
D. Neutrophils, lymphocytes, monocytes, eosinophils, basophils

174. Once released from the bone marrow into the circulation, the average life span of a red blood cell is
about
A. 72 hours B. One week C. 4 months D. 2 years

175. One molecule of hemoglobin


A. Contains two alpha globin proteins and two beta globin proteins
B. Contains one heme group
C. Contains 2 atoms of iron
D. Can carry 12 molecules of oxygen

176. In a red blood cell, carbon dioxide is attached to the


A. Iron of the Hemoglobin C. Plasma membrane
B. Amino acids of the Globin D. Heme of the hemoglobin

177. Erythropoietin is produced by


A. Red blood cells C. Liver
B. Red bone marrow D. Kidneys

178. Red blood cells do not use any of the oxygen that they carry, because
A. Oxygen is bound to heme and cannot be removed
B. They lack mitochondria to use oxygen for aerobic respiration
C. They do not need ATP
D. They use carbon dioxide instead of oxygen for energy production
55
Blood Physiology

179. In which choice are the proteins listed in the order in which they are activated?
A. Prothrombin activator, thrombin, fibrin
B. Fibrin, thrombin, prothrombin activator
C. Thrombin, fibrin, prothrombin activator
D. Thrombin, prothrombin activator, fibrin

180. Platelet plug formation is initiated when platelets come into contact with
A. Undamaged endothelial cells C. Subendothelial collagen fibers
B. Fibrinogen D. Antithrombin

181. Which statement about hemostasis is NOT correct?


A. Positive feedback rapidly increases the amount of activated clotting factors
B. The intrinsic mechanism occurs in response to tissue damage
C. During clotting, blood cells and platelets are trapped in a mesh of protein fibers
D. The extrinsic mechanism is initiated by tissue factor (thromboplastin), released by damaged tissues

182. Repair of damaged blood vessel walls is stimulated by


A. Tissue thromboplastin C. Plasminogen
B. Platelet-derived growth factor D. Serum

183. After preliminary testing, a man who had had a stroke was given tissue plasminogen activator (tPA)
at the hospital. The reason for this treatment is that tPA
A. Activates an enzyme that dissolves clots
B. Prevents the formation of clots by removing calcium from the blood
C. Prevents the formation of thrombin
D. Initiates repair in damaged blood vessels

184. A blood clot that forms in an undamaged vessel is called a(n) _______, and if it breaks loose and
travels through the bloodstream it is called a(n) __________.
A. Hematoma; thrombosis C. Thrombus; embolus
B. Thrombus; hematoma D. Embolus; thrombus

185. The production of prostacyclin by endothelial cells


A. Means that blood clots do not normally form in healthy blood vessels
B. Attracts platelets
C. Exposes the collagen below the endothelial cells
D. Reduces the synthesis of clotting proteins by the liver

186. Which substance is NOT correctly matched to its function?


A. Antithrombin - inactivates thrombin
B. Prostacyclin - prevents platelet adhesion
C. Heparin - interferes with the formation of prothrombin activator
D. Plasminogen - initiates extrinsic mechanism

187. Which of these statements is correct?


A. Thrombin catalyzes the conversion of fibrinogen to fibrin
B. Ca++ is not required for coagulation
C. Tissue factor is always required for activation of factor X
56
Blood Physiology
D. The extrinsic mechanism causes coagulation when there is no tissue damage

188. Which molecule is a potent stimulator of coagulation?


A. Prostacyclin C. Plasmin
B. Tissue factor or thromboplastin D. Tissue plasminogen activator

189. During platelet plug formation, platelets release these substances, which cause more platelets to
adhere to the plug.
A. Prostacyclin and NO C. Heparin and plasmin
B. Erythropoietin and bilirubin D. ADP and thromboxane

190. Which molecule is a potent stimulator of coagulation?


A. Prostacyclin C. Plasmin
B. Tissue factor or thromboplastin D. Tissue plasminogen activator

191. During platelet plug formation, platelets release these substances, which cause more platelets to
adhere to the plug.
A. Prostacyclin and NO C. Heparin and plasmin
B. Erythropoietin and bilirubin D. ADP and thromboxane

192. What happens when fibrinogen is converted to fibrin?


A. A transfusion reaction C. A clot is formed
B. Iron is stored in the liver D. A platelet plug is formed

193. The term that refers to the percentage of packed erythrocytes per unit volume of blood is the:
A. Differential Count B. Hemoglobin C. Hematocrit D. Hemopoiesis

194. Which of the following is a circulating blood cell that is capable of differentiating into a plasma cell?
A. Neutrophil C. B lymphocyte E. Monocyte
B. Basophil D. T lymphocyte

195. The component of plasma responsible for maintaining the osmotic pressure of blood is:
A. Plasmin C. Fibrinogen E. Plasminogen activator
B. Albumin D. Gamma globulin

196. Erythrocytes:
A. Enter the circulation only after becoming fully mature
B. Undergo mitosis in the circulation in response to erythropoietin
C. Are removed from the circulation after about 120 days by macrophages in the spleen, liver, and bone
marrow
D. Have mitochondria and are capable of oxidative respiration
E. None of the above

197. Plasma is:


A. Blood that has no red blood cells
B. The liquid portion of blood including the clotting factors
C. The liquid portion of blood minus the clotting factors
D. The proteins of blood
57
Blood Physiology
198. Excessive destruction of erythrocytes is characteristic of:
A. Thalassemia C. Pernicious anemia
B. Aplastic anemia D. Hemolytic anemia

199. A hematocrit of 80 would be considered:


A. Polycythemia C. Thrombocytopenia
B. Anemia D. Leukemia

200. During hemoglobin recycling in the spleen, heme is initially converted into:
A. Bilirubin B. Stercobilin C. Urobilin D. Urobilinogen

201. The Fe 3+ portion of the hemoglobin is eventually:


A. Converted into transferrin in the large intestine
B. Converted into ferritin in the kidney
C. Excreted from the body
D. All of the above
E. None of the above

202. An increased neutrophil count is typically associated with:


A. An ongoing bacterial infection C. Allergic reactions
B. Neutropenia D. An ongoing parasitic infection

203. ________________ eventually become macrophages.


A. Neutrophils C. Monocytes E. Lymphocytes
B. Basophils D. Macrocytes

204. Hypoxia induces the kidneys to produce:


A. Platelets C. Erythrocytes E. Intrinsic Factor
B. Thrombopoietin D. Erythropoietin

205. The first phase of hemostasis is:


A. Separation of globin and heme C. Platelet aggregation
B. Activation of Prothrombin D. Vascular spasm

206. Which of the following activates platelets during hemostasis?


A. Eosinophil degranulation C. Fibrin thread formation
B. Exposed collagen or endothelial basement D. Thromboplastin
membrane

207. The phase of coagulation that begins with exposed endothelial collagen is the:
A. Extrinsic pathway C. Common pathway
B. Intrinsic pathway D. Fibrin stabilization phase

208. An individual with type B+ blood has which of the following antibodies in their blood?
A. anti-A and anti-O C. anti-A E. anti-Rh
B. anti-B and anti-Rh D. anti-B

209. Which of the following blood cells have some properties similar to connective tissue mast cells?
A. Basophils B. Neutrophils C. Eosinophils
58
Blood Physiology
D. Lymphocytes E. Monocytes

210. Examples of erythrocytes can be found in almost all histological sections. Therefore, knowledge of
the approximate diameter of a red blood cell is useful because it can serve as a built-in ruler on the tissue
section. Which of the following best describes the diameter of red cells?
A. 5-6um C. 7-8um E. 9-10um
B. 6-7um D. 8-9um

211. A. Which of the following might trigger erythropoiesis?


B. Increased tissue demand for O2 D. An increased number of RBCs
C. Decreased tissue demand for O2 E. Moving from a high altitude to a lower altitude

212. An individual who is blood type AB negative:


A. Can receive any blood type in moderate amounts EXCEPT that with the Rh antigen
B. Can donate to all blood types in moderate amounts
C. Can receive types A, B, and AB but not type O
D. Can donate to types A, B, and AB but not to type O

213. Which of the following is not a phase of hemostasis?


A. Vascular spasm C. Platelet plug formation
B. Fibrinolysis D. Coagulation

214. Which of the following is characteristic of all leukocytes?


A. They are nucleated C. They have cytoplasmic granules
B. They have lobed mitochondria D. They are phagocytic

215. Which sequence is correct for the following events?


1. Fibrinogen  Fibrin
2. Clot retraction
3. Formation of thromboplastin
4. Prothrombin --> Thrombin
A. 1,2,3,4 B. 3,4,1,2 C. 4,3,1,2 D. 3,2,1,4

216. Platelets:
A. Stick to the damaged area of a blood vessel and help to seal the break
B. Have a lifespan of about 120 days
C. Are the precursors of leukocytes
D. Have multiple nuclei
E. Two of the above

217. Fred's blood type was determined to be AB+. Which of the following is true of Fred's blood?
A. There are no antibodies to antigens A, B, or Rh in the plasma
B. The RBCs contain the A and B antigens on their nuclei
C. The blood totally lacks the Rh factor
D. He cannot receive blood from someone who is type O negative

218. All of the following conditions impair coagulation EXCEPT:


A. Vascular spasm C. Severe hypocalcemia
B. Vitamin K deficiency D. Liver disease
59
Blood Physiology

219. All leukocytes share the following characteristics EXCEPT:


A. Ability to perform diapedesis
B. Disease-Fighting
C. Distorted, Lobed Nuclei
D. More active in connective tissues than in blood

220. What is the difference between a thrombus and an embolus?


A. One occurs within the cerebral bloodstream while the other does not
B. Emboli occur only once
C. An embolus cannot contribute to the occlusion of a coronary vessel while a thrombus cannot contribute
to a blockage of a cranial arteriole
D. A thrombus must travel to become an embolus

221. The plasma component that forms the fibrous skeleton of a clot is:
A. Platelets B. Fibrinogen C. Thromboplastin D. Thrombin

222. Rank the following in order of abundance:


1. Erythrocyte
2. Basophil
3. Monocyte
4. Platelet
A. 1,2,3,4 B. 1,4,3,2 C. 1,4,2,3 D. 1,3,4,2

223. If Jack has type O+ blood, then which of the following is true?
A. He has no agglutinins at all C. He has no Rh agglutinogens
B. He has no A agglutinogens D. None of the above

224. Anemia is:


A. Any condition in which the body has an abnormally low oxygen carrying capacity
B. Only a genetic disorder
C. Marked by a massive increase in blood viscosity
D. Never the result of nutrient deficiency

225. Which of the following is not involved in coagulation?


A. Gamma globulins C. Vitamin K
B. Calcium D. Prothrombin

226. Which of the following people would have the most anti-B antibodies?
A. An adult male with B+ blood C. An adult female with B-blood
B. An adult male with O- blood D. A female child with AB+ blood

227. If a newborn was blood type O+, which of the following would his blood not contain?
A. Anti-B antibodies
B. Anti-A antibodies
C. Anti-Rh antibodies
D. Erythrocytes which express neither the A nor the B antigen

228. Which of the following is not true?


60
Blood Physiology
A. Blood is a connective tissue
B. Blood contains both a cellular and a non-cellular portion
C. None of the blood cells contain true nuclei
D. The pH of blood is typically between 7.35 and 7.45

229. In a sample of centrifuged blood, the layer immediately beneath the plasma:
A. Can contain neutrophils C. Is called the buffy coat
B. Can contain platelets D. All of the above

230. Water makes up what % of the plasma?


A. 90 B. 70 C. 50 D. 30

231. If you observed a white blood cell with a light microscope and noticed the presence of granules,
how would you classify it?
A. Granulocyte C. It could be either of the above
B. Agranulocyte D. It could be neither of the above

232. Which of the following is true of lymphocytes?


A. They contain a nucleus that occupies only a small portion of the cell
B. They are divided into T and B lymphocytes
C. The majority of them are found within the blood
D. They specific granules visible under the light microscope
E. All of the above

233. Basophils, eosinophils, and neutrophils are all:


A. Leukocytes B. Granulocytes C. Phagocytes D. All of the above

234. In the blood, iron is primarily transported by:


A. Hemosiderin B. Ferritin C. Stercobilin D. Transferrin

235. The stem cells that give rise to all red blood cells, white blood cells, and platelets are known as:
A. Hemocytoblasts C. Hemophilioblasts
B. Reticulocytes D. Myeloid stem cells

236. Which of the following anemia types is an example of a genetic disorder?


A. Sickle cell C. Thalassemia E. 2 of the above
B. Iron deficiency D. Hemorrhagic

237. Why is vitamin B12 necessary for erythropoiesis?


A. It's necessary for the ejection of the RBC nucleus
B. It's necessary for DNA synthesis
C. It's essential for protein aggregation
D. It's essential for oxygen's nucleophilic reaction with hemoglobin

238. Complications of aplastic anemia generally include:


A. Suppressed immunity C. Clotting deficiencies
B. Impaired formation of all formed elements D. All of the above

239. Which of the following females does not have to be concerned with erythroblastosis fetalis?
61
Blood Physiology
A. A female with blood type OB. A female with blood type B+
A female with blood type O+ E. More than one of the above
C. A female with blood type ABD.

240. Which of the following is the most important function of plasma albumin:
A. acting as antibodies
B. contributing to the clotting process
C. contributing to the effective osmotic pressure of plasma
D. acting as binding proteins for thyroid and steroid hormones

241. Which of the following substances makes the greatest contribution to the osmolarity of plasma
(normally about 290 mOsm/L)?
A. protein C. electrolytes (E.g. Na+, Cl−, HCO3−)
B. glucose D. red blood cells

242. Which of the following substances comprise the greatest amount of solute in plasma by weight?
A. electrolytes (E.g. Na+, Cl−, HCO3−) C. glucose
B. protein D. amino acids

243. The hematocrit is a rough measure of the proportion of which of these components of blood?
A. plasma proteins B. deoxyhemoglobin C. red blood cells D. osmotic pressure

244. Which of the following substances is present in plasma but not in serum?
A. globulins B. fibrinogen C. erythrocytes D. inorganic ions

245. Although most of the dissolved solute in plasma by weight is proteins, most of the osmolality of
plasma is provided by its content of inorganic ions. This is because:
A. inorganic ions are charged particles
B. the molarity of inorganic ions in plasma is much greater than the molarity of the proteins
C. inorganic ions cannot pass through capillary walls
D. inorganic ions tend to hydrogen bond with adjacent water molecules

246. When analyzing the osmotic forces acting across the wall of the capillary, plasma albumin is the
greatest contributor even though inorganic ions make the most important contribution to plasma
smolality. This is because:
A. the weight of protein in plasma is greater than the weight of inorganic ions
B. the molecular weights of proteins is greater than the molecular weights of inorganic ions
C. proteins do not carry an electrical charge
D. the capillary wall is not permeable to protein

247. The most abundant cell type in blood is:


A. erythrocytes B. neutrophils C. platelets D. lymphocytes

248. Which of the following conditions would stimulate the synthesis and release of erythropoietin?
A. breathing 100% oxygen C. increasing the dietary intake of iron
B. living at high altitude D. giving an infusion of 500 ml of whole blood

249. Erythropoietin:
A. is secreted by red bone marrow B. stimulates erythrocyte synthesis
62
Blood Physiology
C. is a component of hemoglobin D. all of the above

250. The synthesis and release of erythropoietin is increased in a person who goes to live at high altitude
because the:
A. cardiac output is increased C. partial pressure of oxygen in blood is less
B. hematocrit is increased D. alveolar ventilation rate is increased

251. An increase in the number of red blood cells (polycythemia):


A. increases the viscosity of the blood
B. can occur in someone who goes to live at high altitude
C. can result from a high level of erythropoietin secretion
D. all of the above

252. Erythrocytes:
A. Synthesizes and release erythropoietin
B. have about a 7 day survival time after release into the blood
C. have an abundant supply of mitochondria
D. have a higher cellular concentration of potassium than is in plasma

253. Erythropoietin:
A. is a paracrine C. acts on the spleen
B. is synthesized by the spleen D. is elevated in persons living at high altitude

254. Related to platelets, which of the following is False?


A. They are formed in the bone marrow from megakaryocytes
B. They contain ADP and serotonin
C. They produce prostacyclin
D. They have no nucleus

255. The most abundant cell type in blood is;


A. Erythrocytes B. Neutrophils C. Platelets D. Lymphocytes

256. All the following stimulate erythropoietin production EXCEPT;


A. Living at high altitude C. Androgens
B. Anemia D. Increasing the dietary intake of iron

257. Plasmin;
A. Dissolves fibrin C. Is inhibited by protein C.
B. Is converted to plasminogen D. None of the above is true

258. If a person has group B blood;


A. He may have the genotype AB. C. He has anti-B antibodies in his plasma
B. He has antigen B on his red blood cells D. He has antigen A on his red blood cells

259. Each hemoglobin molecule;


A. Has four heme groups C. Transports four oxygen molecules
B. Has four polypeptide chains D. All of the above are true

260. The major function of neutrophils is;


63
Blood Physiology
A. Antibody production C. Phagocytosis
B. Destruction of parasites D. None of the above

261. Thrombin;
A. Is activated by thrombomodulin C. Inhibits platelet aggregation
B. Activates factors V, VIII, and XIII D. Is inhibited by warfarin

262. Polycythemia;
A. Increases the viscosity of blood
B. Can result from increased erythropoietin production
C. Can occur in someone who goes to live at high altitude
D. All of the above are true

263. The most abundant plasma protein is;


A. Albumin B. Thrombin C. Globulin D. Fibrinogen

264. Plasma proteins;


A. Are responsible for all the buffering capacity of blood
B. Can act as a source for rapid replacement of tissue proteins
C. Of the immunoglobulin variety are manufactured mainly in the liver
D. Are of two major types; globulins and fibrinogen

265. Concerning hemopoiesis, which of the following is True?


A. Colony-stimulating factors (CSFs) stimulate the conversion of multipotent uncommitted stem cells to
committed stem cells
B. CSFs control the production of granulocytes and monocytes
C. CSFs are produced mainly by kidney cells
D. Colony-forming units- megakaryocytes produce macrophages

266. Catabolism of hemoglobin yields;


A. CO B. Iron C. Amino acids D. All of the above

267. Concerning white blood cells, all the following are False EXCEPT;
A. Monocytes after entering the tissues become macrophages
B. Eosinophil count is increased in viral infections
C. B. lymphocytes are responsible for cellular immunity
D. Helper T- cells suppress humoral immunity

268. The intrinsic pathway of blood coagulation begins with the activation of;
A. Factor VII B. Factor XII C. Prothrombin D. Fibrinogen

269. In blood cell production, vitamin B12 is essential for;


A. Formation of hemoglobin C. Activation of erythropoietin
B. Formation of DNA. D. Iron absorption from the intestine

270. Related to B. cells, all the following are True EXCEPT;


A. They are lymphocytes
B. They convert to monocytes in response to antigens
C. They mediate humoral immunity
64
Blood Physiology
D. They can act as antigen presenting cells

271. Concerning platelets, which of the following is True?


A. Platelet aggregation is inhibited by thrombin
B. A fall in platelet count causes failure of clot retraction
C. Platelet aggregation is inhibited by thromboxane A2.
D. Platelets are normally more numerous than red blood cells

272. For blood clotting to occur normally, which of the following is False?
A. Calcium ions must be present
B. The liver must have an adequate supply of vitamin K.
C. There must be a sufficient dietary intake of vitamin C.
D. The liver must be functioning properly

273. Leukocytosis:
A. Is always due to pathological causes
B. Can be caused by exercise
C. Occurs when total white blood cell count is less than 4000/ microliter of blood
D. None of the above is true

274. Increased amounts of erythropoietin might be released from kidney EXCEPT when the:
A. Arterial PO2 is normal and the arterial O2 content is reduced
B. Arterial PO2 and arterial O2 content are reduced
C. Arterial PO2 is low and the saturation of hemoglobin with oxygen is much reduced
D. Tissue PO2 and renal blood flow are both increased

275. Macrocytic normochromic anemia:


A. Can occur after surgical removal of the stomach
B. Occurs in patients with iron deficiency
C. is characterized by low mean cell volume (MCV)
D. All of the above are true

276. Related to hemostasis and blood coagulation, all the following are true EXCEPT:
A. Thrombin decreases platelets aggregation
B. The coagulation mechanism can be initiated through the intrinsic or extrinsic pathway
C. Platelets are necessary for clot retraction to occur
D. Protein C has anticoagulant and fibrinolytic effects

277. Related to the plasma proteins, all of the following are true EXCEPT:
A. They include certain clotting factors
B. They can act as a source for rapid replacement of the tissue proteins
C. They are manufactured only in the liver,
D. They participate in the buffering capacity of the blood

278. Blood platelets:


A. Are formed in the bone marrow C. Have no nucleus
B. Are normally more numerous than leukocytes D. All of the above are true

279. Renal erythropoietic factor:


65
Blood Physiology
A. Slow down the process of erythropoiesis in the bone marrow
B. Inhibits the formation of erythropoietin
C. Is released from the kidney in response to hypoxia
D. Is released from the kidney in response to hyperbaric oxygen

280. Factor XIII and another plasma factor are necessary for the:
A. Activation of prothrombin C. Stabilization of fibrin
B. Conversion of fibrinogen into fibrin monomer D. Conversion of prothrombin into thrombin

281. Microcytic hypochromic type of anemia is most probably caused by a deficiency of:
A. Vitamin B12 B. Intrinsic factor C. Folic acid D. Iron

282. Hb S (sickle cell hemoglobin) is produced when:


A. Glutamic acid is replaced by valine at position 6 on beta chain of the Hb
B. Glutamic acid in replaced by valine at position 5 on alpha chain of the Hb
C. Valine is replaced by lysine at position 6 on beta chain of the Hb
D. Glutamic acid is replaced by lysine at position 6 on beta chain of the Hb

283. Intravascular erythrocyte breakdown represents an ancillary pathway of pigment catabolism


normally accounting for:
A. Less than 15% of the red cells destruction C. More than 50% of the red cells destruction
B. More than 90% of the red cells destruction D. More than 80% of the red cells destruction

284. Hemophilia A (classical hemophilia) is due to deficiency of:


A. Factor X B. Factor IX C. Factor VIII D. Factor XI

285. Normal platelets count and life span in the blood are:
A. 100000/cu.mm - 40 days C. 500000/cu.mm – 129 days
B. 150000/cu.mm – 60 days D. 250000/cu mm – 10 days

286. The most commonly used clinical index of erythropoietic activity in the blood is:
A. RBC level C. Reticulocytes level
B. Hb concentration D. Iron level

287. Fetal hemoglobin has more affinity to O2 than adult hemoglobin because:
A. HbF binds less avidly to 2,3 DPG.
B. HbA binds less avidly to 2,3 DPG.
C. HbF has its alpha chain replaced by S chain
D. HbF has its beta chain replaced by Epsilon chain

288. When MCV is 110 µm3 and MCHC is 20% the cells are:
A. Microcytic hypochromic C. Macrocytic hyperchromic
B. Macrocytic hypochromic D. Normocytic normochromic

289. The common pathway of intrinsic and extrinsic mechanisms of blood coagulation is started with the
activation of:
A. Factor IX B. Factor XI C. Factor VII D. Factor X

290. Erythrocyte sedimentation rate (ESR) increases when there is:


66
Blood Physiology
A. Decreased fibrinogen and globulins C. Sickle cell anemia
B. Increased fibrinogen and globulins D. Decreased rouleaux formation

291. In intrinsic pathway of hemostasis factor ten (X) is activated by:


A. Active factor IX only C. Ca + and active factor IX only
B. Factor VIII and ADP only D. All the above

292. Aggregation of the platelets occurs when there is:


A. Increased prostacyclin and thromboxane
B. Decreased prostacyclin and increased thromboxane
C. Decreased prostacyclin and thromboxane
D. Increased prostacyclin and decreased thromboxane

293. The stored iron in the body which can be used again on demand is in a form of:
A. Transferrin B. Hemosiderin C. Ferritin D. Protoporphyrin

294. Platelets tend to aggregate when:


A. Platelets and endothelial cyclooxygenase pathways are both increased
B. Platelets and endothelial cyclooxygenase pathways are both decreased
C. Platelets cyclooxygenase pathway is decreased while endothelial cyclooxygenase pathway is increased
D. Platelets cyclooxygenase pathway is increased while endothelial cyclooxygenase pathway is decreased

295. what is the MCV of RBCs for the following given data? PCV = 45%, RBC count = 5,340,000/ul, Hb =
15.6 gm/dL.
A. 84.2 fl B. B. 85.2 fl C. C. 86.2 fl D. D. 87.2 fl

296. what is the MCHC of RBCs for the following given data: PCV = 45%, RBC count = 5,340,000/ul, Hb =
15.6 gm/dL.
A. 30.1%. C. 32.9%. E. 34.7%.
B. 31.6%. D. 33.4%.

297. Regarding binding of oxygen with hemoglobin (Hb):


A. Oxygen loosely combines with 2 positive bonds of iron in Hb molecule
B. Oxygen firmly binds with 1 of the coordination bonds of iron atom
C. Oxygen is carried to the tissues in molecular form by hemoglobin
D. Oxygen is released by Hb into the tissues in ionic form
E. Oxygen firmly binds with 2 positive bonds of iron in Hb molecule

298. The 1st step of Heme synthesis takes place in the:


A. Mitochondria D. Peroxisomes
B. Golgi apparatus E. Nucleolus
C. Rough endoplasmic reticulum

299. Regarding the blood coagulation test “Prothrombin time”:


A. It is normally about 12 minutes
B. It indicates concentration of thrombin in blood
C. It involves intrinsic clotting pathway
D. It does not involve oxalation of removed blood
E. It shortens with rise in prothrombin concentration
67
Blood Physiology

300. A 24 year old African American man comes to the emergency room 3 hours after the onset of severe
back and chest pain which started when he was climbing up a mountain. He had an episode of same
symptoms five years ago. His values are Hb: 11 g/dL, Total WBC count: 12,000/mm3, Reticulocyte count:
25%. What is the diagnosis of this patient?
A. Acute blood loss D. End stage kidney disease
B. Sickle cell anemia E. Chronic blood loss
C. Anemia of chronic disease

301. Which of the following treatments would prevent a transfusion reaction?


A. Administration of plasma free blood
B. Administration of washed erythrocytes
C. Treatment with immunoglobulins
D. Treatment with mannitol
E. Proper cross-matching of donor red cells with recipient plasma

302. What happens following the presentation of an antigen by a macrophage?


A. Direct generation of antibodies D. Activation of helper T cells
B. Activation of cytotoxic T cells E. Activation of platelets
C. Increase in phagocytosis

303. What causes the release of histamine in an allergic reaction?


A. Binding of IgM to basophils D. Free radical stimulation of endothelial cells
B. Binding of IgE to mast cells E. Release of histamine by macrophages
C. Release of histamine by helper T cells

304. Which of the following blood units carries the least risk of inducing an immediate reaction in a type
B, Rh+ patient?
A. Type A +ive whole blood D. Type O +ive packed red cells
B. Type O +ive whole blood E. Type AB –ive packed red cells
C. Type AB +ive whole blood

305. Prothrombin level falls in the blood due to lack of


A. Vitamin B12 C. Phospholipids E. Sodium
B. Vitamin K D. Platelets

306. Bluish tint of the polycythemia person is because of excess of


A. Myoglobin C. Oxygenated Hb E. Sulphated Hb
B. Deoxygenated Hb D. Reduced Hb

307. A 30 years old male is brought to the hospital with history of gastrectomy. His skin appears lemon-
yellow. Investigations reveal hemoglobin 10 g/dL, odd shaped RBCs and Serum Vitamin B12 is low. He is
likely to be suffering from:
A. Blood Loss Anemia C. Megaloblastic Anemia E. Aplastic Anemia
B. Pernicious Anemia D. Hemolytic Anemia

308. The mechanism of action of the following substance involves chemotaxis:


A. Bacterial & viral toxins C. Regenerative products of inflamed tissues
B. Fungal & parasitic toxins D. Plasma clotting enzymes
68
Blood Physiology
E. Reaction Products caused by inflammation

309. Ahmad`s blood picture revealed macrocytic anemia of the following type:
A. Sickle cell anemia D. Bone marrow aplasia
B. Hereditary spherocytosis E. Pernicious anemia
C. Blood loss anemia

310. Ghazala was diagnosed as a case of classic hemophilia. The best possibility is that:
A. One of her X chromosomes code for deficiency of Factor VIII
B. Her mother was a hemophilia carrier
C. Her autosomes carry the mutant gene for hemophilia
D. She is suffering from von Willebrand’s disease
E. Her father had hemophilia & mother was a hemophilia carrier

311. A young boy was brought to hospital emergency with complaint of acute pain in right iliac fossa. On
history, examination & clinical investigation, he was diagnosed to have acute appendicitis. His Total WBC
count (per µL) is likely to be:
A. 4,000 C. 8,000 E. 14,000
B. 6,000 D. 10,000

312. A 34-year-old man with schizophrenia has chronic fatigue for 6 months. He has a good appetite but
has developed a dislike for vegetables since last 1 year because he hears voices that tell him that the
vegetables are poisoned. His physical and neurological examinations are normal. His hemoglobin level is
9.1 g/dL, leukocyte count is 10,000/mm and Mean Corpuscular Volume is 122 fl (normal 77-93 fl). Which
of the following is the most likely diagnosis?
A. Acute blood loss C. Hemolytic anemia E. Sickle cell anemia
B. Aplastic anemia D. Folic acid deficiency

313. By the age of 7 months, the primary site of hematopoiesis in a fetus is:
A. Thymus C. Spleen E. Red marrow
B. Liver D. Reticuloendothelial system

314. Iron deficiency anemia:


A. is of hypochromic, macrocytic type
B. Is commonly associated with chronic blood loss
C. Is common cause of anemia in infants below 1 year
D. May increase the number of megaloblasts
E. Is more common in males

315. Polycythemia:
A. Is associated with decrease in blood viscosity
B. May cause an increase in the mass of RBCs in blood
C. Is seen with increase in ESR.
D. Is a contagious disease

316. Beta Thalassemia is associated with:


A. An increase in alpha chain synthesis of hemoglobin molecule
B. A decrease in beta chain synthesis D. An increase in alpha and beta chain synthesis
C. An increase in beta chain synthesis E. More fetal hemoglobin
69
Blood Physiology

317. Bilirubin is:


A. Not bound with Albumin D. Not conjugated in liver
B. Is a useful body pigment E. Converted into colorless urobilinogen
C. Converted into biliverdin in gall bladder

318. Factor VIII is synthesized in:


A. Endothelium C. Kupffer cells E. Platelets
B. Hepatocytes D. Histiocytes

319. Vitamin K dependent coagulation factors are:


A. Factor II, VII and X C. Factor VII and XI E. Factor XI and XII
B. Factor X and XI D. Factor II, VII and XI

320. A lady presented with shock due to post-partum hemorrhage, her blood group was unknown but
became coagulated when mixed with serum containing anti-A antibodies and similarly her serum
coagulated with B positive blood. The most suitable blood group to be transfused in this case is:
A. O negative C. A positive E. B positive
B. O positive D. AB positive

321. A blood smear shows hypochromic, microcytic RBCs with moderate poikilocytosis. Most likely
diagnosis is:
A. Anemia of chronic disease D. Iron deficiency anemia
B. Autoimmune hemolytic anemia E. Microangiopathic hemolytic anemia
C. Folate deficiency anemia

322. A young boy complaining of tiredness and shortness of breath is diagnosed with anemia. His blood
picture shows microcytic and hypochromic anemia with normal count of WBCs & platelets & normal
shape of RBCs. This picture is seen in:
A. Aplastic anemia C. Megaloblastic anemia E. Thalassemia
B. Sickle cell anemia D. Blood loss anemia

323. A survivor of the Hiroshima blast during the Second World War presented to hospital with pale look
and purplish patches on the thin skin area. His hemoglobin is below normal. He is most likely suffering
from:
A. iron deficiency anemia C. megaloblastic anemia E. aplastic anemia
B. sickle cell anemia D. hereditary spherocytosis

324. Nori, a pure vegetarian comes to doctor with complaint of exertional dyspnea. On examination he
has pallor. Initial lab tests show Hb 7.4 g/dL, MCV is 115 fL. What is the most probable cause of
megaloblastic anemia in this patient?
A. Folic acid deficiency C. Iron deficiency E. G6PD deficiency
B. Vitamin B12 deficiency D. Hemolysis

325. Asim, a 5 years old boy falls from his bicycle and receives a cut over the knee. His tissue
macrophages & fibroblasts will move to the site of injury for repair by:
A. Ciliary movement C. Chemotaxis E. Whip- like movement
B. Ameboid movement D. Margination
70
Blood Physiology
326. Long term hypoxia may serve as a growth inducer of the following blood cells:
A. Basophils C. Macrophages E. Platelets
B. Monocytes D. Red blood cells

327. Red blood cells can pass from bone marrow to blood capillaries by diapedesis during:
A. Basophil erythroblast stage D. Proerythroblast stage
B. Mature erythrocyte stage E. Orthochromatic erythroblast stage
C. Reticulocyte stage

328. Regarding Erythropoietin & erythropoiesis:


A. It is formed mainly in liver & partly in kidneys
B. Hypoxia can stimulate erythropoiesis in the absence of erythropoietin
C. Relief from hypoxia inhibits erythropoietin induced erythropoiesis
D. Erythropoietin mainly stimulates production of hematopoietic stem cells
E. Erythropoietin is over stimulated in patients with renal diseases

329. This is a process which removes unneeded cloth when healing has occurred. Without this, blood
vessels would gradually become completely blocked.
A. Hematopoiesis D. Leukopenia
B. Fibrinolysis E. Thrombocytopenia
C. Leukopoiesis

330. This blood disease where hemoglobin becomes spiky and sharp due to a change in just one of the
287 amino acids in a beta chain of a globin molecule. These wrongly shaped blood cells tend to rupture
easily and dam up blood vessels, preventing accurate oxygen flow to the body.
A. Infectious Mononucleosis C. Thrombocytopenia E. Hemophilia
B. Leukemia D. Sickle-Cell Anemia

331. All of the following are types of anemia accept:


A. Hemorrhagic C. Aplastic E. Iron-deficiency
B. Hemolytic D. Reticulocytic

332. This type of lymphocyte functions in the immune response by acting directly against virus-infected
cells and tumor cells.
A. Neutrophils C. Monocytes E. Lymphoblasts
B. T-cells D. B-cells

333. During acute inflammation,


A. The edema fluid has unusually low fibrinogen content
B. Wide gaps appear in venular endothelium
C. Vasoconstriction lowers the capillary filtration pressure
D. The endothelial osmotic reflection coefficient for plasma proteins decreases
E. The hydraulic conductance of the venular wall decreases
F. Histamine is a common, early mediator

334. This type of lymphocyte gives rise to plasma cells, which produce antibodies that are released into
the blood.
A. Neutrophils C. Monocytes E. Lymphoblasts
B. B cells D. T cell
71
Blood Physiology

335. Macrophages are all of the following accept:


A. Actively phagogenic
B. Crucial in the body's defense against viruses, certain intracellular bacterial parasites, and chronic
infections
C. Provide a rough index of the rate at which RBCs are produced
D. Activate lymphocytes to mount the immune response
E. Monocytes differentiate into these

336. Leucopoiesis is which of the following?


A. The production of WBCs stimulated by chemical messengers
B. An abnormally low WBC count, commonly induced by drugs
C. A group of cancerous conditions involving WBCs
D. A type of leukemia involving abnormal myeloblasts
E. The ability of a WBC to slip out of capillary blood vessels

337. Leukopenia is what?


A. The production of WBCs stimulated by chemical messengers
B. An abnormally low WBC count commonly induced by drugs
C. A group of cancerous conditions involving WBCs
D. A type of leukemia involving abnormal myeloblasts
E. The ability of a WBC to slip out of capillary blood vessels

338. Leukemias are:


A. The production of WBCs stimulated by chemical messengers
B. An abnormally low WBC count commonly induced by drugs
C. A group of cancerous conditions involving WBCs
D. WBCs that protect the body from damage by bacteria, viruses, parasites, toxins and tumor cells
E. The ability of a WBC to slip out of capillary blood vessels

339. A leukocyte is:


A. The production of WBCs stimulated by chemical messengers
B. An abnormally low WBC count commonly induced by drugs
C. A group of cancerous conditions involving WBCs
D. WBCs that protect the body from damage by bacteria, viruses, parasites, toxins and tumor cells
E. The ability of a WBC to slip out of capillary blood vessels

340. Meaning "leaping across," this term refers to the ability of WBCs to slip out of capillary blood
vessels.
A. Thrombocytopenia C. Leukopoiesis E. Diapedesis
B. Leukopenia D. Fibrinolysis

341. This is a bleeding disorder where the number of circulating platelets is deficient, causing
spontaneous bleeding from small blood vessels all over the body.
A. Erythropenia C. Leukemia E. Diapedesis
B. Leukopenia D. Thrombocytopenia

342. This term refers to several different hereditary bleeding disorders that involve prolonged bleeding
into tissues from even minor tissue trauma, which can be life threatening.
72
Blood Physiology
A. Anemia C. Hemophilia E. Diapedesis
B. Leukemia D. Thrombocytopenia

343. The percentage of a blood's individual formed elements in comparison to the blood as a whole is
known as:
A. An embolism C. An embolus E. A thrombus
B. The hematocrit D. The heparin

344. This is composed largely of a soft network of reticular connective tissue bordering on wide blood
capillaries. It produces different numbers of each blood type as needed in response to body needs and
regulatory factors.
A. Kidneys C. Liver E. Leukocyte
B. White bone Marrow D. Red Bone marrow

345. This is a drug that inhibits thromboxane A2 formation. Studies show that, over several years, men
taking low doses of this showed a 50% reduction in incidences of heart attack.
A. Embolus C. Protein C E. Vitamin C
B. Thromboxan D. Aspirin

346. The process of RBC production is called:


A. Hemopoeisis C. Erythropoeisis E. Monopoeisis
B. Leukopoeisis D. Hematopoeisis

347. This type of agranulocyte contains a "U" shaped nucleus. They differentiate into macrophages
which protect against viruses, certain intracellular bacterial parasites, and chronic infections.
A. Lymphocyte C. Neutrophil E. Eosinophil
B. Monocyte D. Basophil

348. Which of the following treatments would prevent a transfusion reaction?


A. Administration of plasma free blood
B. Administration of washed erythrocytes
C. Treatment with immunoglobulins
D. Treatment with mannitol
E. Proper cross-matching of donor red cells with recipient plasma

349. Prothrombin level falls in the blood due to lack of


A. Vitamin B12 C. Phospholipids E. Sodium
B. Vitamin K D. Platelets

350. Which of the following is true with regards to reticulocytes?


A. Count is about 1% in peripheral blood
B. They are smaller than red blood cells
C. They are important in cell mediated immunity

351. Which of the following is most likely to give rise to a megaloblastic type of anemia?
A. Dietary deficiency of iron D. Excessive red cell destruction
B. Surgical removal of ileum E. Irradiation of bone marrow
C. Chronic blood loss
73
Blood Physiology
352. Which of the following is best applicable to normal erythrocytes?
A. They are rigid biconcave disks
B. About 10% contain nuclear material
C. They contain carbonic anhydrase
D. They release erythropoietin when destructed
E. They are larger than white cells

353. A WBC count 16000 mm3 with 72% polymorphonuclear leucocytes indicates
A. Normal state C. A chronic illness E. Dehydration
B. Acute infection D. Bone marrow failure

354. Which of the following involve in cell mediated immunity


A. Basophils C. T-lymphocytes E. Monocytes
B. B-lymphocytes D. Eosinophils

355. Which of the following plasma proteins is synthesized outside the liver?
A. Albumin C. α and β globulin E. prothrombin
B. Fibrinogen D. γ globulin

356. What would happen to red blood cells if the heme group were removed from hemoglobin?
A. Red blood cells would not be able to bind oxygen
B. Red blood cells would not be able to reproduce
C. White blood cells would not be able to reproduce
D. Blood clot formation would be inhibited

357. Which of the following statements about erythrocytes is correct?


A. They fight infection C. They lack a nucleus
B. They clot blood D. They are produced in the spleen

358. Where does hematopoiesis take place?


A. Lungs B. Pancreas C. Liver D. Bone marrow

359. The formation of a blood clot is known as which of the following?


A. Coagulation B. Chemotaxis C. Leucopoiesis D. Erythropoiesis

360. The process of coagulation is classically divided into how many pathways?
A. 3 B. 5 C. 2 D. 4

361. Select the statement about red blood cells that is incorrect.
A. Mature red blood cells lack nuclei C. Deoxyhemoglobin carries oxygen
B. Red blood cells contain hemoglobin D. Red blood cells lack mitochondria

362. The precursor of all lines of blood cells is the _______________.


A. Myeloblast B. Hemocytoblast C. Proerythroblast D. Progranulocyte

363. When red blood cells are worn out, part of their components are recycled while others are
disposed. Select the incorrect statement about destruction of red blood cells.
A. The greenish pigment, biliverdin, is recycled to the bone marrow
B. Iron is carried to the bone marrow by a protein called transferrin
74
Blood Physiology
C. Biliverdin and bilirubin impart color to bile
D. Macrophages in the liver and spleen destroy worn out red blood cells

364. Which dietary component(s) is/are needed for DNA synthesis, and thus greatly influence the
production of red blood cells?
A. Calcium C. Vitamin B12 and folic acid
B. Iron D. Protein

365. The type of anemia that is fairly common and caused by insufficient dietary iron is ______________.
A. Aplastic anemia C. Hemolytic anemia
B. Pernicious anemia D. Iron deficiency anemia

366. The type of white blood cell that is often the first to arrive at the site of infection is a granulocyte,
and the member of this group that stains light purple is a _______________.
A. Basophil B. Eosinophil C. Neutrophil D. Monocyte

367. The largest cells in the blood that leave the bloodstream to become macrophages are the
_______________.
A. Eosinophils B. Monocyte C. Basophil D. Neutrophil

368. A person with eosinophilia, or greater than normal numbers of eosinophils, is most likely suffering
from ____________.
A. Allergies or internal parasites C. An autoimmune disease
B. Anemia D. Diabetes

369. An acute infection would show up in a blood count as ______________.


A. Leukopenia C. Too few helper T cells
B. Leukocytosis D. Thrombocytopenia

370. Which plasma proteins help transport lipids in the bloodstream?


A. Albumins C. Fibrinogen
B. Alpha and beta globulins D. Gamma globulins

371. Choose the correct order for the steps of hemostasis.


A. Blood coagulation, platelet plug formation, blood vessel spasm
B. Platelet plug formation, blood coagulation, blood vessel spasm
C. Blood vessel spasm, platelet plug formation, blood coagulation
D. Blood vessel spasm, blood coagulation, platelet plug formation

372. Which clotting factor is released from damaged tissue, and initiates a chain of clotting events?
A. Prothrombin C. Fibrin
B. Thrombin D. Tissue thromboplastin

373. A person with type AB blood would have ____________ antigens on red blood cells, and
___________ antibodies carried in the plasma.
A. A and B; neither anti-A or anti-B C. B; anti-A
B. Neither A nor B; both anti-A and anti-B D. A; anti-B
75
Blood Physiology
374. Erythroblastosis fetalis, also known as hemolytic disease of the newborn, most often occurs in
______________ mothers carrying ______________ fetuses.
A. Rh+; Rh- C. Type A; type O
B. Rh-; Rh+ D. Type B; Type A

375. Which blood cell can be described as being a biconcave disc?


A. Platelet B. Neutrophil C. Eosinophil D. Erythrocyte

376. The liquid portion of blood is referred to as _____.


A. Whole blood B. Hematocrit C. Plasma D. Serum

377. Which of the following does not belong with others?


A. Erythrocyte B. Serum C. Leukocyte D. Platelet

378. In a young female having iron deficiency, her blood tests show hemoglobin=8 g/dL, MCV=75 fl,
MCH=26 pg, MCHC= 28 g/dL. The type of anemia she is suffering from:
A. Hypochromic normocytic D. Normochromic microcytic
B. Hypochromic microcytic E. Normochromic macrocytic
C. Normochromic normocytic

379. During blood coagulation, thromboplastin is released by


A. RBC C. leucocytes
B. blood plasma D. clumped platelets and damaged tissues

380. T lymphocytes gain immunocompetence in the:


A. Thymus
B. Thymus for T-helper cells and Bone marrow for T-killer cells
C. Thymus for T-killer cells and Bone marrow for T-helper cells
D. Bone marrow

381. Vaccinations are an example of:


A. Naturally-acquired active immunity C. Naturally-acquired passive immunity
B. Artificially-acquired active immunity D. Artificially-acquired passive immunity

382. Only _______ and ________ are able to activate complement.


A. IgG and IgA B. IgG and IgE C. IgG and IgM D. IgG and IgD

383. ____________________ are activated by antigen fragments complexed with MHC II proteins.
A. CD8 T cells B. CD4 T cells C. CD8 B cells D. CD4 B cells

384. ____________________ are activated by antigen fragments complexed with MHC I proteins.
A. CD8 T cells B. CD4 T cells C. CD8 B cells D. CD4 B cells

385. Which of the following antibodies is able to confer natural passive immunity?
A. IgD B. IgE C. IgG D. IgM

386. The formation of an antigen-antibody complex can lead to:


I. Agglutination
II. Neutralization
76
Blood Physiology
III. Activation of complement
A. I and II only B. II and III only C. I and III only D. I, II, and III

387. An example of a primary lymphoid organ in humans is the:


A. Thyroid C. Thymus E. 2 of the above are correct
B. Bursa of Fabricius D. Pancreas

388. Macrophages are examples of:


A. Antibody-secreting T lymphocytes C. Activated plasma cells
B. Antigen-presenting cells D. All of the above

389. Which of the following are involved in B cell activation?


A. Antigen C. Cytokines such as Interleukins
B. T-helper cell D. All of the above

390. If Clare has no T-helper cells, she would have:


A. Impaired cell-mediated immunity C. Both a and b
B. Impaired antibody-mediated immunity

391. Which of the following are not phagocytes?


A. Eosinophils B. Microglia C. Mast cells D. Plasma cells

392. Which of the following is associated with complement?


A. Bacterial lysis C. Release of histamine by basophils
B. Opsonization D. All of the above

393. In situation A, you have the body cells displaying class I MHC proteins. In situation B, you have the
body cells displaying class II MHC proteins.
A. in A > in B B. in A < in B C. in A = in B

394. In humans, B lymphocytes typically gain immunocompetence in the:


A. Thymus B. Bursa of Fabricius C. Bone Marrow D. 2 of the above

395. T cells bearing the CD4 surface glycoprotein are known as Helper T cells and only bind to antigens
linked to class II MHC proteins.
A. This statement is true
B. This statement is false because helper T cells display the CD8 glycoprotein
C. This statement is false because helper T cells only bind to antigens linked to class I MHC proteins

Use these 6 possible choices for the next 3 questions.


A. Lymphocytes
B. Macrophages
C. Reticular cells
D. Plasma cells
E. Antigen-presenting cells
396. Non-phagocytic cells that bind antigen to their surfaces and present them to lymphocytes for
recognition and stimulation
397. Derived from blood monocytes
398. Secrete immunoglobulins
77
Blood Physiology

399. Which of the following is the typical site of T and B cell activation?
A. Red marrow C. Lymph nodes E. All of the above
B. Thymus D. Lymph vessels

400. The most abundant immunoglobulin type is:


A. IgA B. IgM C. IgF D. IgG

401. Complement:
A. Interferes with viral replication C. Aids in antigen presentation
B. Is involved in antibody production D. Causes cell lysis

402. Regarding IgG and IgM, all the following are true EXCEPT:
A. IgM is a pentamer while IgG is a monomer D. IgG has 2 antigen-binding sites while IgM has 10
B. IgM is found in lower amounts in the serum E. IgG is made by plasma cells while IgM is not
C. IgG can cross the placenta while IgM cannot

403. An increase in body temperature helps repel pathogens by:


A. Increasing the rate at which WBC's can be mobilized
B. Accelerating repairs
C. Inhibiting pathogen growth
D. All of the above

404. The following may be found in normal adult venous blood EXCEPT:
A. 0.5% carboxyhemoglobin C. 70% oxyhemoglobin
B. 1% methemoglobin D. 5% fetal hemoglobin

Answer the first 45 items with:


a. Increase (I)
b. Decrease (D)
c. Stay the same (S)
405. An increase in plasma levels of erythropoietin will cause blood viscosity to:
406. The activation of plasminogen will cause the size of a blood clot to:
407. Dehydration would cause the hematocrit to:
408. A decline in plasma albumin levels would cause plasma osmotic pressure to:
409. An increase in plasma levels of erythropoietin would cause RBC count to:
410. Aspirin causes the likelihood of platelet activation to:
411. A reduced ability to produce thrombin would cause the time required for blood clot formation to:
412. A decrease in white blood cell count would cause the likelihood of an infection to:
413. A large decrease in RBC count would cause blood viscosity to:
414. During the body’s response to an acute bacterial infection, you would expect the neutrophil count
to:
415. During the vascular spasm phase of hemostasis, the diameter of the affected blood vessel will:
416. If the liver was unable to produce normal quantities of plasma proteins, plasma osmotic pressure
would:
417. A blocked bile duct would cause plasma levels of bilirubin to:
418. In response to a significant increase in altitude, the body’s production of erythropoietin will:
419. In response to a decrease in blood oxygen content, plasma levels of erythropoietin would:
420. As plasma osmotic pressure decreases, interstitial fluid formation will:
78
Blood Physiology
421. As the percentage of blood occupied by plasma increases, blood viscosity will:
422. During an acute bacterial infection, blood neutrophil count will:
423. Prostacyclin causes the likelihood of platelet plug formation to:
424. Plasmin causes plasma fibrin levels to:
425. An inability of the liver to secrete bile could cause plasma bilirubin levels to:
426. In response to infection, plasma leukocyte count would be expected to:
427. In response to damage to the wall of a blood vessel, the level of contraction of that blood vessel’s
smooth muscle will:
428. Calcium chelators cause blood clotting time to:
429. Urokinase is an enzyme that activates plasminogen. Thus, urokinase causes the likelihood of clot
formation to:
430. As erythropoiesis continues, the number of organelles in a developing RBC will:
431. A mismatched transfusion of Type AB blood into an individual with Type O blood would make plasma
bilirubin to:
432. During an infection the size of the buffy coat will:
433. Daily injections of erythropoietin will make the percentage of blood occupied by plasma:
434. During a severe allergic reaction, the number of eosinophils in the body would:
435. As plasma thrombin concentration increases, plasma fibrinogen concentration will:
436. If plasmin was activated prior to clot formation, the time it takes for clot formation to occur would:
437. During leukopenia the body’s ability to prevent bacterial infection will:
438. As the rate of red blood cell destruction increases, plasma bilirubin concentration will:
439. As erythropoietin secretion from the kidneys decreases, hematocrit will:
440. A decrease in intrinsic factor production would cause the blood’s oxygen carrying capacity to:
441. Aplastic anemia would cause the body’s WBC count to:
442. Infection with a parasitic fluke would cause the number of granulocytes in the body to:
443. Large doses of antibiotics would cause the ability of the body to form thrombin to:

444. Which of the following is NOT correctly matched with its function?
A. Albumin – maintains plasma osmotic pressure D. Thrombin – involved in fibrin production
B. Transferrin – transports heme within the plasma E. Ferritin – storage of iron in the liver
C. Hemoglobin – transports oxygen

445. Which of the following would be the greatest source of blood viscosity?
A. Basophils C. Reticulocytes E. Lymphocytes
B. Erythrocytes D. Thrombocytes

446. Which of the following occurs in the red bone marrow?


A. Erythropoiesis C. Thrombopoiesis E. Only 2 of the above
B. Leukopoiesis D. All of the above

447. Which of the following contains granules filled with histamine and heparin?
A. Erythrocyte C. Eosinophil E. None of the above
B. Basophil D. Thrombocyte

448. The smallest formed element is the:


A. RBC C. Macrophage E. Hemocytoblast
B. Lymphocyte D. Platelet

449. Which of the following is the LARGEST?


79
Blood Physiology
A. Number of nuclei in a red blood cell
B. Normal percentage of WBCs in the blood that are neutrophils
C. Number of iron atoms in one hemoglobin protein
D. Number of pulmonary veins that empty into the left atrium
E. Normal temperature of blood in degrees Fahrenheit

450. If an individual’s intact endothelial cells were unable to secrete prostacyclin, his platelet plugs
would most likely be:
A. Larger than normal D. Composed primarily of leukocytes
B. Smaller than normal E. Both C and D are correct
C. Still normal

451. Which of the following is LEAST involved in the normal coagulation process?
A. Calcium chelators C. Prothrombin activator E. Fibrinogen
B. Vitamin K D. Calcium

452. Which of the following is TRUE?


A. T lymphocytes are an example of a polymorphonuclear leukocyte
B. A normal plasma volume for a male is 5 L.
C. Aplastic anemia is caused by destruction of red bone marrow
D. A decreased number of functioning platelets is known as thrombosis
E. None of the above

453. In a condition known as disseminated intravascular coagulation, bacterial toxins activate


thrombin. This would most likely cause:
A. Production of prothrombin activator D. Production of a platelet plug
B. Conversion of thrombin to prothrombin E. All of the above
C. Production of fibrin

454. Which of the following would cause a rise in plasma bilirubin levels?
A. Inability to secrete bile
B. Excessive destruction of white blood cells
C. The presence of vitamin K producing bacteria in the colon
D. Deficiency of erythropoietin

455. A deficiency in _________ could cause clotting problems, while a deficiency in ________ could
cause anemia.
A. Iron: Cobalt C. Copper: Manganese E. Calcium: Iron
B. Calcium: Ferrenium D. Sodium: Iron

456. Which of the following is characteristic of plasma?


A. 5 times as viscous as H2O D. 90% water
B. Scarlet to dark red in color E. All of the above
C. Volume of 5-6 L in males

457. Which of the following is a function of blood?


I. Gas transport
II. Hormone transport
III. Heat transport
80
Blood Physiology
A. I only C. III only E. I, II, and III
B. II only D. II and III only

458. The movement of white blood cells towards a chemical known as LPS would be an example of
________________________.
A. Diapedesis C. Amoebeosis E. None of the above
B. Positive chemotaxis D. Amphoteris

459. Which of the following WBCs contains granules filled with chemicals involved in the inflammatory
process?
A. Basophils C. Platelets E. Eosinophils
B. Lymphocytes D. Neutrophils

460. _______ is produced quickly by the _______ clotting mechanism and slowly by the __________
clotting mechanism.
A. Prothrombin activator – extrinsic – intrinsic D. Thrombin – intrinsic – extrinsic
B. Prothrombin activator – intrinsic – extrinsic E. Plasmin – intrinsic – extrinsic
C. Thrombin – extrinsic – intrinsic

461. Which of the following events associated with coagulation occurs LAST?
A. Clot retraction D. Formation of prothrombin activator
B. Fibrinolysis E. Formation of thrombin
C. Formation of fibrin

462. Which of the following leukocytes exerts the most control over the immune system?
A. Neutrophils C. Granulocytes E. Monocytes
B. Basophils D. Lymphocytes

463. Which of the following is NOT a site of erythropoiesis in the adult male?
A. Liver D. Ribs
B. Proximal epiphysis of the humerus E. Proximal epiphysis of the femur
C. Sternum

464. Which of the following shuttles iron to the liver?


A. Albumin C. Transferrin E. Hemosiderin
B. Stercobilin D. Ferritin

465. Which of the following is a function of blood?


A. Nutrient transport D. Immunity
B. Heat transport E. All of the above
C. Maintaining body temperature

466. Art vanDelay has a blood volume of 5 L. 59% of his blood is composed of RBCs. 1% of his blood is
composed of WBCs and platelets. How many liters of water does his blood contain?
A. 1 L B. 1.8 L C. 2 L D. 2.4 L E. 2.7 L

467. Bilirubin is formed from:


A. Heme C. Transferrin E. Factor X
B. Globin D. Gastroferritin
81
Blood Physiology

468. Hemocytoblasts are stem cells that


A. Can become reticulocytes but not erythrocytes D. Can become granulocytes but not agranulocytes
B. Can become neutrophils but not eosinophils E. None of the above
C. Can become megakaryocytes but not
erythrocytes

469. Hematocrit:
A. Would be expected to decrease in polycythemia
B. Would be expected to increase due to hemolytic anemia
C. Is typically between 28% and 48% in normal adult males
D. Could be increased due to an increase in erythropoietin release by the kidney
E. Is defined as the percentage of plasma that is occupied by packed red blood cells

470. Which of the following is INCORRECT?


A. In a female adult, aged red blood cells are destroyed in the spleen
B. In a female adult, red blood cells are primarily synthesized in the tibia and fibula
C. In a male child, red blood cells function in oxygen transport
D. Red blood cells that lack hemoglobin are less efficient at O2transport
E. Red blood cells lack nuclei

471. In the liver, iron can be stored as:


A. Hemosiderin C. Stercobilin E. Albumin
B. Bilirubin D. Transferrin

472. Which of the following is TRUE?


A. Blood is an example of a specialized epithelial tissue
B. Neutrophils and eosinophils are not formed elements
C. Water is 3 times as viscous as blood
D. Blood temperature is typically 18°C lower than core body temperature
E. Normal blood pH is basic

473. Blood is involved in:


I. Nutrient distribution
II. Transport of nitrogenous wastes
III. Maintaining adequate electrolytes levels in the body
IV. Transport of white blood cells
A. I, II, III, and IV C. II, III and IV E. I, III, and IV
B. I, II, and III D. I, II, and IV

474. Which of the following does NOT belong?


A. Plasma proteins C. Liver E. Calcium
B. Albumin D. Fibrinogen

475. Red blood cells:


A. Are about 7.5mm in diameter and biconcave
B. Function primarily in maintenance of interstitial fluid osmotic pressure
C. Contain more organelles than the normal muscle cell
D. Function in oxygen and carbon dioxide transport
82
Blood Physiology
E. All of the above

476. Which of the following is TRUE?


A. Oxyhemoglobin is more likely to be found in the superior vena cava than in the aorta
B. Deoxyhemoglobin is more likely to be found in the aorta than in the superior vena cava
C. Each molecule of hemoglobin can transport 1 molecule of oxygen
D. The binding of hemoglobin to oxygen is reversible
E. More than 1 of the above

477. Erythropoiesis:
A. Occurs in the yellow bone marrow
B. Is stimulated only by the hormone thrombopoietin
C. Includes cells known as hemocytoblasts, reticulocytes, and erythrocytes
D. Is always stimulated when blood oxygen content is increased
E. None of the above

478. Leukocytes:
A. Are also known as lymphocytes and function in immunity
B. Are capable of moving toward chemicals released by pathogens
C. Are normally found in blood at a count of 5,000-10,000 per 10ml of blood
D. Are formed in the red bone marrow from stem cells known as erythroblasts
E. Are primarily found within the bloodstream and rarely found in connective tissue

479. Monocytes and basophils:


A. Are both granulocytes D. Are both capable of diapedesis
B. Are both agranulocytes E. None of the above
C. Are the 2 least common WBCs in the blood

480. Platelet plug formation:


A. Is primarily a negative feedback process
B. Usually occurs after fibrinolysis
C. Is restricted to the site of injury by prostacyclin released by intact cells
D. Provides a permanent repair to a tear in a blood vessel wall
E. None of the above

481. Blood:
A. Is classified as a combination of epithelial and connective tissues
B. Has an average pH of 7.4, making it very acidic
C. Is 90% water
D. Has an average temperature of 30.9°C
E. None of the above is correct

482. Put the following steps of coagulation in the correct order.


1. Fibrinolysis
2. Prothrombin activator is formed
3. Subendothelial collagen is exposed
4. Thrombin is formed from prothrombin
5. Fibrinogen is converted to fibrin
A. 1-3-4-2-5 B. 3-1-4-2-5 C. 3-2-4-5-1
83
Blood Physiology
D. 3-4-2-5-1 E. 3-2-4-1-5

483. Which of the following can retard coagulation?


I. Low platelet count
II. Aspirin
III. Liver disease
IV. Lack of dietary or enteric vitamin K
A. I, II, III, and IV C. I, II, and IV E. I only
B. I, II, and III D. II, III, and IV

484. Which of the following is TRUE?


A. Platelets are also referred to as thrombocytes and are primarily synthesized in the liver
B. Platelets are fragments of cells known as megakaryocytes and they persist in the circulation for 10-12
days
C. A differential WBC count determines the % of blood occupied by leukocytes, erythrocytes, and platelets
D. Leukopenia, an abnormally high WBC count, is often caused by anticancer drugs
E. Lymphocytes never survive in the body for more than 30-60 minutes

485. Which of the following is NOT a function of blood?


A. Carbon dioxide transport D. Transport of immune proteins
B. Maintaining electrolyte balance in the body E. Hormone transport
C. Heat production

486. Hemoglobin:
A. Transports oxygen but not carbon dioxide D. Can bind both oxygen and carbon monoxide
B. Is contained within the nucleus of the mature RBC E. 2 of the above are true
C. Contains no metals

487. Which of the following is TRUE?


A. Red blood cells are the only formed elements made within the red bone marrow
B. 50% of RBCs began as hemocytoblasts
C. Reticulocytes account for 1-2% of all RBCs in healthy blood
D. As erythropoietin synthesis increases, the rate of RBC production will decrease
E. More than one of the above is correct

488. A typical RBC:


A. Lives for 120 days and then divides to yield 2 new RBCs
B. Could eventually be engulfed by a macrophage in the spleen
C. Lacks organelles but contains a polymorphic nucleus
D. Has a spherical shape, which helps minimize available surface area
E. More than one of the above is correct

489. Which of the following is TRUE of leukocytes?


A. They account for 9-10% of total plasma volume
B. They are not all produced in the red bone marrow
C. They are capable of diapedesis, which is the process by which they leave the bloodstream
D. All of the above
E. 2 of the above
84
Blood Physiology
490. Which WBC is NOT correctly matched with its function?
A. Eosinophils → destroy parasitic worms
B. Basophils → leave the bloodstream and become macrophages
C. T lymphocytes → attack virus-infected cells
D. Neutrophils → involved in bacterial infections
E. None of the above

491. Which of the following is TRUE?


A. Leukopenia is normal during a modest bacterial infection
B. During leukemia, platelet and erythrocyte counts can decrease
C. Platelet plug formation is a negative feedback process
D. Prothrombin activator converts thrombin into plasmin
E. Most clotting factors are synthesized by the spleen

492. Which of the following is TRUE?


A. Neutrophils are referred to as polymorphonuclear leukocytes
B. All monocytes within the bloodstream are referred to as macrophages
C. Normally there are less than one thousand lymphocytes in the body
D. When stained with Wright’s stain, eosinophils contain characteristic dark purple granules
E. All of the above

493. Which of the following is TRUE?


A. A massive increase in blood viscosity would cause the efficiency of blood flow to increase
B. Under conditions of low O2 sickle-cell hemoglobin becomes spiky and sharp and the red blood cells
acquire a crescent shape
C. Surgical removal of the stomach could NOT cause anemia
D. No product of bilirubin breakdown is excreted in the feces
E. All of the above

494. The shape of a red blood cell can best be described as _____________. One advantage of that
shape is that _______________________.
A. Biconcave: it makes the red blood cell more flexible
B. Spherical: it allows the red blood cell to maximize its organelle content
C. Biconcave: it compresses the red blood cell’s nucleus to allow for more frequent DNA transcription
D. Stellate: it increases the red blood cell’s surface area
E. None of the above is correct

495. Suppose a red blood cell contains 1 million molecules of hemoglobin. How many molecules of iron
does it contain?
A. 250,000 B. 1 million C. 4 million D. 10 million E. 1 billion

496. All of the following are PRODUCTS of RBC recycling/disposal EXCEPT:


A. Iron C. Bilirubin E. A and B
B. Amino acids D. Transferrin

497. All of the following are plasma proteins EXCEPT:


A. Albumin C. Fibrinogen E. All of the above are plasma
B. Prothrombin D. Antibodies proteins
85
Blood Physiology
498. Which of the following is the 5th most abundant FORMED ELEMENT in the blood?
A. Eosinophils C. Monocytes E. Neutrophils
B. Lymphocytes D. Platelets

499. What metal ion is essential to virtually all stages of coagulation?


A. Sodium C. Calcium E. Potassium
B. Cadmium D. Strontium

500. All of the following would be considered normal pH values for blood EXCEPT:
A. 7.35 C. 7.86 E. 2 of the above
B. 7.44 D. 7.02

501. Which of the following is NOT TRUE?


A. Platelet plug formation is a positive feedback process
B. An increase in plasma levels of thrombin will cause an increase in plasma levels of fibrin
C. Prostacyclin acts to inhibit platelet plug formation
D. Fibrinogen is primarily produced by the kidneys
E. All of the above

502. Which of the following is NOT TRUE?


A. The most abundant variety of circulating white blood cells are the lymphocytes
B. Bilirubin is transported to the liver by albumin
C. Iron is transported to the liver by transferrin
D. Each hemoglobin contains 4 heme groups
E. 2 of the above

503. Which of the following indicates the relative number of formed elements in a microliter of blood?
A. RBCs > WBCs > Platelets D. Platelets > RBCs > WBCs
B. RBCs > Platelets > WBCs E. None of the above is correct
C. WBCs > Platelets > RBCs

504. Which of the following is TRUE of blood?


A. Blood pH is usually between 7.75 and 7.85.
B. The human body contains a total of 4-6 milliliters of blood
C. Blood is a specialized type of epithelial tissue
D. Blood consists of formed elements and plasma
E. Blood is about 5 times less viscous than water

505. The majority of red blood cell destruction occurs in the:


A. Liver C. Pancreas E. Bile ducts
B. Kidney D. Spleen

506. A single molecule of hemoglobin can transport up to ___ molecules of oxygen.


A. 1 B. 2 C. 4 D. 8 E. 12

507. Which of the following components of plasma functions by preventing pH change?


A. Electrolytes such as K+ C. Wastes such as lactic acid E. Water
B. Nutrients such as glucose D. Buffers such as bicarbonate
86
Blood Physiology
508. All of the following are TRUE of red blood cells EXCEPT:
A. They do not contain a nucleus D. They function primarily in transport of CO2.
B. They are shaped like a biconcave disk E. They are formed in the red bone marrow
C. They contain hemoglobin

509. Which of the following is TRUE?


A. Basophils and lymphocytes are both granulocytes C. All leukocytes are granulocytes
B. Neutrophils and monocytes are both D. All agranulocytes are leukocytes
agranulocytes E. None of the above

510. Globulins produced by plasma cells function as __________, while albumin is important for
__________.
A. Clotting factors; osmotic pressure D. Clotting factors; heat transport
B. Antibodies; oxygen transport E. Leukocytes; clotting
C. Antibodies; osmotic pressure

511. During hemoglobin recycling, iron is transported to the liver by ___________ and stored there as
____________.
A. Transferrin – ferritin D. Transferrin – albumin
B. Ferritin - transferrin E. Basophils – hemoglobin
C. Albumin – transferrin

512. Prostacyclin is released by ____________ cells lining a blood vessel and it functions by __________
platelets.
A. Healthy – repelling D. Damaged – attracting
B. Damaged – repelling E. Damaged – destroying
C. Healthy – attracting

513. During the vascular spasm phase of hemostasis, the __________ muscles in the wall of the
damaged blood vessel will __________. This causes blood vessel diameter to _________.
A. Skeletal – contract – increase D. Smooth – relax – increase
B. Skeletal – relax – decrease E. Smooth – contract – increase
C. Smooth – contract – decrease

514. Which of the following is the only true nucleated cell?


A. Erythrocyte C. Basophil E. Thrombocyte
B. Red blood cell D. Platelet

515. All of the following are TRUE EXCEPT:


A. Platelets contain actin and myosin, which help during clot retraction
B. Plasmin is the chemical responsible for fibrinolysis
C. More prothrombin activator is made via the intrinsic path than via the extrinsic path
D. Calcium plays a role in blood clot formation
E. Leukopoiesis occurs primarily in the white bone marrow

516. Which of the following organs produces most of the plasma proteins?
A. Spleen C. Kidney E. White bone marrow
B. Liver D. Red bone marrow
87
Blood Physiology
517. Which of the following is most necessary for coagulation?
A. Vitamin A C. Vitamin E E. Vitamin Z
B. Vitamin D D. Vitamin K

518. Red blood cells:


A. Measured 15 um in diameter
B. Do not contain mitochondria
C. Have a life span of 120 days in the circulation
D. Are released from the bone marrow as mature erythrocytes
E. Express HLA class II antigens on the cell surface

519. The following are true about the ABO and rhesus (Rh) system:
A. A person of group O is a universal donor
B. A person who is group AB has anti-A and anti-B antibodies
C. The presence of the D antigen means that the subject is Rh positive
D. Rhesus antibodies occur naturally
E. The second rhesus positive baby of a Rh negative mother is at a greater risk than the first

520. The oxygen dissociation curve is shifted to the right with:


A. Polycythemia
B. Pyrexia
C. Respiratory acidosis
D. Sickle cell anemia
E. Decreased concentration of 2,3-DPG in the erythrocytes

521. When neutrophils and macrophages squeeze out of capillaries to fight off infection it is called:
A. Phagocytosis C. Interleukin E. Folliculitis
B. Hemolysis D. Diapedesis

522. During a great battle between your WBC's and an aggressive microbe, an inflammatory response
has been initiated redness and edema has kicked in what else does the body do to protect itself?
A. Histamine cause vasodilation D. Living and dead WBC and bacteria accumulate
B. Hypothalamus raises the thermostat E. All of the above
C. Neutrophils engulf and destroy the microbe

523. Specificity and memory are associated with which body defense mechanism?
A. Inflammatory response
B. Phagocytosis by macrophages and neutrophils
C. Interferon
D. T cell and B cell responses
E. Anatomical barriers in the body

524. An additional chemical defense found in tears and saliva?


A. T lymphocytes B. Saline C. Lysozyme D. B lymphocytes

525. Which of the following does complement protein perform


A. They cause antibody release C. The release if histamine E. Mast cell degranulation
B. T cell development D. Promotes tissue repair
88
Blood Physiology
526. Which substance induces fever?
A. Pyrogen B. Pus C. Monocytes D. Edema E. Interferon

527. Major function(s) of the lymphatic system is/are?


A. Provide route for return of extracellular fluid
B. Act as drain off for inflammatory response
C. Render surveillance, recognition, and protection against foreign materials via lymphocytes, phagocytes,
and antibodies
D. A and C
E. All of the above

528. An antigen is:


A. A chemical messenger that is released by virus infected cells
B. A lymphocyte responsible for cell-mediated immunity
C. Something that coats the inside of lungs, causing infection
D. A protein or other molecule that is recognized as non-self
E. A thick yellow-white fluid

529. A foreign substance, usually a protein that stimulates the immune system to react, such as by
producing antibodies is a ______________.
A. Allergen C. Histamine E. Interferon
B. Antigen D. Mast cell

530. When a macrophage ingests invading bacteria and takes the antigen to a lymph node, what
happens next?
A. The macrophage will present it to the first B-cell it encounters, and the B-cell will in turn change its
surface receptors to match the antigen
B. A b-cell will only become activated if it already has a match for the antigen
C. A matching B-cell will become activated into a cytotoxic T-cell
D. The cells of the lymph node will release histamine
E. The lymph node will increase production of neutrophils

531. What is the most common portal of entry for diseases, into the body?
A. Respiratory system C. Hematocrit system
B. Endocrine system D. Any opening into the body

532. This gland shrinks in size during adulthood, and has hormones that function in maturation of T-
lymphocytes:
A. Lymph nodes B. Thymus C. Spleen D. Tonsils

533. Which of the following is not a mechanical factor to protect the skin and mucous membranes from
infection?
A. Layers of cells B. Tears C. Saliva D. Lysozyme
89
Blood Physiology
1 A 21 C 41 B 61 E 81 D 101 C 121 E

2 A 22 D 42 D 62 A 82 D 102 C 122 D

3 C 23 A 43 D 63 A 83 D 103 C 123 A

4 B 24 A 44 C 64 A 84 D 104 B 124 A

5 B 25 D 45 E 65 D 85 B 105 C 125 C

6 A 26 D 46 C 66 A 86 D 106 C 126 E

7 A 27 C 47 D 67 A 87 C 107 D 127 B

8 D 28 D 48 D 68 B 88 D 108 C 128 A

9 C 29 B 48 B 69 B 89 C 109 B 129 D

10 B 30 C 50 C 70 C 90 D 110 B 130 A

11 B 31 D 51 C 71 D 91 C 111 D 131 D

12 D 32 D 52 A 72 D 92 D 112 C 132 B

13 D 33 D 52 C 73 C 93 B 113 A 133 D

14 B 34 E 54 B 74 B 94 C 114 B 134 C

15 B 35 D 55 C 75 A 95 B 115 D 135 E

16 B 36 A 56 C 76 B 96 A 116 A 136 B

17 B 37 D 57 C 77 C 97 C 117 C 137 A

18 B 38 E 58 C 78 C 98 D 118 A 138 B

19 D 39 C 59 C 79 B 99 A 119 A 139 B

20 C 40 D 60 A 80 E 100 B 120 C 140 D


90
Blood Physiology
141 C 161 A 181 B 201 E 221 B 241 C 261 B

142 D 162 C 182 B 202 A 222 B 242 B 262 D

143 B 163 D 183 A 203 C 223 B 243 C 263 A

144 C 164 C 184 C 204 D 224 A 244 B 264 B

145 C 165 B 185 A 205 D 225 A 245 B 265 B

146 D 166 A 186 D 206 B 226 B 246 D 266 D

147 B 167 C 187 A 207 B 227 C 247 A 267 A

148 C 168 B 188 B 208 C 228 C 248 B 268 B

149 C 169 C 189 D 209 A 229 D 249 B 269 B

150 A 170 B 190 B 210 C 230 A 250 C 270 B

151 A 171 D 191 D 211 A 231 A 251 D 271 B

152 B 172 B 192 C 212 A 232 B 252 D 272 C

153 D 173 D 193 C 213 B 233 D 253 D 273 B

154 A 174 C 194 C 214 A 234 D 254 C 274 D

155 D 175 A 195 B 215 B 235 A 255 A 275 A

156 B 176 B 196 C 216 A 236 E 256 D 276 A

157 B 177 D 197 B 217 A 237 B 257 A 277 C

158 C 178 B 198 D 218 A 238 D 258 B 278 D

159 B 179 A 199 A 219 C 239 E 259 D 279 C

160 A 180 C 200 A 220 D 240 C 260 C 280 C


91
Blood Physiology
281 D 301 E 321 D 341 D 361 C 381 B 401 D

282 A 302 D 322 D 342 C 362 B 382 C 402 E

283 A 303 B 323 E 343 B 363 A 383 B 403 D

284 C 304 D 324 B 344 D 364 C 384 A 404 D

285 D 305 B 325 B 345 D 365 D 385 C 405 I

286 C 306 B 326 D 346 B 366 C 386 D 406 D

287 A 307 C 327 C 347 B 367 B 387 C 407 I

288 B 308 A 328 C 348 E 368 A 388 B 408 D

289 D 309 E 329 B 349 B 369 B 389 D 409 I

290 B 310 E 330 D 350 A 370 B 390 C 410 D

291 D 311 E 331 D 351 B 371 C 391 D 411 I

292 B 312 D 332 B 352 C 372 D 392 D 412 I


B,
293 C 313 E 333 353 B 373 A 393 A 413 D
D, F
294 D 314 B 334 B 354 C 374 B 394 C 414 I

295 A 315 B 335 C 355 D 375 D 395 A 415 D

296 E 316 B 336 A 356 A 376 C 396 E 416 D

297 C 317 E 337 B 357 C 377 B 397 B 417 I

298 A 318 B 338 C 358 D 378 B 398 D 418 I

299 E 319 A 339 D 359 A 379 D 399 C 419 I

300 B 320 C 340 E 360 A 380 A 400 D 420 I


92
Blood Physiology
421 D 441 D 461 B 481 E 501 D 521 D

422 I 442 I 462 D 482 C 502 A 522 E

423 D 443 D 463 A 483 A 503 B 523 D

424 D 444 B 464 C 484 B 504 D 524 C

425 I 445 B 465 E 485 C 505 D 525 E

426 I 446 D 466 E 486 D 506 C 526 A

427 I 447 B 467 A 487 C 507 D 527 E

428 I 448 D 468 E 488 B 508 D 528 D

429 D 449 E 469 D 489 C 509 D 529 B

430 D 450 A 470 B 490 B 510 C 530 B

431 I 451 A 471 A 491 B 511 A 531 A

432 I 452 C 472 E 492 A 512 A 532 B

433 D 453 C 473 A 493 B 513 C 533 D

434 I 454 A 474 C 494 A 514 C

435 D 455 E 475 D 495 C 515 E

436 I 456 D 476 D 496 D 516 B

437 D 457 E 477 C 497 E 517 D

438 I 458 B 478 B 498 C 518 B, C


A, C,
439 D 459 A 479 D 499 C 519
E
B, C,
440 D 460 A 480 C 500 E 520
D
93
Nerves & Muscles Physiology

MCQs in
Nerves & Muscles Physiology
94
Nerves & Muscles Physiology
1. In the resting state the conductance of membrane is more for which of the ions?
A. Na+ B. Ca++ C. K+ D. Cl-
EXCEPT
2. Hyperpolarization in the action potential is caused by:
A. Increased exit of K+ from the cell C. Delay in sodium pump
-
B. Entry of Cl into the cell D. Increased exit of Na+ from the cell

3. Which of the following would carry the membrane potential closer to firing level?
A. Anodal current (negative electrode) C. Cathodal current (positive electrode)
B. Exit of K+ D. Accommodation

4. Repolarization of the membrane begins when:


A. Sodium channels are inactivated C. Sodium entry slows down
B. Potassium channels close D. Sodium channels are activated

5. What would be the effect of decrease in ECF Na+ on the resting membrane potential?
A. Not much effect C. Increase in membrane potential
B. Decrease in membrane potential D. Accommodation

6. Action potential propagation velocity increases with an increase in axon’s:


A. Time constant B. Capacitance C. Myelination D. Axoplasmic resistance

7. Anodal current in the excitable tissue causes:


A. Depolarization B. Hyperpolarization C. Action potential D. Current sink

8. Local potentials in the excitable tissue:


A. Are non-propagatory C. Show temporal and spatial summations
B. Can be graded D. Show all of the above

9. Conductance of membrane for chloride ion in the excitable tissue indicates which of the following?
A. Hyperpolarization C. No effect in the membrane potential
B. Depolarization D. Repolarization

10. Which one of the following nerve fibers is more susceptible to mechanical pressure?
A. A α B. Aγ C. B D. C

11. In ECF, the rise in the concentration of which of the ion can cause decrease in membrane potential?
A. Ca++ B. K+ C. Na+ D. Cl−

12. The ATPase activity to cleave ATP and utilize energy for the contraction of muscle is present in the:
A. Actin B. Myosin C. Troponin D. Tropomyosin

13. After a prolonged work, fatigue occurs in which of the following?


A. Cardiac muscle C. Slow twitch skeletal muscle
B. Visceral smooth muscle D. Fast twitch skeletal muscle

14. Regarding isotonic muscle contraction all of the following are true EXCEPT:
A. Muscle is allowed to shorten C. Length of the muscle remains constant
B. Tension remains constant D. External work is done
95
Nerves & Muscles Physiology

15. The protein that is involved in skeletal muscle contraction but not in smooth muscle contraction is:
A. Troponin B. Actin C. Myosin D. ATPase

16. D tubocurare inhibits neuromuscular transmission as it is a:


A. Cholinesterase inhibitor C. Nicotinic agonist
B. Nicotinic antagonist D. Muscarinic agonist

17. In myasthenia gravis antibodies are produced against:


A. Acetylcholine C. Acetylcholine receptors
B. Acetylcholine esterase D. Ion channels on the endplate

18. Relaxation of muscle occurs by involving all of the following mechanisms EXCEPT:
A. Hydrolysis of ATP in the myosin C. Attachment of ATP to myosin head
++
B. Removal of Ca from troponin C D. Active pumping of Ca++ into the cisternae

19. All of the following would lead to contraction of skeletal muscle EXCEPT:
A. Depolarization of T tubules C. Ca++ release from cisternae
++
B. Ca release from troponin C D. Depolarization of sarcolemma

20. During muscle shortening, decrease in width of all of the following occurs EXCEPT:
A. A band B. I band C. H zone D. Sarcomere

21. Which one of the following would increase acetylcholine concentration at the neuromuscular
junction when administered?
A. Curare B. Neostigmine C. Botulinum toxin D. Succinylcholine

22. During muscle shortening power stroke movement is produced by:


A. Actin B. Myosin C. Troponin D. Tropomyosin

23. In a nerve fiber, inactivation of sodium channels gives rise to:


A. Absolute refractory period D. Repolarization
B. Depolarization E. Relative refractive period
C. Hyperpolarization

24. Hyperpolarization results if in the extracellular fluid, there is decreased concentration of:
A. Calcium C. Magnesium E. Sodium
B. Chloride D. Potassium

25. Compound action potential is recorded from a:


A. Motor nerve fiber C. Nerve trunk E. Single nerve fiber
B. Myelinated nerve fiber D. Sensory nerve fiber

26. The muscle protein which covers the active sites on the actin filament at rest is:
A. Actin C. Tropomyosin E. Troponin-I
B. Myosin D. Troponin-T

27. During the muscle contraction, there is shortening of:


A. A-band B. Actin filaments C. Myosin
96
Nerves & Muscles Physiology
D. Sarcomere E. Z-disc

28. The visceral smooth muscle:


A. Control is mainly through nervous stimuli D. Is present in the iris of eye
B. Has contact type of neuromuscular junctions E. Shows contraction when it is stretched
C. Is composed of individual muscle fibers

29. During the neuromuscular transmission, end plate potential is produced. It is:
A. Due to Ca++ influx D. Obeys all or none law
B. Having a refractory period E. Of low amplitude in myasthenia gravis
C. Localized hyperpolarization

30. In isometric muscle contraction:


A. ATP is utilized in large amount
B. Muscle length decreases during the contraction
C. Much work is performed by the muscle
D. Shortening of the contractile component does not occur
E. There is stretching of the elastic component

31. The Nernst equation represents the potential at which:


A. Electrical neutrality exists
B. Concentration of ions on each side of membrane equal
C. Potential at which there is no net movement of ions
D. Balance of chemical & electrical forces
E. Both sides are equiosmolar

32. In skeletal muscle:


A. Relaxation is due to passive Ca++ uptake by sarcoplasmic reticulum
B. Contraction is due to Ca++ release from T tubules
C. Contraction is due to Ca++ binding to tropomyosin from the myosin head binding site
D. Z lines move together in contraction

33. In smooth muscle, all the following are true EXCEPT:


A. Spontaneous pacemaker potentials are generated
B. An action potential is required for contraction
C. Ca++ is released from sarcoplasmic reticulum
D. Longer twitch contraction time relative to skeletal muscles

34. In skeletal muscle neuromuscular junctions, all the following are false EXCEPT:
A. The motor end plate is the motor nerve terminal
B. Spontaneous (miniature) potentials may be recorded in the motor nerve terminal
C. Motor nerve terminals have vesicles containing norepinephrine
D. There is a high concentration of acetylcholinesterase
E. Transmission is facilitated by botulinum toxin

35. A skeletal muscle fiber, all the following are true EXCEPT:
A. Membrane is negatively charged on the inside with respect to the outside at rest
B. Contains intracellular stores of calcium ions
C. Is normally innervated by one motor neuron
97
Nerves & Muscles Physiology
D. Becomes more excitable as its resting membrane potential falls
E. Becomes less excitable as the extracellular ionized calcium levels fall

36. An excitatory post-synaptic potential, all the following are false EXCEPT
A. It is the depolarization of a post-synaptic nerve cell membrane that occurs when a presynaptic neuron is
stimulated
B. Involves reversal of polarity across the post-synaptic nerve cell membrane
C. Is propagated at the same rate as an action potential
D. Is caused by the electrical field induced by activity in the pre-synaptic nerve terminals

37. Nerve impulses, all the following are false EXCEPT


A. Can travel in one direction only in a nerve fiber
B. Can travel in two directions across a synapse
C. Travel at the speed of an electric current
D. Correspond in duration to that of the nerve refractory period

38. In skeletal muscle, all are false EXCEPT


A. Contraction occurs when its pacemaker cells depolarize sufficiently to reach the threshold for firing
B. Calcium is taken up by the sarcotubular system when it contracts
C. Actin and myosin filaments shorten when it contracts
D. The sarcomeres shorten during contraction
E. Contraction strength does not affected by the initial length of the muscle fibers

39. Saltatory conduction, all are true EXCEPT


A. Occurs only in myelinated fibers
B. Does not depend on depolarization of the nerve membrane
C. Has a slower velocity in cold than in warm conditions
D. Is faster than non-saltatory conduction in nerve fibers with diameters around 10 µm
E. Transmits impulses with a velocity proportional to fiber diameter

40. Nerve fibers continue to conduct impulses when


A. Extracellular sodium is replaced by potassium
B. Extracellular sodium is replaced by a non-diffusible cation
C. Temperature is lowered from 37 to 30oC
D. Temperature is lowered to below 0oC provided freezing does not occur

41. When a nerve cell membrane is depolarized by 5 mV, all the following are true EXCEPT
A. Its permeability to sodium increases
B. Sodium ions move into the cell to cause further depolarization
C. Potassium ions move outwards down their electrochemical gradient
D. Chloride ions move inwards down their electrochemical gradient
E. An action potential is generated

42. An action potential in a nerve fiber, all the fallowing are false EXCEPT
A. Occurs when its membrane potential is hyperpolarized to a critical level
B. Is associated with a transient decrease in membrane permeability to sodium
C. Is associated with a transient decrease in membrane permeability to potassium
D. Induces local electrical currents in adjacent segments of the fiber
E. Has amplitude which varies directly with the strength of stimulus
98
Nerves & Muscles Physiology

43. Non-myelinated axons differ from myelinated axons in that they are
A. Not sheathed in Schwann cells
B. Not capable of regeneration after section
C. Found only in the autonomic nervous system
D. More excitable
E. Refractory for a longer period after excitation

44. Resting nerve cell membranes are more permeable to


A. Organic anions than to Cl− anions
B. K+ ions than to Cl− ions
C. Na+ ions than to K+ ions
D. Glucose molecules than to oxygen molecules
E. Water molecules than to H+ ions

45. All the following are true for the differences of visceral smooth muscle from skeletal muscle EXCEPT
A. It contracts when stretched
B. It is not paralyzed when its motor nerve supply is cut
C. Its cells have unstable resting membrane potentials
D. It contains no Actin or myosin
E. Excitation depends more on influx of extracellular calcium than release of calcium from endoplasmic
reticulum

46. A property shared by


A. Skeletal and cardiac muscle is their striated microscopical appearance
B. Cardiac and visceral smooth muscle is their spontaneous activity when denervated
C. Skeletal and cardiac ventricular muscle is their stable resting membrane potential
D. All varieties of muscle are that contraction strength is related to their initial length
E. All of the above are true
F. “A” and “C” are true
G. “B” and “D” are true

47. Histological and physiological study of skeletal muscle shows that the
A. Distance between two Z lines remains constant during contraction
B. Width of the A band is constant during contraction
C. Tension developed is maximal when actin and myosin molecules just fail to overlap
D. The T system of transverse tubules opens into the terminal cisterns of the sarcoplasmic reticulum

48. Which of the following has the slowest conduction velocity?


A. Aα fibers B. A β fibers C. A γ fibers D. B fibers E. C fibers

49. The functions of tropomyosin in skeletal muscle include


A. sliding on actin to produce shortening
B. releasing Ca++ after initiation of contraction
C. binding to myosin during contraction
D. acting as a "relaxing protein" at rest by covering up the sites where myosin binds to actin
E. generating ATP, which it passes to the contractile mechanism

50. The cross-bridges of the sarcomere in skeletal muscle are made up of


99
Nerves & Muscles Physiology
A. Actin B. Myosin C. Troponin D. Tropomyosin E. Myelin

51. A single contraction of skeletal muscle is most likely to be terminated by which of the following
actions?
A. Closure of the postsynaptic nicotinic acetylcholine receptor
B. Removal of acetylcholine from the neuromuscular junction
C. Removal of Ca++ from the terminal of the motor neuron
D. Removal of sarcoplasmic Ca++
E. Return of the dihydropyridine receptor to its resting conformation

52. Which of the following best describes an attribute of visceral smooth muscle not shared by skeletal
muscle?
A. Contraction is ATP dependent D. High rate of cross-bridge cycling
B. Contracts in response to stretch E. Low maximal force of contraction
C. Does not contain actin filaments

53. The resting potential of a myelinated nerve fiber is primarily dependent on the concentration
gradient of which of the following ions?
A. Ca++ B. Cl− C. HCO3− D. K+ E. Na+

54. Calmodulin is most closely related, both structurally and functionally, to which of the following
proteins?
A. G-actin B. Myosin light chain C. Tropomyosin D. Troponin C

55. During a demonstration for medical students, a neurologist uses magnetic cortical stimulation to
trigger firing of the ulnar nerve in a volunteer. At relatively low-amplitude stimulation, action potentials
are recorded only from muscle fibers in the index finger. As the amplitude of the stimulation is increased,
action potentials are recorded from muscle fibers in both the index finger and the biceps muscle. What is
the fundamental principle underlying this amplitude-dependent response?
A. Large motor neurons that innervate large motor units require a larger depolarizing stimulus
B. Recruitment of multiple motor units requires a larger depolarizing stimulus
C. The biceps muscle is innervated by more motor neurons
D. The motor units in the biceps are smaller than those in the muscles of the fingers
E. The muscles in the fingers are innervated only by the ulnar nerve

56. A neurotransmitter activates its receptor on an ion channel of a neuron, which causes the water-
filled channel to open. Once the channel is open, ions move through the channel down their respective
electrochemical gradients. A change in membrane potential follows. Which of the following best
describes the type of channel and mechanism of ion transport?
Type of Channel : Mechanism of Transport
A. Ligand gated : Facilitated diffusion
B. Ligand gated : Simple diffusion
C. Ligand gated : Secondary active transport
D. Voltage gated : Facilitated diffusion
E. Voltage gated : Simple diffusion
F. Voltage gated : Secondary active transport

57. Which of the following decreases in length during the contraction of a skeletal muscle fiber?
A. A band of the sarcomere B. I band of the sarcomere C. Thick filaments
100
Nerves & Muscles Physiology
D. Thin filaments E. Z disks of the sarcomere

58. Tetanic contraction of a skeletal muscle fiber results from a cumulative increase in the intracellular
concentration of which of the following?
A. ATP B. Ca++ C. K+ D. Na+ E. Troponin

59. Weight lifting can result in a dramatic increase in skeletal muscle mass. This increase in muscle mass
is primarily attributable to which of the following?
A. Fusion of sarcomeres between adjacent myofibrils
B. Hypertrophy of individual muscle fibers
C. Increase in skeletal muscle blood supply
D. Increase in the number of motor neurons
E. Increase in the number of neuromuscular junctions

60. The figure shows the change in membrane


potential during an action potential in a giant
squid axon. Which of the following is primarily
responsible for the change in membrane potential
between points B and D?
A. Inhibition of the Na+- K+-ATPase
B. Movement of K+ into the cell
C. Movement of K+ out of the cell
D. Movement of Na+ into the cell
E. Movement of Na+ out of the cell

61. The figure shows the change in membrane


potential during an action potential in a giant
squid axon. Which of the following is primarily
responsible for the change in membrane potential between points D and E?
A. Inhibition of the Na+ - K+-ATPase D. Movement of Na+ into the cell
+
B. Movement of K into the cell E. Movement of Na+ out of the cell
C. Movement of K+ out of the cell

62. The figure shows the change in membrane potential during an action potential in a giant squid axon.
Which of the following is primarily responsible for the change in membrane potential between points A
and B?
A. Inhibition of the Na+ - K+-ATPase D. Movement of Na+ into the cell
B. Movement of K+ into the cell E. Movement of Na+ out of the cell
+
C. Movement of K out of the cell

63. The figure shows the change in membrane potential during an action potential in a giant squid axon.
Which of the following is primarily responsible for the change in membrane potential between points F
and G?
A. Inhibition of the Na+ - K+-ATPase D. Movement of Na+ into the cell
+
B. Movement of K into the cell E. Movement of Na+ out of the cell
C. Movement of K+ out of the cell
101
Nerves & Muscles Physiology
64. The axon of a neuron is stimulated experimentally with a 25-millivolt pulse, which initiates an action
potential with a velocity of 50 meters per second. The axon is then stimulated with a 100-millivolt pulse.
What is the action potential velocity after the 100-millivolt stimulation pulse (in meters per second)?
A. 25 B. 50 C. 100 D. 150 E. 200

65. The delayed onset and prolonged duration of smooth muscle contraction, as well as the greater force
generated by smooth muscle compared with skeletal muscle, are all consequences of which of the
following?
A. Greater amount of myosin filaments present in smooth muscle
B. Higher energy requirement of smooth muscle
C. Physical arrangement of actin and myosin filaments
D. Slower cycling rate of the smooth muscle myosin cross-bridges
E. Slower uptake of Ca++ ions after contraction

66. An experimental drug is being tested as a potential therapeutic treatment for asthma. Preclinical
studies have shown that this drug induces the relaxation of cultured porcine tracheal smooth muscle cells
pre-contracted with acetylcholine. Which of the following mechanisms of action is most likely to induce
this effect?
A. Decreased affinity of troponin C for Ca++
B. Decreased plasma membrane K+ permeability
C. Increased plasma membrane Na+ permeability
D. Inhibition of the sarcoplasmic reticulum Ca++-ATPase
E. Stimulation of adenylate cyclase

A 55-year-old woman visits her physician because of double vision, eyelid droop, difficulty chewing and
swallowing, and general weakness in her limbs. All these symptoms worsen with exercise and occur more
frequently late in the day. The physician suspects myasthenia gravis and orders a Tensilon test. The test
is positive. Use this information when answering Questions 67–68.

67. The increased muscle strength observed during the Tensilon test is due to an increase in which of the
following?
A. Amount of acetylcholine (ACh) released from the motor nerves
B. Levels of ACh at the muscle end plates
C. Number of ACh receptors on the muscle end plates
D. Synthesis of norepinephrine

68. What is the most likely basis for the symptoms described in this patient?
A. Autoimmune response
B. Botulinum toxicity
C. Depletion of voltage-gated Ca++ channels in certain motor neurons
D. Development of macro motor units after recovery from poliomyelitis
E. Overexertion

69. Which of the following drugs would likely alleviate this patient’s symptoms?
A. Atropine B. Botulinum toxin antiserum C. Curare D. Halothane E. Neostigmine

70. Smooth muscle contraction is terminated by which of the following?


A. Dephosphorylation of myosin kinase
B. Dephosphorylation of myosin light chain
102
Nerves & Muscles Physiology
C. Efflux of Ca++ ions across the plasma membrane
D. Inhibition of myosin phosphatase
E. Uptake of Ca++ ions into the sarcoplasmic reticulum

A 56-year-old man sees a neurologist because of weakness in his legs that improves over the course of
the day or with exercise. Extracellular electrical recordings from a single skeletal muscle fiber reveal
normal miniature end plate potentials. Low-frequency electrical stimulation of the motor neuron,
however, elicits an abnormally small depolarization of the muscle fibers. The amplitude of the
depolarization is increased after exercise. Use this information to answer Questions 71–73.

71. Based on these findings, which of the following is the most likely cause of this patient’s leg
weakness?
A. Acetylcholinesterase deficiency
B. Blockade of postsynaptic acetylcholine receptors
C. Impaired presynaptic voltage-sensitive Ca++ influx
D. Inhibition of Ca++ re-uptake into the sarcoplasmic reticulum
E. Reduced acetylcholine synthesis

72. A preliminary diagnosis is confirmed by the presence of which of the following?


A. Antibodies against the acetylcholine receptor
B. Antibodies against the voltage-sensitive Ca++ channel
C. Mutation in the gene that codes for the ryanodine receptor
D. Relatively few vesicles in the presynaptic terminal
E. Residual acetylcholine in the neuromuscular junction

73. The molecular mechanism underlying these symptoms is most similar to which of the following?
A. Acetylcholine C. Curare E. Tetrodotoxin
B. Botulinum toxin D. Neostigmine

74. Trace A in the above figure represents a typical action potential recorded under control conditions
from a normal nerve cell in response to a
depolarizing stimulus. Which of the
following perturbations would explain the
conversion of the response shown in trace
A to the action potential shown in trace
B?
A. Blockade of voltage-sensitive Na+
channels
B. Blockade of voltage-sensitive K+
channels
C. Blockade of Na-K “leak” channels
D. Replacement of the voltage-sensitive K+
channels with “slow” Ca++ channels
E. Replacement of the voltage-sensitive Na+ channels with “slow” Ca++ channels

75. Which of the following perturbations would account for the failure of the same stimulus to elicit an
action potential in trace C?
A. Blockade of voltage-sensitive Na+ channels
B. Blockade of voltage-sensitive K+ channels
103
Nerves & Muscles Physiology
C. Blockade of Na-K “leak” channels
D. Replacement of the voltage-sensitive K+ channels with “slow” Ca++ channels
E. Replacement of the voltage-sensitive Na+ channels with “slow” Ca++ channels

76. A 17-year-old soccer player sustained a fracture to the left tibia. After her lower leg has been in a
cast for 8 weeks, she is surprised to find that the left gastrocnemius muscle is significantly smaller in
circumference than it was before the fracture. What is the most likely explanation?
A. Decrease in the number of individual muscle fibers in the left gastrocnemius
B. Decrease in blood flow to the muscle caused by constriction from the cast
C. Temporary reduction in actin and myosin protein synthesis
D. Increase in glycolytic activity in the affected muscle
E. Progressive denervation

77. Smooth muscle that exhibits rhythmical contraction in the absence of external stimuli also
necessarily exhibits which of the following?
A. “Slow” voltage-sensitive Ca++ channels
B. Intrinsic pacemaker wave activity D. Hyperpolarized membrane potential
C. Higher resting cytosolic Ca++ concentration E. Action potentials with “plateaus”

78. A 24-year-old woman is admitted as an emergency to University Hospital after an automobile


accident in which severe lacerations to the left wrist severed a major muscle tendon. The severed ends of
the tendon were overlapped by 6 cm to facilitate suturing and reattachment. Which of the following
would be expected after 6 weeks compared with the preinjured muscle? Assume that series growth of
sarcomeres cannot be completed within 6 weeks.
Passive Tension : Maximal Active Tension
A. Decrease : Decrease C. Increase : Increase E. No change : No change
B. Decrease : Increase D. Increase : Decrease

79. Which of the following best describes the correct temporal order of events for skeletal muscle?
A. Muscle action potential  Muscle contraction  Nerve action potential
B. Muscle action potential  Nerve action potential  Muscle contraction
C. Muscle contraction  Muscle action potential  Nerve action potential
D. Muscle contraction  Nerve action potential  Muscle action potential
E. Nerve action potential  Muscle action potential  Muscle contraction
F. Nerve action potential  Muscle contraction  Muscle action potential

80. Which of the following best describes a physiological difference between the contraction of smooth
muscle compared with the contraction of cardiac muscle and skeletal muscle?
A. Ca++ independent C. Requires more energy
B. Does not require an action potential D. Shorter in duration

81. The resting cardiac muscle cell is most permeable to:


A. Na B. K C. Ca D. Cl

82. The membrane potential of cardiac muscle cells is most affected by even a small change in plasma
concentration of:
A. Na B. K C. Cl D. Ca

83. Hypokalemia would be expected to result in:


104
Nerves & Muscles Physiology
A. Increased neuronal excitability C. No change in RMP
B. A more negative RMP D. A decrease in firing level of neurons

84. The number of Na channels per square micrometer of membrane in myelinated mammalian neurons
is maximum in the:
A. Cell body B. Dendritic zone C. Initial segment D. Node of Ranvier

85. The site of origin in the nerve of conducted impulses is the:


A. Dendritic zone B. Axon hillock C. Node of Ranvier D. Terminal buttons

86. In motor neurons, the portion of the cell with the lowest threshold for the production of a full-
fledged action potential is:
A. Initial segment B. Soma C. Dendritic zone D. Node of Ranvier

87. Action potential conduction velocity is influenced by:


A. Axon diameter B. Temperature C. Myelination D. All of the above

88. In the CNS, the membranes that wrap around myelinated neurons are those of:
A. Schwann cells B. Oligodendroglia C. Endothelial cells D. Astrocytes

89. The duration of action potential in a skeletal muscle fiber is typically:


A. 5 ms B. 25 ms C. 200 ms D. 250 ms

90. When heart rate is about 75 beats per minute, what is the mean duration of action potential in
ventricular muscle cells?
A. 5 ms B. 25 ms C. 200 ms D. 250 ms

91. Regarding the force of muscle contraction, all the fallowing are true EXCEPT:
A. increasing the frequency of activation of motor units
B. increasing the number of motor units activated
C. increasing the amplitude of action potentials in motor neurons
D. recruiting larger motor units

92. An excitable cell has an RMP of -70 mV and a firing level of - 50 mV. This cell would be unexcitable
when its membrane potential is:
A. - 30 mV B. - 55 mV C. - 70 mV D. - 90 mV

93. An excitable cell has an RMP of -70 mV and a firing level of - 50 mV. This cell would be most excitable
when its membrane potential is:
A. - 30 mV B. - 55 mV C. - 70 mV D. - 90 mV

94. Which one of the following statements about graded potential is incorrect?
A. They are graded responses C. They may be depolarizing or hyperpolarizing
B. They are local (non-propagated) responses D. They are produced by a threshold stimulus

95. Thin filaments do not contain:


A. actin B. myosin C. troponin D. tropomyosin
105
Nerves & Muscles Physiology
96. The ATPase activity of which of the following proteins is altered to regulate skeletal muscle
contraction?
A. Actin B. Myosin C. Troponin D. Tropomyosin

97. The activity of which contractile protein is altered to regulate smooth muscle contraction?
A. Actin B. Myosin C. Calmodulin D. Tropomyosin

98. Smooth muscle lacks:


A. Actin B. Myosin C. Troponin D. Tropomyosin

99. The major source of calcium for contraction of skeletal muscle is:
A. ECF B. cytosol C. mitochondria D. sarcoplasmic reticulum (SR)

100. Ryanodine receptor is located in the:


A. sarcolemma C. terminal cisterns of sarcoplasmic reticulum
B. T-tubule D. cytosol

101. Rigor mortis is due to:


A. damage to actin & myosin C. increased myosin ATPase
B. rapid sequestration of Ca in ER D. ATP depletion

102. Staircase phenomenon (Treppe) is due to:


A. increased availability of calcium C. tetanus
B. summation D. increased excitability

103. Which of the following statements is incorrect?


A. Contraction against a constant load with approximation of the ends of the muscle is called isotonic
contraction
B. Contraction can occur without an appreciable decrease in the length of the muscle
C. Isometric contractions do work whereas isotonic contractions do not
D. Muscles can lengthen while doing work

104. Regarding the ionic basis of action potential in cardiac muscle cells, which one of the following is
incorrect?
A. Phase 0: Na influx B. Phase 1: K influx C. Phase 2: Ca influx D. Phase 3: K efflux

105. When heart rate is about 75 beats/min, the duration of absolute refractory period of cardiac muscle
is:
A. 50 ms B. 100 ms C. 200 ms D. 300 ms

106. Which of the following muscle types is the ‘fastest’?


A. Skeletal muscle B. Smooth muscle C. Cardiac muscle

107. Which of the following characteristics is exhibited only by skeletal muscle?


A. Gradation of force production C. Staircase phenomenon
B. Refractoriness D. None of the above

108. Non-linearity of length-tension relationship is most evident in:


A. skeletal muscle B. smooth muscle C. cardiac muscle
106
Nerves & Muscles Physiology

109. The latch-bridge mechanism in smooth muscle is responsible for:


A. fast muscle twitch C. excitation-contraction coupling
B. sustained muscle contraction D. unstable membrane potential

110. In a muscle fiber light bands:


A. Contain myosin filaments D. Show interaction among actin and myosin
B. Are produced due to Z disc filaments
C. Have only actin filaments

111. During Isometric muscle contraction:


A. Muscle does not shorten D. Muscle gets shorter against a fixed load
B. Shortening of muscle occurs E. Actual body movements occur
C. Tension on muscle remains constant

112. Rigor mortis:


A. Occurs immediately after death D. Results in autolysis of muscles
B. Is due to abundance of ATP. E. Occurs 25 hrs after death
C. Leads to rigidity of body muscles

113. Acetylcholine receptors in muscle fibre membrane are:


A. Carbohydrate in nature
B. Present only in smooth muscles
C. Cause destruction of Acetylcholine esterase
D. Acetylcholine gated ion channels
E. Having strong positive charges upon their surface

114. Myasthenia Gravis is:


A. Due to rapid transmission of nerve signals
B. An acute inflammatory disease
C. A chronic infection of nerve fibers at motor end plate
D. An auto immune disease
E. Due to decrease secretion of acetylcholine at neuromuscular junction

115. Which of the following characteristics of an axon is most dependent on its diameter?
A. The magnitude of its resting potential D. The overshoot of its action potential
B. The duration of its refractory period E. The activity of its sodium-potassium pump
C. The conduction velocity of its action potential

116. If the extracellular K+ concentration is increased from 4 meq/L to 10 meq/L,


A. The membrane potential will become more negative
B. The sodium conductance will increase
C. The potassium conductance will increase
D. The membrane will become more excitable
E. The Na-K pump will become inactivated

117. Inactivation of the sodium-potassium pump will cause


A. An increase in the intracellular volume
B. An increase in the intracellular potassium concentration
107
Nerves & Muscles Physiology
C. Hyperpolarization of the membrane potential
D. An increase in the excitability of nerve cells
E. An increase in the flow of sodium out of the cell

118. Membrane excitability will be increased by the greatest amount by


A. Increasing extracellular Na+ D. Decreasing extracellular Ca++
B. Increasing extracellular K+ E. Decreasing extracellular H+

C. Decreasing extracellular Cl

119. The resting potential of a nerve membrane is primarily dependent on the concentration gradient of
A. Potassium B. Sodium C. Calcium D. Chloride E. Bicarbonate

120. Which one of the following muscle proteins plays an important role in contraction of both smooth
and striated muscle?
A. Calmodulin B. Troponin C. Tropomyosin D. Actin E. Myosin light chains

121. Which of the following words or phrases is most closely associated with an end-plate potential at
the neuromuscular junction?
A. “All-or-none response” C. Hyperpolarization E. Electrically excitable gates
B. Depolarization D. Action potential

122. In a nerve, the magnitude of the action potential overshoot is normally a function of the
A. Magnitude of the stimulus D. Resting membrane potential
B. Intracellular potassium concentration E. Diameter of the axon
C. Extracellular sodium concentration

123. Nicotinic receptors are responsible for


A. Producing the skeletal muscle end-plate potential
B. Decreasing the rate of phase-4 depolarization at the SA node
C. Increasing the force of stomach contractions
D. Delaying the emptying of liquids from the stomach
E. Decreasing the excitability of sympathetic postganglionic neurons

124. Which of the following statements about synaptic transmission at the neuromuscular junction is
true?
A. It is enhanced by high levels of cholinesterase
B. It is caused by an influx of potassium ions through the muscle membrane
C. It is depressed by abnormally low levels of magnesium
D. It is depressed by increased parasympathetic nerve activity
E. It is produced by the release of acetylcholine from the alpha motoneuron

125. When comparing the contractile responses in smooth and skeletal muscle, which of the following is
most different?
A. The source of activator calcium
B. The role of calcium in initiating contraction
C. The mechanism of force generation
D. The source of energy used during contraction
E. The nature of the contractile proteins
108
Nerves & Muscles Physiology
126. The amount of force produced by a skeletal muscle can be increased by
A. Increasing extracellular Mg2+ D. Decreasing the interval between contractions
++
B. Decreasing extracellular Ca E. Increasing the preload beyond 2.2 µm
C. Increasing the activity of acetylcholine esterase

127. The velocity of nerve conduction is increased with a decrease in the


A. Diameter of the nerve fiber C. Capacitance of the nerve fiber membrane
B. Degree of myelinization D. Resting membrane potential

128. Periodic hyperkalemic paralysis is characterized by high potassium concentration and muscle
weakness. Which of the following is likely to cause muscle weakness as a result of increased extracellular
potassium concentration?
A. Hyperpolarization of muscle cells
B. Inactivation of sodium channels in muscle cells
C. Increased release of neurotransmitters from alpha motoneurons
D. Decreased potassium conductance in muscle cells
E. Increased duration of action potentials produced by alpha motoneurons

129. The flow of calcium into the cell is an important component of the upstroke phase of action
potentials in
A. Cardiac ventricular muscle D. Nerve cell bodies
B. Intestinal smooth muscle E. Presynaptic nerve terminals
C. Skeletal muscle fibers

130. The membrane potential will depolarize by the greatest amount if the membrane permeability
increases for
A. K B. Na and K C. Cl D. K and Cl E. Na

131. Preventing the inactivation of sodium channels will decrease


A. The relative refractory period of nerve cells
B. The upstroke velocity of nerve cell action potentials
C. The downstroke velocity of nerve cell action potentials
D. The magnitude of the overshoot in nerve cell action potentials
E. The duration of nerve cell action potentials

132. An increase in sodium conductance is associated with


A. The plateau phase of the ventricular muscle action potential
B. The downstroke of the skeletal muscle action potential
C. The upstroke of the smooth muscle action potential
D. The refractory period of the nerve cell action potential
E. The end-plate potential of the skeletal muscle fiber

133. The sodium gradient across the nerve cell membrane is


A. A result of the Donnan equilibrium
B. Significantly changed during an action potential
C. Used as a source of energy for the transport of other ions
D. An important determinant of the resting membrane potential
E. Maintained by a Na/Ca exchanger
109
Nerves & Muscles Physiology
134. Increasing the extracellular potassium concentration will
A. Increase the threshold for eliciting an action potential
B. Hyperpolarize the membrane potential
C. Decrease potassium permeability
D. Decrease the activity of the sodium-potassium pump
E. Make the equilibrium potential for potassium more negative

135. Which of the following would cause an immediate reduction in the amount of potassium leaking
out of a cell?
A. Increasing the permeability of the membrane to potassium
B. Increasing the intracellular potassium concentration
C. Increasing (hyperpolarizing) the membrane potential
D. Reducing the activity of the sodium-potassium pump
E. Decreasing the potassium equilibrium potential

136. Excitation-contraction coupling in smooth muscle is initiated when calcium binds to


A. Myosin light chains B. Calmodulin C. Troponin D. Tropomyosin E. Protein kinase A

137. Synaptic transmission between pain fibers from the skin and spinal cord neurons is mediated by
A. Acetylcholine B. Substance P C. Endorphins D. Somatostatin E. Serotonin

Use the diagram to answer the questions 138-139.


138. At which point on the action potential does the Na+ current
exceed the K+ current?
A. Point A
B. Point B
C. Point C
D. Point D
E. Point E

139. At which point on the action potential is the membrane


closest to the Na+ equilibrium potential?
A. Point A B. Point B C. Point C D. Point D E. Point E

140. When skeletal muscle is in its resting state, myosin cross-bridges are prevented from binding to
actin molecules by
A. Calmodulin D. Titin
B. Troponin E. Phospholamban
C. Tropomyosin
110
Nerves & Muscles Physiology

Use the diagram to answer the questions 141-142


141. Which one of the following enzymes is responsible for step 1 in the diagram?
A. Calmodulin D. Phospholipase C
B. Protein kinase A E. Actomyosin ATPase
C. Myosin light chain kinase

142. Which one of the steps in the diagram is responsible for the formation of latch bridges?
A. Step 1 B. Step 2 C. Step 3 D. Step 4 E. Step 5

143. An action potential in a nerve


A. Is terminated by influx of Na+ excessive receptors
B. Is terminated by efflux of K+
C. Is initiated by efflux of Na+
D. Is initiated by influx of K+

144. Saltatory conduction


A. Is seen only in myelinated nerve fibers
B. Is slower that non saltatory conduction
C. Is not affected if a local anesthetic is applied to the node of ranvier
D. None of the above

145. Myelin sheath is produced by


A. Axoplasm B. Mitochondria C. Schwann cell D. Muscle cell

146. The action potential of skeletal muscle


A. Has a prolonged plateau phase
B. Spreads inwards to all parts of the muscle via T tubules
C. Is longer than the action potential of cardiac muscle
D. Is not essential for muscle conduction

147. Smooth muscle need help of


A. Calmodulin for contraction C.K+ for contraction
B. Acetyl choline for contraction D. Monoamine oxidase for contraction

148. The cross bridges of the sarcomere in skeletal muscle are components of
A. Actin B. Myosin C. Troponin D. Tropomyosin
111
Nerves & Muscles Physiology
149. The likely mechanism through which neostigmine acts in improving muscular weakness is
A. It blocks action of acetylcholine C. It enhances the action of catecholamines
B. It interferes with action of mono-amine D. It blocks the action of acetyl choline esterase
oxidase

150. A skeletal muscle


A. Obeys all or none phenomenon
B. Becomes less excitable when its membrane becomes hyperpolarized
C. Has a resting membrane potential positive inside
D. Contains excessive Na+ in intracellular compartment

151. The function of tropomyosin in skeletal muscle is


A. Sliding on actin to produce shortening
B. Releasing Ca++ after initiation of contraction
C. Binding to myosin during contraction
D. Covering up the actin binding sites of myosin at rest

152. Regarding the release of neurotransmitter from synaptic vesicles, all the following are true EXCEPT
A. Takes place by exocytosis C. Is quantal
B. Is controlled by neuronal calcium influx D. Requires neuronal sodium ions influx

153. The following are true about acetylcholine EXCEPT:


A. It has a strong affinity for nicotinic receptors
B. Is derived from acetyl CoA and choline
C. Is synthesized by a reaction involving choline acetyl transferase
D. Is inactivated by reuptake

154. The effect of calcium ions on neurotransmitter release at synapses include the following EXCEPT:
A. Vesicular fusion
B. Tonic depolarization of the presynaptic neuron
C. Post-tetanic potentiation
D. Transport of synaptic vesicles to the presynaptic active zone of release
E. Reuptake of noradrenaline

155. Regarding the neuronal resting membrane of the human brain is, all the following are true, EXCEPT:
A. Maintained by the sodium pump
B. Around -70mV
C. Maintained by using ATP for energy
D. Achieved by active extrusion of potassium ions out of the neuron

156. The cells which carry electrical signals are


A. Axons C. Acinar cells E. Neurons
B. Processes D. Glial cells

157. The chemical signals released into synapses are called


A. Anticodons C. Amino acids E. Hormones
B. Neurotransmitters D. Codons

158. The portion of the nervous system that directly controls skeletal muscles is the
112
Nerves & Muscles Physiology
A. Somatic motor division D. Autonomic system
B. Visceral nervous system E. Peripheral nervous system
C. Central nervous system

159. The part of the peripheral nervous system which brings information to the central nervous system is
A. The motor C. Somatic E. Afferent
B. Efferent D. Autonomic

160. Nerve and muscle cells are classified as excitable tissues because they
A. Generate electrical signals and propagate electrical signals
B. Generate electrical signals, propagate electrical signals, and store electrical capacity
C. Generate electrical signals
D. Propagate electrical signals
E. Store electrical capacity

161. Interneurons
A. Carry only sensory impulses
B. Are found between neurons, synapsing with their effectors
C. Carry only motor impulses
D. Are found only in the central nervous system
E. Only connect motor neurons to other motor neurons

162. Axonal transport refers to ______________ down the length of the axon.
A. Sodium and potassium movement D. Mitochondrial transport
B. Movement of proteins packaged in vesicles E. None of the above are true
C. Impulse transmission

163. Glial cells which manufacture myelin include


A. Schwann cells and oligodendrocytes
B. Schwann cells, oligodendrocytes, and astrocytes
C. Oligodendrocytes
D. Schwann cells
E. Astrocytes

164. Each of these is a true statement EXCEPT one. Identify the exception.
A. EPSP's can make action potentials more likely to occur
B. IPSP's depolarize the membrane
C. EPSP's that reach threshold can initiate an action potential
D. The trigger zone is the integrating part of the neuron
E. All of the above are true statements

165. The ion that plays a key role in initiating electrical signals in neurons is
A. K+ B. Ca++ C. Na+ D. Cl−

166. Saltatory conduction refers to


A. The smooth, uninterrupted conduction of action potentials down an axon
B. The transfer of heat between two axons
C. The abnormal conduction of action potentials in someone with demyelinating disease
D. The jumping of action potentials from one node of Ranvier to the next
113
Nerves & Muscles Physiology

167. Application of the poison tetrodotoxin (TTX) to a neuron blocks voltage-gated Na+ channels. What
effect will this have on an action potential in the neuron?
A. TTX will lengthen the depolarization phase of the action potential
B. TTX will not affect action potentials because voltage-gated Na+ channels have no role in them
C. TTX will diminish or abolish the depolarization phase of the action potential
D. TTX will increase the amplitude of the action potential

168. Action potentials are non-directional. Why do they only travel from the cell body of a neuron to the
terminal and never go backward?
A. This statement is false They do travel backwards
B. Both sodium gates and potassium gates are closed once an action potential passes
C. The sodium channel is at normal resting state once an action potential passes
D. The membrane behind the region of action potential is at refractory period once an action potential
passes
E. Too much of a depolarization would inhibit the action potential, so it doesn't travel backwards so that it
won't open up more sodium channels

169. Sarcoplasmic reticulum is the name given to which of the following?


A. Rough endoplasmic reticulum in smooth muscle cells
B. Smooth endoplasmic reticulum in cells of the epimysium
C. Smooth endoplasmic reticulum in all muscle cells
D. Rough endoplasmic reticulum in cardiac muscle cells

170. An overlap of actin and myosin filaments occurs in the:


A. A Band B. I Band C. Z Line D. H Band E. M Line

171. Which of the following does not describe skeletal muscle fibers?
A. Striated B. Typically voluntary C. Multinucleate D. Branched

172. Intercalated disks:


A. Are found only in smooth muscle D. Are located at the M line
B. Are found in skeletal and cardiac muscle E. Contain desmosomes and gap junctions
C. Are part of the neuromuscular junction?

173. Motor units:


A. Are found only in cardiac muscle
B. Are largest in muscles responsible for delicate movements
C. Consist of a muscle fiber and all the nerves that supply it
D. Consist of a motor neuron and all the muscle fibers it supplies
E. Are the same as neuromuscular junctions

174. In muscle tissue, neurotransmitter receptors are located:


A. In synaptic vesicles C. In the synaptic cleft
B. On the motor neuron axon terminals D. On the motor end plate

175. An action potential is:


A. A migrating region of membrane potential reversal
B. A flow of electrons along the sarcolemma
114
Nerves & Muscles Physiology
C. A nucleophilic reaction between Na and K ions
D. Something that is only caused by acetylcholine

176. Acetylcholinesterase:
A. Produces acetylcholine
B. Is the acetylcholine receptor in muscle tissue
C. Is responsible for smooth but not skeletal muscle contraction
D. Degrades the neurotransmitter which is found in the neuromuscular junction

177. Tetanus toxin causes convulsive paralysis by:


A. Blocking acetylcholine from binding to the muscarinic acetylcholine receptor
B. Inhibiting acetylcholinesterase
C. Causing motor neurons to release massive amounts of acetylcholine
D. Blocking acetylcholine from being released by motor neurons

178. A person suffering from nerve gas exposure is given atropine to counter the effects because:
A. Atropine will bind to and electrophilically inactivate the nerve gas
B. Atropine blocks the nerve gas receptor
C. Atropine blocks the acetylcholine receptor which prevents the lingering excess ach from having adverse
effects
D. Atropine inactivates acetylcholinesterase

179. Skeletal muscle is described by all of the following EXCEPT:


A. Striated B. Voluntary C. Multinucleate D. Autorhythmic E. Contractile

180. The walls of hollow organs and some blood vessels contain this type of muscle tissue.
A. Striated B. Skeletal C. Cardiac D. Voluntary E. Smooth

181. Which of the following is unique to cardiac muscle tissue?


A. Involuntary B. Striated C. Non-striated D. Contains actin AND myosin E. Contains intercalated
disks

182. The energy of muscle contraction is derived from the following EXCEPT:
A. ATP B. muscle glycogen C. lactic acid D. Creatine phosphate

183. Depolarization:
A. is associated with increase in membrane permeability to Na+
B. is terminated with closure of voltage activated K+ channels
C. is followed by muscle relaxation
D. is caused by K+ efflux

184. Action potential:


A. is a graded potential
B. is produced by sub threshold stimulus
C. starts with repolarization caused by outward movement of Cl
D. is conducted slower in thin nerve fibers

185. RMP of a nerve:


A. is caused by equal distribution of ions along both sides of the membrane
115
Nerves & Muscles Physiology
B. is caused by selective permeability of the membrane to the ions
C. Na+- K+ pump has no role in RMP.
D. is caused mainly by inward movement of Na+ ions

186. As regard conduction of action potential in a nerve:


A. In thick myelinated nerve fibers can reach up to 120 meter / second
B. Can be increased by increase calcium
C. Can be increased by cooling
D. Is conducted with decrement

187. Pale (fast) fiber:


A. Contains much blood capillaries C. Contains low concentration of myoglobin
B. Doesn't show fatigue D. Depends on aerobic oxidation

188. About cellular sheath (sheath of Schwann):


A. It acts as an electric insulator around the soma of the neuron
B. It surrounds the axons of all neurons inside and outside the CNS
C. It is responsible for the color of the white matter of the spinal cord
D. It is essential for regeneration of the damaged verve fibers

189. Myelin sheath:


A. Present in the myelinated and unmyelinated nerve fibers
B. Formed of lipoprotein complex and acts as electric insulator
C. It is formed of successive wrappings of the membrane of Schwann cells
D. It is the cause of decreased conduction of nerve impulse

190. During depolarization:


A. voltage activated Na+ channels open
B. the membrane becomes impermeable to Na+
C. when membrane potential reaches 55 mv Na+ & K+ fluxes occur at the same time
D. K+ ions diffuse outside

191. The resting membrane potential is caused by:


A. Diffusion of K+ ions outside the nerve fibers C. Opening of the chemically activated ion channels
B. Diffusion of Na+ ions inside the nerve fibers D. Opening of the voltage activated ion channels

192. Repolarization:
A. Occurs at first gradual then becomes fast
B. Results from closure of sodium gates and opening of potassium gates
C. is represented by the ascending limb of the spike
D. is followed by appearance of response

193. Continuous conduction:


A. occurs in myelinated nerve fibers
B. occurs by jumping of charges from one node of Ranvier to another
C. is relatively slow 0.5.2.0 meter / second
D. occurs in the neuromuscular junction

194. Saltatory conduction:


116
Nerves & Muscles Physiology
A. occurs in unmyelinated nerve fibers
B. may reach up to 120 meter / second
C. occurs by jumping from one neuron to another
D. decreases gradually with distance till it disappears

195. Local excitatory state is characterized by the following EXCEPT:


A. its magnitude is directly proportional with the intensity of the sub-minimal stimulus
B. does not obey all or non-rule
C. can be summated
D. propagated without decrement

196. About A fibers all are true, EXCEPT:


A. They have the greatest diameter
B. They conduct impulses with the greatest velocity (10-120 m/sec)
C. They include somatic sensory and motor fibers
D. They are very sensitive to local anesthetic drugs

197. In all or non-rule:


A. A minimal stimulus produces a maximal response
B. The response in a single nerve fiber increases with increase intensity of stimulus
C. The nerve trunk either responds maximally or not responds at all
D. Minimal stimulus produces minimal response

198. Excitation contraction coupling involves all the following EXCEPT:


A. Release of Ca++ from troponin
B. Formation of cross bridges between actin and myosin
C. Spread of depolarization along the transverse tubules
D. Hydrolysis of ATP to ADP

199. Muscle fatigue is due to:


A. Inability of the action potential to spread over the muscle
B. Failure of transmission in the motor nerve
C. Failure of neuromuscular transmission
D. Depletion of energy stores

200. The motor end plate potential is produced by:


A. Opening of Na+ channels
B. Opening of Na+ channels then opening of K+ channels
C. Opening of Na+ and K+ channels at the same time
D. Opening of Ca++ channels

201. As regard neuromuscular transmission all are true EXCEPT:


A. It shows fatigue due to depletion of acetylcholine vesicles
B. Occurs from nerve to muscle i.e. One way conduction
C. It is stimulated by succinyl choline
D. It is blocked by botulinum toxins

202. Red (slow) fibers are characterized by the following EXCEPT:


A. Contains much blood capillaries B. Glycogen stores is low
117
Nerves & Muscles Physiology
C. Contains high concentration of myoglobin D. Depends on anaerobic oxidation

203. All about the Sarcomere are true EXCEPT:


A. Is the distance between myosin and actin C. Is the contractile unite of the muscle
B. Is the distance between two z membranes D. Shorten when the muscle contracts

204. Excitability of nerve fibers:


A. Is increased by decreased temperature C. Is decreased by decrease Ca++ ions
B. Is increased by decreased Na+ . D. Is completely lost by local anesthetic drugs

205. Contraction in multi–unit smooth muscle is:


A. Myogenic B. Neurogenic C. Both D. None of above

206. In intestinal smooth muscle, cold:


A. Increases the tone and decreases the rhythm C. Increases the tone and increases the rhythm
B. Decreases the tone and increases the rhythm D. Decreases the tone and rhythm

207. Rigor- mortis is due to depletion of:


A. ADP. B. ATP. C. Both D. Creatine phosphate

208. Muscle hypertrophy is due to:


A. Very weak muscular activity C. Moderate muscular activity
B. Very forceful muscular activity C. None of above

209. The electromyogram (EMG) is a record of the electrical activity of the:


A. Brain B. Heart C. Retina D. Skeletal muscle

210. The release of the neurotransmitter from the synaptic vesicles depends mainly on:
A. Ca++ B. K+ C. Na+ D. Mg2+

211. The action potential of the skeletal muscle:


A. Has a prolonged plateau phase
B. Is not essential for muscle contraction
C. Causes the immediate efflux of Ca++ from terminal cisternae
D. Is initiated spontaneously by the muscle itself

212. During the upstroke of the action potential:


A. There is net outward current and the cell interior becomes more negative
B. There is net outward current and the cell interior becomes less negative
C. There is net inward current and the cell interior becomes more negative
D. There is net inward current and the cell interior becomes less negative

213. The correct sequence of events at the neuromuscular junction is:


A. Action potential in the motor nerve  depolarization of the muscle end plate  uptake of Ca+2 into the
presynaptic nerve terminal
B. Uptake of Ca+2 into the presynaptic terminal  release of acetylcholine (Ach) depolarization of the
muscle end plate
C. Release of Achaction potential in the motor nerve action potential in the muscle
118
Nerves & Muscles Physiology
D. Uptake of Ca+2 into the motor end plate action potential in the motor end plateaction potential in
the muscle
E. Release of Ach action potential in the muscle end plate action potential in the muscle

214. Which characteristic or component is shared by skeletal muscle and smooth muscle?
A. Thick and thin filaments arranged in sarcomeres
B. Troponin
C. Elevation of intracellular Ca+2 concentration for excitation-contraction coupling
D. Spontaneous depolarization of the membrane potential
E. High degree of electrical coupling between cells

215. Repeated stimulation of a skeletal muscle fiber causes tetanic contraction because the intracellular
concentration of which solute increases and remains at high levels?
A. Na+ D. Mg+2 G. Calmodulin
+ ++
B. K E. Ca H. ATP
C. Cl− F. Troponin

216. During a nerve action potential, a second stimulus is delivered as indicated by the arrow shown in
figure 1. In response to the stimulus, a second action potential:

A. Of smaller magnitude will occur


B. Of normal magnitude will occur
C. Of normal magnitude will occur, but will be delayed
D. Will occur, but will not have an overshoot
E. Will not occur

217. The rate of conduction of action potentials along a nerve will be increased by:
A. Stimulating the Na+ - K+ pump
B. inhibiting the Na+ - K+ pump
C. Decreasing the diameter of the nerve
D. Myelinating the nerve
E. Lengthening the nerve fiber

218. A drug completely blocks Na+ channels in nerves. Which of following effects on the action potential
would it be expected to produce?
A. Block the occurrence of action potentials
B. Increase the rate of increase of the upstroke of the action potential
C. Shorten the absolute refractory period
D. Abolish the hyperpolarizing after-potential
E. Increase the Na+ equilibrium potential
119
Nerves & Muscles Physiology
219. In figure 2, at which labeled point on the action potential the relative refractory period is located?

A. 1 & 2. B. 3. C. 4. D. 4 & 5 E. 5.

220. In figure 2, what process is responsible for the change in membrane potential that occurs between
point 1 and point 3?
A. Movement of Na+ into the cell D. Movement of K+ out the cell
+
B. Movement of Na out of the cell E. Activation of the Na+ - K+ pump
+
C. Movement of K into the cell F. Inhibition of the Na+ - K+ pump

221. In figure 2, what process is responsible for the change in membrane potential that occurs between
point 3 and point 4?
A. Movement of Na+ into the cell D. Movement of K+ out of the cell
+
B. Movement of Na out the cell E. Activation of the Na+ - K+ pump
+
C. Movement of K into the cell F. Inhibition of the Na+ - K+ pump

222. In figure 2, what process is responsible for the change in membrane potential that occurs at area 5?
A. Movement of Na+ into the cell D. Movement of K+ out of the cell
+
B. Movement of Na out the cell E. Activation of the Na+ - K+ pump
+
C. Movement of K into the cell F. Inhibition of the Na+ - K+ pump

223. In figure 2, at which labeled point on the action potential the absolute refractory period is located?
A. 1 & 2. B. 2 & 3. C. 4. D. 4 & 5 E. 5.

224. An inhibitory postsynaptic potential:


A. Depolarizes the postsynaptic membrane by opening Na+ channels
B. Depolarizes the postsynaptic membrane by opening K+ channels
C. Hyperpolarizes the postsynaptic membrane by opening Ca++ channels
D. Hyperpolarizes the postsynaptic membrane by opening Cl – channels

225. Which of the following sequence of events is correct for excitation – contraction coupling in skeletal
muscle?
A. Increased intracellular Ca 2+ concentration  action potential in the muscle membrane cross- bridge
formation
B. Action potential in the muscle membrane  depolarization of the T tubules  release of Ca++ from the
sarcoplasmic reticulum (SR)
C. Action potential in the muscle membrane  splitting of adenosine triphosphate (ATP)  binding of Ca 2+
to troponin C.
D. Release of Ca++ from the SR  depolarization of the T tubules  binding of Ca 2+ to troponin C.
120
Nerves & Muscles Physiology

226. Which of the following events occurs before depolarization of the T tubules in skeletal muscle in the
mechanism of excitation – contraction coupling?
A. Depolarization of the sarcolemmal membrane
B. Opening of Ca 2+ release channels on the sarcoplasmic reticulum (SR)
C. Uptake of Ca++ into the SR by Ca 2+ - adenosine triphosphatase (ATP) ase
D. Binding of Ca++ to troponin C.
E. Binding of actin and myosin

227. Which of the following is always an inhibitory neurotransmitter in the central nervous system
(CNS)?
A. Norepinephrine D. Serotonin
B. Glutamate E. Histamine
C. Gamma-aminobutyric acid (GABA)

228. Which of the following statements is not correct? If the sodium-potassium pump were suddenly
poisoned:
A. The resting membrane potential would become less negative
B. No further transmission of nerve impulses could occur
C. The intracellular potassium concentration would decrease
D. The intracellular sodium concentration would increase

229. Which of the following statements is not correct? Release of acetylcholine at the neuromuscular
junction:
A. Produces an endplate potential
B. Increases sodium movement into the muscle fiber
C. Always causes the muscle fiber to contract
D. Is followed by rapid destruction of acetylcholine

230. In skeletal muscle, the:


A. Contraction precedes the action potential
B. Action potential lasts as long as the contraction
C. Action potential lasts longer than the contraction
D. Contraction and action potential begin simultaneously
E. Action potential precedes the contraction

231. During relaxation of skeletal muscle, ATP is required for:


A. Release of Ca++ B. Reuptake of Ca++ C. Reuptake of Mg2+ D. Release of K+

232. The _________ portion of the motor division of the nervous system conducts action potentials from
the central nervous system to skeletal muscle fibers.
A. Afferent B. Autonomic C. Somatic D. Sympathetic

233. The _________ division of the nervous system conducts actions potentials from the central nervous
system to cardiac muscle.
A. Afferent B. Autonomic C. Sensory D. Somatic

234. Which of the following effectors in NOT innervated by the autonomic nervous system?
A. Cardiac muscle B. Glands C. Skeletal muscle D. Smooth muscle
121
Nerves & Muscles Physiology

235. Which of the following types of neurons has a single process extending from the cell body which
later divides a short distance away into central and peripheral processes?
A. Unipolar B. Bipolar C. Multipolar D. None of the above

236. Collections of neuron cell bodies in the central nervous system are called
A. Myelin B. Axons C. Dendrites D. Nuclei

237. Which of the following is involved in the myelination of axons of the central nervous system?
A. Microglia B. Astrocytes C. Oligodendrocytes D. Schwann cells

238. Microglia are


A. Specialized motor neurons in the peripheral nervous system
B. Star-shaped cells that help form the blood-brain barrier
C. Specialized macrophages of the CNS that mobilize in response to inflammation
D. Found in choroid plexuses, and produce cerebrospinal fluid

239. Nodes of Ranvier are


A. Interruptions in the myelin sheath along the course of a myelinated axon
B. Collections of immune cells in the CNS
C. Satellite cells that support neuron cell bodies in ganglia
D. Gaps between choroids plexuses where cerebral spinal fluid emerges

240. Which type of propagation occurs along a myelinated axon?


A. Continuous B. Treppe C. Saltatory D. Migratory

241. Which of the following statements is NOT true concerning the resting membrane potential?
A. The membrane is relatively more permeable to potassium ions than to sodium ions
B. Potassium ions are constantly moving out of the cell down their concentration gradient
C. Sodium ions are in relatively higher concentration inside the cell than outside
D. The membrane is relatively permeable to chloride ions

242. During the repolarization phase of an action potential, which of the following is the primary
activity?
A. Sodium ions are flowing out of the cell C. Potassium ions are flowing into the cell
B. Sodium ions are flowing into the cell D. Potassium ions are flowing out of the cell

243. The threshold of a neuron is the


A. Total amount of neurotransmitter it takes to cause an action potential
B. Voltage that triggers activation of voltage-gated channels
C. Time between binding of the neurotransmitter and firing of an action potential
D. Voltage across the resting cell membrane

244. During the depolarization phase of an action potential, which of the following situations exists?
A. The membrane is becoming less permeable to all ions
B. The inside of the membrane is becoming more negative with respect to the outside
C. The membrane potential remains constant
D. The inside of the membrane is becoming more positive with respect to the outside
122
Nerves & Muscles Physiology
245. Local potentials (graded potential) have ALL of the following characteristics EXCEPT
A. Propagation C. Comparatively long duration
B. Variation of amplitude D. No refractory period

246. The one-way nature of action potential propagation down an axon is primarily due to
A. Sodium ion depletion C. The refractory period
B. Potassium ion depletion D. ATP loss

247. In the resting state of the membrane, which of the following situations exists for voltage-gated
sodium ion channels?
A. The activation and inactivation gates alternate being closed and open
B. The inactivation gate is open and the activation gate is closed
C. The activation and inactivation gates are both closed
D. The activation gate is open and the inactivation gate is closed

248. A supramaximal stimulus produces which of the following?


A. The fastest muscle contraction possible
B. A muscle contraction that is slower than one produced by a maximal stimulus
C. A greater frequency of action potentials than a maximal stimulus
D. None of the above
249. Which of the following types of stimuli produces only a local potential, but not an action potential?
A. Maximal B. Supramaximal C. Threshold D. Subthreshhold

250. Which of the following stimuli produces a depolarization that propagates from one end of the
membrane to the other?
A. Supramaximal B. Threshold C. Maximal D. All of the Above

251. Release of which neurotransmitter leads to the excitation and contraction of skeletal muscles?
A. Acetylcholine B. Epinephrine C. Glutamate D. Serotonin

252. Which of the following is NOT true concerning an electrical synapse?


A. Connexons carry ions between the two communicating cells
B. Information travels in one direction only
C. This type of synapse is characteristic of cardiac muscle
D. Information transfer does not involve the release of neurotransmitters

253. When a depolarization wave reaches the synaptic end bulb of a presynaptic neuron, the NEXT event
is
A. Uptake of neurotransmitter from the synaptic cleft
B. Active transport of calcium ions out of the cell
C. Diffusion of calcium ions into the cell
D. Immediate release of neurotransmitter

254. A neurotransmitter that allows sodium ions to leak into a postsynaptic neuron causes
A. No changes in the resting potential
B. Damage to the myelin sheath
C. Excitatory postsynaptic potentials
D. An alteration of the membrane threshold
123
Nerves & Muscles Physiology

255. The voltage across a neuron's membrane has changed from its resting potential of -70 mv to -80 mv.
Which of the following is most likely to be TRUE regarding this situation?
A. The neuron has bound a neurotransmitter that increases membrane permeability to sodium ions
B. Large pores have opened in the membrane to allow release of large intracellular anions
C. The neuron has bound a neurotransmitter that increases the permeability to chloride ions
D. Additional myelin has been added to the membrane to provide greater electrical insulation

256. The enzyme acetylcholinesterase insures that


A. Additional neurotransmitter is produced for skeletal muscle action
B. The neurotransmitter at the neuromuscular junction has a short duration of action
C. Excess GABA is destroyed rapidly
D. The inhibitory effect of glycine is prolonged

257. Neurons have cytoplasmic extensions that connect one neuron to another neuron. Given these
structures:
1. Axon
2. Dendrite
3. Dendritic spine
4. Presynaptic terminal
Choose the arrangement that lists the structures in the order they are found between two neurons.
A. 1,4,2,3 B. 1,4,3,2 C. 4,1,2,3 D. 4,1,3,2 E. 4,3,2,1

258. a neuron with many short dendrites and a single long axon is a ______________ neuron.
A. Multipolar B. Unipolar C. Bipolar

259. unmyelinated axons within nerves may have which of these associated with them?
A. Schwann cells B. Nodes of Ranvier C. Oligodendrocytes D. All of the above

260. action potentials are conducted more rapidly


A. In small-diameter axons than in large-diameter axons
B. In unmyelinated axons than in myelinated axons
C. Along axons that have nodes of Ranvier
D. All of the above

261. concerning concentration difference across the plasma membrane of muscle fiber, there are
A. More K+ and Na+ outside the cell than inside
B. More K+ and Na+ inside the cell than outside
C. More K+ outside the cell than inside and more Na+ inside the cell than outside
D. More K+ inside the cell than outside and more Na+ outside the cell than inside

262. compared to the inside of the resting plasma membrane, the outside surface of the membrane is
A. Positively charged
B. Electrically neutral
C. Negatively charged
D. Continuously reversing so that it is positive one second and negative the next
E. Negatively charged whenever the Na+ –K+ pump is operating

263. non-gated ion channels


124
Nerves & Muscles Physiology
A. Open in response to small voltage changes
B. Open when a ligand binds to its receptor
C. Are responsible for the ion permeability of the resting plasma membrane
D. Allow substances to move into the cell but not out
E. All of the above

264. the resting membrane potential results when the tendency for _____________ to diffuse out
Of the cell is balanced by their attraction to opposite charges inside the cell.
A. Na+ B. K+ C. Cl− D. Negatively charged proteins

265. if the permeability of the plasma membrane to K+ increases, resting membrane potential
_____________. This is called _____________.
A. Increases, hyperpolarization C. Decreases, hyperpolarization
B. Increases, depolarization D. Decreases, depolarization

266. decreasing the extracellular concentration of K+ affects the resting membrane potential by causing
A. Hyperpolarization B. Depolarization C. No change

267. which of these terms are correctly matched with their definition or description?
A. Depolarization: membrane potential becomes more negative
B. Hyperpolarization: membrane potential becomes more negative
C. Hypopolarization: membrane potential becomes more negative

268. local potentials


A. Spread over the plasma membrane in decremental fashion
B. Are not propagated for long distances
C. Are graded
D. Can summate
E. All of the above

269. during the depolarization phase of an action potential, the permeability of the membrane
A. To K+ is greatly increased C. To Ca++ is greatly increased
+
B. To Na is greatly increased D. Is unchanged

270. during the repolarization phase of an action potential, the permeability of the membrane
A. To K+ is greatly increased C. To Ca++ is greatly increased
+
B. To Na is greatly increased D. Is unchanged

271. the absolute refractory period


A. Limits how many action potentials can be produced during a given period of time
B. Prevents an action potential from starting another action potential at the same point on the plasma
membrane
C. Is the period of time when a strong stimulus can initiate a second action potential
D. Both A and B.
E. All of the above

272. A subthreshold stimulus


A. Produces an after-potential
B. Produces a local potential
125
Nerves & Muscles Physiology
C. Causes an all-or-none response
D. Produces more action potentials than a submaximal stimulus

273. in a chemical synapse,


A. Action potentials in the presynaptic terminal cause voltage-gated Ca++ channels to open
B. Neurotransmitters can cause ligand-gated Na+ channels to open
C. Neurotransmitters can be broken down by enzymes
D. Neurotransmitters can be taken up by the presynaptic terminal
E. All of the above

274. an inhibitory presynaptic neuron can affect a postsynaptic neuron by


A. Producing an IPSP in the postsynaptic neuron
B. Hyperpolarizing the plasma membrane of the postsynaptic neuron
C. Causing K+ to diffuse out of the postsynaptic neuron
D. Causing Cl− to diffuse into the postsynaptic neuron
E. All of the above

275. summation
A. Is caused by combining two or more local potentials
B. Occurs at the trigger zone of the postsynaptic neuron
C. Results in an action potential if it reaches the threshold potential
D. Can occur when two action potentials arrive in close succession at a single Presynaptic terminal
E. All of the above

276. in convergent pathways,


A. The response of the postsynaptic neuron depends on the summation of EPSPs and IPSPs
B. A smaller number of presynaptic neurons synapse with a larger number of postsynaptic neurons
C. Information transmitted in one neuronal pathway can go into two or more pathways
D. All of the above

277. one of the characteristics of muscle tissue is that is has conductivity. Which of the following best
describes this characteristic?
A. Muscles contract when stimulated
B. Muscles have local electrical charges that are capable of moving along the muscle fiber
C. Muscles only pull; they cannot push
D. Muscles can stretch when needed

278. one of the characteristics of muscle tissue is that is has extensibility. Which of the following best
describes this characteristic?
A. Muscles contract when stimulated
B. Muscles have local electrical charges that move along the muscle fiber
C. Muscles only pull; they cannot push
D. Muscles can stretch when needed

279. one of the characteristics of muscle tissue is that is has excitability. Which of the following best
describes this characteristic?
A. Muscles contract when stimulated
B. Muscles have local electrical charges that move along the muscle fiber
C. Muscles only pull; they cannot push
126
Nerves & Muscles Physiology
D. Muscles can stretch when needed

280. which of the following groups are considered to be parts of the series-elastic components of
muscle?
A. Actin and myosin
B. Ligaments and tendons
C. Thick and thin filaments
D. Endomysium, perimysium, epimysium, fascia, and tendons

281. which of the following best describes the function of the series-elastic component of muscle?
A. These components help the muscle contract
B. The components help maintain muscle tone
C. These are not excitable or contractile, but are extensible and elastic
D. These assist the thick and thin filaments to contract

282. what part of a muscle is the muscle fiber?


A. Actin and myosin component C. Muscle cell itself
B. Series-elastic component D. Nuclei and the mitochondria

283. the molecule within the cytoplasm of a muscle fiber that supplies quick energy for contraction is
A. Fructose B. Glycogen C. Sucrose D. ADP

284. thick filaments within the myofibril are composed of


A. Actin B. Myosin C. Titin D. Sarconin

285. thin filaments within the myofibril are composed primarily of


A. Actin B. Myosin C. Titin D. Sarconin

286. in the myofibril, ATP is used for which of the following?


A. Muscle tone B. Muscle contraction C. Muscle relaxation D. All of the above

287. which of the following tissues has the greatest ability to repair itself?
A. Skeletal muscle B. Cardiac muscle C. Smooth muscle D. All have about the same ability

288. of the following muscles types, which one has long, cylindrical-shaped cells?
A. Skeletal muscle B. Cardiac muscle C. Smooth muscle D. All are about the same

289. of the following muscle types, which one has only one nucleus?
A. Skeletal muscle B. Cardiac muscle C. Smooth muscle D. All of the above

290. the recovery stroke in muscle contraction involves which of the following?
A. Movement of the myosin molecule while the cross-bridge is attached
B. Return of the myosin head to its original position after the cross bridge releases
C. Actin slides on myosin
D. Myosin slides on actin

291. the striations in striated muscle correspond with actual molecular structures. Which of the
following is the I band?
A. The band where only actin molecules are found
127
Nerves & Muscles Physiology
B. The band where only myosin molecules are found
C. An attachment point for actin molecules
D. An attachment point for myosin molecules

292. the striations in striated muscle correspond with actual molecular structures. Which of the
following is the h band?
A. The band where only actin molecules are found
B. The band where only myosin molecules are found
C. An attachment point for actin molecules
D. An attachment point for myosin molecules

293. during an action potential, calcium is released into the sarcoplasm, near the actin and myosin
molecules. Which of the following is the best explanation of what the calcium does?
A. The calcium binds to the actin and myosin and makes them work together
B. The calcium breaks apart ATP to ADP and P.
C. The calcium removes the tropomyosin block
D. The calcium causes the release of acetylcholine

294. when the myosin head (of the thick filament) flexes into a bend, while releasing ADP and inorganic
phosphate, pulling the thin filament along with it, this is called the
A. Action reaction B. Power stroke C. Recovery stroke D. Muscle tone

295. intercalated discs are capable of doing which of the following?


A. Allow for re-growth of damaged muscle cells
B. Pass the resting potential to other cardiac cells
C. Pass electrical stimulation to other cardiac cells
D. Allow for cardiac tissue to expand and contract properly

296. which of the following is unique about cardiac muscle tissue?


A. The cells have large mitochondria C. There are very few nuclei within the cells
B. The cells are large and wide D. There is no neuron innervation

297. which of the following is true of cardiac tissue?


A. It contains only small amounts of myoglobin and glycogen
B. It has few nerve endings within the tissue
C. The fibers are not arranged in bundles
D. It has abundant glycogen and myoglobin and is aerobic

298. When the neuron stimulates a muscle fiber, what molecule diffuses across the gap between them?
A. Calcium B. Acetylcholine C. Sodium D. Sodium and potassium

299. Muscle fibers and neurons are called electrically excitable cells because
A. Their membranes respond to electricity
B. Their plasma membranes exhibit voltage changes in response to stimulation
C. Their surfaces respond to the release of acetylcholine
D. They have junctional folds in the membranes that can trap electrical charges

300. Movement of which of the following proteins causes exposure of the active sites on the actin
molecule?
128
Nerves & Muscles Physiology
A. Troponin B. Tropomyosin C. Myosin D. Acetylcholine

301. Binding of acetylcholine with its receptor on muscle cells causes


A. Opening of a ligand-gated sodium channel
B. A gap in the cell membrane for calcium diffusion
C. Vesicle release of sodium ions
D. Opening of a ligand-gated calcium channel

302. What is the voltage of the resting potential of the sarcolemma?


A. 70 mv B. -70 mv C. -45 mv D. -15 mv

303. Which of the following is a molecule that binds to a receptor?


A. Acetylcholine B. Tropomyosin C. Acetylcholinesterase D. All of the above

304. Which of the following assures quick removal of neurotransmitter at the junction between a nerve
and a muscle?
A. Acetylcholine B. Dopamine C. Acetylcholinesterase D. PABA

305. A buildup of acetylcholine in the synaptic cleft without breakdown may cause which of the
following?
A. Spastic paralysis B. Treppe C. Myasthenia gravis D. Flaccid paralysis

306. In a muscle twitch, which of the following represents the time between application of a stimulus
and the beginning of contraction?
A. Treppe B. Tetany C. Lag (latent) D. Relaxation

307. Which of the following determines the height of the peaks in multiple motor unit summation?
A. The speed of stimuli
B. The amount of tension produced by the number of motor units responding
C. The strength of each stimulus
D. None of the above

308. Which of the following would represent incomplete tetanus?


A. Multiple contractions with no relaxation
B. Multiple contractions with slight relaxation
C. Multiple contractions followed by a long rest period
D. A long rest period before contractions start

309. Which of the following explains why treppe takes place in muscle contractions?
A. Decreased calcium levels around the myofibrils
B. Increased sodium around the myofibrils
C. Increased calcium levels around the myofibrils
D. Decreased potassium around the myofibrils

310. Active tension in a muscle is defined as


A. The tension applied to load when the muscle is stretched but not stimulated
B. The tension applied to a load when myofilaments within sarcomeres minimally overlap
C. The tension applied to a load when the muscle contracts
D. Muscle contraction produced wholly from complete tetanus
129
Nerves & Muscles Physiology

311. The sum of active and passive tension is called


A. Concentric tension B. Total tension C. Sarcomere tension D. Treppe

312. Which of the following energy storages source is most commonly used during the first 10 seconds of
exercise?
A. ATP B. Creatine phosphate C. Lactic acid D. None of the above

313. During anaerobic respiration, what is the net amount of ATP produced?
A. 4 ATP B. 6 ATP C. 20 ATP D. 2 ATP

314. All of the following are characteristics of slow-twitch fibers EXCEPT


A. Slow contraction B. Large diameter C. Good blood supply D. Many mitochondria

315. All of the following are true about fast-twitch fibers EXCEPT
A. High amount of myoglobin C. Break down ATP rapidly
B. Rapid contraction D. Fewer mitochondria than slow-twitch fibers

316. Which of the following is true about distribution of fast and slow-twitch fibers in the body?
A. Fast-twitch fibers are primarily found in long distance runners
B. Slow-twitch fibers are usually more resistant to fatigue
C. There is a clear separation of slow- and fast-twitch fibers in individual muscles
D. Training does not influence the balance of fast- and slow-twitch fibers

317. Unitary smooth muscle is found in all of the following locations in the body EXCEPT?
A. Digestive tract B. Reproductive tract C. Urinary tract D. Iris of the eye

318. Which of the following is true about smooth muscle?


A. The resting membrane potential is less negative than in skeletal muscle
B. It responds in an "all or none" fashion
C. It does not respond to hormones
D. It has a faster speed of contraction than skeletal muscle

319. Which of the following are the most common neurotransmitters in smooth muscle?
A. Dopamine and epinephrine C. Acetylcholine and norepinephrine
B. Acetylcholine and epinephrine D. Acetylcholine and dopamine

320. All of the following are properties of cardiac muscle cells EXCEPT
A. Intercalated discs
B. Involuntary
C. Branching fibers
D. Shorter in duration of contraction than skeletal muscles

321. During contraction, muscles shorten __________, and lengthen __________.


A. Actively, actively B. Actively, passively C. Passively, actively D. Passively, passively

322. Which of these is NOT a part of the actin myofilament?


A. Actin B. Cisternae C. Tropomyosin D. Troponin
130
Nerves & Muscles Physiology
323. Which of these locations has the ability to break down ATP, which releases energy?
A. Globular actin molecule D. Tropomyosin molecule
B. Head of myosin molecule E. Troponin-tropomyosin complex
C. Troponin molecule

324. Troponin binds to


A. Actin B. Calcium ions C. Tropomyosin D. All of these

325. Which of these units contains the Z disk?


A. A band B. H zone C. I band D. M line

326. The resting membrane potential occurs when


A. Sodium ions diffuse into the cell D. Potassium ions diffuse out of the cell
B. Sodium ions diffuse out of the cell E. Large negative molecules diffuse out of the cell
C. Potassium ions diffuse into the cell

327. If the permeability of the plasma membrane to Na+ ions increases, the result is
A. Depolarization of the plasma membrane
B. Repolarization of the plasma membrane
C. Little, if any, change in the resting membrane potential of the plasma membrane

328. In the neuromuscular junction, the axon is also called the


A. Postsynaptic membrane C. Synaptic cleft
B. Presynaptic terminal D. Synaptic vesicle

329. Given these events:


1. Acetylcholine is released from synaptic vesicles
2. An action potential reaches the presynaptic terminal
3. Ca++ ions diffuse into the cell
4. Acetylcholine is secreted from the presynaptic terminal by exocytosis
Choose the arrangement that lists the events in the order they occur when an action potential travels
through the axon of a motor neuron.
A. 1,2,3,4 B. 1,3,4,2 C. 2,1,3,4 D. 2,3,1,4 E. 3,2,1,4

330. Given these events:


1. Acetylcholine binds to receptors on the postsynaptic membrane
2. Acetylcholinesterase breaks down acetylcholine
3. Choline is reabsorbed by presynaptic membrane
4. Depolarization of postsynaptic membrane occurs
5. Ligand-gated Na+ ion channels open
Choose the arrangement that lists the events in the order they occur when an action potential travels
through the axon of a motor neuron.
A. 1,2,3,4,5 C. 2,1,3,4,5 E. 5,4,1,2,3
B. 1,5,4,2,3 D. 4,3,2,1,5

331. The drug atropine blocks neurotransmitter receptors on the postsynaptic terminal of the
neuromuscular junction. This cause
A. Flaccid paralysis B. Spastic paralysis C. No change in muscle function
131
Nerves & Muscles Physiology
332. The enlarged areas of the sarcoplasmic reticulum near the T tubules are the
A. Sarcolemma C. Terminal cisternae E. Tropomyosin molecules
B. Sarcomeres D. Troponin molecules

333. Given these events:


1. Action potential travels along the sarcolemma
2. T tubules undergo depolarization
3. Voltage-gated Ca++ ion channels in sarcoplasmic reticulum open
4. Ca++ ions diffuse into the sarcoplasm
5. Ca++ ions bind to troponin molecules
Choose the arrangement that lists these events in the order they occur following a single stimulation of a
skeletal muscle cell.
A. 1,2,3,4,5 B. 1,3,5,4,2 C. 2,1,3,4,5 D. 3,1,5,2,4 E. 4,5,12,3

334. Cross bridges form during muscle contraction when myosin comes in contact with active sites on
A. Troponin molecules C. Calcium ions E. Actin molecules
B. Tropomyosin molecules D. Acetylcholine molecules

335. Given these events:


1. Active sites on actin myofilament are exposed
2. Actin myofilament slides over myosin myofilament
3. Ca++ ion binds to troponin
4. Myosin heads move
5. Cross bridges form
Choose the arrangement that lists the correct order in which they occur during a single stimulation of a
skeletal muscle.
A. 4,3,2,1,5 B. 3,1,5,4,2 C. 3,2,5,4,1 D. 2,4,3,5,1 E. 1,2,3,4,5

336. For cross bridge release, it is necessary for __________ to attach to the myosin head.
A. A tropomyosin molecule C. Ca++ ions E. The sarcolemma
B. ATP D. A troponin molecule

337. Which of these is (are) required for the relaxation of a muscle fiber?
A. ATP D. No additional stimulation of the neuron
B. Ca++ ion uptake into the sarcoplasmic reticulum E. All of these
C. Acetylcholinesterase activity

338. Even though each muscle fiber responds in all-or-none fashion, a whole muscle can contract with
varying force because of
A. Different threshold values of each fiber D. Rigor mortis
B. Total tetanus of the muscle E. The latent phase of contraction
C. Multiple motor unit summation

339. Which of these processes results in an increase in the force of contraction of a whole muscle?
A. Multiple motor unit summation C. Treppe
B. Multiple wave summation D. All of these

340. A muscle contraction in which the muscle produces an increasing tension, but the length remains
constant is a(n)
132
Nerves & Muscles Physiology
A. Isometric contraction C. Concentric contraction
B. Isotonic contraction D. Eccentric contraction

341. Given these sources of energy during exercise:


1. Anaerobic respiration
2. Aerobic respiration
3. ATP and creatine phosphate in the cell
The correct sequence for the use of these energy sources by muscle cells when an individual starts
exercising is
A. 1,2,3 B. 1,3,2 C. 2,1,3 D. 3,2,1 E. 3,1,2

342. Fast-twitch muscle fibers


A. Are more resistant to fatigue than slow-twitch fibers
B. Have a richer blood supply than slow-twitch fibers
C. Have a larger amount of stored glycogen than slow-twitch fibers
D. Have more mitochondria than slow-twitch fibers
E. Have more myoglobin than slow-twitch fibers

343. Which of these characteristics is typical of a slow-twitch muscle fiber?


A. Large number of mitochondria D. Largely anaerobic
B. Few capillaries E. B, C, and D
C. Low myoglobin content

344. Which of these is correctly matched?


A. Multiunit smooth muscle -- spontaneous contractions
B. Visceral smooth muscle -- contracts when stretched
C. Skeletal muscle -- slower, longer lasting contractions than smooth muscle
D. Multiunit smooth muscle -- numerous gap junctions; autorhythmic
E. All of these

345. Visceral smooth muscle


A. Responds in an all-or-none fashion to action potentials
B. Has slow waves of depolarization and repolarization
C. Has depolarization occur when the cell becomes more permeable to Na+ ions and K+ ions
D. Requires a stimulus from a neuron or hormone
E. Produces strong contractions that only last for a few milliseconds

346. Which of these is true of skeletal muscle?


A. Spindle-shaped cells D. Forms the walls of hollow internal organs
B. Under involuntary control E. May be autorhythmic
C. Many peripherally located nuclei per muscle cell

347. Which of these is not a major functional characteristic of muscle?


A. Contractility B. Elasticity C. Excitability D. Extensibility E. Secretability

348. Each myofibril


A. Is made up of many muscle fibers D. Contains T tubules
B. Contains sarcoplasmic reticulum E. Is the same thing as a muscle fiber
C. Is made up of many sarcomeres
133
Nerves & Muscles Physiology

349. Which of these statements about the molecular structure of myofilaments is true?
A. Tropomyosin has a binding site for Ca++
B. The head of the myosin molecule binds to an active site on G actin
C. ATPase is found on troponin
D. Troponin binds to the rodlike portion of myosin
E. Actin molecules have a hingelike portion that bends and straightens during contraction

350. The part of the sarcolemma that invaginates into the interior of the skeletal muscle cells is the
A. T tubule system D. Terminal cisternae
B. Sarcoplasmic reticulum E. Mitochondria
C. Myofibrils

351. Given these events:


1. Acetylcholine broken down into acetic acid and choline
2. Acetylcholine moves across the synaptic cleft
3. Action potential reaches the terminal branch of the motor neuron
4. Acetylcholine combines with a receptor molecule
5. Action potential is produced on the muscle fiber’s plasma membrane
Choose the arrangement that lists the events in the order they occur at a neuromuscular junction.
A. 2,3,4,1,5 B. 3,2,4,5,1 C. 3,4,2,1,5 D. 4,5,2,1,3 E. 5,1,2,4,3

352. Acetylcholinesterase is an important molecule in the neuromuscular junction because it


A. Stimulates receptors on the presynaptic terminal
B. Synthesizes acetylcholine from acetic acid and choline
C. Stimulates receptors within the postsynaptic membrane
D. Breaks down acetylcholine
E. Causes the release of Ca++ from the sarcoplasmic reticulum

353. Given these events:


1. Sarcoplasmic reticulum releases Ca++
2. Sarcoplasmic reticulum takes up Ca++
3. Ca++ diffuse into myofibrils
4. Action potential moves down the T tubule
5. Sarcomere shortens
6. Muscle relaxes
Choose the arrangement that lists the events in the order they occur following a single
Stimulation of a skeletal muscle cell.
A. 1,3,4,5,2,6 C. 4,1,3,5,2,6 E. 5,1,4,3,2,6
B. 2,3,5,4,6,1 D. 4,2,3,5,1,6 F. 4,1,3,5,2,6

354. Given these events:


1. Ca++ combine with tropomyosin
2. Ca++ combine with troponin
3. Tropomyosin pulls away from actin
4. Troponin pulls away from actin
5. Tropomyosin pulls away from myosin
6. Troponin pulls away from myosin
7. Myosin binds to actin
134
Nerves & Muscles Physiology
Choose the arrangement that lists the events in the order they occur during muscle
Contraction.
A. 1,4,7 B. 2,5,6 C. 1,3,7 D. 2,4,7 E. 2,3,7

355. With stimuli of increasing strength, which of these is capable of a graded response?
A. Nerve axon B. Muscle fiber C. Motor unit D. Whole muscle

356. Jerry Jogger’s 3 mile run every morning takes about 30 minutes. Which of these sources provides
most of the energy for his run?
A. Aerobic respiration C. Creatine phosphate
B. Anaerobic respiration D. Stored ATP

357. Which of these increases the least as a result of muscle hypertrophy?


A. Increase in number of sarcomeres D. Increase in blood vessels and mitochondria
B. Increase in number of myofibrils E. Increase in connective tissue
C. Increase in number of fibers

358. Which of these conditions would one expect to find within the leg muscle cells of a world-class
marathon runner?
A. Myoglobin-poor C. Primarily anaerobic E. Large deposits of glycogen
B. Contract very quickly D. Numerous mitochondria

359. Relaxation in smooth muscle occurs when


A. Myosin kinase attaches phosphate to the myosin head
B. Ca++ binds to calmodulin
C. Myosin phosphatase removes phosphate from myosin
D. Ca++ channels open
E. Ca++ is released from the sarcoplasmic reticulum

360. The ______ is a depression in the sarcolemma that receives a motor nerve ending.
A. T tubule B. Terminal cisterna C. Sarcomere D. Motor end plate E. Synapse

361. Before a muscle fiber can contract, ATP must bind to


A. A Z disc B. The myosin head C. Tropomyosin D. Troponin E. Actin

362. Smooth muscle cells have______, whereas skeletal muscle fibers do not.
A. Sarcoplasmic reticulum C. Calmodulin E. Myosin ATPase
B. Tropomyosin D. Z discs

363. Single-unit smooth muscle cells can stimulate each other because they have
A. A latch-bridge C. Gap junctions E. Calcium pumps
B. Diffuse junctions D. Tight junctions

364. Warm-up exercises take advantage of ______ to enable muscles to perform at peak strength.
A. The stress-relaxation response D. Oxygen debt
B. The length-tension relationship E. Treppe
C. Excitatory junction potentials

365. Choose one correct answer.


135
Nerves & Muscles Physiology
A. Heart muscle cells receive motor neuron innervation
B. Heart cells are held together at the intercalated disk
C. Transverse tubules directly connect the interstitial fluid to the cytoplasm of the cell
D. None are correct

366. Choose one correct answer.


A. Gap junctions are part of the intercalated disk
B. The A-band is composed of actin filaments
C. Action potentials cannot propagate into the t-tubules of muscle
D. None are correct

367. Choose one correct answer.


A. Skeletal muscle contracts because the intracellular calcium goes up inside transverse tubules
B. Blockade of SR Ca release channels will increase the magnitude of the calcium transient in heart muscle
C. Epinephrine will decrease the strength of contraction in heart muscle
D. None are correct

368. Choose one correct answer.


A. Cardiac muscle requires beat-to-beat Ca influx to trigger contraction
B. A DHP blocker will increase the strength of contraction in heart muscle
C. SR Ca release channels are called DHP receptors
D. None are correct

369. Choose one correct answer.


A. Digitalis decreases the strength of contraction of heart muscle
B. Na-Ca exchange requires ATP directly and normally acts to pump Ca into heart cells
C. Blockade of SR Ca pumps will increase the rate of relaxation of heart and skeletal muscle
D. None are correct

370. Choose one correct answer.


A. Calcium binds directly to myosin to promote crossbridge formation
B. In a resting heart muscle cell, tropomyosin acts to shield the myosin binding sites on actin
C. Binding of ATP to actin acts to detach myosin from actin
D. None are correct

371. Choose one correct answer.


A. During an isometric contraction the tension goes up and the muscle shortens
B. Heart muscle cells can normally develop tetanic contractions
C. Skeletal muscle cells are normally held at a muscle length that maximizes their ability to generate force
D. None are correct
Correct Answer: C

372. Choose one correct answer.


A. Inositol triphosphate (IP3) inhibits SR Ca release in smooth muscle
B. The calcium–calmodulin complex in smooth muscle inhibits myosin-light chain kinase
C. Smooth muscle can develop both tonic and phasic contractions
D. None are correct

373. During an action potential the repolarization phase is the result of:
136
Nerves & Muscles Physiology
A. passive transport of Na+ out of the cell
B. passive transport of K+ out of the cell
C. active transport of Na+ out of the cell
D. active transport of K+ out of the cell

374. When an action potential occurs in a neuron, the after hyperpolarization is caused by:
A. a burst of activity of the Na+ /K+ pump
B. inhibition of the Na+ /K+ pump
C. increased permeability of the cell membrane to K+
D. increased permeability of the cell membrane to Na+

375. All other things being equal, which of the following axons will have the fastest conduction velocity?
A. a thick unmyelinated fiber
B. a thin unmyelinated fiber
C. a thick myelinated fiber
D. a thin myelinated fiber

Use the following graph to answer the next 5 questions:

376. The depolarization phase of the action potential


(indicated by arrow P) is caused by:
A. Increased permeability of the cell membrane to K+
B. Increased permeability of the cell membrane to Na+
C. Decreased permeability of the cell membrane to Na+
D. Inhibition of the Na+ /K+ pump

377. At which point on this graph is the cell membrane most


permeable to K+?
A. Point P B. Point Q C. Point R D. Point S
378. If the cell membrane were treated with a drug which completely inhibited the diffusion of Na+
across the cell membrane, the resting membrane potential of the nerve cell would:
A. Go to line A B. Go to line B C. Go to line C D. Not change

379. The relative refractory period is the time:


A. When the nerve cell cannot be stimulated irrespective of the strength of the stimulus
B. When the nerve cell can be stimulated but requires an above normal stimulus
C. Before the arrival of the action potential
D. Which roughly corresponds to the period of increased Na+ permeability

380. Which of the lines corresponds to the threshold?


A. Line A B. Line B C. Line C D. None of them

381. An excitatory post synaptic potential:


A. Is a partial depolarization of the postsynaptic membrane
B. Is a partial hyperpolarization of the post synaptic membrane
C. Can be a partial depolarization or hyperpolarization of the post synaptic membrane
D. Shows the changes in membrane potential typical of an action potential

382. Action potentials do not occur in the dendrites of neurons because:


137
Nerves & Muscles Physiology
A. The cell membrane of dendrites does not contain voltage gated Na+ channels
B. The cell membrane of dendrites does not contain ligand gated Na+ channels
C. The resting membrane potential of dendrites is too (cell) negative
D. Axon terminals of presynaptic neurons rarely terminate on the dendrites

383. A graded potential has all of the following properties EXCEPT:


A. Can be a hyperpolarization or a depolarization
B. Decreases in amplitude as it spreads across the cell membrane
C. Can add to other graded potentials
D. Has a refractory period of about 1 millisecond

384. Which of the following statements about the axon hillock (initial segment) is true?
A. Is the site on the neuron where summation can occur
B. Is the site where a graded potential can be transformed into one or more action potentials
C. Does not exist in neurons which have myelinated axons
D. Is the site of storage of synaptic vesicles

385. Where in the membrane of a nerve cell would you expect to find the highest concentration of
voltage regulated Na+ channels?
A. Cell body B. Dendrites C. Presynaptic membrane D. Axon hillock (initial segment)

386. The axon membrane propagates action potentials whereas the dendrite membrane only propagates
post synaptic (graded) potentials because the axon membrane:
A. Has a more negative resting membrane potential
B. Is mostly covered by a layer of myelin
C. Contains ligand regulated ion channels
D. Contains voltage regulated ion channels

387. Which of the following properties is characteristic of a graded potential?


A. Has an absolute refractory period of about 1 millisecond
B. Decreases in amplitude as it passes along the cell membrane
C. Can be a depolarization or a hyperpolarization
D. Has a threshold about 15 mv less negative than the resting membrane of the cell

388. At steady state all of the following processes are occurring in the unexcited nerve cell EXCEPT:
A. K+ is leaving the cell down its electrochemical gradient
B. K+ is being actively transported into the cell
C. The cell membrane is more permeable to K+ than to Na+
D. Na+ is leaving the cell down its electrochemical gradient

389. If a neuron was treated with a chemical that blocked the passive movement of K+, the resting
membrane potential of the cell would:
A. Change to the threshold potential of the neuron
B. Change to the K+ equilibrium potential
C. Change to the Na+ equilibrium potential
D. Change to zero (i.e. No membrane potential)

390. Which of the following changes will cause the membrane potential of a neuron to hyperpolarize
(become more negative)?
138
Nerves & Muscles Physiology
A. Reducing the extracellular concentration of K+
B. Increasing the permeability of the cell membrane to Na+
C. Increasing the extracellular concentration of Na+
D. Treating the cell with ouabain

391. If the concentration of KCl outside a nerve cell were increased, the resting membrane potential of
the cell would:
A. Not be affected because the cell membrane is relatively impermeable to K+
B. Not be affected because K+ is pumped out of the cell
C. Hyperpolarize (become more negative)
D. Depolarize (become less negative)

392. Which of the following actions would reduce the resting membrane potential of the nerve cell (i.e.
make it less negative)?
A. Increase the extracellular concentration of K+
B. Decrease the extracellular concentration of Na+
C. Increase the permeability of the membrane to K+
D. None of the above would reduce the resting membrane potential

393. If the permeability of a nerve cell to K+ were increased, the resting membrane potential of the cell
would:
A. becomes more negative (hyperpolarize)
B. becomes less negative (depolarize)
C. briefly becomes less negative and then returns to its original value
D. not change because the membrane is virtually impermeable to K+

394. Which of the following statements about synapses is not true?


A. Each neuron can have a synapse with only one other neuron
B. The neurotransmitter of a synapse is stored in small sacs called synaptic vesicles
C. There are often enzymes in the synaptic cleft which destroy neurotransmitters
D. The post-synaptic membrane contains specific binding sites for the neurotransmitters

395. Which of the following statements about chemical synapses is not true?
A. They always have acetyl choline as a neurotransmitter
B. They allow transmission of nerve impulses in only one direction
C. They can be inhibitory or excitatory
D. They can be found inside the spinal cord

396. Which of the following properties is characteristic of a chemical synapse?


A. Allows the transmission of nerve impulses in both direction across the synapse
B. Always results in the depolarization of the post synaptic cell
C. Has specific cell surface receptors on the post synaptic membrane
D. Generally occurs between a muscle cell and another muscle cell

397. Removal of a neurotransmitter from the synaptic cleft can occur by:
A. Reuptake of the neurotransmitter by the pre-synaptic neuron
B. Diffusion of the neurotransmitter away from the synaptic cleft
C. Destruction of the neurotransmitter by enzymes located on the post-synaptic membrane
D. Any of the above mechanisms
139
Nerves & Muscles Physiology

398. The function of the initial segment (axon hillock) of a neuron is to:
A. Convert graded post synaptic potentials into action potentials
B. Synthesize the neurotransmitter of the neuron
C. Receive incoming impulses from other neurons
D. Release neurotransmitter when depolarized

399. An inhibitory neuron could affect the neuron with which it synapses by:
A. Producing an IPSP within the neuron D. Increasing Cl− influx into the neuron
B. Hyperpolarizing the neuron E. All of the above
C. Increasing K+ efflux from the neuron F. 2 of the above

400. Summation:
A. Is caused by a combining of several local potentials
B. Can occur when 2 action potentials arrive simultaneously at 2 different presynaptic terminals
C. Can occur when 2 action potentials arrive in very close succession at a single presynaptic terminal
D. All of the above

401. Following production of ADP during contraction of skeletal muscle, the most rapid way of
regenerating ATP is:
A. Phosphorylation of ADP by creatine phosphate
B. Anaerobic glycolysis of cell stores of glycogen
C. Anaerobic glycolysis of glucose from the blood
D. Oxidative phosphorylation of fatty acids from the blood

402. The cytoplasmic Ca++ concentration of skeletal muscle would be expected to be highest in:
A. Relaxed muscle
B. During unfused tetanus
C. During fused tetanus
D. At the point during a single twitch when maximum muscle tension is developed

403. A skeletal muscle contracting isometrically generates the greatest tension at its normal resting
length because at that length:
A. The cellular concentration of ATP is highest
B. Sarcoplasmic reticulum is most precisely aligned with the thick filaments
C. The rate of conduction of the action potential across the cell membrane is most rapid
D. The greatest number of cross bridges is aligned with actin binding sites

404. When contracting isometrically, a skeletal muscle will develop its greatest tension:
A. At its normal resting length
B. At 60% of its normal resting length
C. At 150% of its normal resting length
D. The tension developed is constant, irrespective of its length

405. The force of contraction of a skeletal muscle depends on all of the following factors EXCEPT:
A. Its initial length C. The type of muscle fiber it contains
B. The number of motor units activated D. The extracellular Ca++ concentration
140
Nerves & Muscles Physiology
406. Most of the energy for a skeletal muscle functioning for 20 minutes at 25% of its maximum output is
supplied by:
A. Stored ATP C. Anaerobic metabolism of glycogen
B. Stored creatine phosphate D. Aerobic metabolism of fatty acids

407. A motor unit is defined as:


A. A single neuron and all the muscle cells which it innervates
B. A single muscle cell and all the neurons which innervate it
C. All the muscles involved in movement of a single joint
D. All the muscles which have their insertions on a single bone

408. Which of the following statements about the motor unit is not true?
A. Motor units in the coulometer muscles are the smallest in the body
B. The muscle fibers of a motor unit tend to be located next to each other
C. When the motor nerve fires an action potential, all muscle fibers in the motor unit contract
D. Smaller motor units tend to fire earliest as muscle tension is increased

409. When compared with oxidative slow muscle fibers, fast glycolytic fibers would have:
A. Glycogen as their most important energy source C. Smaller motor units
B. A higher myoglobin content D. A richer capillary blood supply

410. Which of the following types of motor unit would be the first to contract if a skeletal muscle were
given progressively more powerful stimuli?
A. Those containing slow oxidative fibers C. Those with the largest diameter fibers
B. The largest motor units D. Those with the greatest glycogen content

411. An exercise regimen which includes running and swimming long distances will promote an increase
in the:
A. Number of fast glycolytic fibers in skeletal muscle
B. Size of existing fast glycolytic fibers in skeletal muscle
C. Number of slow oxidative fibers in skeletal muscle
D. Mitochondrial content and blood supply of slow oxidative fibers in skeletal muscle

412. An exercise regimen which consisted of a daily 90 minute leisurely swim would be expected to have
all of the following effects EXCEPT:
A. An increased number of fast glycolytic fibers
B. An increased blood supply to muscles
C. A lowered resistance to blood flow
D. An increased concentration of myoglobin within muscle cells

413. The neurotransmitter at the neuromuscular junction of skeletal muscle:


A. Can be nor-epinephrine
B. Can generate an inhibitory or and excitatory post synaptic membrane
C. Binds to a receptor which activates adenylate cyclase in the post synaptic cell
D. Is hydrolyzed by an enzyme located on the post synaptic membrane

414. The neuromuscular junction of skeletal muscle has all of the following properties EXCEPT:
A. Has acetyl choline as its neurotransmitter
B. Has an enzyme in its post-synaptic membrane which degrades the neurotransmitter
141
Nerves & Muscles Physiology
C. The neurotransmitter causes receptor-operated Cl− channels to open in the post-synaptic membrane
D. The end plate potential is an excitatory post synaptic potential

415. Which of these statements about the neuromuscular junction in skeletal muscle is true?
A. The neurotransmitter could be norepinephrine
B. A single muscle cell can be supplied by a number of different motor neurons
C. The neurotransmitter causes opening of voltage gated ion channels on the motor end plate
D. The neurotransmitter can be hydrolyzed by an enzyme on the post-synaptic membrane

416. Binding of the neurotransmitter to the post-synaptic membrane at the neuromuscular junction of
skeletal muscle has which of the following effects?
A. Opening of voltage gated Na+ channels C. Opening of ligand gated cation channels
B. Opening of voltage gated K+ channels D. Opening of ligand gated Cl− channels

417. A cross section of a relaxed skeletal muscle made immediately adjacent to the Z line of the
sarcomere would be expected to show:
A. Thin filaments only C. Thin filaments and thick filaments
B. Thick filaments only D. Neither thin filaments nor thick filaments

418. Troponin has a binding site for all of the following substances EXCEPT:
A. Tropomyosin B. Actin C. ATP D. Calcium

419. The function of tropomyosin in skeletal muscle during contraction or relaxation is to:
A. Bind Ca++ during contraction
B. Bind Ca++ during relaxation
C. Block the cross bridge binding sites on actin
D. Conduct action potentials to the sarcoplasmic reticulum

420. The function of the transverse tubules in the skeletal muscle cell is to:
A. Conduct the action potential to the central part of the cell
B. Conduct nutrients to the central part of the cell
C. Store Ca++ during muscle relaxation
D. Form a docking site for synaptic vesicles at the neuromuscular junction

421. Which of the following proteins has the enzymatic ability to hydrolyze ATP?
A. Myosin B. Actin C. Troponin D. Tropomyosin

422. Which of the following proteins contains a binding site for Ca++?
A. Actin B. Myosin C. Troponin D. Tropomyosin

423. In the relaxed skeletal muscle the highest concentration of Ca++ is to be found:
A. In synaptic vesicles C. Bound to troponin
B. Within the sarcoplasmic reticulum D. In the cytoplasm of the myofibrils

424. The following events occur as part of the process of skeletal muscle contraction:
A. Depolarization of the transverse tubules
B. Ca++ release by the sarcoplasmic reticulum
C. Increased permeability of the muscle cell membrane to Na+ and K+
D. Binding of acetyl choline to the muscle cell membrane
142
Nerves & Muscles Physiology
These events occur in which of the following sequences?
A. A,D,B,C B. D,A,C,B C. D,C,A,B D. D,C,B,A

425. During skeletal muscle contraction and relaxation the function of ATP is to:
A. Energize the transport of Ca++ from the cytoplasm of the cell into sarcoplasmic reticulum
B. Separate myosin and actin by binding to the myosin cross bridge
C. Energize the myosin cross bridge when hydrolyzed
D. All of the above

426. Contraction of smooth muscle can be caused by a variety of stimuli. All of these stimulatory
processes involve:
A. Movement of Ca++ into the cytoplasm of the cell from extracellular fluid
B. Activation of the sympathetic nervous system
C. An extracellular chemical stimulus (neurotransmitter or hormone)
D. Activation of troponin

427. Contraction of smooth muscle involves all of the following processes EXCEPT:
A. Movement of Ca++ into the cytoplasm from sarcoplasmic reticulum and extracellular fluid
B. Binding of Ca++ to troponin
C. Phosphorylation of myosin
D. Interaction of myosin cross bridges with binding sites on actin

428. Which of the following properties is possessed by smooth muscle but not by skeletal muscle?
A. Contains both actin and myosin
B. Its contraction is triggered by an increase in cytoplasmic Ca++ concentration
C. The thick filament consists of myosin
D. Ca++ action is mediated by its binding to calmodulin

429. Contraction of smooth muscle can be caused by all of the following EXCEPT:
A. Stimulation by the sympathetic nervous system
B. Stimulation by the somatic nervous system
C. Stretch of the muscle
D. Contraction can be spontaneous (without outside stimulation)

430. The direct action of Ca++ during smooth muscle contraction is to:
A. Bind to calmodulin
B. Stimulate release of Ca++ from sarcoplasmic reticulum
C. Bind to troponin
D. Stimulate the activation of a G protein

431. The correct temporal sequence for events at the neuromuscular junction is
A. Action potential in the motor nerve; depolarization of the muscle end plate; uptake of Ca++ into the
presynaptic nerve terminal
B. Uptake of Ca++ into the presynaptic terminal; release of acetylcholine (ACh); depolarization of the muscle
end plate
C. Release of ach; action potential in the motor nerve; action potential in the muscle
D. Uptake of Ca++ into the motor end plate; action potential in the motor end plate; action potential in the
muscle
143
Nerves & Muscles Physiology
E. Release of ach; action potential in the muscle end plate; action potential in the muscle

432. Repeated stimulation of a skeletal muscle fiber causes a sustained contraction (tetanus).
Accumulation of which solute in intracellular fluid is responsible for the tetanus?
A. Na+ C. Cl– E. Ca++ G. Calmodulin
+ 2+
B. K D. Mg F. Troponin H. ATP

433. A 42-year-old man with myasthenia gravis notes increased muscle strength when he is treated with
an acetylcholinesterase (AChE) inhibitor. The basis for his improvement is increased
A. Amount of acetylcholine (ACh) released from motor nerves
B. Levels of ACh at the muscle end plates C. number of ACh receptors on the muscle end plates
D. Amount of norepinephrine released from motor nerves
E. Synthesis of norepinephrine in motor nerves

434. The velocity of conduction of action potentials along a nerve will be increased by
A. Stimulating the Na+ –K+ pump D. Myelinating the nerve
+ +
B. Inhibiting the Na –K pump E. Lengthening the nerve fiber
C. Decreasing the diameter of the nerve

435. A newly developed local anesthetic blocks Na+ channels in nerves. Which of the following effects on
the action potential would it be expected to produce?
A. Decrease the rate of rise of the upstroke of the action potential
B. Shorten the absolute refractory period
C. Abolish the hyperpolarizing after - potential
D. Increase the Na+ equilibrium potential
E. Decrease the Na+ equilibrium potential

436. At the muscle end plate, acetylcholine (ACh) causes the opening of
A. Na+ channels and depolarization toward the Na+ equilibrium potential
B. K+ channels and depolarization toward the K+ equilibrium potential
C. Ca++ channels and depolarization toward the Ca++ equilibrium potential
D. Na+ and K+ channels and depolarization to a value halfway between the Na+ and K+ equilibrium potentials
E. Na+ and K+ channels and hyperpolarization to a value halfway between the Na+ and K+ equilibrium
potentials

437. An inhibitory postsynaptic potential


A. Depolarizes the postsynaptic membrane by opening Na+ channels
B. Depolarizes the postsynaptic membrane by opening K+ channels
C. Hyperpolarizes the postsynaptic membrane by opening Ca++ channels
D. Hyperpolarizes the postsynaptic membrane by opening Cl– channels

438. Which of the following temporal sequences is correct for excitation– contraction coupling in skeletal
muscle?
A. Increased intracellular Ca++; action potential in the muscle membrane; cross-bridge formation
B. Action potential in the muscle membrane; depolarization of the T tubules; release of Ca++ from the
sarcoplasmic reticulum (SR)
C. Action potential in the muscle membrane; splitting of adenosine triphosphate (ATP); binding of Ca++ to
troponin C
D. Release of Ca++ from the SR; depolarization of the T tubules; binding of Ca++ to troponin C
144
Nerves & Muscles Physiology

439. In skeletal muscle, which of the following events occurs before depolarization of the T tubules in
the mechanism of excitation–contraction coupling?
A. Depolarization of the sarcolemmal membrane
B. Opening of Ca++ release channels on the sarcoplasmic reticulum (SR)
C. Uptake of Ca++ into the SR by Ca++-adenosine triphosphatase (ATPase)
D. Binding of Ca++ to troponin C
E. Binding of actin and myosin

440. Adenosine triphosphate (ATP) is used indirectly for which of the following processes?
A. Accumulation of Ca++ by the sarcoplasmic reticulum (SR)
B. Transport of Na+ from intracellular to extracellular fluid
C. Transport of K+ from extracellular to intracellular fluid
D. Transport of H+ from parietal cells into the lumen of the stomach
E. Absorption of glucose by intestinal epithelial cells

441. Which of the following causes rigor in skeletal muscle?


A. No action potentials in motoneurons D. An increase in ATP level
B. An increase in intracellular Ca++ level E. A decrease in ATP level
++
C. A decrease in intracellular Ca level

442. A 56-year-old woman with severe muscle weakness is hospitalized. The only abnormality in her
laboratory values is an elevated serum K+ concentration. The elevated serum K+ causes muscle weakness
because
A. the resting membrane potential is hyperpolarized
B. the K+ equilibrium potential is hyperpolarized
C. the Na+ equilibrium potential is hyperpolarized
D. K+ channels are closed by depolarization
E. K+ channels are opened by depolarization
F. Na+ channels are closed by depolarization
G. Na+ channels are opened by depolarization

443. The conduction velocity of the nerve fibers is increased by:


A. Decreased temperature
B. Increased concentration of the external sodium ions
C. Increased axon diameter
D. Myelination
E. Increased serum calcium ions

444. In the neuron, all the following are true EXCEPT:


A. The magnitude of the action potential is dependent on the strength of the stimulus
B. Impulses can travel in both directions
C. Depolarization is accompanied by increased permeability of the cell membrane to potassium ions
D. During depolarization, the potential of the neuron changes from -70mV to +40mV

445. Acetylcholine is a neurotransmitter at:


A. Sweat glands C. Parasympathetic ganglia
B. The adrenal medulla D. The parotid gland
145
Nerves & Muscles Physiology
E. The neuromuscular junction

446. The sequence of events in muscle contraction, all the following are true EXCEPT:
A. Action potential depolarize the T-tubules
B. Depolarization of T-tubules release calcium from sarcoplasmic reticulum
C. Calcium binds to the troponin-tropomyosin complex
D. Actin combines with myosin ATP leading to crossbridge activation
E. Calcium moves back into sarcoplasmic reticulum by passive transport

447. What activity should be taking place at the postsynaptic membrane, if its membrane potential
changes from – 65 mv to – 50 mv?
A. EPSP B. IPSP C. End plate potential D. Graded potentials

448. Electrical response to subminimal stimuli in the excitable tissue would produce all of the following
except:
A. Temporal summation B. Spatial summation C. Action potential D. Graded potentials

449. A nerve cell in which phase cannot respond to a new stimulus?


A. Resting potential B. Action potential C. Repolarization D. Refractory period

450. The release of neurotransmitter at a chemical synapse in the central nervous system is dependent
upon which of the following?
A. Synthesis of acetylcholinesterase
B. Hyperpolarization of the synaptic terminal
C. Opening of ligand-gated ion calcium channels
D. Influx of calcium into the presynaptic terminal

451. The excitatory or inhibitory action of a neurotransmitter is determined by which of the following?
A. The function of its postsynaptic receptor
B. Its molecular composition
C. The shape of the synaptic vesicle in which it is contained
D. The distance between the pre- and postsynaptic membranes

452. Which of the following is characteristic of the events occurring at an excitatory synapse?
A. There is a massive efflux of calcium from the presynaptic terminal
B. Synaptic vesicles bind to the postsynaptic membrane
C. Voltage-gated potassium channels are closed
D. Ligand-gated channels are opened to allow sodium entry into the postsynaptic neuron

453. Post-tetanic facilitation is thought to be the result of which of the following?


A. Opening of voltage-gated sodium channels
B. Opening of transmitter-gated potassium channels
C. A buildup of calcium in the presynaptic terminal
D. Electrotonic conduction

454. Which electrical event is characteristic of inhibitory synaptic interactions?


A. A neurotransmitter agent that selectively opens ligand-gated chloride channels is the basis for an
inhibitory postsynaptic potential
146
Nerves & Muscles Physiology
B. Because the Nernst potential for chloride is about −70 mV, chloride ions tend to move out of the cell
along its electrochemical gradient
C. A neurotransmitter that selectively opens potassium channels will allow potassium to move into the cell
D. An increase in the extracellular sodium concentration usually leads directly to an inhibitory postsynaptic
potential

455. Prolonged changes in neuronal activity are usually achieved through the activation of which of the
following?
A. Voltage-gated chloride channels
B. Transmitter-gated sodium channels
C. G-protein–coupled channels
D. Voltage-gated potassium channels

456. Which statement concerning synaptic transmission is correct?


A. When a specific population of synaptic terminals is spread over the considerable surface of a neuron,
their collective effects cannot spatially summate and lead to initiation of an action potential
B. Even if the successive discharges of an excitatory synapse occur sufficiently close in time, they cannot
temporally summate and initiate an action potential
C. A neuron is “facilitated” when its membrane potential is moved in the less negative or depolarizing
direction
D. Even when rapidly stimulated by excitatory synaptic input for a prolonged period, neurons typically do
not exhibit synaptic fatigue

457. Which statement concerning the generation of an action potential is correct?


A. When the membrane potential in the soma/axon hillock dips below “threshold,” an action potential is
initiated
B. The action potential is initiated in synaptic buttons
C. The least number of voltage-gated sodium channels in an axon is found near the node of Ranvier
D. Once an action potential is initiated, it will always run its course to completion

458. Which of the following types of cells work as scavenger cells in the CNS?
A. Microglia B. Oligodendroglia C. Ependymal cells D. Astrocytes

459. Which is the commonest type of synapse in the CNS?


A. Axosomatic synapse B. Axodendritic synapse C. Axoaxonal synapse

460. Excitatory postsynaptic potentials (EPSP) are not produced by:


A. Opening of Na channels C. Opening of calcium channels
B. Closure of K channels D. Opening of chloride channels

461. Inhibitory postsynaptic potentials (IPSP) may be produced by:


A. Closure of Na channels C. Opening of K channels
B. Closure of Ca channels D. Opening Cl channels

462. The minimum time for transmission across one synapse is:
A. 0.5 ms B. 1 ms C. 1.5 ms D. 2 ms

463. Inhibitory postsynaptic potentials are an example of:


A. Postsynaptic inhibition B. Presynaptic inhibition
147
Nerves & Muscles Physiology
C. Direct inhibition D. Indirect inhibition

464. Presynaptic inhibition occurs at:


A. Axoaxonal
B. Axosomatic
C. Axodendritic synapses

465. Which of the following is/are examples of ligand-gated ion channels?


A. Nicotinic Ach receptor C. IP3 receptor
B. GABA-activated Cl channels D. Glycine receptor

466. Which of the following statements about synaptic potentials is incorrect?


A. They are propagated down the postsynaptic neuron
B. They undergo spatiotemporal summation
C. They are analogous to generator potentials and end-plate potentials
D. IPSP hyperpolarize the postsynaptic neuron
E. They are proportional to the amount of transmitter released by the presynaptic neuron.

467. Most excitatory neurotransmission in the brain is mediated by:


A. Glutamate
B. Glycine
C. GABA
D. GnRH

468. The inhibitory amino acid neurotransmitters in the CNS are:


A. Glutamate and glycine
B. Glutamate and aspartate
C. GABA and glycine
D. Aspartate and glycine

469. The GABA-A receptor is a:


A. Na channel
B. Cl ion channel
C. Ca channel
D. Cation channel

470. IPSPs due to chloride influx are produced by:


A. acetylcholine
B. GABA
C. Glutamate
D. Substance P

471. Which of the following neurotransmitters has both excitatory and inhibitory effects?
A. Glycine
B. GABA
C. Aspartate
D. Glutamate
148
Nerves & Muscles Physiology
1 C 21 B 41 E 61 C 81 B 101 D 121 B

2 A 22 B 42 D 62 D 82 B 102 A 122 C

3 C 23 A 43 E 63 C 83 B 103 C 123 A

4 A 24 D 44 E 64 B 84 D 104 B 124 E

5 A 25 C 45 D 65 D 85 B 105 C 125 B

6 C 26 C 46 E 66 E 86 A 106 A 126 D

7 B 27 D 47 B 67 B 87 D 107 D 127 C

8 D 28 E 48 E 68 A 88 B 108 B 128 B

9 A 29 E 48 D 69 E 89 A 109 B 129 B

10 A 30 E 50 B 70 B 90 D 110 C 130 E

11 B 31 C 51 D 71 C 91 C 111 A 131 C

12 B 32 D 52 B 72 B 92 A 112 C 132 E

13 D 33 B 52 D 73 B 93 B 113 D 133 C

14 C 34 D 54 D 74 E 94 D 114 E 134 A

15 A 35 E 55 A 75 A 95 B 115 C 135 C

16 B 36 A 56 B 76 C 96 B 116 C 136 B

17 C 37 D 57 B 77 B 97 B 117 A 137 B

18 A 38 D 58 B 78 D 98 C 118 D 138 D

19 B 39 B 59 B 79 E 99 D 119 A 139 A

20 A 40 C 60 D 80 B 100 C 120 D 140 C


149
Nerves & Muscles Physiology
141 C 161 D 181 E 201 C 221 D 241 C 261 D

142 D 162 B 182 C 202 D 222 D 242 D 262 A

143 B 163 A 183 A 203 A 223 B 243 B 263 C

144 A 164 C 184 D 204 D 224 D 244 D 264 B

145 C 165 C 185 B 205 B 225 B 245 A 265 A

146 B 166 D 186 A 206 C 226 A 246 C 266 A

147 A 167 C 187 C 207 B 227 C 247 B 267 B

148 B 168 D 188 D 208 B 228 B 248 D 268 E

149 D 169 C 189 B 209 D 229 C 249 D 269 B

150 A 170 A 190 A 210 A 230 E 250 D 270 A

151 D 171 D 191 A 211 C 231 B 251 A 271 D

152 D 172 E 192 B 212 D 232 C 252 B 272 B

153 D 173 D 193 C 213 B 233 B 253 C 273 E

154 E 174 D 194 B 214 C 234 C 254 C 274 E

155 D 175 A 195 D 215 E 235 A 255 C 275 E

156 E 176 D 196 D 216 E 236 D 256 B 276 A

157 B 177 C 197 A 217 D 237 C 257 E 277 B

158 A 178 C 198 A 218 A 238 C 258 A 278 D

159 E 179 D 199 A 219 D 239 A 259 A 279 A

160 B 180 E 200 C 220 A 240 C 260 C 280 D


150
Nerves & Muscles Physiology
281 C 301 A 321 B 341 E 361 B 381 A 401 A

282 C 302 B 322 B 342 C 362 C 382 A 402 C

283 B 303 A 323 B 343 A 363 C 383 D 403 D

284 B 304 C 324 D 344 B 364 E 384 B 404 A

285 A 305 A 325 C 345 B 365 B 385 D 405 D

286 D 306 C 326 D 346 C 366 A 386 D 406 D

287 C 307 B 327 A 347 E 367 D 387 B 407 A

288 A 308 B 328 B 348 C 368 A 388 D 408 D

289 C 309 C 329 D 349 B 369 D 389 A 409 C

290 B 310 C 330 B 350 A 370 B 390 A 410 C

291 A 311 B 331 A 351 B 371 C 391 D 411 D

292 B 312 B 332 C 352 D 372 C 392 A 412 A

293 C 313 D 333 A 353 C 373 B 393 A 413 D

294 B 314 B 334 E 354 E 374 C 394 A 414 C

295 C 315 A 335 B 355 D 375 C 395 A 415 D

296 A 316 B 336 B 356 A 376 B 396 C 416 A

297 D 317 D 337 E 357 C 377 C 397 D 417 A

298 B 318 A 338 C 358 D 378 D 398 A 418 C

299 B 319 C 339 D 359 C 379 B 399 E 419 C

300 B 320 D 340 A 360 D 380 B 400 D 420 A


151
Nerves & Muscles Physiology
421 A 441 E 461 All 481 501 521 541

422 C 442 F 462 A 482 502 522 542


B, C,
423 B 443 463 A, C 483 503 523 543
D
424 C 444 A 464 A 484 504 524 544

425 D 445 All 465 All 485 505 525 545

426 A 446 E 466 A 486 506 526 546

427 B 447 A 467 A 487 507 527 547

428 D 448 C 468 C 488 508 528 548

429 B 449 D 469 B 489 509 529 549

430 A 450 D 470 B 490 510 530 550

431 B 451 A 471 A 491 511 531 551

432 E 452 D 472 492 512 532 552

433 B 453 C 473 493 513 533 553

434 D 454 A 474 494 514 534 554

435 A 455 C 475 495 515 535 555

436 D 456 C 476 496 516 536 556

437 D 457 D 477 497 517 537 557

438 B 458 A 478 498 518 538 558

439 A 459 B 479 499 519 539 559

440 E 460 D 480 500 520 540 560


209
Respiratory System Physiology
1. Which of the following is a function of the nasopharynx?
A. Filter dust
B. Warm and moisten inspired air
C. Provide openings for the right and left Eustachian tubes
D. Contain olfactory receptors responsible for the sense of smell

2. Which structure is responsible for directing food and liquids into the esophagus during swallowing?
A. Glottis B. Epiglottis C. Adam’s apple D. Conchae

3. What purpose do the cartilaginous rings around the trachea serve?


A. Keep the trachea from collapsing during inhalation
B. Attach the trachea firmly to the esophagus
C. Protect the trachea from trauma
D. They serve no purpose

4. Inhaled food or foreign objects are most likely to lodge in which part of the respiratory system?
A. Pharynx B. Right bronchus C. Left bronchus D. Bronchioles

5. What is the purpose of surfactant?


A. Facilitate in the diffusion of oxygen across the respiratory membrane
B. Transport oxygen to the alveoli
C. Purify the air entering the respiratory tract
D. Keep alveoli from collapsing

6. What is one of the purposes of the fluid in the pleural cavity?


A. Lubricate the pleural surfaces to allow them to glide painlessly during lung expansion and contraction
B. Warm and moisten lung tissue
C. Prevent bacteria from entering lung tissue
D. Assist in the diffusion of oxygen across the respiratory membrane

7. The main muscle responsible for pulmonary ventilation is:


A. The abdominals C. The diaphragm
B. The external intercostals D. The internal intercostals

8. Which gas is the primary regulator of respiration?


A. Oxygen B. Carbon dioxide C. Nitrogen D. Bicarbonate

9. When pressure in the lungs drops lower than atmospheric pressure, what occurs?
A. Air flows out of the lungs
B. Air flows into the lungs
C. A pneumothorax forms, collapsing the lungs
D. The bronchioles constrict, causing respiratory distress

10. The primary way oxygen is transported in the blood is:


A. In the form of bicarbonate C. In the form of oxyhemoglobin
B. In the form of carbaminohemoglobin D. Dissolved in plasma

11. Which of the following lung volumes or capacities can be measured by spirometry?
A. Functional residual capacity FRC. C. Residual volume RV.
B. Physiologic dead space D. Total lung capacity TLC.
210
Respiratory System Physiology
E. Vital capacity VC.

12. An infant born prematurely in gestational week 25 has neonatal respiratory distress syndrome.
Which of the following would be expected in this infant?
A. Arterial PO2 of 100 mm Hg D. Normal breathing rate
B. Collapse of the small alveoli E. Lecithin/sphingomyelin ratio of greater than
C. Increased lung compliance 2:1 in amniotic fluid

13. In which vascular bed does hypoxia cause vasoconstriction?


A. Coronary B. Pulmonary C. Cerebral D. Muscle E. Skin

Questions 14 and 15
A 12−year−old boy has a severe asthmatic attack with wheezing. He experiences rapid breathing and
becomes cyanotic. His arterial PO2 is 60 mm Hg and his PCO2 is 30 mm Hg.

14. Which of the following statements about this patient is most likely to be true?
A. Forced expiratory volume/forced vital capacity (FEV1/FVC) is increased
B. Ventilation/perfusion (V/Q) ratio is increased in the affected areas of his lungs
C. His arterial PCO2 is higher than normal because of inadequate gas exchange
D. His arterial PCO2 is lower than normal because hypoxemia is causing him to hyperventilate
E. His residual volume (RV) is decreased

15. To treat this patient, the physician should administer


A. an α1−adrenergic antagonist D. A muscarinic agonist
B. A β1−adrenergic antagonist E. A nicotinic agonist
C. A β2−adrenergic agonist

16. Which of the following is true during inspiration?


A. Intrapleural pressure is positive
B. The volume in the lungs is less than the functional residual capacity (FRC)
C. Alveolar pressure equals atmospheric pressure
D. Alveolar pressure is higher than atmospheric pressure
E. Intrapleural pressure is more negative than it is during expiration

17. Which volume remains in the lungs after a tidal volume (TV) is expired?
A. Tidal volume (TV) E. Functional residual capacity (FRC)
B. Vital capacity (VC) F. Inspiratory capacity
C. Expiratory reserve volume (ERV) G. Total lung capacity
D. Residual volume (RV)

18. A 35−year−old man has a vital capacity (VC) of 5 L, a tidal volume (TV) of 0.5 L, an inspiratory capacity
of 3.5 L, and a functional residual capacity (FRC) of 2.5 L. What is his expiratory reserve volume (ERV)?
A. 4.5 L B. 3.9 L C. 3.6 L D. 3.0 L E. 2.5 L F. 2.0 L G. 1.5 L

19. When a person is standing, blood flow in the lungs is


A. Equal at the apex and the base
B. Highest at the apex owing to the effects of gravity on arterial pressure
C. Highest at the base because that is where the difference between arterial and venous pressure is
greatest
D. Lowest at the base because that is where alveolar pressure is greater than arterial pressure
211
Respiratory System Physiology

20. Which of the following is illustrated in the graph showing


volume versus pressure in the lung–chest wall system?
A. The slope of each of the curves is resistance
B. The compliance of the lungs alone is less than the
compliance of the lungs plus chest wall
C. The compliance of the chest wall alone is less than the
compliance of the lungs plus chest wall
D. When airway pressure is zero atmospheric. The volume of
the combined system is the functional residual capacity (FRC)
E. When airway pressure is zero atmospheric. Intrapleural
pressure is zero

21. Which of the following is the site of highest airway


resistance?
A. Trachea B. Largest bronchi C. Medium−sized bronchi D. Smallest bronchi E. Alveoli

22. A 49−year−old man has a pulmonary embolism that completely blocks blood flow to his left lung. As
a result, which of the following will occur?
A. Ventilation/perfusion (V/Q) ratio in the left lung will be zero
B. Systemic arterial PO2 will be elevated
C. V/Q ratio in the left lung will be lower than in the right lung
D. Alveolar PO2 in the left lung will be approximately equal to the PO2 in inspired air
E. Alveolar PO2 in the right lung will be approximately equal to the PO2 in venous blood

Questions 23 and 24

23. In the hemoglobin–O2 dissociation curves shown, the shift from curve A to curve B could be caused
by
A. increased pH D. HbF
B. decreased 2,3−DPG concentration E. CO poisoning
C. strenuous exercise

24. The shift from curve A to curve B is associated with


A. increased P50 D. increased O2−carrying capacity of hemoglobin
B. increased affinity of hemoglobin for O2 E. decreased O2−carrying capacity of hemoglobin
C. impaired ability to unload O2 in the tissues

25. Which volume remains in the lungs after a maximal expiration?


A. Tidal volume (TV) C. Expiratory reserve volume (ERV)
B. Vital capacity (VC) D. Residual volume (RV)
212
Respiratory System Physiology
E. Functional residual capacity (FRC) G. Total lung capacity
F. Inspiratory capacity

26. Compared with the systemic circulation, the pulmonary circulation has a
A. higher blood flow D. higher capillary pressure
B. lower resistance E. higher cardiac output
C. higher arterial pressure

27. A healthy 65−year−old man with a tidal volume (TV) of 0.45 L has a breathing frequency of 16
breaths/min. His arterial PCO2 is 41 mm Hg, and the PCO2 of his expired air is 35 mm Hg. What is his
alveolar ventilation?
A. 0.066 L/min B. 0.38 L/min C. 5.0 L/min D. 6.14 L/min E. 8.25 L/min

28. Compared with the apex of the lung, the base of the lung has
A. a higher pulmonary capillary PO2 C. a higher ventilation/perfusion (V/Q) ratio
B. a higher pulmonary capillary PCO2 D. the same V/Q ratio

29. Hypoxemia produces hyperventilation by a direct effect on the


A. phrenic nerve D. medullary chemoreceptors
B. J receptors E. carotid and aortic body chemoreceptors
C. lung stretch receptors

30. Which of the following changes occurs during strenuous exercise?


A. Ventilation rate and O2 consumption increase to the same extent
B. Systemic arterial PO2 decreases to about 70 mm Hg
C. Systemic arterial PCO2 increases to about 60 mm Hg
D. Systemic venous PCO2 decreases to about 20 mm Hg
E. Pulmonary blood flow decreases at the expense of systemic blood flow

31. If an area of the lung is not ventilated because of bronchial obstruction, the pulmonary capillary
blood serving that area will have a PO2 that is
A. equal to atmospheric PO2 D. higher than inspired PO2
B. equal to mixed venous PO2 E. lower than mixed venous PO2
C. equal to normal systemic arterial PO2

32. In the transport of CO2 from the tissues to the lungs, which of the following occurs in venous blood?
A. Conversion of CO2 and H2O to H+ and HCO3– in the red blood cells RBCs
B. Buffering of H+ by oxyhemoglobin
C. Shifting of HCO3– into the RBCs from plasma in exchange for Cl–
D. Binding of HCO3– to hemoglobin
E. Alkalinization of the RBCs

33. Which of the following causes of hypoxia is characterized by a decreased arterial PO2 and an
increased difference between alveolar PO2 and arterial PO2?
A. Hypoventilation D. Carbon monoxide poisoning
B. Right−to−left cardiac shunt E. Ascent to high altitude
C. Anemia
213
Respiratory System Physiology
34. A 42−year−old woman with severe pulmonary fibrosis is evaluated by her physician and has the
following arterial blood gases: pH = 7.48, PO2 = 55 mm Hg, and PCO2 = 32 mm Hg. Which statement best
explains the observed value of PCO2?
A. The increased pH stimulates breathing via peripheral chemoreceptors
B. The increased pH stimulates breathing via central chemoreceptors
C. The decreased PCO2 inhibits breathing via peripheral chemoreceptors
D. The decreased PCO2 stimulates breathing via peripheral chemoreceptors
E. The decreased PCO2 stimulates breathing via central chemoreceptors

35. A 38−year−old woman moves with her family from New York City sea level to Leadville Colorado
(10,200 feet above sea level). Which of the following will occur as a result of residing at high altitude?
A. Hypoventilation E. Pulmonary vasodilation
B. Arterial PO2 greater than 100 mm Hg F. Hypertrophy of the left ventricle
C. Decreased 2,3−DPG concentration G. Respiratory acidosis
D. Right shift of the Hb–O2 dissociation curve

36. The pH of venous blood is only slightly more acidic than the pH of arterial blood because
A. CO2 is a weak base
B. There is no carbonic anhydrase in venous blood
C. The H+ generated from CO2 and H2O is buffered by HCO3– in venous blood
D. The H+ generated from CO2 and H2O is buffered by deoxyhemoglobin in venous blood
E. Oxyhemoglobin is a better buffer for H+ than is deoxyhemoglobin

37. In a maximal expiration, the total volume expired is


A. Tidal volume (TV) E. Functional residual capacity (FRC)
B. Vital capacity (VC) F. Inspiratory capacity
C. Expiratory reserve volume (ERV) G. Total lung capacity
D. Residual volume (RV)

38. A person with a ventilation/perfusion V/Q. defect has hypoxemia and is treated with supplemental
O2. The supplemental O2 will be most helpful if the person’s predominant V/Q defect is
A. Dead space B. Shunt C. High V/Q D. Low V/Q E. V/Q = 0

39. Which person would be expected to have the largest difference between alveolar PO2 and arterial
PO2?
A. Person with pulmonary fibrosis
B. Person who is hypoventilating due to morphine overdose
C. Person at 12,000 feet above sea level
D. Person with normal lungs breathing 50% O2
E. Person with normal lungs breathing 100% O2

40. Stability to alveoli is given by all of the following EXCEPT:


A. Intrapleural pressure B. Lung surfactant C. Surface tension D. Lung elastin

41. Airway resistance is increased by all of the following EXCEPT:


A. Lung Surfactant B. Histamine C. Prostaglandin D. Leucotriene

42. Chloride shift shows all of the following EXCEPT:


A. Occurs in the lung capillaries C. Rises chloride level in the RBC
B. Occurs to maintain ionic equilibrium D. Necessary for CO2 transport
214
Respiratory System Physiology

43. At what level of alveolar oxygen pressure the peripheral chemoreceptors are activated? (Pressure in
mmHg)
A. 150 B. 100 C. 90 D. 60

44. The affinity of hemoglobin for oxygen is increased if there is an increase in:
A. pH B. temperature C. 2,3 DPG D. PCO2

45. Thickening of alveolar capillary membrane causes:


A. Emphysema B. Pulmonary edema C. Hypoxemia D. Atelectasis

46. During inspiration, all of the following show an increase EXCEPT:


A. Intrapleural pressure B. Stroke volume C. Venous return D. Intrapulmonary pressure

47. The stimulation of ventilation during exercise is mainly caused by:


A. Hypoxia B. Hypercapnia C. Proprioceptive afferents D. Acidosis

48. If the Intrapleural pressure becomes atmospheric, the changes in the lungs would include:
A. Increased compliance C. Decreased air way resistance
B. Decreased work of breathing D. Collapse of alveoli

49. Rise in capillary hydrostatic pressure at the base of lungs can result in:
A. Pulmonary embolism B. Pulmonary edema C. Dyspnea D. B and C

50. Which of the following relaxes bronchial muscle?


A. Acetylcholine B. Leucotrienes C. Histamine D. Epinephrine

51. Voluntary hyperpnea can lead to:


A. Respiratory acidosis B. Respiratory alkalosis C. Metabolic acidosis D. Metabolic alkalosis

52. The volume of air that is present in the lungs after a forceful expiration is called:
A. Tidal volume C. Expiratory reserve volume
B. Residual volume D. Functional residual capacity

53. Automatic respiration stops after:


A. Bilateral vagotomy C. Transection below medulla
B. Transection above pons D. Lesion of pneumotaxic center

54. Alveolar surface tension is caused by:


A. Air in the alveoli B. Fluid in the alveoli C. Air fluid interface D. Lung surfactant

55. Physiological dead space is increased in all of the following EXCEPT:


A. Venous arterial shunt B. Emphysema C. Hyperpnea D. Pulmonary edema

56. In chronic obstructive pulmonary disease an increase in which of the following cannot occur?
A. Work of breathing B. Air way resistance C. Vital capacity D. Residual volume of lungs

57. Maximum voluntary ventilation is reduced in:


A. Old age B. Obstructive lung disease C. Restrictive lung disease D. All of the above
215
Respiratory System Physiology
58. In which of the following the residual volume of lungs is included?
A. Tidal volume B. Vital capacity C. Functional residual capacity D. Inspiratory capacity

59. Pulmonary edema mostly occurs at the base of the lungs because of:
A. Less PO2 at the base
B. More PCO2 at the apex
C. Increased capillary hydrostatic pressure at the base
D. Increased vascular resistance at the base

60. A decrease in arterial blood oxygen content could occur by:


A. An increase in 2, 3 DPG B. Rise in pH C. Decrease in PCO2 D. Decrease in temperature

61. 100% oxygen administration is useful in all of the following EXCEPT:


A. Loss of cabin pressure in airplane C. Pulmonary edema
B. Hypoventilation D. Venous arterial shunt

62. Respiratory acidosis occurs in:


A. Pneumonia B. Diarrhea C. High altitude D. Hyperpnea

63. In obstructive pulmonary disease there is a high:


A. Forced expiratory flow rate C. Functional residual capacity
B. Ventilation perfusion ratio D. Vital capacity

64. During the inspiration, the vertical dimension of the thoracic cavity is increased due to contraction
of:
A. Diaphragm D. Serritus anterior
B. External intercostals E. Sternocleidomastoid
C. Scalene

65. Surfactant deficiency is not likely to produce:


A. Collapse of alveoli
B. Increased surface tension of the fluid lining alveoli
C. Increased compliance of the lungs
D. Pulmonary edema
E. Respiratory distress syndrome in newborns

66. The difference of total lung capacity and vital capacity is:
A. Expiratory reserve volume D. Residual volume
B. Functional residual capacity E. Tidal volume
C. Inspiratory reserve volume

67. The rate of diffusion of gases through the respiratory membrane is inversely proportional to:
A. Diffusion capacity of the respiratory membrane for the gas
B. Pressure gradient across the respiratory membrane
C. Surface area of the respiratory membrane
D. Solubility of the gas
E. Thickness of the respiratory membrane

68. Dissociation of oxygen from oxyhemoglobin facilitated by:


A. Decreased temperature B. Decreased 2:3 DPG concentration
216
Respiratory System Physiology
C. Decreased H+ concentration E. Exercise
D. Decreased PCO2

69. The rate and duration of inspiratory ramp signals from the dorsal respiratory medullary neurons is
controlled by impulses from:
A. Cerebral cortex D. Pneumotaxic center
B. Hypothalamus E. Ventral respiratory neurons
C. Peripheral chemoreceptors

70. A patient having hypoxia shows his arterial PO2 – 60 mmHg and hemoglobin concentration 15 g/dl.
He is most likely suffering from:
A. Anemic hypoxia D. Hypoxic hypoxia
B. Carbon monoxide poisoning E. Stagnant hypoxia
C. Histologic hypoxia

71. During the acclimatization at high altitude, the change not likely to occur is:
A. Accelerated erythropoiesis
B. Increased hemoglobin concentration
C. Increased rate of pulmonary ventilation
D. Increased concentration of 2:3 diphosphoglycerate
E. Respiratory acidosis

72. In a person breathing normally at rest with an environmental temperature of 25°C, regarding the
partial pressure, all the following are true EXCEPT
A. Of CO2 in alveolar air is about 40 mm Hg
B. Of water vapor in alveolar air is less than half the alveolar PCO2 level
C. Of water vapor in alveolar air is greater than that in room air even at 100 per cent humidity
D. Of O2 in expired air is greater than in alveolar air
E. Of CO2 in mixed venous blood is greater than in alveolar air

73. As blood passes through systemic capillaries, all the following are true EXCEPT
A. pH falls
B. HCO3− ions pass from red cells to plasma
C. Cl− ion concentration in red cells falls
D. Its oxygen dissociation curve shifts to the right
E. Its ability to deliver oxygen to the tissues is enhanced

74. Regarding the respiratory center, all the following are true EXCEPT
A. Is in the medulla oblongata
B. Sends impulses to inspiratory muscles during quiet breathing
C. Sends impulses to expiratory muscles during quiet breathing
D. Is involved in the swallowing reflex
E. Is involved in the vomiting reflex

75. Regarding the carotid bodies, all the following are true EXCEPT
A. Are stretch receptors in the walls of the internal carotid arteries
B. Have the greatest flow rate/unit volume in the body
C. Are influenced more by blood PO2 than by its oxygen content
D. Generate more afferent impulses when blood H+ ion concentration rises
E. And the aortic bodies are mainly responsible for the increased ventilation in hypoxia
217
Respiratory System Physiology

76. Pulmonary surfactant increases


A. The surface tension of the fluid lining alveolar walls
B. Lung compliance
C. In effectiveness as the lungs are inflated
D. In amount when pulmonary blood flow is interrupted

77. As people age, there is usually a decrease in the following EXCEPT


A. Ratio of lung residual volume to vital capacity
B. Percentage of vital capacity expelled in one second
D. Lung elasticity
E. Resting arterial blood PO2

78. During inspiration


A. Intrapleural pressure is lowest at mid−inspiration
B. Intrapulmonary pressure is lowest around mid−inspiration
C. Intraesophageal pressure is lowest at mid−inspiration
D. The rate of air flow is greatest at end−inspiration
E. The lung volume/intrapleural pressure relationship is the same as in expiration

79. Carbon dioxide


A. Is carried as carboxyhemoglobin on the hemoglobin molecule
B. Uptake by the blood increases its oxygen−binding power
C. Uptake by the blood leads to similar increases in Hand HCO3− ion concentrations
D. Stimulates ventilation when breathed at a concentration of 20 per cent
E. Content is greater than oxygen content in arterial blood

80. Regarding the normal lungs, all the following are true EXCEPT
A. The rate of alveolar ventilation at rest exceeds the rate of alveolar capillary perfusion
B. The ventilation/perfusion (V/P) ratio exceeds 1.0 during maximal exercise
C. The V/P ratio is higher at the apex than at the base of the lungs when a person is standing
D. Oxygen transfer can be explained by passive diffusion
E. Dead space increases during inspiration

81. Alveolar ventilation is increased by breathing


A. 21 per cent O2 and 79 per cent N2
B. 17 per cent O2 and 83 per cent N2
C. 2 per cent CO2 and 98 per cent O2
D. 10 per cent CO2 and 90 per cent O2
E. A gas mixture which raises arterial PCO2 by 10 per cent

82. Bronchial smooth muscle contracts in response to the following EXCEPT


A. Bronchial mucosal irritation C. Inhalation of cold air
B. A fall in bronchial PCO2 D. Circulating noradrenaline

83. In early inspiration there is a fall in the following EXCEPT


A. Intrapulmonary pressure D. Dead space PO2
B. Intrathoracic pressure E. Pressure in the superior vena cava
C. Intra−abdominal pressure
218
Respiratory System Physiology
84. Compliance of the lungs is greater in the following EXCEPT
A. When they are expanded above their normal tidal volume range
B. In adults than in infants
C. Than the compliance of the lungs and thorax together
D. When they are filled with normal saline than when they are filled with air
E. In standing than in recumbent subjects

85. At a high altitude where atmospheric pressure is halved, there is an increase in


A. Pulmonary ventilation D. Arterial pH
B. Alveolar H2O vapor pressure E. Cerebral blood flow
C. Arterial PO2

86. During inspiration, all the following are true EXCEPT


A. Venous return to the heart is increased
B. More energy is expended than during expiration
C. Lung expansion is assisted by surface tension forces in the alveoli
D. Lung expansion begins when intrapleural pressure falls below atmospheric
E. Lung expansion ends when intrapulmonary pressure falls to atmospheric

87. The residual volume is


A. The gas remaining in the lungs at the end of a full expiration
B. Greater on average in men than in women
C. 3–4 liters on average in young adults
D. Measured directly using a spirometer
E. Smaller in old than in young people

88. A rise in arterial PCO2 leads to an increase in all the following EXCEPT
A. Ventilation due to stimulation of peripheral baroreceptors
B. Ventilation due to stimulation of central chemoreceptors
C. Arterial pressure
D. Cerebral blood flow
E. The plasma bicarbonate level

89. Ventilation is increased during


A. Periods when cerebrospinal fluid pH is reduced
B. Chronic renal failure
C. Periods when plasma bicarbonate level is raised
D. Deep sleep
E. Exercise because of the ensuing fall in arterial PO2

90. Voluntarily hyperventilation increases the


A. Negative charge on the plasma proteins
B. Level of ionized calcium in blood
C. Alveolar PO2 three−fold when ventilation is increased three−fold
D. Arterial blood oxygen saturation by 10–15 per cent when ventilation is increased by 10–15 per cent
E. The renal excretion of bicarbonate

91. If the carotid and aortic chemoreceptors are denervated


A. Increasing alveolar PCO2 by 25 per cent fails to stimulate ventilation
B. Halving the alveolar PO2 fails to stimulate ventilation
219
Respiratory System Physiology
C. The resting ventilation rate is depressed by more than 40 per cent
D. Ventilation does not increase during exercise
E. The ability to adapt to life at high altitude is impaired

92. Pulmonary
A. Arterial mean pressure is about one−sixth systemic mean arterial pressure
B. Blood flow/minute is similar to systemic blood flow/minute
C. Vascular resistance is about 50 per cent that of systemic vascular resistance
D. Vascular capacity is similar to systemic vascular capacity
E. Arterial pressure increases by about 50 per cent when cardiac output rises by 50 per cent

93. Carbon dioxide is carried in the blood in the following forms EXCEPT
A. Combination with the hemoglobin molecule
B. Combination with plasma proteins
C. Physical solution in plasma
D. Greater quantity in red blood cells than in plasma
E. Greater quantity as HCO3− ions than as other forms

94. A shift of the oxygen dissociation curve of blood to the right EXCEPT
A. Occurs in the pulmonary capillaries
B. Occurs if blood temperature rises
C. Favors oxygen delivery to the tissues
D. Favors oxygen uptake from the lungs by alveolar capillary blood
E. Increases the P

95. The work of breathing increases in all the following EXCEPT


A. When lung compliance increases
B. When the subject exercises
C. When the rate of breathing increases even though the minute volume stays constant
D. When the subject lies down
E. When functional residual capacity increases

96. The compliance of the lungs and chest wall is


A. Expressed as volume change per unit change in pressure
B. Minimal during quiet breathing
C. Increased by the surface tension of the fluid lining the alveoli
D. Increased by surfactant
E. Changed by parallel displacement of the line relating lung volume to distending pressure

97. Respiratory dead space


A. Saturates inspired air with water vapor before it reaches the alveoli
B. Removes all particles from inspired air before it reaches the alveoli
C. Decreases when blood catecholamine levels rise
D. Decreases during a deep inspiration
E. Decreases during a cough

98. Regarding vital capacity, all are true EXCEPT


A. It is the volume of air expired from full inspiration to full expiration
B. It is reduced as one grows older
C. It is greater in men than in women of the same age and height
220
Respiratory System Physiology
D. It is related more to total body mass than to lean body mass
E. It is the sum of the inspiratory and expiratory reserve volumes

99. In pulmonary capillary blood


A. Carbonic anhydrase in erythrocytes catalyzes the formation of Hand HCO3−.
B. Hydrogen ions dissociate from hemoglobin
C. The rise in PO2 is of greater magnitude than the fall in PCO2
D. The oxygen content is linearly related to alveolar PO2
E. The pH is lower than in blood in the pulmonary artery

100. Regarding Oxygen debt, all the following are true, EXCEPT
A. It is the amount of O2 consumed after cessation of exercise
B. It is incurred because the pulmonary capillary walls limit O2 uptake during exercise
C. It is possible since skeletal muscle can function temporarily without oxygen
D. It is associated with a rise in blood lactate
E. It is associated with metabolic acidosis

101. The CO2 dissociation curve for whole blood shows that
A. Its shape is sigmoid
B. Blood saturates with CO2 when PCO2 exceeds normal alveolar levels
C. Blood contains some CO2 even when the PCO2 is zero
D. Oxygenation of the blood drives CO2 out of the blood
E. Adding CO2 to the blood drives O2 out of the blood

102. The oxygen content of mixed venous blood can be all the following EXCEPT
A. Measured using blood sampled from the right atrium
B. Increased during generalized muscular exercise
C. Increased in a warm environment
D. Increased in cyanide poisoning
E. Decreased in circulatory failure

103. Bronchial asthma is likely to be relieved by all the following EXCEPT


A. Stimulation of cholinergic receptors
B. Stimulation of beta adrenoceptors
C. Histamine aerosols
D. Drugs which stabilize mast cell membranes
E. Glucocorticoids

104. Regarding air in the pleural cavity (pneumothorax), all the following are true EXCEPT
A. Allows intrapleural pressure to rise to atmospheric pressure
B. Causes the underlying lung to collapse by compressing it
C. Increases the functional residual capacity
D. Leads to a slight outward movement of the chest wall
E. Reduces vital capacity

105. What tends to decrease airway resistance?


A. Asthma C. Treatment with acetylcholine
B. Stimulation by sympathetic fibers D. Exhalation to residual volume
221
Respiratory System Physiology
106. The pleural pressure of a normal 56−year−old woman is approximately −4 cm H2O during resting
conditions immediately before inspiration (i.e., at functional residual capacity, FRC). What is the pleural
pressure (in cm H2O) during inspiration?
A. +1 B. +4 C. 0 D. −3 E. −6

107. A healthy, 25−year−old medical student participates in a 10−kilometer charity run for the American
Heart Association. Which muscles does the student use (contract) during expiration?
A. Diaphragm and external intercostals D. Internal intercostals and abdominal recti
B. Diaphragm and internal intercostals E. Scaleni
C. Diaphragm only F. Sternocleidomastoid muscles

108. Which of the following would be expected to increase the measured airway resistance?
A. Stimulation of parasympathetic nerves to the lungs
B. Low lung volumes D. Forced expirations
C. Release of histamine by mast cells E. All of the above

109. Several students are trying to see who can generate the highest expiratory flow. Which muscle is
most effective at producing a maximal effort?
A. Diaphragm D. Rectus abdominis
B. Internal intercostals E. Sternocleidomastoid
C. External intercostals

110. The figure on the right shows three different compliance curves (S, T, and U) for isolated lungs
subjected to various transpulmonary pressures. Which of the following best describes the relative
compliances for the three curves?

A. S < T < U B. S < T > U C. S − T − U D. S > T < U E. S > T > U

Direction: Use the figure below to answer the following two questions:

111. Assuming a respiratory rate of 12 breaths/min, calculate the minute ventilation.


A. 1 L/min B. 2 L/min C. 4 L/min D. 5 L/min E. 6 L/min
222
Respiratory System Physiology

112. A 22−year−old woman inhales as much air as possible and exhales as much air as she can, producing
the spirogram shown in the figure above. A residual volume of 1.0 liter was determined using the helium
dilution technique. What is her FRC (in liters)?
A. 2.0 B. 2.5 C. 3.0 D. 3.5 E. 4.0 F. 5.0

113. With a slow decrease in left heart function, which of the following will minimize the formation of
pulmonary edema?
A. An increase in plasma protein concentration due to fluid loss
B. Increase in the negative interstitial hydrostatic pressure
C. Increased pumping of lymphatics
D. Increase in the concentration of interstitial proteins

114. A 22−year−old woman has a pulmonary compliance of 0.2 L/cm H2O and a pleural pressure of −4 cm
H2O. What is the pleural pressure (in cm H2O) when the woman inhales 1.0 liter of air?
A. −6 B. −7 C. −8 D. −9 E. −10

115. A preterm infant has a surfactant deficiency. Without surfactant, many of the alveoli collapse at the
end of each expiration, which in turn leads to pulmonary failure. Which set of changes is present in the
preterm infant compared with a normal infant?
Alveolar Surface Tension Pulmonary Compliance
A Decreased Decreased
B Decreased Increased
C Decreased No change
D Increased Decreased
E Increased Increased
F Increased No change
G No change No change

116. A patient has a dead space of 150 ml, FRC of 3 liters, tidal volume (VT) of 650 ml, expiratory reserve
volume (ERV) of 1.5 liters, total lung capacity (TLC) of 8 liters, and respiratory rate of 15 breaths/min.
What is the residual volume (RV)?
A. 500 ml B. 1000 ml C. 1500 ml D. 2500 ml E. 6500 ml

117. A patient has a dead space of 150 milliliters, FRC of 3 liters, VT of 650 milliliters, ERV of 1.5 liters,
TLC of 8 liters, and respiratory rate of 15 breaths/min. What is the alveolar ventilation (VA)?
A. 5 L/min B. 7.5 L/min C. 6.0 L/min D. 9.0 L/min

118. The various lung volumes and capacities include the total lung capacity (TLC), vital capacity (VC),
inspiratory capacity (IC), tidal volume (VT), expiratory capacity (EC), expiratory reserve volume (ERV),
inspiratory reserve volume (IRV), functional residual capacity (FRC), and residual volume (RV). Which of
the following lung volumes and capacities can be measured using direct spirometry without additional
methods?
TLC VC IC VT EC ERV IRV FRC RV
A No No Yes No Yes No Yes No No
B No Yes Yes Yes Yes Yes Yes No No
C No Yes Yes Yes Yes Yes Yes Yes No
D Yes Yes Yes Yes Yes Yes Yes No Yes
E Yes Yes Yes Yes Yes Yes Yes Yes Yes
223
Respiratory System Physiology

119. What happens during exercise?


A. Blood flow is uniform throughout the lung
B. Lung−diffusing capacity increases because blood flow is continuous in all pulmonary capillaries
C. Pulmonary blood volume decreases
D. The transit time of blood in the pulmonary capillaries does not change from rest

120. A 34−year−old man sustains a bullet wound to the chest that causes a pneumothorax. What best
describes the changes in lung volume and thoracic volume in this man compared with normal?
Lung Volume Thoracic Volume
A Decreased Decreased
B Decreased Increased
C Decreased No change
D Increased Decreased
E Increased Increased
F No change Decreased

121. A healthy 10−year−old boy breathes quietly under resting conditions. His tidal volume is 400
milliliters and his ventilation frequency is 12/min. Which of the following best describes the ventilation
of the upper, middle, and lower lung zones in this boy?

122. An experiment is conducted in two persons (subjects T and V) with identical tidal volume (VTs) of
1000 milliliters, dead space volumes (200 milliliters), and ventilation frequencies of 20 breaths per
minute. Subject T doubles his VT and reduces his ventilation frequency by 50%. Subject V doubles his
ventilation frequency and reduces his VT by 50%. What best describes the total ventilation (also called
minute ventilation) and alveolar ventilation (VA) of subjects T and V?

123. A person with normal lungs has an oxygen (O2) consumption of 750 ml O2/min. The hemoglobin
(HB) concentration is 15 g/dl. The mixed venous saturation is 25%. What is the cardiac output?
A. 2500 ml/min C. 7500 ml/min E. 20,000 ml/min
B. 5000 ml/min D. 10,000 ml/min

124. A cardiac catheterization is performed in a healthy adult. The blood sample withdrawn from the
catheter shows 60% O2 saturation, and the pressure recording shows oscillations from a maximum of 27
mm Hg to a minimum of 12 mm Hg. Where was the catheter tip located?
A. Ductus arteriosus B. Foramen ovale C. Left atrium
224
Respiratory System Physiology
D. Pulmonary artery E. Right atrium

125. If alveolar surface area is decreased 50% and pulmonary edema leads to a doubling of diffusion
distance, how does diffusion of O2 compare with normal?
A. 25% increase C. 25% decrease E. 75% decrease
B. 50% increase D. 50% decrease

126. Which of the following sets of differences best describes the hemodynamics of the pulmonary
circulation when compared with the system circulation?

127. A 67−year−old man is admitted emergently to the hospital because of severe chest pain. A
Swan−Ganz catheter is floated into the pulmonary artery, the balloon is inflated, and the pulmonary
wedge pressure is measured. The pulmonary wedge pressure is used clinically to monitor which
pressure?
A. Left atrial pressure D. Pulmonary artery systolic pressure
B. Left ventricular pressure E. Pulmonary capillary pressure
C. Pulmonary artery diastolic pressure

128. Which diagram in the figure below best illustrates the pulmonary vasculature when the cardiac
output has increased to a maximum extent?

A. A B. B C. C D. D E. E

129. A human experiment is being performed in which forearm blood flow is being measured under a
variety of conditions. The forearm is infused with a vasodilator, resulting in an increase in blood flow.
Which of the following occurs?
A. Tissue interstitial partial pressure of oxygen (PO2) will increase
B. Tissue interstitial partial pressure of carbon dioxide (PCO2) will increase
C. Tissue pH will decrease

130. Blood gas measurements are obtained in a resting patient who is breathing room air. The patient
has an arterial content of 19 ml O2/dl with a PO2 of 95. The mixed venous O2 content is 4 ml O2/dl blood.
Which condition does the patient have?
A. An increase in physiological dead space C. A low Hb concentration
B. Pulmonary edema D. A low cardiac output
225
Respiratory System Physiology
131. A normal male subject has the following initial conditions (in the steady state):
Arterial PO2 = 92 mm Hg Cardiac output = 5600 ml/min
Arterial O2 saturation = 97% O2 consumption = 256 ml/min
Venous O2 saturation = 20% Hb concentration = 12 gm/dl
Venous PO2 = 30 mm Hg
If you ignore the contribution of dissolved O2 to the O2 content, what is the venous O2 content?
A. 2.2 ml O2/100 ml blood E. 6.2 ml O2/100 ml blood
B. 3.2 ml O2/100 ml blood F. 10.8 ml O2/100 ml blood
C. 4 ml O2/100 ml blood G. 16 ml O2/100 ml blood
D. 4.6 ml O2/100 ml blood

132. A man fell asleep in his running car. He was unconscious when he was brought into the emergency
department. With carbon monoxide (CO) poisoning, you would expect his alveolar O2 partial pressure
(PCO2) would be _______, while his arterial O2 content (CaO2) would be ______.
A. Normal, decreased C. Increased, normal
B. Decreased, decreased D. Increased, normal

133. A 30−year−old woman performs a Valsalva maneuver about 30 minutes after eating lunch. Which
option best describes the changes in pulmonary and systemic blood volumes that occur in this woman?

134. A child who is eating round candies approximately 1 and 1.5 cm in diameter inhales one down his
airway, blocking his left bronchiole. Which of the following describes the changes that occur?

135. The forces governing the diffusion of a gas through a biological membrane include the pressure
difference across the membrane (ΔP), the cross−sectional area of the membrane (A), the solubility of the
gas (S), the distance of diffusion (D), and the molecular weight of the gas (MW). Which changes increase
the diffusion of a gas through a biological membrane?
226
Respiratory System Physiology

136. A person’s normal VT is 400 milliliters with a dead space of 100 milliliters. The respiratory rate is 12
breaths/min. The person undergoes ventilation during surgery, and the VT is 700 with a rate of 12. What
is the approximate alveolar PCO2 for this person?
A. 10 B. 20 C. 30 D. 40 E. 45

137. Arterial PO2 is 100 mm Hg and arterial PCO2 is 40 mm Hg. Total blood flow to a muscle is 700
ml/min. There is a sympathetic activation resulting in a decrease in blood flow of this muscle to 350
ml/min. There is no neuromuscular activation, and thus no contraction of the muscle. Which of the
following will occur?

138. A 45−year−old man at sea level has an inspired O2 tension of 149 mm Hg, nitrogen tension of 563
mm Hg, and water vapor pressure of 47 mm Hg. A small tumor pushes against a pulmonary blood vessel,
completely blocking the blood flow to a small group of alveoli. What are the O2 and carbon dioxide (CO2)
tensions of the alveoli that are not perfused (in mm Hg)?

139. In which conditions is alveolar PO2 increased and alveolar PCO2 decreased? (Va = Alveolar
ventilation)?
A. Increased Va and unchanged metabolism C. Increased metabolism and unchanged Va
B. Decreased Va and unchanged metabolism D. Proportional increase in metabolism and Va

140. The diffusing capacity of a gas is the volume of gas that will diffuse through a membrane each
minute for a pressure difference of 1 mm Hg. Which gas is often used to estimate the O2−diffusing
capacity of the lungs?
A. CO2 B. CO C. Cyanide gas D. Nitrogen E. O2
227
Respiratory System Physiology
141. The O2−CO2 diagram below shows a ventilation perfusion (V/Q) ratio line for the normal lung.
Which of the following best describes the effect of decreasing V/Q ratio on the alveolar PO2 and PCO2?

142. A 23−year−old medical student has mixed venous O2 and CO2 tensions of 40 mm Hg and 45 mm Hg,
respectively. A group of alveoli are not ventilated in this student because mucus blocks a local airway.
What are the alveolar O2 and CO2 tensions distal to the mucus block (in mm Hg)?

Questions 41 and 42

143. A 67−year−old man has a solid tumor that pushes against an airway, partially obstructing air flow to
the distal alveoli. Which point on the V/Q line of the O2−CO2 diagram below corresponds to the alveolar
gas of these distal alveoli?

A. A B. B C. C D. D E. E
228
Respiratory System Physiology
144. A 55−year−old man has a pulmonary embolism that completely blocks the blood flow to his right
lung. Which point on the V/Q line of the O2−CO2 diagram above corresponds to the alveolar gas of his
right lung?
A. A B. B C. C D. D E. E

145. The figure below shows two lung units (S and T) with their blood supplies. Lung unit S has an ideal
relationship between blood flow and ventilation. Lung unit T has a compromised blood flow. What is the
relationship between total dead space (TDS), physiologic dead space (PDS) and anatomic dead space
(ADS) for these lung units?

Lung unit S Lung unit T


A TDS < ADS TDS = ADS
B TDS = PDS TDS > PDS
C TDS = ADS TDS < ADS
D TDS = ADS TDS > ADS
E TDS > ADS TDS < ADS

146. A 32−year−old medical student has a fourfold increase in cardiac output during strenuous exercise.
Which curve on the figure below most likely represents the changes in O2 tension that occur as blood
flows from the arterial end to the venous end of the pulmonary capillaries in this student?

A. A B. B C. C D. D E. E

147. The figure below shows changes in the partial pressures of O2 and CO2 as blood flows from the
arterial (Art) end to the venous (Ven) end of the pulmonary capillaries. Which diagram best depicts the
normal relationship between PO2 (red line) and PCO2 (green line) during resting conditions?
229
Respiratory System Physiology

A. A B. B C. C D. D E. E

148. Which of the following would be true if the blood lacked red blood cells and just had plasma and
the lungs were functioning normally?
A. The arterial PO2 would be normal
B. The O2 content of arterial blood would be normal
C. Both A and B
D. Neither A nor B

149. The figure below shows a normal O2−Hb dissociation curve. What are the approximate values of Hb
saturation (% Hb−O2), PO2, and O2 content for oxygenated blood leaving the lungs and reduced blood
returning to the lungs from the tissues?

150. A person with anemia has a Hb concentration of 12 g/dl. He starts exercising and uses 12 ml O2/dl.
What is the mixed venous PO2?
A. 0 mm Hg C. 20 mm Hg E. 100 mm Hg
B. 10 mm Hg D. 40 mm Hg

151. Which points on the figure below represent arterial blood in a severely anemic person?
230
Respiratory System Physiology

152. A stroke that destroys the respiratory area of the medulla would be expected to lead to which of
the following?
A. Immediate cessation of breathing
B. Apneustic breathing
C. Ataxic breathing
D. Rapid breathing (hyperpnea)
E. None of the above (breathing would remain normal)

153. Which of the below O2−Hb dissociation curves corresponds to normal blood (red solid line) and
blood containing CO (green dotted line)?

A. A B. B C. C D. D E. E F. F
231
Respiratory System Physiology
154. Using the figure of the previous question, which of the above O2−Hb dissociation curves
corresponds to blood during resting conditions (red solid line) and blood during exercise (green dotted
line)?
A. A B. B C. C D. D E. E F. F

155. Using the figure of the previous question, which of the above O2−Hb dissociation curves
corresponds to blood from an adult (red solid line) and blood from a fetus (green dotted line)?
A. A B. B C. C D. D E. E F. F

156. Arterial PO2 is 100 mm Hg and arterial PCO2 is 40 mm Hg. Total blood flow to all muscle is 700
ml/min. There is a sympathetic activation resulting in a decrease in blood flow to 350 ml/min. What will
occur?

157. What is the most important pathway for the respiratory response to systemic arterial CO2 (PCO2)?
A. CO2 activation of the carotid bodies
B. Hydrogen ion (H+) activation of the carotid bodies
C. CO2 activation of the chemosensitive area of the medulla
D. H+ activation of the chemosensitive area of the medulla
E. CO2 activation of receptors in the lungs

158. The basic rhythm of respiration is generated by neurons located in the medulla. What limits the
duration of inspiration and increases respiratory rate?
A. Apneustic center D. Pneumotaxic center
B. Dorsal respiratory group E. Ventral respiratory group
C. Nucleus of the tractus solitarius

159. When the respiratory drive for increased pulmonary ventilation becomes greater than normal, a
special set of respiratory neurons that are inactive during normal quiet breathing then becomes active,
contributing to the respiratory drive. These neurons are located in which structure?
A. Apneustic center D. Pneumotaxic center
B. Dorsal respiratory group E. Ventral respiratory group
C. Nucleus of the tractus solitarius

160. A 26−year−old medical student on a normal diet has a respiratory exchange ratio of 0.8. How much
O2 and CO2 are transported between the lungs and tissues of this student (in ml gas/100 ml blood?
232
Respiratory System Physiology

161. CO2 is transported from the tissues to the lungs predominantly in the form of bicarbonate ion.
Compared with arterial red blood cells, which of the following options best describes venous red blood
cells?

162. The afferent (sensory) endings for the Hering−Breuer reflex are mechanoreceptors located in the
A. Carotid arteries C. External intercostals E. Diaphragm
B. Alveoli D. Bronchi and bronchioles

163. An anesthetized man is breathing with no assistance. He then undergoes artificial ventilation for 10
minutes at his normal VT but at twice his normal frequency. He undergoes ventilation with a gas mixture
of 60% O2 and 40% nitrogen. The artificial ventilation is stopped and he fails to breathe for several
minutes. This apneic episode is due to which of the following?
A. High arterial PO2 suppressing the activity of the peripheral chemoreceptors
B. Decrease in arterial pH suppressing the activity of the peripheral chemoreceptors
C. Low arterial PCO2 suppressing the activity of the medullary chemoreceptors
D. High arterial PCO2 suppressing the activity of the medullary chemoreceptors
E. Low arterial PCO2 suppressing the activity of the peripheral chemoreceptors

164. Which of the following describes a patient with constricted lungs compared with a normal patient?

165. Which diagram in the figure below best describes the relationship between alveolar ventilation (Va)
and arterial CO2 tension (PCO2) when the PCO2 is changed acutely over a range of 35 to 75 mm Hg?
233
Respiratory System Physiology

A. A B. B C. C D. D E. E F. F

166. Which diagram in the figure below best describes the relationship between alveolar ventilation (Va)
and arterial O2 tension (PO2) when the PO2 is changed acutely over a range of 0 to 160 mm Hg and the
arterial PCO2 and H+ concentration remain normal?

A. A B. B C. C D. D E. E F. F

167. At a party, a 17−year−old male places a paper bag over his mouth and breathes in and out of the
bag. As he continues to breathe into this bag, his rate of breathing continues to increase. Which of the
following is responsible for the increased ventilation?
A. Increased alveolar PO2 B. Increased alveolar PCO2 C. Decreased arterial PCO2 D. Increased pH

168. Alveolar ventilation (Va) increases severalfold during strenuous exercise. Which factor is most likely
to stimulate ventilation during strenuous exercise?
A. Collateral impulses from higher brain centers D. Decreased mean venous PO2
B. Decreased mean arterial pH E. Increased mean arterial PCO2
C. Decreased mean arterial PO2
234
Respiratory System Physiology

169. During strenuous exercise, O2 consumption and CO2 formation can increase as much as 20−fold.
Alveolar ventilation (Va) increases almost exactly in step with the increase in O2 consumption. Which
option best describes what happens to the mean arterial O2 tension (PO2), CO2 tension (PCO2), and pH in
a healthy athlete during strenuous exercise?

170. A 54−year−old woman with advanced emphysema due to long−term cigarette smoking is admitted
to the hospital for shortness of breath. She is diagnosed with pulmonary hypertension. Her arterial blood
gases are
PO2 = 75 mm Hg
PCO2 = 45 mm Hg
pH = 7.35
What is the cause of the pulmonary hypertension in this woman?
A. Increased alveolar PCO2 C. Decreased alveolar PO2
B. Increased sympathetic tone D. Decreased pulmonary capillary number

171. Cheyne−Stokes breathing is an abnormal breathing pattern characterized by a gradual increase in


the depth of breathing, followed by a progressive decrease in the depth of breathing that occurs again
and again approximately every minute. Which time points on the figure below (V−Z) are associated with
the highest PCO2 of lung blood and highest PCO2 of the neurons in the respiratory center?

172. A 45−year−old man inhaled as much air as possible and then expired with a maximum effort until
no more air could be expired. This action produced the maximum expiratory flow−volume (MEF−V) curve
shown in the figure below. What is the forced vital capacity (FVC) of this man (in liters)?
235
Respiratory System Physiology

A. 1.5 B. 2.5 C. 3.5 D. 4.5 E. 5.5 F. 6.5

173. The maximum expiratory flow−volume (MEF−V) curve shown in the figure below is used as a
diagnostic tool for identifying obstructive and restrictive lung diseases. At which point on the curve does
airway collapse limit maximum expiratory air flow?

A. A B. B C. C D. D E. E

174. The maximum expiratory flow−volume (MEF−V) curves shown in the figure below were obtained
from a healthy person (red solid curve) and a 57−year−old man with shortness of breath (green dotted
curve). The man with shortness of breath likely has which disorder?
236
Respiratory System Physiology

A. Asbestosis B. Emphysema C. Kyphosis D. Scoliosis E. Silicosis F. Tuberculosis

175. A 62−year−old man reports difficulty breathing. The figure below shows an MEFV (maximum
expiratory flow−volume curve from the patient (green solid line) and from a typical healthy individual
(red dotted curve). Which of the following best explains the MEFV curve of the patient?

A. Asbestosis B. Asthma C. Bronchospasm D. Emphysema E. Old age

176. The MEFV (maximum expiratory flow−volume curve) shown in the figure below (red solid line) was
obtained from a 75−year−old man who smoked 40 cigarettes per day for 60 years. The green doted
flow−volume curve was obtained from the man during resting conditions. Which set of changes is most
likely to apply to this man?
237
Respiratory System Physiology

177. The figure below shows a forced expiration for a healthy person (curve X) and a person with a
pulmonary disease (curve Z). What is the forced expiratory volume in the first second of expiration
(FEV1)/FVC ratio (as a percent) in these persons?

178. The figure below shows forced expirations from a person with healthy lungs (curve X) and from a
patient (curve Z). The patient most likely has which condition?
238
Respiratory System Physiology

A. Asthma B. Bronchospasm C. Emphysema D. Old age E. Silicosis

179. Which of the following describes blood gases during consolidated pneumonia where the lung is
filled with fluid and cellular debris?

180. Which of the following occurs during atelectasis of one lung?


A. Increase in arterial PCO2
B. A 40% decrease in PO2
C. Normal blood flow in the lung with atelectasis
D. Slight decrease in arterial O2 content

181. The volume–pressure curves in the figure below were obtained from a normal subject and a patient
with a pulmonary disease. Which abnormality is most likely present in the patient?
239
Respiratory System Physiology

A. Asbestosis C. Mitral obstruction E. Silicosis


B. Emphysema D. Rheumatic heart disease F. Tuberculosis

182. A 34−year−old medical student generates the flow−volume curves shown in the figure below. Curve
W is a normal maximum expiratory flow-volume (MEFV) curve generated when the student was healthy.
Which of the following best explains curve X?

A. Asthma attack C. Heavy exercise F. Pneumonia


B. Aspiration of meat into the D. Light exercise G. Tuberculosis
trachea E. Normal breathing at rest

183. Which of the following best describes comparison of the lung compliance and surfactant levels in a
premature infant with respiratory distress syndrome versus a normal full−term infant?
240
Respiratory System Physiology
184. Compared with a normal healthy person, how do TLC and maximum expiratory flow (MEF) change
with restrictive lung disease?

185. A 78−year−old man who smoked 60 cigarettes per day for 55 years reports shortness of breath. The
patient is diagnosed with chronic pulmonary emphysema. Which set of changes is present in this man
compared with a healthy nonsmoker?

186. While breathing room air, a patient with chronic obstructive pulmonary disease, has a systemic
arterial PCO2 of 65 mm Hg and a PO2 of 40 mm Hg. Supplemental oxygen is administered at a 40%
fractional concentration of oxygen in inspired gas (FiO2), which resulted in an increase of PO2 to 55 mm
Hg and PCO2 to 70 mm Hg. Which of the following describes the supplemental O2?
A. Restored arterial dissolved O2 to normal
B. Did not change breathing
C. Reduced the hypoxic stimulation of breathing
D. Increased the pulmonary excretion of CO2

187. Which of the following describes diffusing capacity of O2 in the lung?


A. Does not change during exercise
B. Is greater than diffusing capacity for CO2
C. Is greater in residents at sea level than in residents at 3000 meters altitude
D. Is directly related to alveolar capillary surface area

188. When he was in his early 40s, a 75−year−old man worked for 5 years in a factory where asbestos
was used as an insulator. The man is diagnosed with asbestosis. Which set of changes is present in this
man compared with a person with healthy lungs?

189. The amount of air present in the lungs at the end of a tidal breath is:
241
Respiratory System Physiology
A. Residual volume B. Expiratory reserve volume C. Functional residual capacity

190. The largest lung volume is:


A. Residual volume C. Expiratory residual volume
B. Inspiratory reserve volume D. Tidal volume

191. By spirometry, one cannot determine:


A. Vital capacity C. Residual volume
B. Functional residual capacity D. Total lung capacity

192. Functional residual capacity can be determined using:


A. Spirometer C. Helium dilution technique
B. Computerized spirometry D. Whole body plethysmography

193. A person breathes into and from a spirometer (volume 12 liters) containing 10% helium gas
mixture. After equilibration, helium concentration of expired gas was found to be 6.7%. His vital capacity
is 4.2 liters. What is his residual volume?
A. 1000 ml B. 1200 ml C. 1500 ml D. 1800 ml

194. The maximum amount of gas that can be exhaled after a full inspiration is called:
A. Expiratory reserve volume C. Total lung capacity
B. Vital capacity D. Functional residual capacity

195. Residual volume is normally about ____% of total lung capacity.


A. 10 B. 20 C. 30 D. 40

196. The largest volume of gas that can be moved into and out of the lungs in 1 minute by voluntary
effort is called:
A. Respiratory minute volume C. Maximal voluntary ventilation
B. Minute ventilation D. Vital capacity

197. The normal compliance of the human lungs and chest wall is:
A. 0.1 L/cm H2O B. 0.2 L/cm H2O C. 0.3 L/cm H2O D. 0.4 L/cm H2O

198. The major constituent of surfactant is:


A. Neutral lipid C. Phosphatidylglycerol
B. Dipalmitoyl phosphatidyl choline D. Surfactant protein A & D

199. Intrapleural pressure is positive during:


A. Deep inspiration C. Forced expiration
B. Tidal expiration D. Tidal inspiration

200. Which of the following produce bronchodilation?


A. VIP C. Leukotriene C4
B. Epinephrine D. PAF (platelet activating factor)

201. Bronchodilation is mediated by:


A. α1 receptors C. M2 receptors
B. α2 receptors D. β2 adrenoceptors
242
Respiratory System Physiology
202. Bronchial tone is highest at about:
A. 6 AM B. 10 AM C. 2 PM D. 6 PM

203. Which of the following produces bronchoconstriction?


A. VIP
B. PAF (platelet activating factor) D. Epinephrine
C. LTB4 (leukotrienes 4) E. Nitric oxide

204. Which of the following is least likely to produce bronchoconstriction?


A. NO B. Sulfur dioxide C. Cool air D. Exercise

205. The FEV1/FVC ratio is normally greater than:


A. 0.8 B. 0.7 C. 0.9 D. 0.5

206. Peak expiratory flow rate (PEFR), an index of airways resistance, is reduced in:
A. Restrictive lung disease B. Bronchial asthma

207. The most sensitive index of small airways resistance in a patient with bronchial asthma is:
A. vital capacity
B. FEV1
C. FEV1/FVC
D. PEFR
E. Maximal Mid-Expiratory Flow Rate (Forced Expiratory Flow 25%–75%)

208. In chronic obstructive lung disease:


A. FRC increases C. VC decreases
B. TLC increases D. RV increases

209. Which of the following statements is correct?


A. Normally, ventilation is greater at the apex than at the lung bases
B. In the upright position, lower lung zones are more compliant compared to upper lung zones
C. In the upright position, blood flow is greater at the lung bases than at the apices
D. Positive intrapleural pressures during forced expiration prevent airways from closing

210. Anatomic dead space is determined using:


A. Whole body plethysmography C. Single-breath nitrogen curve
B. Spirometry D. Single-breath carbon monoxide method

211. Under basal conditions, the amount of oxygen consumed per minute is about:
A. 100 ml B. 250 ml C. 350 ml D. 500 ml

212. Under basal conditions, the respiratory quotient (i.e., the volume of CO2 produced / volume of
oxygen consumed) is about:
A. 1 B. 0.8 C. 0.7 D. 1.2

213. Under basal conditions, respiratory exchange ratio is normally:


A. 0.7 B. 0.8 C. 0.9 D. 1.0

214. Alveolar oxygen tension (PO2) is influenced by:


A. Barometric pressure
243
Respiratory System Physiology
B. Fraction of oxygen in inspired air (FiO2)
C. PCO2
D. Respiratory exchange ratio

215. At rest, the normal diffusing capacity of the lungs for carbon monoxide is about:
A. 25 ml/min/mm Hg C. 75 ml/min/mm Hg
B. 50 ml/min/mm Hg D. 100 ml/min/mm Hg

216. The mean systemic arterial pressure and the mean pulmonary artery pressure are respectively
90 and 15 mm Hg. Can you tell the ratio of systemic and pulmonary vascular resistances?
A. 1 B. 10 C. 6 D. Data inadequate

217. Ventilation-perfusion ratio (whole lung) is normally:


A. 1 B. 0.8 C. 0.6 D. 1.2

218. In the upright position, V/Q ratio is highest in the:


A. Lung apices B. Middle zone C. Lung bases

219. In the upright position, the V/Q ratio at the lung apex is about:
A. 1 B. 0.7 C. 0.5 D. 3

220. The major buffer of carbon dioxide in blood is:


A. Bicarbonate B. Albumin C. Hemoglobin

221. If blood Hb concentration is 15 g/dL, how much oxygen would be transported in blood?
A. Approximately 20 ml/dL B. Approximately 15 ml/dL C. Data inadequate

222. If Hb were completely absent, how much plasma would be required to transport oxygen required
for basal metabolism?
A. 10 liters B. 27 liters C. 52 liters D. 84 liters

223. Which of the following shifts the oxyhemoglobin dissociation curve to the right?
A. CO2 C. Increase in tissue temperature
B. 2, 3-BPG D. Decrease in tissue pH

224. Suppose you administer 100% oxygen at 4 atmospheres, then how much oxygen would be
transported in 100 ml of plasma?
A. 0.3 ml B. 3 ml C. 6 ml D. 9 ml

225. The decrease in oxygen affinity of Hb when the pH of blood falls is:
A. Bohr effect C. Hawthorne effect
B. Haldane effect D. Hamburger effect

226. Which of the following shifts the oxyhemoglobin dissociation curve to the left?
A. CO C. Increase in tissue temperature
B. Increase in 2,3 DPG in RBC D. Decrease in tissue pH

227. PO2 may be normal in hypoxia due to:


A. Hypoventilation C. Severe circulatory shock E. Cyanide poisoning
B. Diffusion limitation D. Anemia
244
Respiratory System Physiology

228. Administration of O2 rich gas mixtures improves tissue oxygenation most in:
A. Hypoxic hypoxia C. Hypokinetic hypoxia
B. Anemic hypoxia D. Histotoxic hypoxia

229. CO2 is chiefly transported in blood:


A. As bicarbonate C. As dissolved CO2
B. Bound to hemoglobin D. In combination with plasma proteins

230. The major stimulus for spontaneous respiration is:


A. CO2 B. O2 C. Lactate D. HCO3−

231. Central chemoreceptors are chiefly stimulated by:


A. An increase in CSF H+ concentration C. A decrease in PCO2
B. A fall in PO2 D. All of the above

232. Systemic arterial chemoreceptors (commonly called peripheral chemoreceptors) are mainly
stimulated by:
A. A decline in PO2 C. A decrease in blood flow through them
+
B. An increase in H concentration of arterial D. An increase in PCO2
plasma

233. Forced hyperventilation to exhaustion may result in:


A. A decrease in serum ionized calcium C. An increase in blood pH
B. An increase in plasma protein level

234. Hypocapnia is usually due to:


A. Hypoxemia C. Decreased CO2 production
B. Alkalosis D. Alveolar hyperventilation

235. Normally, PCO2 chiefly depends on:


A. Total lung capacity C. Alveolar ventilation
B. PO2 D. Rate of CO2 production

236. J receptors are located in:


A. Pulmonary interstitium C. Bronchial mucosa
B. Alveoli D. Roof of the fourth ventricle

237. Stimulation of J receptors results in:


A. Apnea C. Hyperpnea
B. Tachypnea D. Apnea followed by tachypnea

238. Which of the following statements regarding Cheyne-Stokes respiration are correct?
A. It is characterized by periods of hyperpnea punctuated by apnea
B. It is attributed to increased sensitivity of the respiratory center to CO2
C. It may occur if lung-to-brain circulation time is prolonged
D. It occurs in severe congestive heart failure

239. During mouth-to-mouth resuscitation, what is the oxygen concentration of the resuscitating gas
mixture?
245
Respiratory System Physiology
A. 16% B. 21% C. 28% D. 35%

240. Hyperventilating allows one to hold one’s breath for a longer period of time, because
hyperventilation
A. Increases the oxygen reserve of systemic arterial blood
B. Decreases the PCO2 of systemic arterial blood
C. Decreases the pH of systemic arterial blood
D. Increases brain blood flow
E. None of the above, since hyperventilation reduces the time one can voluntarily stop breathing

241. The carotid bodies:


A. Have a low blood flow per gram of tissue
B. Contain baroreceptors
C. Respond to changes in pH
D. Respond to small changes in PO2
E. Are located on the external carotid arteries

242. If oxygen is added to inspired air to increase its partial pressure from 150 mm Hg to 450 mm Hg:
A. Dissolved oxygen will increase approximately three-fold
B. The oxygen content of the blood will increase approximately three-fold
C. The PN2 will remain the same
D. The PO2 will increase approximately one-fold
E. Hypercarbia will be prevented

243. Pulmonary vascular resistance is increased by the following EXCEPT:


A. Serotonin C. Hypoxia E. Adrenaline
B. Hypocarbia D. A fall in pH

244. The following cause a decrease in the arterial partial pressure of oxygen:
A. Anemia D. A rise in physiological dead-space
B. Carbon monoxide E. Old age
C. Hyperventilation

245. In normal human lungs:


A. A low PO2 produces pulmonary vasodilatation
B. Beta-2 agonists cause bronchoconstriction
C. Pulmonary vascular resistance is increased by serotonin
D. Pulmonary vascular resistance is decreased by histamine
E. Pulmonary vascular resistance is decreased by noradrenaline

246. Regarding the surfactant, all the following are true, EXCEPT :
A. Is a mucopolypeptide
B. Causes a decrease in surface tension
D. Causes an increase in compliance
E. Production is reduced after a prolonged reduction in pulmonary blood flow

247. A pressure-volume curve can be used for measuring:


A. The work of breathing D. Compliance
B. Functional residual capacity E. Respiratory quotient
C. Anatomical dead space
246
Respiratory System Physiology

248. Vital capacity:


A. Is the volume of air expired from full inspiration to full expiration
B. Increases gradually with age in adults
C. Is greater in men than in women of similar age and height
D. Is equal to the sum of the inspiratory and expiratory reserve volumes
E. May be measured by spirometry

249. Hyperventilation in a normal subject for 24 hours will produce a:


A. Fall in PCO2 D. Fall in cerebrospinal fluid bicarbonate
B. Rise in PCO2 E. Rise in plasma bicarbonate
C. Rise in ionized calcium

250. The P5O is:


A. The oxygen saturation when the arterial partial pressure of oxygen is 50 mm Hg
B. The arterial oxygen tension when hemoglobin is 50% saturated
C. An indicator of the position of the oxygen dissociation curve
D. Raised in fetal blood
E. Lowered in chronic anemia

251. An increase in the 2.3-DPG concentration in red blood cells occurs in the following EXCEPT:
A. Anemia C. Stored blood E. Cyanotic heart disease
B. Acclimatization to altitude D. Trained athletes

252. On ascending to an altitude of 6000 m, changes include all the following EXCEPT:
A. An increase in minute volume D. A fall in arterial PO2
B. An initial increase in plasma pH E. An increase in cerebral blood flow
C. A rise in urine pH

253. Regarding chemoreceptors in the arterial system, all the following EXCEPT:
A. Have a higher rate of oxygen consumption per gram than brain tissue
B. Respond to changes in oxygen tension and not content
C. Respond to changes in pH
D. Conduct afferent information via the glossopharyngeal and vagus nerves
E. Are found in the carotid sinus

254. Regarding surfactant, all the following EXCEPT:


A. Contains phospholipids
B. Prevents edema formation in the alveolar wall
D. Produces a monomolecular layer
E. Stabilizes the size of an alveolus

255. The oxyhemoglobin dissociation curve is shifted to the left by:


A. An increase in arterial PCO2 D. Carbon monoxide
B. Acidosis E. A fall in temperature
C. Chronic anemia

256. In a healthy individual breathing spontaneously, which of the following pressures is positive with
reference to atmospheric pressure?
A. Alveolar pressure during inspiration B. Alveolar pressure during expiration
247
Respiratory System Physiology
C. Intrapleural pressure during inspiration D. Intrapleural pressure during expiration

257. Which muscle does not contract during forced expiration?


A. Internal intercostals C. Transversus abdominus
B. External intercostals D. Rectus abdominis

258. In a healthy individual with a total lung volume of 6 liters, the amount of oxygen present in the
lungs at the end of a tidal expiration is about:
A. 100 ml B. 210 ml C. 400 ml D. 1000 ml E. 2300 ml

259. A person breathes into and from a spirometer (volume 12 liters) containing 10% helium gas
mixture. After equilibration, helium concentration of expired gas was found to be 6.7%. His vital
capacity is 4.2 liters. What is his residual volume?
A. 1000 ml B. 1200 ml C. 1500 ml D. 1800 ml

260. For a respiratory minute volume of 6 liters, which of the following combinations of breathing rate
and tidal volumes allows for maximum alveolar ventilation in a healthy individual?
A. 10 breaths per minute; and 600 ml C. 20 breaths per minute; and 300 ml
B. 15 breaths per minute; and 400 ml D. 30 breaths per minute; and 200 ml

261. During a tidal inspiration, air flow velocity is least in:


A. Nasopharynx B. Trachea C. Major bronchi D. Bronchioles

262. If the PO2 of blood in the pulmonary capillaries is 100 mm Hg and it is 98 mm Hg in the left ventricle,
which of the following is most likely to account for this?
A. Some bronchial venous blood drains into pulmonary veins
B. Some return from the coronary veins occurs into the chambers of the left side of the heart
C. Pulmonary arteriovenous anastomoses
D. Patent foramen ovale

263. In the adult, which of the following is most different between the systemic and pulmonary
circulations?
A. Volume of blood flowing through it C. Capillary hydrostatic pressure
B. Stroke volume D. Oncotic pressure

264. Intrapleural pressure is normally negative because:


A. Intrapulmonary pressure is always negative
B. Chest wall and lungs recoil in opposite directions at functional residual capacity
C. Surfactant prevents lung collapse
D. Transpulmonary pressure determines the negativity

265. Which of the following is true at functional residual capacity (relaxation volume)?
A. Intrapulmonary pressure = atmospheric pressure
B. Intrapleural pressure > atmospheric pressure
C. Intrapulmonary pressure = intrapleural pressure
D. Transrespiratory pressure is positive

266. Intrapleural pressure is positive during:


A. Deep inspiration B. Tidal expiration C. Forced expiration D. Tidal inspiration
248
Respiratory System Physiology
267. Lung compliance is increased in:
A. Pulmonary emphysema C. Surfactant deficiency
B. Pulmonary fibrosis D. Acute pulmonary edema

268. In advanced emphysema:


A. VC is decreased B. RV is increased C. FRC is increased D. TLC is increased

269. Breathing in individuals with restrictive lung disease is typically:


A. Slow and deep B. Slow and shallow C. Rapid and deep D. Rapid and shallow

270. Pulmonary vascular resistance is lowest when lung volume is:


A. Total lung capacity B. Residual volume C. Functional residual capacity

271. Inhaled particles of which size generally reach the alveoli?


A. 50-100 microns B. 10-20 microns C. 5-10 microns D. < 2 microns

272. What is the maximum amount of oxygen a gram of Hb A can bind?


A. 1 ml B. 1.34 ml C. 15 ml D. 20 ml

273. If blood Hb is 10 g/dl, PO2 is 100 mm Hg, and hemoglobin is 50% saturated with oxygen, the volume
of oxygen contained in 100 ml of blood is approximately:
A. 5.6 ml B. 6.7 ml C. 9.5 ml D. 19.5 ml

274. Which of the following has the highest P50?


A. Myoglobin B. Hb F C. Hb A

275. The normal value of P50 on the oxyhemoglobin dissociation curve in an adult is:
A. 13.5 mm Hg B. 20.3 mm Hg C. 26 mm Hg D. 33.8 mm Hg

276. What is/are the consequence(s) of a selective lesion of the pneumotaxic center?
A. Respiratory frequency increases D. Tidal volume becomes greater
B. Tidal volume decreases E. Prolonged inspiratory spasms
C. Respiration becomes slower

277. Apneusis occurs typically after:


A. Damage to phrenic motor neurons C. Lesions of pneumotaxic center and vagotomy
B. Damage to pneumotaxic center D. Denervation of peripheral chemoreceptors

278. When someone inhales a gas mixture consisting of 10% CO2 for 2 minutes, which of the following is
least likely?
A. PCO2 remains at 40 mm Hg C. Alveolar PO2 decreases
B. Respiratory minute volume increases D. pH of brain interstitial fluid drops

279. Following removal of both carotid bodies:


A. Ventilatory response to hypoxia is increased C. Breath holding time is increased
B. Ventilatory response to CO2 is increased

280. Which of the following structures or mechanisms is / are least important in the regulation of
breathing at rest in humans?
A. Medullary chemoreceptors B. Carotid and aortic bodies C. Hering-Breuer reflexes
249
Respiratory System Physiology

281. Life is impossible without pressurization at an altitude greater than:


A. 12000 m B. 14000 m C. 17000 m D. 19000 m

282. Choose all correct answers. Severe hypoxia:


A. Leads to accumulation of lactate ions in tissues D. Induces erythropoietin secretion
B. Stimulates the sympathetic nervous system E. Increases synthesis of 2,3-BPG
C. Decreases cerebral vascular resistance F. Increases P50 of Hb

283. Which one of the following has been recently postulated to function as a sensor of oxygen levels
and a facilitator of oxygen transport in the brain?
A. Myoglobin B. Neuroglobin C. Nitric oxide D. Cytochrome oxidase c

284. Which of the following combinations of arterial blood gas results is most likely in a normal person
after a month’s residence at 4000-meter altitude? Arterial pH, PCO2 and HCO3‒ (mmol/L) respectively
would be:
A. 7.36, 36 mm Hg, 30 C. 7.46, 26 mm Hg, 19
B. 7.4. 40 mmHg, 24 D. 7.6, 20 mm Hg, 40

285. The arterial blood gas values, pH 7.58, PCO2 23 mm Hg, PO2 300 mm Hg and oxygen saturation of
hemoglobin 60% are most consistent with a diagnosis of:
A. Carbon monoxide poisoning C. Voluntary hyperventilation
B. Ventilatory (type 2 respiratory) failure D. Methyl alcohol poisoning

286. Which of the following conditions leads to tissue hypoxia without an alteration of oxygen content
of blood?
A. Carbon monoxide poisoning C. Cyanide poisoning
B. Methemoglobinemia D. Respiratory acidosis

287. What is the treatment of choice for air embolism?


A. Continuous positive airway pressure C. Hyperbaric oxygenation
B. Gradual decompression D. Positive pressure ventilation

288. Hyperbaric oxygen therapy is useful in the management of:


A. Cyanide poisoning C. Diabetic leg ulcers E. Decompression sickness
B. Carbon monoxide poisoning D. Gas gangrene

289. Vital capacity:


A. Equals to inspiratory reserved volume + residual volume
B. Is increased in pulmonary tuberculosis
C. Is maximum amount of air inspired after tidal inspiration
D. Is not affected in obstructive lung diseases
E. Normally is 4600 ml

290. 97% of oxygen from lungs to tissues is carried in chemical combination with:
A. Carbon dioxide C. Hemoglobin E. Plasma proteins
B. Hydrogen ions D. Water

291. Reaction between water and carbon dioxide with in Red Blood Cells is catalyzed by the enzymes:
A. Peroxidase B. Catalase C. Collagenase
250
Respiratory System Physiology
D. Carbonic anhydrase E. Esterase

292. Dorsal respiratory group of neurons:


A. Controls the depth of breathing
B. Causes expiration
C. Is located in superior portion of pons
D. Emits inspiratory ramp signals
E. Remains inactive during normal resting breathing

293. Increased respiratory rate at initiation of exercise results from:


A. Neurogenic signals D. Decreased PCO2
B. Increased PCO2 E. Increased body temperature
C. Increased hydrogen ion concentration

294. The basic respiratory rhythm is generated in the


A. Apneustic center D. Pneumotaxic center
B. Nucleus parabrachialis E. Cerebrum
C. Dorsal medulla

295. At the end of a quiet inspiration, intra-alveolar pressure is normally


A. 240 cmH2O D. 14 cmH2O
B. 24 cmH2O E. 140 cmH2O
C. 0 cm H2O

For the following three questions 296–298


The diagram below represents a spirometry tracing illustrating the changes in lung volume that occurred
when a subject inhaled maximally and then rapidly exhaled as much gas as possible.

296. If the patient’s total lung capacity is 6 L, what is the functional residual capacity?
A. 1 L B. 2 L C. 3 L D. 4 L E. 5 L

297. What is the FEV1?


A. 1.5 L B. 2.5 L C. 3.5 L D. 4.5 L E. 5.5 L

298. What is the inspiratory capacity?


A. 1.0 L B. 1.5 L C. 2.0 L D. 2.5 L E. 3.0 L
251
Respiratory System Physiology

For the following two questions 299–300


Use the respiratory data below to answer the following questions.
Tidal volume = 400 mL
Dead space volume = 100 mL
Breathing frequency = 10 breaths/min
PCO2 = 50 mmHg (arterial blood)

299. The patient’s alveolar ventilation is


A. 3 L/min B. 4 L/min C. 5 L/min D. 6 L/min E. 7 L/min

300. If the patient doubles his tidal volume without changing his CO2 production, his PCO2 will be
A. 15 mmHg B. 20 mmHg C. 25 mmHg D. 30 mmHg E. 35 mmHg

301. A woman has a respiratory rate of 18, a tidal volume of 350 mL, and a dead space of 100 mL. What
is her alveolar ventilation?
A. 4.0 L B. 4.5 L C. 5.0 L D. 5.5 L E. 6.0 L

302. The woman in the preceding question has a normal PCO2 of 40 mmHg. If she increases her tidal
volume by 75 mL, her PCO2 will become approximately
A. 15 mmHg B. 20 mmHg C. 25 mmHg D. 30 mmHg E. 35 mmHg

303. Which one of the following would increase in obstructive, but not in restrictive, lung disease?
A. Vital capacity C. Functional residual capacity
B. FEV1 D. Breathing frequency

304. During the early stages of an asthmatic attack,


A. Arterial carbon dioxide tension decreases C. Airway resistance decreases
B. Lung compliance increases D. Arterial oxygen tension increases

305. Which one of the following will decrease in a person with ventilation-perfusion (V/Q)
abnormalities?
A. Anion gap D. Alveolar-arterial gradient for oxygen
B. Arterial pH E. Alveolar ventilation
C. Arterial carbon dioxide tension

306. Which one of the following is higher at the apex of the lung than at the base when a person is
standing?
A. V/Q ratio C. Ventilation E. Lung compliance
B. Blood flow D. PCO2

307. In areas of the lung with lower than normal V/Q ratios, the
A. Capillary CO2 tension is lower than normal
B. Pulmonary vascular resistance is higher than normal
C. Alveolar O2 tension is higher than normal
D. Water vapor pressure is higher than normal
E. Gas exchange ratio is higher than normal

308. Very small particles are removed from the respiratory system by
A. Bulk flow B. Diffusion C. Expectoration D. Phagocytosis E. Ciliary transport
252
Respiratory System Physiology

309. Which of the following conditions causes a decrease in arterial O2 saturation without a decrease in
O2 tension?
A. Anemia D. Hypoventilation
B. Carbon monoxide poisoning E. Right-to-left shunt
C. A low V/Q ratio

310. The bulk of CO2 is transported in arterial blood as


A. Dissolved CO2 C. Carbaminohemoglobin E. Carboxyhemoglobin
B. Carbonic acid D. Bicarbonate

311. The partial pressure of oxygen inhaled from a tank containing 21% O2 by divers at a depth of 100 ft
below sea level (4 atmospheres) is approximately
A. 40 mmHg B. 100 mmHg C. 200 mmHg D. 600 mmHg E. 1200 mmHg

For the following three questions 312–314


Use the pulmonary values below to answer the following questions.
Tidal volume = 500 mL Expired PCO2 = 40 mmHg
Respiratory rate = 15/min Alveolar PCO2 = 50 mmHg

312. The patient’s dead space is (The dead space can be calculated using the Bohr equation: Dead space
volume (VD) = Tidal volume (VT) * (Alveolar PCO2 − Expired PCO2) ÷ Alveolar PCO2]
A. 80 mL B. 90 mL C. 100 mL D. 110 mL E. 120 mL

313. The patient’s alveolar ventilation is


A. 4.0 L/min B. 4.5 L/min C. 5.0 L/min D. 5.5 L/min E. 6.0 L/min

314. To return the PCO2 value to normal (40 mmHg), the patient would have to increase her alveolar
ventilation to
A. 6.5 L/min B. 7.0 L/min C. 7.5 L/min D. 8.0 L/min E. 8.5 L/min

315. At what point during the tidal breath illustrated below is the alveolar PCO2 at its highest value?

A. A B. B C. C D. D E. E

316. Peripheral and central chemoreceptors may both contribute to the increased ventilation that occurs
as a result of
A. A decrease in arterial oxygen content B. A decrease in arterial blood pressure
253
Respiratory System Physiology
C. An increase in arterial carbon dioxide tension E. An increase in arterial pH
D. A decrease in arterial oxygen tension

317. Complete transection of the brainstem above the pons would


A. Result in cessation of all breathing movements
B. Prevent any voluntary holding of breath
C. Prevent the central chemoreceptors from exerting any control over ventilation
D. Prevent the peripheral chemoreceptors from exerting any control over ventilation
E. Abolish the Hering-Breuer reflex

For the following two questions 318–319


The diagram below illustrates the change in intrapleural pressure during a single breath.

318. At which point on the diagram is inspiratory airflow the greatest?


A. A B. B C. C D. D E. E

319. At which point on the diagram is lung volume the greatest?


A. A B. B C. C D. D E. E

320. The following diagram illustrates a flow-volume loop obtained from a normal patient. At which one
of the points on the curve will airflow remain constant despite an increased respiratory effort?

A. A B. B C. C D. D E. E
254
Respiratory System Physiology

321. A deficiency of pulmonary surfactant would


A. Decrease surface tension in the alveoli
B. Decrease the change in intrapleural pressure required to achieve a given tidal volume
C. Decrease lung compliance
D. Decrease the work of breathing
E. Increase functional residual capacity (FRC)

For the following two questions 322–323


Use the following diagram to answer the questions.

322. What is the P50 of the oxyhemoglobin curve labeled A in the diagram?
A. 80 mmHg B. 60 mmHg C. 40 mmHg D. 30 mmHg E. 20 mmHg

323. Which of the following conditions is most likely to shift the above oxyhemoglobin curve from A to
B?
A. Increased temperature D. Hyperventilation
B. Exercise E. Metabolic acidosis
C. Acclimatization to high altitude

324. Which one of the following is higher at total lung capacity than it is at residual volume?
A. Anatomical dead space
B. Lung compliance C. Airway resistance D. Alveolar pressure

325. Which one of the following components of a pulmonary function test will be closest to normal in a
patient with restrictive lung disease?
A. FEV1 B. FVC C. FEV1/FVC D. TLC

326. A man breathing room air at sea level has a PCO2 of 48 mmHg. His alveolar oxygen tension (PO2) is
(can be calculated using the modified alveolar gas equation, Alveolar PO2 = PiO2 − (alveolar PCO2/R),
where R is the respiratory exchange ratio (VCO2 produced /VO2 consumed), which depends on the diet
and is normally 0.8. PiO2 equals the fraction of oxygen in room air (0.21) times the barometric pressure at
sea level (760 mmHg) minus the water vapor pressure (47 mmHg) of saturated tracheal gas at body
temperature (37°C)).
A. 150 mmHg B. 110 mmHg C. 100 mmHg D. 90 mmHg E. 60 mmHg
255
Respiratory System Physiology

327. A young skier with normal pulmonary function (minute volume 4 L; pulmonary blood flow 5 L/min)
who is recovering from a tibial fracture suddenly develops right-sided chest pain and tachypnea. Embolic
occlusion of the right pulmonary artery is suspected. Which of the following alveolar gas measurements
would immediately confirm the diagnosis?
PO2 (mmHg) PCO2 (mmHg)
A. 125 60
B. 125 20
C. 100 40
D. 80 20
E. 80 60

328. Measurement of the lecithin-sphingomyelin (L-S) ratio in amniotic fluid assesses


A. The placenta’s ability to oxygenate the fetus D. Fetal brain development
B. Fetal adrenal function E. Fetal lung maturity
C. Fetal kidney development

329. When the respiratory muscles are relaxed, the lungs are at
A. Residual volume (RV) D. Inspiratory reserve volume (IRV)
B. Expiratory reserve volume (ERV) E. Total lung capacity (TLC)
C. Functional residual capacity (FRC)

330. Which one of the following is the most likely cause of a high arterial PCO2?
A. Increased metabolic activity D. Alveolar capillary block
B. Increased alveolar dead space E. Increased alveolar ventilation
C. Depressed medullary respiratory centers

331. Pulmonary vascular resistance decreases if


A. The lungs are inflated to total lung capacity
B. Sympathetic stimulation to the pulmonary vessels is increased
C. Alveolar oxygen tension is decreased
D. Plasma hydrogen ion concentration is decreased
E. Cardiac output is increased

332. Which of the following would normally be less in the fetus than in the mother?
A. PCO2 D. PO2
B. Pulmonary vascular resistance E. Arterial hydrogen ion concentration
C. Affinity of hemoglobin for oxygen

333. An increase in the P50 of an oxyhemoglobin curve would result from a decrease in
A. Metabolism B. pH C. Temperature D. Oxygen E. 2,3-DPG

334. During moderate aerobic exercise,


A. PO2 increases D. Alveolar ventilation increases
B. PCO2 decreases E. Blood lactate level increases
C. Arterial pH decreases

335. Reduction of functional hemoglobin associated with anemia, methemoglobinemia, or carbon


monoxide poisoning does not produce hyperpnea because the
A. Blood flow to the carotid body is decreased
256
Respiratory System Physiology
B. Total arterial oxygen content is maintained within the normal range
C. Carotid body chemoreceptors are stimulated
D. Central chemoreceptors are stimulated
E. PO2 of arterial blood is normal

336. Pulmonary alveoli are kept dry by factors that include the
A. Phagocytic activity of alveolar macrophages
B. Negative interstitial fluid pressure
C. Low vapor pressure of water in inspired air
D. Lack of surfactant
E. Tight junctions between the alveolar capillary endothelial cells

337. In which one of the following conditions will the diffusing capacity of the lung increase?
A. Formation of pulmonary emboli D. Congestive heart failure
B. Fibrotic lung disease E. COPD
C. Polycythemia

338. The percentage of hemoglobin saturated with oxygen will increase if


A. The arterial PCO2 is increased
B. The hemoglobin concentration is increased
C. The temperature is increased
D. The arterial PO2 is increased
E. The arterial pH is decreased

339. Which of the following will return toward normal during acclimatization to high altitude?
A. Arterial hydrogen ion concentration D. Arterial hemoglobin concentration
B. Arterial carbon dioxide tension E. Alveolar ventilation
C. Arterial bicarbonate ion concentration

340. Which one of the following statements characterizes pulmonary compliance?


A. It decreases with advancing age
B. It is inversely related to the elastic recoil properties of the lung
C. It increases in patients with pulmonary edema
D. It increases when there is a deficiency of surfactant

341. The activity of the central chemoreceptors is stimulated by


A. An increase in the PCO2 of blood flowing through the brain
B. A decrease in the PO2 of blood flowing through the brain
C. A decrease in the oxygen content of blood flowing through the brain
D. A decrease in the metabolic rate of the surrounding brain tissue
E. An increase in the pH of the CSF

342. In an acclimatized person at high altitudes, oxygen delivery to the tissues may be adequate at rest
because of
A. An increase in hemoglobin concentration
B. The presence of an acidosis
C. A decrease in the number of tissue capillaries
D. The presence of a normal arterial PO2
E. The presence of a lower-than-normal arterial PCO2
257
Respiratory System Physiology
343. Which of the following will increase as a result of stimulating parasympathetic nerves to the
bronchial smooth muscle?
A. Lung compliance D. Resistive work of breathing
B. Airway diameter E. Anatomic dead space
C. Elastic work of breathing

344. During a normal inspiration, more air goes to the alveoli at the base of the lung than to the alveoli
at the apex of the lung because
A. The alveoli at the base of the lung have more surfactant
B. The alveoli at the base of the lung are more compliant
C. The alveoli at the base of the lung have higher V/Q ratios
D. There is a more negative intrapleural pressure at the base of the lung
E. There is more blood flow to the base of the lung

345. A spirometer can be used to measure directly


A. Functional residual capacity D. Total lung capacity
B. Inspiratory capacity E. Physiological dead space
C. Residual volume

346. Which of the following conditions is most likely to produce the change from the normal maximum
flow-volume curve illustrated below?

A. Asthma B. Emphysema C. Bronchiolitis D. Fibrosis E. Fatigue

347. The affinity of hemoglobin for oxygen is increased by


A. Metabolic acidosis C. Hypoxemia E. Carbon monoxide poisoning
B. Exercise D. Anemia

348. The oxygen consumption of the respiratory muscles is decreased by


A. A decrease in lung compliance
B. A decrease in airway resistance
C. An increase in the rate of respiration
D. A decrease in the production of pulmonary surfactant
E. An increase in tidal volume
258
Respiratory System Physiology
349. Which one of the following characteristics is the most similar between the pulmonary and skeletal
muscle capillaries?
A. The interstitial protein concentration D. The capillary hydrostatic pressure
B. The interstitial hydrostatic pressure E. The capillary permeability to proteins
C. The capillary oncotic pressure

350. An increase in pulmonary blood flow during exercise


A. Causes alveolar oxygen tension to decrease
B. Causes the V/Q ratio at the top of the lung to increase
C. Causes pulmonary arterial resistance to decrease
D. Causes diffusion capacity to decrease
E. Causes arterial oxygen saturation to decrease

351. Which one of the following will be greater than normal in a patient with an areas of low V/Q ratio?
A. PCO2 D. Oxygen dissolved in blood
B. PO2 E. Oxygen combined with hemoglobin
C. Alveolar-arterial gradient

352. A patient with inadequate surfactant will have a relatively normal


A. FEV1 B. FVC C. FEV1/FVC D. V/Q ratio

353. Enzymes within the lung are responsible for the activation of
A. Angiotensin II B. Bradykinin C. Prostaglandins D. Serotonin E. Leukotrienes

354. When a person ascends to a high altitude, alveolar ventilation increases. Alveolar ventilation
continues to increase over the next several days because
A. The central chemoreceptors become more sensitive to low oxygen tensions
B. The peripheral chemoreceptors increase their firing rate
C. The plasma concentration of 2,3-DPG increases
D. Adaptation of central chemoreceptors to low pH of the cerebrospinal fluid
E. The oxygen-carrying capacity of hemoglobin increases

355. The clinical sign of cyanosis is caused by


A. An increase in the affinity of hemoglobin for oxygen
B. A decrease in the percent of red blood cells (hematocrit)
C. An increase in the concentration of carbon monoxide in the venous blood
D. A decrease in the concentration of iron in the red blood cells
E. An increase in the concentration of deoxygenated hemoglobin

356. Which one of the following gases diffuses across the alveoli-capillary membrane by a diffusion-
limited transport process?
A. Oxygen C. Carbon dioxide E. Nitrous oxide (N2O)
B. Nitrogen D. Carbon monoxide

357. A person ascends to the top of a mountain where the atmospheric pressure is below normal. Which
one of the following blood gases was drawn from the person at the top of the mountain?
PO2 PCO2
A. 50 30
B. 60 40
C. 80 50
259
Respiratory System Physiology
D. 100 40
E. 120 30

358. Hyperventilation in response to a stressful situation leads to


A. A decrease in the blood flow to the brain
B. An increase in the activity of the central chemoreceptors
C. A decrease in pH of the arterial blood
D. An increase in the resistance of the pulmonary blood vessels
E. A decrease in the excitability of nerve and muscle cells

359. Patients with chronic lung disease are often divided into “blue bloaters” (those who are cyanotic)
and “pink puffers” (those who are not cyanotic). The presence of cyanosis in blue bloaters but not in pink
puffers results from difference in their
A. V/Q ratios C. Airway resistances E. Expiratory flow rates
B. Vital capacities D. Total lung capacities

For the following two questions 360–361


Use the following diagram of oxyhemoglobin saturation curves to answer the next two questions. “N”
curve is the normal oxyhemoglobin saturation curve.

360. Which one of the above oxyhemoglobin saturation curves was obtained from fetal blood?
A. A B. B C. C D. D E. E

361. Which one of the above oxyhemoglobin saturation curves was obtained from blood exposed to
carbon monoxide?
A. A B. B C. C D. D E. E

For the following two questions 362–363


The following table shows several combinations of arterial blood-gas changes.
pH PCO2 PO2
A. ↑ ↑ ↑
B. ↓ ↑ ↑
C. ↑ ↑ ↓
D. ↓ ↓ ↑
E. ↑ ↓ ↓

362. Which one of the above changes in arterial blood gas values was obtained from a patient with a
large intrapulmonary shunt?
260
Respiratory System Physiology
A. A B. B C. C D. D E. E

363. Which one of the above changes in arterial blood gas values was obtained from a patient with
metabolic acidosis?
A. A B. B C. C D. D E. E

364. A patient with reduced VC, FRC, and RV is found to have a normal pH. A tentative diagnosis of
diffuse interstitial fibrosis is made. Which of the following characteristics are consistent with this
disease?
A. An increase in lung compliance D. A decrease in PCO2
B. A decrease in respiratory rate E. An increase in the FEV1/FVC ratio
C. An increase in the V/Q ratio

365. Airway resistance is lowest


A. During a forced expiration D. During vagal stimulation
B. At the total lung capacity E. When breathing gas with low oxygen
C. At the residual volume

366. In Caissons disease pain in the joints and muscles is due to


A. Formation of N2 bubbles C. Due to fatigue
B. Formation of CO2 bubbles D. Due to increase in barometric pressure

367. Normal value of FEV1 in an adult is


A. 95% B. 80% C. 65% D. 50%

368. The most important gas maintaining alveolar ventilation is


A. Oxygen B. Hydrogen C. Carbon dioxide D. N2

369. Hyperbaric oxygen is useful in all EXCEPT


A. Congenital heart disease C. Co poisoning
B. Gas gangrene D. N2 toxicity

370. Administration of O2 is of value in all EXCEPT


A. Cytotoxic hypoxia C. Anaemic hypoxia
B. Stagnant hypoxia D. Histotoxic hypoxia

371. As one ascends to higher than 3000 meters above sea level changes in alveolar PO2 and PCO2 are as
follows
A. Decrease in PO2, increase in PCO2 C. Increase in both PO2 and PCO2
B. Decrease in PO2, decrease in PCO2 D. Increase in PO2, decrease in PCO2

372. Surfactant is secreted by


A. Type 1 pneumatocytes C. Goblet cells
B. Type 2 pneumatocytes D. Pulmonary vessels

373. Which of the following discharge spontaneously during quiet breathing


A. Stretch receptors in lung C. Dorsal respiratory group of neurons
B. Motor neurons in respiratory muscles D. Ventral respiratory group of neurons

374. Pneumatic center functions primarily to


261
Respiratory System Physiology
A. Limit inspiration C. Decrease rate
B. Prolong expiration D. Discharge inspiratory action potentials

375. Airway resistance


A. Increases in asthma C. Increases in paraplegic patients
B. Decreases in emphysema D. Does not affect work of breathing

376. Decrease on PCO2, decrease in H+ and increased PO2 causes


A. Hyperventilation B. Hypoventilation C. Hypercapnoea D. Hypoxia

377. Herring-Breur inflation reflex in human being


A. Decreases the rate of respiration
B. Is not activated until the tidal volume increases above 1.5 lit
C. Is an important factor in normal control of ventilation
D. Is activated only when tidal volume is less than 1 lit

378. Total vital capacity is decreased but timed vital capacity is normal in
A. Bronchial asthma B. Scoliosis C. Chronic bronchitis D. All the above

379. The intrapleural pressure at the end of deep inspiration is


A. - 4mm Hg B. + 4 mm Hg C. - 6mm hg D. + 6 mm Hg

380. Figure 3–13 shows the oxyhemoglobin dissociation curves for a healthy patient and for an anemic
patient. Which of the following statements concerning these patients is true?

A. Patient A is anemic
B. Arterial PO2 is likely to be similar for both subjects
C. Venous PO2 of the anemic subject will be greater than that of the normal subject at rest or during
exercise
D. If cardiac output is identical, then oxygen delivery will be identical in subjects A and B.

381. A 25-year-old, 70-kg man broke several ribs as a result of a fall from a ladder. His treatment at a
nearby hospital included stabilizing his chest with bandages. The bandages were tied in a way that
reduced his tidal volume by 50%. To compensate, he doubled his respiratory rate. Two hours later, an
arterial blood sample was taken. Which of the following conditions would have been observed?
A. Increased PO2 and decreased PCO2 C. Decreased PO2 and increased PCO2
B. No change in PO2 or PCO2 D. Increased PO2 and increased PCO2

382. An individual’s total lung capacity (TLC) is 6.5 L, and her inspiratory capacity (FRC −TLC) is 3.55 L. At
the end of a normal expiration, her lung volume is 4.45 L. The individual’s tidal volume (VT) is
262
Respiratory System Physiology
A. 1.50 L B. 3.00 L C. 0.500 L D. 0.750 L E. 0.900 L

383. Following infusion of lactic acid into the blood of a healthy subject, arterial pH falls to 7.35. Which
of the following would be expected to occur?
A. A decrease in ventilation D. A rise in arterial PCO2
B. A rise in the pH of the cerebrospinal fluid E. A decrease in the ratio
C. A decrease in arterial PO2

384. Which of the following causes of brain hypoxia would most strongly stimulate the aortic and carotid
chemoreceptors?
A. Carbon monoxide poisoning
B. Severe anemia
C. Formation of methemoglobin
D. A marked decrease in pulmonary diffusing capacity
E. Acute respiratory alkalosis

385. Which of the following is NOT a function of the lungs?


A. Metabolism
B. Serves as a reservoir of blood for the left ventricle
C. It is a filter to protect the systemic vasculature
D. Facilitates the exchange of O2 and CO2 between air and blood
E. All of the above are true

386. Which of the following is in the correct path of CO2 from the tissue to the atmosphere?
A. Reaction with H2O to make H2CO3, dissociation to H+ and HCO3-, H+ combines with imidazole side chain of
hemoglobin, carried back to lungs as HHb+ and HCO3-, reverse reaction forms CO2
B. O2 is metabolized to CO2, reaction with H2O to make H2CO3, H2CO3 combines with imidazole side chain of
hemoglobin, H2CO3-Hb+ is carried back to the lungs, reverse reaction forms CO2
C. Reaction with H2O to make H2CO3, dissociation to H+ and HCO3-, HCO3- combines with imidazole side
chain of hemoglobin, carried back to the lungs as HCO3--Hb+ and H+, reverse reaction forms CO2
D. O2 is metabolized to CO2, reaction with H2O to make H2CO3, dissociation to H+ and HCO3-, carried back to
lungs in this form, reverse reaction forms CO2

387. Which of the following is the first branching of the bronchial tree that has gas exchanging
capabilities?
A. Terminal bronchioles C. Alveoli E. Alveolar ducts
B. Respiratory bronchioles D. Segmental bronchi

388. Which of the following could NOT be part of an acinus?


A. alveolar sacs C. Terminal bronchioles
B. Alveolar ducts D. Respiratory bronchiole

389. If you increased the left atrial pressure from 5 mmHg to 15 mmHg, what effect would that have on
pulmonary circulation?
A. It would force blood the opposite direction
B. It would increase the speed at which blood moves through the pulmonary circulation
C. No change
D. Blood flow would almost or completely stop
263
Respiratory System Physiology
390. Which of the following concerning average lung volumes and capacities of a person at rest is TRUE?
A. TLC>VC>TV>FRC C. TLC>VC>FRC>TV
B. TLC>FRC>VC>TV D. TLC>FRC>TV>VC

391. Which of the following is NOT a normal occurrence with increasing age?
A. Vital capacity of the lung decreases D. Inspiratory capacity decreases
B. Residual volume increases E. Expiratory reserve volume increases
C. Functional residual capacity increases

392. Which of the following spirometry measurements has the greatest sensitivity for detecting early air
flow obstruction?
A. FVC B. FEV1 C. FFE D. FEF25-75

393. Which of the following does NOT happen during inspiration?


A. The ribs move upward
B. The diaphragm lifts up
C. The antero-posterior dimensions of the chest are increased
D. The transverse dimensions of the thorax are increased
E. The scalene and sternocleidomastoid muscles can be recruited for inspiration

394. During inspiration, how does alveolar pressure compare to atmospheric pressure?
A. Alveolar pressure is greater than atmospheric
B. Alveolar pressure is less than atmospheric
C. Alveolar pressure is the same as atmospheric
D. Alveolar pressure is one of the few pressures where the reference pressure is not atmospheric

395. Which of the following represents the pressure difference that acts to distend the lungs?
A. Alveolar pressure C. Transthoracic pressure E. Esophageal pressure
B. Airway opening pressure D. Transpulmonary pressure

396. If a patient had a progressive lung disease that required an ever increasing pressure to fill the same
volume of lung, how would the lung's compliance be affected?
A. It would increase it
B. It would stay the same
C. It would decrease it
D. These variables do not affect lung compliance

397. An asthma sufferer finds she has to breathe at twice her normal rate. How does that affect her
dynamic compliance?
A. It stays the same
B. It decreases
C. It increases
D. Static compliance, not dynamic, is the variable affected by asthma

398. According to the Law of Laplace, air should flow from the smaller alveoli to the larger, collapsing
them. In the lungs, several factors counter that tendency, and stabilize the alveolar structures. Which of
the following is NOT one of them?
A. Surfactant lowers surface tension to a greater degree when it is on a smaller surface area, allowing the
smaller alveoli to stay open
B. Mechanical stability is given by surrounding alveoli
264
Respiratory System Physiology
C. Transpulmonary pressure is lower for smaller alveoli, allowing them to stabilize in comparison to the
bigger ones
D. Without surfactant, surface tension at the gas-liquid interface increases as alveolar surface area
decreases

399. Which of the following is FALSE concerning the production and role of lung surfactant?
A. It is part of a lipoprotein called dipalmitoyl phosphatidyl-choline
B. It is synthesized by alveolar type I cells
C. As the alveolar surface area decreases during the compression curve, the surfactant decreases the
surface tension at a constant rate
D. When surfactant density is decreased during expansion, surface tension initially rises rapidly, and then
slows down until it reaches the starting point

400. Which of the following is NOT true concerning respiratory distress syndrome in premature infants?
A. Their ability to synthesize dipalmitoyl phosphatidyl-choline is limited
B. Higher pressures are required to ventilate the lungs
C. Lung compliance is low
D. Positive pressure respirators are often used to assist them in breathing
E. Alveoli tend to overexpand and sometimes burst at the end of inspiration

401. Which of the following is NOT true at FRC?


A. It is about 75% TLC.
B. The elastic recoil of the chest wall is outward
C. The elastic recoil of the lung is inward
D. The relaxation pressure of the lung and chest wall combined is at atmospheric pressure

402. If the lung were punctured, which of the following would happen?
A. The lung would collapse on the side of the puncture
B. Both the lung and the chest wall would collapse on the side of the puncture
C. The relaxation pressure of the chest wall would increase until it surpassed the atmospheric
D. The relaxation pressure of the chest wall would increase, but stop before it reached atmospheric
pressure

403. Which of the following is FALSE concerning the airflow in the lungs?
A. During inspiration and expiration, the flow in the trachea and larger bronchi is turbulent
B. Towards the middle of the bronchial tree, the flow is turbulent at the branches and laminar in between
C. Near the end of the bronchial tree, the flow is laminar
D. The acini have very small radii which significantly increases the total air flow resistance of the bronchial
tree

404. Which of the following is FALSE concerning airway resistance?


A. Up to 50% is in the nose
B. The maximum resistance in the bronchial tree occurs in medium-sized bronchi
C. In the later generations, the radii are smaller; increasing the total resistance at each successive
generation
D. Airway resistance can be increased by loss of tissue elasticity and contraction of bronchial smooth
muscles

405. Which of the following does NOT apply to the alveoli at the base of the lungs?
A. They are less elastic than the alveoli at the apex
265
Respiratory System Physiology
B. The pleural pressure is lower
C. At FRC they are less inflated than the alveoli at the apex
D. They are closed at RV.
E. They have a greater volume change than alveoli at the apex during inspiration from FRC.

406. Which of the following is TRUE if a patient breathes slower than normal with increased tidal
volumes?
A. More resistive work is done C. More elastic work is done
B. The total work done decreases D. Compliance is decreased

407. Which of the following is INCORRECT concerning the efficiency of breathing and the oxygen
consumption of the respiratory muscles?
A. Efficiency is defined as the ratio of mechanical work done to move air to the amount of metabolic energy
used by the respiratory muscles
B. The respiratory system uses less than 3% of the body's total oxygen consumption at rest
C. Respiratory muscles are more efficient than large muscle groups
D. Emphysema increases the oxygen requirement for respiratory muscles
E. Hyperventilation can increase the oxygen consumption of respiratory muscles to 30%.

408. If the FIO2 is 0.21, the FEO2 is 0.16, the VT is 0.5 L, and the frequency of breathing is 12. What is the
VO2? The equations are VO2=VI * (FIO2 - FEO2) and VI = VT * f.
VO2 = oxygen consumption, FIO2 = Percent Oxygen in Inspired Air, FEO2 = Percent Oxygen in Expired Air,
VT = Tidal volume, VI = Volume of expired air = TV * F (frequency)
A. 3.0 L/min B. 0.75 L/min C. -0.75 L/min D. 0.3 L/min

409. How do you calculate how much inspired air actually ventilates the alveoli during one minute?
A. Subtract the volume of dead space from the tidal volume
B. Subtract both the dead space volume that was already in the lungs plus the dead space of the inspired air
that won't reach the alveoli from the tidal volume
C. Subtract the volume of dead space from the tidal volume and multiply it by the number of breaths per
minute
D. It is equal to the tidal volume times the frequency of breathing

410. Which of the following is NOT a function of dead space?


A. Warms expired air to body temperature
B. Saturates inspired air with water vapor
C. Removes bacteria and other particulate matter
D. Conducts the warmed air to the respiratory membranes

411. To which of the following is alveolar PCO2 directly proportional?


A. Rate of CO2 production and alveolar ventilation
B. Rate of CO2 production and rate of O2 consumption
C. Alveolar ventilation and rate of O2 consumption
D. Alveolar ventilation, rate of O2 consumption, and rate of CO2 production

412. If a patient's blood carries 10 grams of Hb per deciliter, what is the O2 carrying capacity of his
blood?
A. 18 ml per deciliter C. 10 ml per deciliter
B. 20 ml per deciliter D. 13 ml per deciliter
266
Respiratory System Physiology
413. Which of the following definitions is FALSE?
A. O2 content of blood is the actual amount of O2 in one deciliter of blood
B. O2 saturation of blood is the ratio of O2 content to its O2 capacity
C. The O2 uptake curve of blood is the functional relationship between O2 content and PO2
D. The O2 content of blood depends completely on the amount of Hb in the blood

414. Which of the following statements about Hb is FALSE?


A. A higher P50 than normal means that the O2 binds less tightly to Hb
B. An increase in 2,3-DPG shifts the O2 uptake curve to the left
C. An increase in PCO2 causes a right shift of the O2 uptake curve
D. An decrease in pH increases P50.
E. An increase in temperature shifts the O2 uptake curve to the right

415. Which of the following is NOT a form by which CO2 can be transported in the blood?
A. As bicarbonate C. Bound to the amino end groups in proteins
B. Dissolved in the blood D. Bound to the imidazole ring of glutamate

416. Which of the following pairs is NOT a pulmonary mechanoreceptor paired to a possible stimulus?
A. Stretch receptor: inflation
B. Irritant receptor: inhaled dust
C. Juxtacapillary receptors: decreases interstitial fluid volume in alveolar walls
D. Bronchial C receptors: large inflations

417. Which of the following is INCORRECT concerning the O2 / CO2 movement and processing through
the lungs and tissues?
A. Binding of O2 to Hb changes its configuration so that CO2 and H+ ions are more likely to dissociate
B. When CO2 diffused into the alveoli, the arterial PCO2 is lowered
C. Carbonic acid is an intermediate in the reaction combining H+ with HCO3- to form H2O and CO2
D. Arterial blood flows to the tissues where H+ ions combine with HCO3- to form H2O and CO2

418. Which of the following can cause stagnant hypoxia?


A. COPD C. Cyanide poisoning
B. Shock or heart failure D. Carbon monoxide poisoning

419. If you blocked the blood supply to an alveolus, which of the following would NOT occur as a result?
A. The ventilation perfusion ratio would be 0.
B. The alveolar PO2 would be greater than normal
C. The alveolar PCO2 would be 0.
D. All of the above are true

420. Which of the following is FALSE concerning the ventilation and perfusion of different regions of the
lung?
A. Alveoli at the top of the lung have a smaller dynamic compliance
B. The Hb moving through the base of the lung is less saturated than that at the apex of the lung
C. Alveolar PO2 at the apex of the lung is higher than that at the base of the lung
D. Regional variation in ventilation-perfusion is more efficient for oxygenating blood than is uniform
ventilation-perfusion

421. Which of the following is FALSE concerning the relationships of the variables in diffusion of O2
across a membrane?
267
Respiratory System Physiology
A. Doubling the thickness of the membrane would cut the total flow of O2 in half
B. Doubling the area of the membrane would double the total flow of O2
C. If you increased the alveolar concentration of O2, you would increase the total flow of O2 across the
alveolar membrane
D. The lower the diffusion coefficient, the higher the total flow
E. Increasing the arterial concentration of O2 would decrease the total flow of O2

422. If the blood moved slower than normal through the alveolar capillaries, which of the following
would have an increased uptake?
A. Carbon dioxide C. Oxygen
B. Carbon monoxide D. None of the above

423. Which of the following is FALSE concerning diffusion in the lungs?


A. CO is used to measure diffusing capacity because its uptake is diffusion limited
B. Actual diffusion time includes time required for an O2 molecule to diffuse from the alveolus, through the
membrane, the plasma, and into a RBC.
C. Reaction time is the time it takes the O2 molecule to react with Hb
D. The diffusion time is greater than the reaction time of an O2 molecule

424. Which of the following pairs is INCORRECT concerning central nervous systems and a factor they
respond to by affecting respiration?
A. Cerebellum: Mechanoreceptor input
B. Limbic system: emotional states
C. Cerebral cortex: voluntary control
D. Cerebral motor cortex: exercise

425. Which of the following will NOT increase the minute ventilation?
A. An increase in arterial pH.
B. An increase in arterial partial pressure of carbon dioxide
C. Increase in alveolar pressure of carbon dioxide
D. Exercise
E. Hypoxia

426. Which of the following is the primary regulating variable of the central chemoreceptors?
A. Arterial PO2 C. arterial pH
B. Arterial PCO2 D. Input from stretch receptors

427. Which of the following is FALSE concerning chemoreceptor input to the respiratory centers?
A. CSF is a poor buffer and a drop in PCO2 produces a large change in pH initiating a change in respiration
B. The body can adjust to chronic hypercapnea by using an active HCO3- transport process in the choroid
plexus
C. The carotid and aortic bodies detect increases in arterial PCO2 and pH, and decreases in arterial PO2
D. 75% of ventilatory response is regulated by chemoreceptors in the CSF and 25% by the carotid and aortic
bodies
E. Central chemoreceptors tend to respond slowly over time, while carotid bodies react quickly to
immediate needs

428. Considering the events associated with inspiration, all the fallowing are true EXCEPT
A. In order to fill the lungs with air, the alveolar pressure must be decreased relative to the atmospheric
pressure
268
Respiratory System Physiology
B. The contraction of the respiratory diaphragm increases the vertical dimension of the thorax, increasing
the volume of the thorax
C. The internal intercostal muscles contract during inspiration
D. Inspiration is dependent upon the contraction of the inspiratory muscles

429. Considering the events of the neural control of respiration,


A. The apneustic area is located in the medulla
B. The autorhythmicity of breathing is due to the intrinsic excitability of the neurons of the expiration
center
C. The pneumotaxic area prolongs inspiration
D. The carotid and aortic bodies contain important chemoreceptors sensitive to blood pH.
E. The respiratory diaphragm is innervated by the intercostal nerves
F. The expiratory center is inactive during normal, quiet breathing

430. Each lung is ventilated by a


A. Primary bronchus C. Tertiary bronchus E. terminal bronchiole
B. Secondary bronchus D. Respiratory bronchiole

431. Which of the following statements regarding the bronchial tree is TRUE?
A. The epithelium decreases in height as you proceed from the trachea toward the terminal bronchioles
B. Cartilage gradually replaces smooth muscle as the structural support as you approach the terminal
bronchioles
C. No bronchioles are capable of gas exchange
D. An aspirated "beer nut" would most likely be lodged in the left primary bronchus
E. A bronchopulmonary segment is ventilated by one secondary bronchus

432. The laryngopharynx


A. Serves only a respiratory function D. Is posterior to the trachea
B. Contains the true vocal folds (cords) E. Extends from the level of the hyoid bone to
C. Is the location of the pharyngeal tonsils the cricoid cartilage

433. Which of the following structures is NOT found within the hilus of the lung?
A. Lymphatic vessels C. Pulmonary artery E. Autonomic nerves
B. Pulmonary vein D. Tertiary bronchus

434. Which of the following is NOT a part of the respiratory membrane?


A. Capillary endothelium D. Capillary basement membrane
B. Alveolar basement membrane E. Type I alveolar cells
C. Smooth muscle cells

435. The lungs will collapse if the


A. Atmospheric pressure is less than alveolar pressure
B. Atmospheric and alveolar pressures are equal
C. Atmospheric pressure is greater than alveolar pressure
D. Intrapleural pressure is less than alveolar pressure
E. Intrapleural pressure is greater than alveolar pressure

436. The exchange of gases between the alveoli and the pulmonary capillaries is called
A. Pulmonary ventilation C. Internal respiration E. Compliance
B. External respiration D. Airway resistance
269
Respiratory System Physiology

437. Which of the following statements regarding normal, quiet breathing is FALSE?

A. Tidal volume typically equals 500 mL


B. Muscles of expiration are NOT used
C. Muscles of inspiration are used
D. All the inspired air reaches the alveoli
E. Dr. Haroian's anatomical dead space is greater than average because of his big Armenian nose

438. Which of the following statements regarding the lungs is FALSE?


A. The base of the lung rests on the respiratory diaphragm
B. Both lungs have an oblique fissure
C. The apex of each lung extends into the neck
D. The hilus of the lung is located on the costal surface of the lung
E. The pleural membranes secrete a serous fluid

439. During normal, quiet expiration, the


A. Thoracic volume increases D. Diaphragm contracts
B. Alveolar pressure increases E. Muscles of expiration contract
C. Intrapleural pressure decreases

440. Which of the following structures is a part of the conduction portion of the respiratory system?
A. Alveoli C. Alveolar ducts E. None of the above
B. Terminal bronchioles D. Respiratory bronchioles

441. Surfactant is secreted by


A. The parietal pleura C. The type i alveolar cells E. A serous membrane
B. The visceral pleura D. The type ii alveolar cells

442. Smooth muscle contributes to the structure of the


A. Respiratory membrane C. Terminal bronchiole E. Alveoli
B. Respiratory bronchiole D. Alveolar ducts

443. Calculate the expiratory reserve volume from the following data:
Tidal volume = 525 mL
Inspiratory reserve volume = 3175 mL
Vital capacity = 4975 mL
A. 1275 mL
B. 1800 mL
C. 2650 mL
D. 3700 mL
E. This calculation CAN NOT be made from the data presented

444. During internal respiration, CO2 diffuses into the


A. Alveoli C. Pulmonary capillaries
B. Systemic (tissue) capillaries D. Atmosphere

445. Which of the following factors has the most important influence on how much O 2 combines with
hemoglobin?
A. A low, but normal hematocrit B. The thickness of the respiratory membrane
270
Respiratory System Physiology
C. Fluctuations in tidal volume-alveolar D. Partial pressure of oxygen
ventilation E. The surface area for gas exchange

446. Which of the following statements regarding the transport of O2 in blood is FALSE?
A. The higher the PO2, the more hemoglobin binds with O2
B. The majority of O2 in the blood is in the form of oxyhemoglobin
C. Oxyhemoglobin is found within plasma
D. O2 doesn't dissolve easily in H2O.
E. Only dissolved O2 in the blood can diffuse from tissue capillaries into the tissue

447. Which of the following statements regarding CO2 transport in the blood is FALSE?
A. The greatest amount of CO2 is transported in the blood plasma as HCO3-.
B. During internal respiration, Cl- diffuses into the red blood cell
C. HbCO2 dissociates into Hb and CO2 within the pulmonary capillaries
D. The least amount of CO2 is transported in the blood plasma as dissolved CO2
E. In external respiration, CO2 is converted to HCO3-.

448. Which of the following represents a "right-to-left shunt"?


A. Pulmonary blood flow through a region of lung atelectasis (alveolar collapse)
B. Blood flow from the left ventricle to the right ventricle through a hole in the interventricular septum
C. Blood flow from skin arteries to skin veins which does not pass through skin capillaries
D. Blood flow from the aorta into the pulmonary artery through the ductus arteriosis
E. All of the above

449. A patient has an alveolar PO2 of 60 mmHg and a systemic arterial PO2 of 56 mmHg. The most likely
explanation for these values is
A. Hypoventilation
B. Diffusion limitation D. Ventilation-to-perfusion nonuniformity
C. Right-to-left shunt E. Either B, C, or D above

450. All of the following are possible causes of a higher than normal alveolar-arterial oxygen difference
EXCEPT
A. Pulmonary edema
B. Mismatching of alveolar ventilation and pulmonary perfusion
C. Chronic bronchitis
D. A cardiac septal defect that allows blood to flow from the right atrium to the left atrium
E. None of the above; that is, none are exceptions, since all could lead to a high alveolar-arterial oxygen
difference

451. Lack of oxygen equilibration due to diffusion limitation ("alveolar-capillary block") can be evaluated
by measuring the
A. Distribution of an inhaled gas mixture containing a radioactive marker
B. Diffusing capacity of CO (carbon monoxide)
C. Diffusing capacity of CO2 (carbon dioxide)
D. Diffusing capacity of N2 (nitrogen) when inhaling air
E. FEV-1/VC when inhaling pure oxygen

452. Blood gas measurements in an hypoxic patient indicates that the patient’s systemic arterial oxygen
content is normal but his systemic venous oxygen content is low. This is characteristic of
A. Diffusion limitation
271
Respiratory System Physiology
B. Right-to-left shunt
C. Pulmonary ventilation/perfusion nonuniformity
D. Anemic hypoxia (low Hb concentration)
E. Stagnant hypoxia (low cardiac output)

453. A patient has a normal oxygen partial pressure and content in pulmonary venous blood but his
systemic arterial blood shows a significantly lower than normal oxygen partial pressure and content. This
is diagnostic of
A. Diffusion limitation
B. Right-to-left shunt
C. Pulmonary ventilation/perfusion nonuniformity
D. Stagnant hypoxia (low cardiac output)
E. A, B. Or C above

454. Cyanosis (a bluish color of the skin and mucous membranes) indicates a higher than normal blood
concentration of
A. Carbon dioxide C. Hydrogen ion E. Reduced hemoglobin
B. Carbon monoxide D. Diphosphoglycerate (DPG)

455. As blood passes through systemic capillaries, what happens to the affinity of hemoglobin for oxygen
and what happens to the Hb-O2 dissociation curve?
A. Hb affinity for O2 increases and the dissociation curves shifts to the left
B. Hb affinity for O2 increases and the dissociation curves shifts to the right
C. Hb affinity for O2 decreases and the dissociation curves shifts to the left
D. Hb affinity for O2 decreases and the dissociation curves shifts to the right
E. Neither Hb affinity for O2 nor the Hb-O2 dissociation curve change

456. Suppose a person has a genetic defect causing him to continue to produce fetal hemoglobin (rather
than normal hemoglobin) throughout adult life. What would be the expected systemic arterial oxygen
partial pressure and saturation (compared to a normal person)?
A. Higher arterial PO2 and higher Hb O2 saturation
B. Higher arterial PO2 but lower Hb O2 saturation
C. Lower arterial PO2 but higher Hb O2 saturation
D. Lower arterial PO2 and lower Hb O2 saturation
E. Normal arterial PO2 but higher Hb O2 saturation
F. Normal arterial PO2 but lower Hb O2 saturation

457. Compared with systemic arterial blood, pulmonary arterial blood has a higher
A. Oxygen content C. HCO3− ion concentration E. None of the above
B. pH D. Hb concentration

458. As blood passes through systemic capillaries, the enzyme carbonic anhydrase catalyzes
A. Conversion of dissolved CO2 to carbonic acid
B. Conversion of carbonic acid to bicarbonate ion
C. Conversion of gaseous CO2 to dissolved CO2
D. Binding of carbon dioxide to hemoglobin, thus displacing oxygen
E. All of the above

459. What would be the expected systemic arterial oxygen content of a normal person inhaling pure
oxygen (100% O2) for an hour or so?
272
Respiratory System Physiology
A. 100 ml O2 / dl blood
B. 40 ml O2 / dl blood
C. 22 ml O2 / dl blood
D. 11 ml O2 / dl blood
E. None of the above, since pure oxygen is toxic and would cause death within the hour

460. An oxyhemoglobin saturation of mixed systemic venous blood of 25% for a person at rest is
A. Above normal B. Below normal C. Within the normal range

461. A hemoglobin concentration in systemic venous blood of 20 gm/dl blood is


A. above normal B. below normal C. within the normal range

462. What would be the expected effect of pulmonary edema on the pulmonary diffusing capacity for
oxygen (DO2) and carbon dioxide (DCO2)
A. Reduce DO2 and reduce DCO2
B. Reduce DO2 but no effect on DCO2
C. Reduce DCO2 but no effect on DO2
D. No effect on either DO2 or DCO2
E. No effect on either DO2 or DCO2 unless diffusing area is reduced also

463. At which of the following sites is the partial pressure of carbon dioxide (PCO2) highest?
A. Exhaled gas D. Systemic venous blood
B. Alveolar gas E. About the same in all of the above (40 mmHg)
C. Systemic arterial blood

464. At which of the following sites is the partial pressure of oxygen (PO2) highest?
A. Exhaled gas D. Alveolar gas
B. Anatomical dead space at the end of expiration E. About the same in all of the above (100 mmHg)
C. Anatomical dead space at the end of
inspiration

465. An individual who breaths through a hose or tube while keeping his tidal volume normal would be
expected to have an increased (compared to normal) of which of the following?
A. Dead space D. All of the above
B. Wasted ventilation E. Only a and b above
C. Systemic arterial carbon dioxide content

466. At the top of a 3000 meter high mountain, which of the following alveolar partial pressures would
be expected to be lower than normal? (Remember that barometric pressure, PB, decreases as one goes
higher)
A. Alveolar PO2 C. Alveolar PH2O E. only A and B above
B. Alveolar PCO2 D. all of the above

467. Reduction of the pulmonary diffusing capacity to one-fourth of its normal value would be expected
to have what effect on systemic arterial oxygen and carbon dioxide partial pressures (compared to
normal)?
A. decrease PO2 and decrease PCO2 D. increase PO2 and increase PCO2
B. decrease PO2 and increase PCO2 E. decrease arterial PO2 but no change in arterial
C. increase PO2 and decrease PCO2 PCO2
273
Respiratory System Physiology
468. At rest, wasted ventilation as a percent of total ventilation of 30% is
A. above normal B. below normal C. within the normal range

469. When is the resistance to blood flow of the pulmonary vascular bed lowest?
A. When a person is at rest sitting up
B, When a person is at rest lying down
C. When a person is breathing air at high altitude
D. When a person is exercising maximally
E. None of the above because pulmonary vascular resistance is constant

470. Which of the following might be expected to lead to pulmonary edema?


A. Decrease pulmonary arterial pressure (pulmonary hypotension)
B. Decrease in systemic arterial pressure (systemic hypotension)
C. Decrease volume of blood in the pulmonary circulation (as in hemorrhage)
D. Increase systemic venous pressure (as in right heart failure)
E. Increase pulmonary capillary permeability to plasma proteins (as in pulmonary inflammation)

471. In a person standing upright, which region of the lungs has the highest ventilation rate and which
region has the highest circulatory perfusion rate?
A. Highest ventilation: Apex; highest perfusion: Apex
B. Highest ventilation: Apex; highest perfusion: Base
C. Highest ventilation: Base; highest perfusion: Apex
D. Highest ventilation: Base; highest perfusion: Base
E. There is no "highest" region as the apex and base have equal ventilation and perfusion rates

472. Which one of the below is not a function of the respiratory system:
A. Allows oxygen from the air to enter the blood and carbon dioxide to leave the blood and enter the air
B. Can alter the pH by changing oxygen levels
C. Provides protection against some microorganisms by preventing their entry into the body and by
removing them from respiratory surfaces
D. Allows for speech and sound generation

473. Using the following formula: F = (P1 – P2)/R. Where:


F = airflow
P1 = pressure in the alveoli
P2 = atmospheric pressure outside the body
R = resistance to airflow
If R is kept constant, what would allow for the greatest airflow (F)?
A. P1 = P2
B. P1 > P2
C. P2 > P1
D. The relationship between P1 and P2 does not matter to airflow

474. Using the following formula: F = (P1 – P2)/R. Where:


F = airflow
P1 = pressure in the alveoli
P2 = atmospheric pressure outside the body
R = resistance to airflow
In an asthma attack, you would expect the airflow (F) to
A. Increase B. Decrease C. Not change D. Stop
274
Respiratory System Physiology

475. Assume that the atmospheric pressure outside the body is 760 mm Hg. Assuming you are neither
inhaling nor exhaling, the percent of gases in your alveoli is as follows:
N2 = 74.9%
O2 = 13.6%
CO2 = 5.3%
According to Dalton's Law, the partial pressure of oxygen in your alveoli is approximately
A. 104 mm B. 136 mm C. 569 mm D. 760 mm

476. Carbon dioxide has a solubility coefficient 24 times higher than oxygen. This means that carbon
dioxide
A. Has a partial pressure that is 24 times higher than oxygen
B. Diffuses 24 times faster in air than oxygen
C. Is 24 times more soluble in water than oxygen
D. Diffuses across the respiratory membrane 24 times faster than oxygen

477. In premature infants with respiratory distress syndrome or hyaline membrane disease, surfactant is
not produced in adequate quantities and a respirator may be required. This is because surfactant
A. Decreases the compliance of the lungs
B. Decreases the surface tension of the fluid that lines the alveoli
C. Decreases the elasticity of the elastic fibers in the alveolar walls
D. Increases the ability of hemoglobin to bind to oxygen

478. At the end of a normal respiration, the pleural pressure is -4 mm Hg and the alveolar pressure is 0
mm Hg. The lungs do not pull away from the pleural wall but create a suction effect. This suction effect (-
4 mm Hg) is caused by
A. Contraction of the internal intercostals
B. Elastic recoil of the lungs and surface tension of the fluid lining the alveoli
C. Elastic recoil of the central tendon
D. Contraction of the diaphragm

479. Emphysema causes destruction of elastic lung tissue. As a result of emphysema, the elastic recoil of
the lungs would ___________ and the compliance of the lungs would _____________.
A. Decrease, decrease C. Increase, decrease
B. Decrease, increase D. Increase, increase

480. The amount of air inspired or expired in a normal inhalation or exhalation is called __________ and
has a volume of about ____________ mL.
A. Tidal volume, 4600 C. Residual volume, 1200
B. Vital capacity, 4600 D. Tidal volume, 500

481. The maximum amount of moveable air in the lungs is called ______________ and has a volume of
about _____________mL.
A. Vital capacity, 4600 C. Inspiratory reserve volume, 3000
B. Total lung capacity, 5800 D. Inspiratory capacity, 3500.

482. In a normal inspiration, the amount of air that is actually available for exchange of oxygen and
carbon dioxide is about ____________mL.
A. 150 B. 350 C. 500 D. 650
275
Respiratory System Physiology
483. You went snorkeling for the first time and noticed that breathing through the tube was more
labored than breathing without the tube. This labored breathing was caused by:
A. Increase in residual volume C. Increase in expiratory reserve volume
B. Decrease in vital capacity D. Increase in dead air space

484. A resting individual has a respiratory rate of 12 breaths per minute. How much air is wasted (not
involved in gas exchange) per minute?
A. 500 mL/min C. 4200 mL/min
B. 1800 mL/min D. 6000 mL/min

485. Nitrogen is not normally exchanged in the respiratory tract, yet the partial pressure of nitrogen
varies in inhaled air, alveolar air and expired air. The differences can be explained because
A. Of random diffusing of n2 across the respiratory membranes
B. Nitrogen is the only gas not to obey Dalton's law
C. Nitrogen is added to the alveoli but removed from other areas of the respiratory tract
D. Oxygen is removed and carbon dioxide is added to the alveoli and water is added all along the
respiratory tract

486. Individuals with tuberculosis or pneumonia have difficulty breathing because


A. Solubility coefficients for oxygen and carbon dioxide are increased and the surface area of the
respiratory Membrane is decreased
B. The thickness of the respiratory membrane is increased while the surface area is decreased
C. The thickness of the respiratory membrane is increased and the partial pressure difference across the
respiratory membrane for oxygen and carbon dioxide is increased
D. The partial pressure differences across the respiratory membrane for oxygen and carbon dioxide are
increased while the surface area is decreased

487. Toxins that decrease the abundance of cilia in the respiratory tract tend to increase the
A. Partial pressures of oxygen and carbon dioxide
B. Thickness of the respiratory membrane
C. Coughing reflex
D. Surface area of the respiratory membrane

488. After vigorous exercise, you increase your rate and depth of breathing. This will cause the
A. Surface area of the respiratory membrane to increase
B. Partial pressure differences for oxygen and carbon dioxide to increase across the respiratory membrane
C. Thickness of the respiratory membrane to decrease
D. Solubility coefficients of oxygen and carbon dioxide to increase

489. Deoxygenated blood from the bronchi and bronchioles mix with oxygenated blood in the
pulmonary veins. This is called the
A. Physiologic shunt C. Pulmonary vein
B. Pulmonary artery D. Anatomical shunt

490. Blood entering the lungs has a PO2 of 40 mm Hg while blood immediately leaving the lung
capillaries has a PO2 of _______ mm Hg because it has reached equilibrium with the PO2 in the
____________.
A. 45, interstitial fluid C. 104, interstitial fluid
B. 45, alveoli D. 104, alveoli
276
Respiratory System Physiology
491. Arterial blood entering the tissues has a PO2 = 95 mm Hg and a PCO2 = 40 mm Hg. Venous blood
leaving the tissues has a PO2 = _________ mm Hg and a PCO2 = __________ mm Hg.
A. 104, 40 B. 95, 40 C. 40, 45 D. 20, 46

492. In the lungs, as a result of CO2 diffusing into the alveoli, you would expect hemoglobin's ability to
bind to oxygen to
A. Be unstable B. Stay the same C. Decrease D. Increase

493. In carbon monoxide (CO) poisoning, CO binds to the heme of hemoglobin much easier than oxygen.
What effect will this have on the ability of the tissues to get oxygen?
A. More carbonic acid will be produced, and the hemoglobin will give up more oxygen to the tissue
B. Hemoglobin will bind to less oxygen, and hemoglobin will not release as much oxygen to the tissue
C. Carbon monoxide makes hemoglobin give up oxygen more readily in the tissues
D. Carbon monoxide ties up hemoglobin's ability to bind to oxygen, but more hemoglobin will be produced
so no decline in oxygen in the tissue will be noticed

494. People living at high altitudes have increased levels of 2,3-bisphosphoglycerate (DPG) in their red
blood cells. This high level of DPG will
A. Increase the Bohr effect and make oxygen more available to the tissue
B. Bind to hemoglobin and reduce the amount of oxygen available to the tissue
C. Bind to hemoglobin and increase the amount of oxygen available to the tissue
D. Shift the oxygen-hemoglobin dissociation curve to the left and reduce the amount of oxygen available to
the tissue

495. People with anemias involving disorders of hemoglobin such as thalassemia and sickle cell anemia
are not getting enough oxygen to tissues. They have chemoreceptors in the walls of the large arteries
going to the head that monitor oxygen concentration in the blood. However, in spite of tissues getting
inadequate oxygen, the chemoreceptors do not reflexly increase heart rate and blood pressure to
provide more rapid movement of blood to the tissues. What might explain this apparent paradox;
chemoreceptors monitor oxygen in the blood but apparently they do not respond in people who have
thalassemia or sickle cell anemia to provide adequate oxygen to the tissues.
A. Chemoreceptors only monitor oxygen in arterial blood so what happens in the tissues has no effect
B. Chemoreceptors only monitor dissolved oxygen in the blood and that remains unchanged in the anemia
C. Chemoreceptors are made up of similar amino acids as hemoglobin and are also damaged in the anemia
D. Chemoreceptors actually inhibit the heart rate and blood pressure when oxygen falls below a certain
level in hemoglobin

496. Hemoglobin that has released its oxygen binds more readily to carbon dioxide than hemoglobin
that is highly saturated with oxygen. This is called
A. The Bohr effect C. Boyle's Law
B. Dalton's Law D. The Haldane effect

497. Bicarbonate ions are produced in red blood cells and then diffuse across the membrane and travel
in the blood dissolved in the plasma. Carrier molecules move bicarbonate ions across the red blood cell
membrane in exchange for ______ ions.
A. Sodium B. Potassium C. Hydrogen D. Chloride

498. The dorsal and ventral respiratory groups make up the respiratory center and are located in the
A. Pons C. Mesencephalon
B. Medulla oblongata D. Hypothalamus
277
Respiratory System Physiology

499. The group in the respiratory center that is most active during inspiration is the
A. Dorsal group C. Pontine respiratory group
B. Ventral group D. Pons

500. when stretch receptors are firing in the bronchi and bronchioles of the lungs, you would expect
A. Stimulation of the dorsal respiratory group
B. Inhibition of the dorsal respiratory group and stimulation of the expiratory center of the ventral
respiratory group
C. Inhibition of both the dorsal respiratory group and the entire ventral respiratory group
D. Stimulation of both the dorsal respiratory group and the entire ventral respiratory group

501. Which of the following muscles is stimulated by the dorsal respiratory group?
A. Sternocleidomastoid
B. Internal intercostals
C. Diaphragm
D. Rectus abdominus

502. A person holds his or her breath until he or she passes out. The person will resume breathing
because a(n)
A. Decrease in oxygen is detected by chemoreceptors in the carotid and aortic arteries, which inhibits the
inspiratory neurons of the respiratory center
B. Increase in pH is detected by chemoreceptors in the carotid and aortic arteries, which stimulates the
inspiratory neurons of the respiratory center
C. Increase in carbon dioxide and a decrease in pH are detected by chemoreceptors in the walls of carotid
and aortic arteries as well as chemoreceptors that detect changes in the pH of cerebrospinal fluid. These
chemoreceptors excite the inspiratory neurons of the respiratory center
D. Decrease in carbon dioxide and an increase in oxygen are detected by chemoreceptors that monitor the
blood and cerebrospinal fluid. These chemoreceptors inhibit the inspiratory neurons of the respiratory
center

503. Chemoreceptors are located in the


A. Walls of the carotid and aortic arteries and in the medulla oblongata
B. Walls of the carotid and aortic arteries only
C. Medulla oblongata and in the walls of the superior and inferior vena cavas
D. Walls of the carotid and aortic arteries and in the walls of the superior and inferior vena cavas

504. Strong emotions can sometimes stimulate hyperventilation. The best treatment for
hyperventilation is to
A. Just let the person relax and normal breathing will resume
B. Give the person oxygen
C. Have the person increases his or her rate of breathing
D. Have the person breath into a bag

505. A drop in PO2 has little effect on the respiratory centers until the arterial PO2 drops by about 40%.
However, a 10% rise in arterial PCO2 will cause the respiratory rate to double. This can be explained
because
A. Hemoglobin carries great reserves of oxygen while PCO2 changes also cause changes in pH
B. Oxygen is not as important to the cells as is carbon dioxide
C. Chemoreceptors are not sensitive to PO2 but are sensitive to PCO2 and pH
278
Respiratory System Physiology
D. Oxygen cannot diffuse from the blood into the cerebrospinal fluid while carbon dioxide can, and the
chemoreceptors in the medulla oblongata are much more important in regulating respiration

506. A decrease in oxygen in the tissues below normal levels is called


A. Apnea C. Hypoxia
B. Hypocapnia D. Hypercapnia

507. During an asthma attack, the patient has difficulty breathing because of constriction of the
A. Trachea C. Terminal bronchioles E. Respiratory membrane
B. Bronchi D. Alveoli

508. During quiet expiration, the


A. Abdominal muscles relax D. Thorax and lungs passively recoil
B. Diaphragm moves inferiorly E. All of the above
C. External intercostal muscles contract

509. Contraction of the bronchiolar smooth muscle has which of these effects?
A. A smaller pressure gradient is required to get the same rate of airflow when compared to normal
bronchioles
B. Increases airflow through the bronchioles
C. Increases resistance to airflow
D. Increases alveolar ventilation

510. During the process of expiration, the alveolar pressure is


A. Greater than the pleural pressure C. Less than the barometric pressure
B. Greater than the barometric pressure D. Unchanged

511. The lungs do not normally collapse because of


A. Surfactant B. Pleural pressure C. Elastic recoil D. Both A and B.

512. The rate of diffusion of a gas across the respiratory membrane increases as the
A. Respiratory membrane becomes thicker
B. Surface area of the respiratory membrane decreases
C. Partial pressure difference of the gas across the respiratory membrane increases
D. Diffusion coefficient of the gas decreases
E. All of the above

513. In which of these sequences does PO2 progressively decrease?


A. Arterial blood, alveolar air, body tissues D. Alveolar air, arterial blood, body tissues
B. Body tissues, arterial blood, alveolar air E. Arterial blood, body tissues, alveolar air
C. Body tissues, alveolar air, arterial blood

514. The partial pressure of carbon dioxide in the venous blood is


A. Greater than in the tissue spaces C. Less than in the alveoli
B. Less than in the tissue spaces D. Less than in arterial blood

515. The chloride shift


A. Occurs primarily in pulmonary capillaries
B. Occurs when Cl- replaces HCO3- within red blood cells
C. Decreases the formation of bicarbonate ions
279
Respiratory System Physiology
D. Decreases the number of hydrogen ions

516. Which of these parts of the brainstem is correctly matched with its main function?
A. Ventral respiratory groups—stimulate the diaphragm
B. Dorsal respiratory groups—limit inflation of the lungs
C. Pontine respiratory group—switching between inspiration and expiration
D. All of the above

517. The chemosensitive area


A. Stimulates the respiratory center when blood carbon dioxide levels increase
B. Stimulates the respiratory center when blood pH increases
C. Is located in the pons
D. Stimulates the respiratory center when blood oxygen levels increase
E. All of the above

518. Blood oxygen levels


A. Are more important than carbon dioxide in the regulation of respiration
B. Need to change only slightly to cause a change in respiration
C. Are detected by sensory receptors in the carotid and aortic bodies
D. All of the above

519. The Hering-Breuer reflex


A. Limits inspiration
B. Limits expiration
C. Occurs in response to changes in carbon dioxide levels in the blood
D. Is stimulated when oxygen decreases in the blood
E. Does not occur in infants

520. At the onset of exercise, respiration rate and depth increases primarily because of
A. Increased blood carbon dioxide levels
B. Decreased blood oxygen levels
C. Decreased blood pH
D. Input to the respiratory center from the cerebral motor cortex and proprioceptors

521. In response to exercise training,


A. The tidal volume at rest does not change
B. Minute ventilation during maximal exercise increases
C. The brain learns to match ventilation to exercise intensity
D. All of the above

522. Besides lubricating the visceral and parietal pleura, pleural fluid also
A. Holds the visceral and parietal pleural membranes together
B. Prevents the lungs from overinflating
C. Helps fill the pleural cavity
D. Increases diffusion rates in the lungs
E. Prevents the lungs from leaking

523. If alveolar volume increases, alveolar pressure


A. Decreases B. Increases C. Is unchanged
280
Respiratory System Physiology
524. For inspiration to occur, barometric pressure must be __________ alveolar pressure.
A. Greater than B. Less than C. Equal to

525. During expiration, the volume of the thorax __________ as the diaphragm __________ .
A. Decreases, contracts C. Increases, contracts
B. Decreases, relaxes D. Increases, relaxes

526. Lung recoil occurs because of elastic fibers in the alveolar walls and
A. Barometric pressure
B. Pleural pressure
C. Surface tension of the fluid that lines the alveoli
D. Surfactant secretion in the alveoli
E. The pneumothorax principle

527. Which of these conditions produces increased compliance of the lungs?


A. Airway obstruction C. Fibrosis of lungs E. All of these
B. Emphysema D. Pulmonary edema

528. In which of these areas is the partial pressure of oxygen (PO2) normally the greatest?
A. Inspired air C. Expired air E. Tissue capillaries
B. Alveolar air D. Pulmonary capillaries

529. The partial pressure of oxygen in the air is __________ than the partial pressure of carbon dioxide;
the solubility of oxygen in water is __________ than the solubility of carbon dioxide in water.
A. Lower, lower C. Greater, lower
B. Lower, higher D. Greater, higher

530. Which of these conditions causes a decreased rate of diffusion through the respiratory membrane?
A. Increased fluid in the lungs D. Decreased partial pressure gradient
B. Decreased diffusion coefficient E. All of these
C. Decreased respiratory surface area

531. If pH decreases, PCO2 increases, or temperature increases, the amount of oxygen bound to
hemoglobin
A. Increases B. Decreases C. Remains unchanged

532. Oxygen and carbon dioxide move through the respiratory membrane and into and out of cells by
the process of
A. Active transport C. Diffusion E. Facilitated diffusion
B. Cotransport D. Exocytosis

533. Hyperventilation produces which of these effects?


A. Decreased blood PCO2 D. Increase in pH
B. Vasodilation of peripheral blood vessels E. All of these
C. Decreased blood pressure

534. The most important factor for regulating respiratory rate is


A. Bicarbonate level in the blood C. CO2 level in the blood
B. Oxygen level in the blood D. Urea concentration in the blood
281
Respiratory System Physiology
535. Which of these statements concerning respiration is NOT true?
A. Higher brain centers can modify the activity of the respiratory center
B. A decrease in pH of the blood increases respiration rate
C. The Bohr effect allows carbon dioxide to bind more easily to hemoglobin that has released its oxygen
D. An increase in carbon dioxide in the blood causes pH to decrease
E. Low oxygen levels in the blood increase respiration rate

536. What test measures the amount of gas expelled when one takes a deep breath and exhales
maximally and rapidly?
A. Forced expiratory volume test C. Forced residual capacity test
B. Forced vital capacity test D. Forced internal thoracic volume assessment

537. Stimulating the cephalic end of a cut vagus nerve would cause respiration to:
A. Cease because of the lung stretch receptor reflex
B. Increase in rate because of the increased activity of the dorsal respiratory group
C. Increase in depth because of the increased sympathetic activity
D. None of the above

538. A premature baby usually has trouble breathing. However the respiratory system is completely
developed by:
A. 17 wks B. 22 wks C. 24 wks D. 28 wks E. 20 wks

539. Damage to which of the following would result in cessation of breathing?


A. The pneumotaxic center C. Lung stretch receptors E. The pons
B. The medulla D. The apneustic center

540. What happens when the V/Q of a lung unit decreases? The alveoli in that unit develop a
A. Higher PO2 D. Higher PCO2
B. Lower PO2 and a lower PCO2 E. Higher PN2 and a higher PO2
C. Higher PO2 and a lower PCO2 F. Lower PO2 and a Higher PCO2

541. Which is the letter that represents anemia in the following Hb-O2 dissociation curve?

1. A
2. B
3. C
4. D
5. E
282
Respiratory System Physiology
542. Stimulation of the reticular activating system of the brainstem (RAS) plays what role in ventilation?
A. Stimulates ventilation
B. Inhibits ventilation
C. Stimulates inhalation but inhibits exhalation (apneustic breathing)
D. Stimulates exhalation but inhibits inhalation
E. Causes ventilation to cease entirely ("Ondine’s Curse")

Regarding physiologic changes during moderate isotonic exercise in a healthy young adult, which of the
following are true and which are false?
543. PO2 declines
544. PCO2 increases
545. pH of arterial blood decreases (eventually during intense exercise)
546. A-V O2 difference increases
547. SvO2 increases
548. Plasma K+ concentration increases
549. The impulse rate from carotid bodies increases
550. During maximal exercise, O2 consumption may reach up to 4 L /min, about 16 times basal oxygen
consumption
551. VO2 max (maximum oxygen consumption during intense exercise) is increased by training
552. Heart rate may remain elevated for as long as 1 hour after severe exercise

553. The urge to breathe comes in direct response to:


A. How long it has been since you last took a breath
B. The oxygen concentration of your surrounding environment
C. The buildup of nitrogen within your blood stream
D. The pH of your blood
E. The buildup of blood pressure that occurs when you don't breathe

554. Involuntary breathing is caused by the


A. Pituitary gland C. Cerebral cortex E. Endocrine gland
B. Exocrine gland D. Medulla oblongata

555. Carbon monoxide is dangerous because


A. It binds strongly to hemoglobin, making it unavailable to oxygen
B. It binds strongly to plasma, making it unavailable to carbon dioxide
C. It raises the blood’s pH level, causing a person to hyperventilate
D. Carbon monoxide is not harmful; we have it in our bodies normally

556. The need to breathe is caused by


A. A decrease in blood pH C. A decrease in blood oxygen levels
B. An increase in blood pH D. A decrease in carbon dioxide levels

557. A person more susceptible to Chronic Obstructive Pulmonary Disease would be


A. A long time smoker D. A farmer that deals with pesticides
B. A long time fireman E. All of the above
C. A child whose parents smoke

558. The exchange of gases between the blood within the capillaries and tissue fluid surrounding the
body's cells is called?
283
Respiratory System Physiology
A. external respiration C. cellular respiration
B. cell metabolism D. internal respiration

559. The medulla oblongata and pons regulate and measure what?
A. The pH level of your blood C. The amount of O2 in your blood
B. Your body temperature D. The amount of air in your lungs

560. About how many alveoli are there in the lungs?


A. 300 million C. 300 trillion E. None of the above
B. 300 billion D. 300 thousand

561. In relation to atmospheric pressure, intrapleural pressure is:


A. More pressurized B. Less pressurized C. About the same

562. Hemoglobin gives up oxygen when the environment is more _______.


A. Acidic B. Alkaline C. Icey D. Open

563. The sac that surrounds your lungs is called what?


A. Diaphragm C. Pulmonary Thorax
B. Visceral Pleura D. None of the above

564. In what cellular organelle is the oxygen actually consumed and carbon dioxide produced?
A. Nucleus C. Microfilaments
B. Cytoplasm D. Mitochondria

565. Which of these are protective reflexes?


A. Hiccuping B. Sobbing C. Sneezing D. Itching

566. Where does gas exchange take place?


A. Bronchioles C. Pulmonary Capillaries
B. Conchae D. Roots of the Lungs

567. When you hyperventalate you release large amounts of CO2 and drop your O2 levels. As a result you
lose the urge to breathe and may pass out. This is called what?
A. Chronic Obstructive Pulmonary Disease C. Shallow water black out
B. Asthma D. Pulmonary Fibrosis

568. Acidosis is when you blood pH is below?


A. 7.05 B. 7.15 C. 7.25 D. 7.35

569. When we exhale deeply some air is still left in the lungs, this air left is called?
A. Tidal Volume C. Expiratory reserve Volume
B. Vital Capacity D. Residual Volume
284
Respiratory System Physiology
1 C 21 C 41 A 61 D 81 C&E 101 D 121 D

2 B 22 D 42 A 62 A 82 D 102 A&B 122 E

3 A 23 C 43 D 63 C 83 C&D 103 A&C 123 B

4 B 24 A 44 A 64 A 84 A 104 B&C 124 D

5 D 25 D 45 C 65 C 85 A&D 105 B 125 E

6 A 26 B 46 D 66 D 86 C&D 106 E 126 F

7 C 27 D 47 C 67 E 87 A&B 107 D 127 A

8 B 28 B 48 D 68 E 88 A 108 E 128 A

9 B 29 E 48 D 69 D 89 A&B 109 D 129 A

10 C 30 A 50 D 70 D 90 A&E 110 E 130 D

11 E 31 B 51 B 71 E 91 B&E 111 E 131 B

12 B 32 A 52 B 72 B 92 A&B 112 C 132 A

13 B 33 B 52 C 73 C 93 D 113 C 133 B

14 D 34 D 54 C 74 C 94 A&D 114 D 134 E

15 C 35 D 55 C 75 A 95 A 115 D 135 E

16 E 36 D 56 C 76 B 96 A&C 116 C 136 B

17 E 37 B 57 D 77 A 97 A&E 117 B 137 B

18 G 38 D 58 C 78 B 98 D&E 118 B 138 B

19 C 39 A 59 C 79 E 99 B&C 119 B 139 A

20 D 40 C 60 A 80 A 100 A&B 120 B 140 B


285
Respiratory System Physiology
141 D 161 F 181 B 201 D 221 C 241 C 261 D

142 C 162 D 182 C 202 A 222 D 242 A 262 A&B


B&
143 A 163 C 183 C 203 223 All 243 B&E 263 C
C
144 E 164 F 184 B 204 A 224 D 244 E 264 B

145 D 165 F 185 E 205 A 225 A 245 C 265 A


A&
146 E 166 F 186 C 206 226 A 246 A 266 C
B
D&
147 A 167 B 187 D 207 E 227 247 A&D 267 A
E
148 A 168 A 188 C 208 All 228 A 248 A, C, E 268 All
B&
149 C 169 E 189 C 209 229 A 249 A 269 D
C
150 C 170 D 190 B 210 C 230 A 250 B&C 270 C
B, C,
151 D 171 B 191 211 B 231 A 251 C&D 271 D
D
C& A, B,
152 A 172 D 192 212 B 232 252 E 272 B
D C
A&
153 E 173 D 193 D 213 B 233 253 A&E 273 B
C
154 B 174 B 194 B 214 All 234 D 254 D 274 C

155 C 175 A 195 B 215 A 235 C 255 D&E 275 C

156 B 176 B 196 C 216 C 236 A 256 B 276 C&D

157 D 177 A 197 B 217 B 237 D 257 B 277 C

158 D 178 E 198 B 218 A 238 All 258 C 278 A

159 E 179 D 199 C 219 D 239 A 259 D 279 C


A&
160 C 180 D 200 220 C 240 B 260 A 280 C
B
286
Respiratory System Physiology
281 B 301 B 321 C 341 A 361 C 381 C 401 A

282 All 302 D 322 D 342 A 362 E 382 A 402 A

283 B 303 C 323 D 343 D 363 D 383 B 403 D

284 C 304 A 324 A 344 B 364 E 384 D 404 C

285 A 305 C 325 C 345 B 365 B 385 E 405 A

286 C 306 A 326 D 346 D 366 A 386 A 406 C

287 C 307 B 327 B 347 E 367 B 387 B 407 C

288 All 308 D 328 E 348 B 368 C 388 C 408 D

289 E 309 B 329 C 349 C 369 D 389 D 409 C

290 C 310 D 330 C 350 C 370 D 390 C 410 A

291 D 311 C 331 C 351 C 371 B 391 E 411 B

292 D 312 C 332 D 352 C 372 A 392 D 412 D

293 A 313 E 333 B 353 A 373 C 393 B 413 D

294 C 314 C 334 D 354 D 374 A 394 B 414 B

295 C 315 A 335 E 355 E 375 A 395 D 415 D

296 C 316 C 336 B 356 D 376 B 396 C 416 C

297 C 317 B 337 C 357 A 377 B 397 B 417 D

298 E 318 B 338 D 358 A 378 B 398 C 418 B

299 A 319 C 339 A 359 A 379 A 399 B 419 A

300 B 320 C 340 B 360 A 380 B 400 E 420 D


287
Respiratory System Physiology
421 D 441 D 461 A 481 A 501 C 521 D 541 B

422 B 442 C 462 A 482 B 502 C 522 A 542 A

423 D 443 A 463 D 483 D 503 A 523 A 543 F

424 A 444 B 464 C 484 B 504 D 524 A 544 F

425 A 445 D 465 D 485 D 505 A 525 B 545 T

426 B 446 C 466 E 486 B 506 C 526 C 546 T

427 C 447 E 467 E 487 C 507 C 527 B 547 F

428 C 448 A 468 C 488 B 508 D 528 A 548 T


D
429 449 A 469 D 489 D 509 C 529 C 549 T
&F
430 A 450 C 470 E 490 D 510 B 530 E 550 T

431 A 451 B 471 D 491 C 511 D 531 B 551 T

432 E 452 E 472 B 492 D 512 C 532 C 552 T

433 D 453 B 473 C 493 B 513 D 533 E 553 D

434 C 454 E 474 B 494 C 514 B 534 C 554 D

435 E 455 D 475 A 495 B 515 B 535 C 555 A

436 B 456 E 476 C 496 D 516 C 536 B 556 A

437 D 457 C 477 B 497 D 517 A 537 A 557 E

438 D 458 A 478 B 498 B 518 C 538 D 558 D

439 B 459 C 479 B 499 A 519 A 539 B 559 A

440 B 460 B 480 D 500 B 520 D 540 F 560 A


288
Respiratory System Physiology
561 B

562 A

563 B

564 D

565 C

566 C

567 C

568 D

569 D

570

571

572

573

574

575

576

577

578

579

580
289
Cardiovascular System Physiology

MCQs in
Cardiovascular System Physiology
290
Cardiovascular System Physiology
1. The plateau of action potential recorded from ventricular muscle is caused by opening of which
voltage gated channel?
A. Na+ B. K+ C. Slow Ca++ D. Long lasting Ca++

2. The release of acetylcholine from vagal endings on the heart increases the conductance for:
A. Na+ B. K+ C. Ca++ D. Cl–

3. Autorhythmicity of cardiac muscle is demonstrated in:


A. Refractory period B. All or none law C. Starling's law D. Pacemaker potentials

4. Capacitance vessels are:


A. Veins B. Arteries C. Arterioles D. Capillaries

5. Which interval of the ECG can be used to determine heart rate?


A. PR B. RR C. ST D. QT

6. The normal pressure in the right atrium is close to:


A. 0 mmHg B. 8 mmHg C. 10 mmHg D. 20 mmHg

7. If preload to the heart is increased, rise in each of the following will occur EXCEPT:
A. Heart rate B. Cardiac output C. End systolic volume D. End diastolic volume

8. The strength of pulse pressure depends on:


A. Stroke volume B. Heart rate C. Arterial compliance D. A and C

9. Ejection fraction is increased in:


A. Muscular exercise B. Myocardial infarction C. Hemorrhage D. Myocarditis

10. Stimulation of baroreceptors would cause a rise in:


A. Vasomotor tone B. Vagal discharge C. Heart rate D. Peripheral resistance

11. Systolic pressure is maintained by:


A. Total peripheral resistance C. Venous return
B. Cardiac output D. Windkessel effect

12. Tachycardia which occurs due to increase in venous return is caused by:
A. Sino aortic reflex C. Bainbridge reflex
B. Cushing reflex D. Bezold Jarisch reflex

13. A positive inotropic effect on the heart causes increase in:


A. Preload B. Afterload C. Force of contraction D. Rate of contraction

14. In which type of circulatory shock, warm skin is observed?


A. Hemorrhagic B. Cardiogenic C. Obstructive D. Anaphylactic

15. Total peripheral resistance depends on all of the following EXCEPT:


A. Elasticity of the vessel B. End diastolic volume C. Caliber of the vessel D. Viscosity of the blood

16. Transmission of cardiac impulse is faster in:


A. SA node B. AV node C. Bundle of His D. Purkinje fibers
291
Cardiovascular System Physiology

17. In the first degree heart block:


A. PR interval is shortened C. ST segment is elevated
B. PR interval is lengthened D. QT interval is increased

18. Change of posture from lying down to standing shows increase in all of the following EXCEPT:
A. Blood pressure B. Heart rate C. Vagal tone D. Peripheral resistance

19. Coronary blood flow is least in which phase of the cardiac cycle?
A. Ventricular filling C. Isovolumetric relaxation
B. Isovolumetric contraction D. Ejection

20. Maximum fall in pressure occurs in which segment of the vascular system?
A. Aorta B. Arteries C. Arterioles D. Veins

21. Blood flow in a vessel is increased by all of the following EXCEPT:


A. Increase in pressure gradient C. Decrease in radius of vessel
B. Decrease in viscosity of blood D. Decrease in vasomotor tone

22. Which of the following is likely to cause bradycardia?


A. Moderate exercise C. Carotid sinus massage
B. Hypovolemia D. Vagal inhibition

23. During exercise an increase does not occur in:


A. Systolic pressure C. Cardiac output
B. Heart rate D. Peripheral resistance

24. R wave of ECG precedes:


A. I heart sound B. II heart sound C. III heart sound D. IV heart sound

25. Vasoconstriction is caused by:


A. Histamine B. Angiotensin II C. Bradykinin D. Nitric oxide

26. T wave of ECG is caused by which of the following ion currents in the ventricle?
A. K+ influx B. K+ exit C. Na+ influx D. Na+ exit

27. Stimulation of vagus nerve to the heart would decrease the conduction of impulses in :
A. SA node C. Ventricular myocardium
B. Bundle of His D. AV node

28. Intravenous infusion of fluid will not improve the condition of which type of shock?
A. Hemorrhagic B. Anaphylactic C. Traumatic D. Burn

29. The point of maximum impulse of the heart is at the:


A. Mediastinum B. Base C. Apex D. Aorta

30. The portion of the heart wall that lines the heart’s chambers is the:
A. Myocardium B. Pericardium C. Endocardium D. Epicardium

31. Which heart valve controls the flow of blood between the left atrium and the left ventricle?
292
Cardiovascular System Physiology
A. Pulmonary valve B. Aortic valve C. Tricuspid valve D. Mitral valve

32. What is the name of the great vessel that supplies blood to the right atrium?
A. Superior and inferior vena cavae C. Pulmonary artery
B. Aorta D. Pulmonary veins

33. How does the myocardium receive its blood supply?


A. It receives its supply of blood from the left ventricle
B. It receives its blood through the coronary sinus
C. It doesn’t require any additional blood other than what flows through the chambers
D. It receives its blood through the right and left coronary arteries

34. What is the heart’s primary pacemaker?


A. The atrioventricular (AV) node C. The sympathetic nervous system
B. The Purkinje fibers D. The sinoatrial (SA) node

35. On an electrocardiogram, the QRS complex represents:


A. Atrial depolarization
B. Ventricular depolarization
C. Ventricular repolarization
D. Impulse transmission from the atria to the ventricles

36. The cardiac cycle is:


A. The amount of blood pumped By the heart in 1 minute
B. The heart’s ability to beat spontaneously
C. The period of time when the left ventricle ejects its volume of blood
D. The series of events that occur from the beginning of one heartbeat to the beginning of the next

37. Cardiac output equals:


A. Heart rate times stroke volume
B. Stroke volume times ejection fraction
C. Age times heart rate
D. The percentage of blood ejected by the ventricles with each contraction

38. The parasympathetic nervous system sends impulses to the heart via the vagus nerve, which:
A. Slows the heart rate
B. Increases the heart rate
C. Doesn’t affect the heart rate
D. Raises the blood pressure, which, in turn, slows the heart rate

39. What is the term used to describe the amount of tension, or stretch, in the ventricular muscle just
before it contacts?
A. Contractility B. Afterload C. Ascites D. Preload

40. The vessels that carry blood away from the heart are:
A. Capillaries B. Sinusoids C. Veins D. Arteries

41. The innermost layer of blood vessels is composed of what type of material?
A. Simple squamous epithelium C. Connective tissue
B. Smooth muscle D. Elastic tissue
293
Cardiovascular System Physiology

42. Which arteries are called elastic arteries because of their ability to expand when blood surges into
them?
A. Distributing arteries B. Conducting arteries C. Arterioles D. Metarterioles

43. Veins are called capacitance vessels because they have the:
A. Capacity to pulsate with the heart’s contractions
B. Capacity to dilate and constrict to regulate blood pressure
C. Ability to stretch, giving them a great capacity for storing blood
D. Capacity to direct blood flow to organs and tissues in need

44. What is the purpose of the valves in veins?


A. Keep oxygenated blood from mixing with unoxygenated blood
B. Prevent the backflow of blood
C. Aid in the redirection of blood flow
D. Aid in the pumping of blood

45. What are the exchange vessels of the circulatory system, where nutrients, wastes, and hormones are
transferred between blood and tissues?
A. Arterioles B. Capillaries C. Veins D. Venules

46. The reason blood constantly circulates is because of:


A. The beating of the heart C. Pressure gradients
B. Low blood viscosity D. Vasoconstriction and dilation

47. What is the main reason blood flow is slowest in the capillaries?
A. Capillaries have a greater cross-sectional area
B. Capillaries have a higher osmotic pressure
C. Capillary blood has a higher viscosity than venous blood
D. Capillaries have to fight the pull of gravity

48. A 53-year-old woman is found, by arteriography, to have 50% narrowing of her left renal artery.
What is the expected change in blood flow through the stenotic artery?
A. Decrease to 1⁄2 C. Decrease to 1⁄8 E. No change
B. Decrease to 1⁄4 D. Decrease to 1⁄16

49. When a person moves from a supine position to a standing position, which of the following
compensatory changes occurs?
A. Decreased heart rate D. Decreased cardiac output
B. Increased contractility E. Increased PR intervals
C. Decreased total peripheral resistance (TPR)

50. At which site is systolic blood pressure the highest?


A. Aorta C. Pulmonary artery E. Renal artery
B. Central vein D. Right atrium F. Renal vein

51. A person’s electrocardiogram (ECG) has no P wave, but has a normal QRS complex and a normal T
wave. Therefore, his pacemaker is located in the
A. Sinoatrial (SA) node C. Bundle of his E. Ventricular muscle
B. Atrioventricular (av) node D. Purkinje system
294
Cardiovascular System Physiology

52. If the ejection fraction increases, there will be a decrease in


A. Cardiac output C. Heart rate E. Stroke volume
B. End-systolic volume D. Pulse pressure F. Systolic pressure

53. An increase in contractility is demonstrated on a Frank–Starling diagram by


A. Increased cardiac output for a given end-diastolic volume
B. Increased cardiac output for a given end-systolic volume
C. Decreased cardiac output for a given end-diastolic volume
D. Decreased cardiac output for a given end-systolic volume

For the following four questions

54. On the graph showing left ventricular volume and pressure, Isovolumetric contraction occurs from
point
A. 4 → 1 B. 1 → 2 C. 2 → 3 D. 3 → 4

55. The aortic valve closes at point


A. 1 B. 2 C. 3 D. 4

56. The first heart sound corresponds to point


A. 1 B. 2 C. 3 D. 4

57. If the heart rate is 70 beats/min, then the cardiac output of this ventricle is closest to
A. 3.45 L/min C. 5.25 L/min E. 9.85 L/min
B. 4.55 L/min D. 8.00 L/min

58. The tendency for blood flow to be turbulent is increased by


A. Increased viscosity C. Partial occlusion of a blood vessel
B. Increased hematocrit D. Decreased velocity of blood flow

59. Following a sympathectomy, a 66-yearold man experiences orthostatic hypotension. The explanation
for this occurrence is
A. An exaggerated response of the renin–angiotensin–aldosterone system
B. A suppressed response of the renin–angiotensin–aldosterone system
C. An exaggerated response of the baroreceptor mechanism
D. A suppressed response of the baroreceptor mechanism
295
Cardiovascular System Physiology
60. The ventricles are completely depolarized during which isoelectric portion of the electrocardiogram
(ECG)?
A. PR interval C. QT interval E. T wave
B. QRS complex D. ST segment

61. The change indicated by the dashed lines on the cardiac output/venous return curves shows

A. Decreased cardiac output in the “new” steady state


B. Decreased venous return in the “new” steady state
C. Increased mean systemic pressure
D. Decreased blood volume
E. Increased myocardial contractility

62. A 30-year-old female patient’s electrocardiogram (ECG) shows two P waves preceding each QRS
complex. The interpretation of this pattern is
A. Decreased firing rate of the pacemaker in the sinoatrial (SA) node
B. Decreased firing rate of the pacemaker in the atrioventricular (AV) node
C. Increased firing rate of the pacemaker in the sa node
D. Decreased conduction through the AV node
E. Increased conduction through the His Purkinje system

63. An acute decrease in arterial blood pressure elicits which of the following compensatory changes?
A. Decreased firing rate of the carotid sinus nerve
B. Increased parasympathetic outflow to the heart
C. Decreased heart rate
D. Decreased contractility
E. Decreased mean systemic pressure

64. Inspiration “splits” the second heart sound because


A. The aortic valve closes before the pulmonic valve
B. The pulmonic valve closes before the aortic valve
C. The mitral valve closes before the tricuspid valve
D. The tricuspid valve closes before the mitral valve
E. Filling of the ventricles has fast and slow components
296
Cardiovascular System Physiology
65. During exercise, total peripheral resistance (TPR) decreases because of the effect of
A. The sympathetic nervous system on splanchnic arterioles
B. The parasympathetic nervous system on skeletal muscle arterioles
C. Local metabolites on skeletal muscle arterioles
D. Local metabolites on cerebral arterioles
E. Histamine on skeletal muscle arterioles

For the following two questions

66. Curve A in the figure represents


A. Aortic pressure B. Ventricular pressure C. Atrial pressure D. Ventricular volume

67. Curve B in the figure represents


A. Left atrial pressure B. Ventricular pressure C. Atrial pressure D. Ventricular volume

68. An increase in arteriolar resistance, without a change in any other component of the cardiovascular
system, will produce
A. A decrease in total peripheral resistance (TPR) C. An increase in arterial pressure
B. An increase in capillary filtration D. A decrease in afterload

69. Which of the following is the result of an inward Na+ current?


A. Upstroke of the action potential in the sinoatrial (SA) node
B. Upstroke of the action potential in Purkinje fibers
C. Plateau of the action potential in ventricular muscle
D. Repolarization of the action potential in ventricular muscle
E. Repolarization of the action potential in the SA node
297
Cardiovascular System Physiology
For the following two questions

70. The dashed line in the figure illustrates the effect of


A. Increased total peripheral resistance (TPR) D. A negative inotropic agent
B. Increased blood volume E. Increased mean systemic pressure
C. Increased contractility

71. The x-axis in the figure could have been labeled


A. End-systolic volume C. Pulse pressure E. Heart rate
B. End-diastolic volume D. Mean systemic pressure

72. The greatest pressure decrease in the circulation occurs across the arterioles because
A. They have the greatest surface area
B. They have the greatest cross-sectional area
C. The velocity of blood flow through them is the highest
D. The velocity of blood flow through them is the lowest
E. They have the greatest resistance

73. Pulse pressure is


A. The highest pressure measured in the arteries
B. The lowest pressure measured in the arteries
C. Measured only during diastole
D. Determined by stroke volume
E. Decreased when the capacitance of the arteries decreases
F. The difference between mean arterial pressure and central venous pressure

74. In the sinoatrial (SA) node, phase 4 depolarization (pacemaker potential) is attributable to
A. An increase in K+ conductance D. A decrease in Ca2+ conductance
+
B. An increase in Na conductance E. Simultaneous increases in K+ and Cl− conductances
C. A decrease in Cl− conductance

75. Which receptor mediates constriction of arteriolar smooth muscle?


A. α1 Receptors B. β1 Receptors C. β2 Receptors D. Muscarinic receptors

76. During which phase of the cardiac cycle is aortic pressure highest?
A. Atrial systole C. Rapid ventricular ejection
B. Isovolumetric ventricular contraction D. Reduced ventricular ejection
298
Cardiovascular System Physiology
E. Isovolumetric ventricular relaxation G. Reduced ventricular filling (diastasis)
F. Rapid ventricular filling

77. Myocardial contractility is best correlated with the intracellular concentration of


A. Na+ B. K+ C. Ca2+ D. Cl− E. Mg2+

78. Which of the following is an effect of histamine?


A. Decreased capillary filtration
B. Vasodilation of the arterioles
C. Vasodilation of the veins
D. Decreased Pc
E. Interaction with the muscarinic receptors on the blood vessels

79. Carbon dioxide (CO2) regulates blood flow to which one of the following organs?
A. Heart C. Brain E. Skeletal muscle during exercise
B. Skin D. Skeletal muscle at rest

80. Cardiac output of the right side of the heart is what percentage of the cardiac output of the left side
of the heart?
A. 25% B. 50% C. 75% D. 100% E. 125%

81. The physiologic function of the relatively slow conduction through the atrioventricular (AV) node is
to allow sufficient time for
A. Runoff of blood from the aorta to the arteries D. Contraction of the ventricles
B. Venous return to the atria E. Repolarization of the ventricles
C. Filling of the ventricles

82. Blood flow to which organ is controlled primarily by the sympathetic nervous system rather than by
local metabolites?
A. Skin C. Brain
B. Heart D. Skeletal muscle during exercise

83. Which of the following parameters is decreased during moderate exercise?


A. Arteriovenous O2 difference
B. Heart rate D. Pulse pressure
C. Cardiac output E. Total peripheral resistance (TPR)

84. When propranolol is administered, blockade of which receptor is responsible for the decrease in
cardiac output that occurs?
A. α1 Receptors C. β2 Receptors E. Nicotinic receptors
B. β1 Receptors D. Muscarinic receptors

85. During which phase of the cardiac cycle is ventricular volume lowest?
A. Atrial systole E. Isovolumetric ventricular relaxation
B. Isovolumetric ventricular contraction F. Rapid ventricular filling
C. Rapid ventricular ejection G. Reduced ventricular filling (diastasis)
D. Reduced ventricular ejection

86. Which of the following changes will cause an increase in myocardial O2 consumption?
A. Decreased aortic pressure
299
Cardiovascular System Physiology
B. Decreased heart rate
C. Decreased contractility
D. Increased size of the heart
E. Increased influx of Na+ during the upstroke of the action potential

87. A 24-year-old woman presents to the emergency department with severe diarrhea. When she is
supine (lying down), her blood pressure is 90/60 mm Hg (decreased) and her heart rate is 100 beats/min
(increased). When she is moved to a standing position, her heart rate further increases to 120 beats/min.
Which of the following accounts for the further increase in heart rate upon standing?
A. Decreased total peripheral resistance D. Increased afterload
B. Increased venoconstriction E. Decreased venous return
C. Increased contractility

For the following three questions

88. During which phase of the ventricular action potential is the membrane potential closest to the K+
equilibrium potential?
A. Phase 0 B. Phase 1 C. Phase 2 D. Phase 3 E. Phase 4

89. During which phase of the ventricular action potential is the conductance to Ca2+ highest?
A. Phase 0 B. Phase 1 C. Phase 2 D. Phase 3 E. Phase 4

90. Which phase of the ventricular action potential coincides with diastole?
A. Phase 0 B. Phase 1 C. Phase 2 D. Phase 3 E. Phase 4

91. Propranolol (β receptor blocker) has which of the following effects?


A. Decreases heart rate
B. Increases left ventricular ejection fraction
C. Increases stroke volume
D. Decreases splanchnic vascular resistance
E. Decreases cutaneous vascular resistance

92. Which receptor mediates slowing of the heart?


A. α1 Receptors C. β2 Receptors
B. β1 Receptors D. Muscarinic receptors

93. Which of the following agents or changes has a negative inotropic effect on the heart?
A. Increased heart rate B. Sympathetic stimulation C. Norepinephrine
300
Cardiovascular System Physiology
D. Acetylcholine (ACh) E. Cardiac glycosides

94. The low-resistance pathways between myocardial cells that allow for the spread of action potentials
are the
A. Gap junctions C. Sarcoplasmic reticulum E. Mitochondria
B. T tubules D. Intercalated disks

95. Which agent is released or secreted after a hemorrhage and causes an increase in renal Na+
reabsorption?
A. Aldosterone D. Antidiuretic hormone (ADH)
B. Angiotensin I E. Atrial natriuretic peptide
C. Angiotensinogen

96. During which phase of the cardiac cycle does the mitral valve open?
A. Atrial systole
B. Isovolumetric ventricular contraction E. Isovolumetric ventricular relaxation
C. Rapid ventricular ejection F. Rapid ventricular filling
D. Reduced ventricular ejection G. Reduced ventricular filling (diastasis)

97. A hospitalized patient has an ejection fraction of 0.4, a heart rate of 95 beats/min, and a cardiac
output of 3.5 L/min. What is the patient’s end-diastolic volume?
A. 14 mL B. 37 mL C. 55 mL D. 92 mL E. 140 mL

98. Cardiac muscle acts as a functional syncytium because it:


A. Consists of single nucleated cells D. Has gap junctions between adjacent cells
B. Has a long refractory period E. Is involuntary
C. Has larger T-tubules

99. During the cardiac cycle, the maximal right ventricular systolic pressure (mmHg) is:
A. 5 B. 8 C. 25 D. 80 E. 120

100. Vagal stimulation results into slowing of the heart rate. This is due to increased permeability of the
sinoatrial nodal fiber membrane to:
A. Calcium C. Potassium E. Sodium and calcium
B. Chloride D. Sodium

101. In a patient having first degree atrioventricular block, the ECG shows prolonged:
A. PR interval D. RR interval
B. QRS interval E. VAT (ventricular activation time)
C. QT interval

102. QRS complex is the ECG:


A. Has a normal duration of 0.3 second
B. Has a normal shape in cases of bundle branch block
C. Is due to ventricular depolarization
D. Is due to ventricular systole
E. Represents atrial repolarization

103. The cardiac index is:


A. Cardiac output per square meter body surface area
301
Cardiovascular System Physiology
B. Increased in old age
C. Normally about 5 liters/minute/m2.
D. Stroke volume multiplied by the heart rate
E. Stroke volume per square meter body surface area

104. Normally about 65% of the total blood volume is present in:
A. Capillaries C. Arteriovenous anastomosis E. Veins
B. Arteries D. Arterioles

105. When arterial blood pressure falls to very low level(50 mmHg), the nervous mechanism which helps
to regulate it is mainly:
A. Baroreceptor reflex C. CNS ischemic response E. Venoconstriction
B. Chemoreceptors D. Cushing’s reflex

106. Coronary blood flow is regulated mainly by:


A. Autonomic nerve C. Heart rate E. Local metabolism
B. Diastolic aortic pressure D. Hormones

107. A patient having valvular heart disease shows collapsing pulse (water hammer). He is most likely to
have:
A. Aortic regurgitation C. Mitral regurgitation E. Tricuspid regurgitation
B. Aortic stenosis D. Pulmonary stenosis

108. During fetal life, the blood vessel having maximally oxygenated blood is:
A. Aorta C. Inferior vena cava E. Umbilical vein
B. Ductus arteriosus D. Pulmonary artery

109. During the exercise, skeletal muscle blood flow increases 15-25 times the resting value. This
increase is mainly due to:
A. Increased arterial blood pressure
B. Increased secretion of catecholamines
C. Local metabolic factors
D. Stimulation of sympathetic vasoconstrictor nerves
E. Stimulation of sympathetic vasodilator nerves

110. Coronary blood flow to the left ventricle increases during


A. Early systole
B. Myocardial hypoxia
C. Hypothermia
D. Stimulation of sympathetic nerves to the heart
E. Arterial hypertension

111. Local metabolic activity is the chief factor determining the rate of blood flow to the
A. Heart C. Skeletal muscle E. Kidney
B. Skin D. Lung

112. The pressure


A. Drop along large veins is similar to that along large arteries
B. Drop across the hepatic portal bed is similar to that across the splenic vascular bed
C. In the hepatic portal vein exceeds that in the inferior vena cava
302
Cardiovascular System Physiology
D. Drop across the vascular bed in the foot is greater when a subject is in the vertical than when he is in the
horizontal position
E. In foot veins is lower when walking than when standing still

113. The second heart sound differs from the first heart sound in that it is
A. Related to turbulence set up by valve closure
B. Longer lasting than the first sound
C. Higher in frequency
D. Occasionally split
E. Heard when the ventricles are relaxing

114. Ventricular filling


A. Depends mainly on atrial contraction
B. Begins during isometric ventricular relaxation
C. Gives rise to a third heart sound in some healthy people
D. Can occur only when atrial pressure is greater than atmospheric pressure
E. Is most rapid in the first half of diastole

115. Veins
A. Contain most of the blood volume
B. Have a sympathetic vasoconstrictor innervation
C. Receive nutrition from vasa vasorum arising from their lumen
D. Respond to distension by contraction of their smooth muscle
E. Undergo smooth muscle hypertrophy when exposed to high pressure through an arteriovenous fistula

116. In the heart


A. The left atrial wall is about three times thicker than the right atrial wall
B. Systolic contraction normally begins in the left atrium
C. Excitation spreads directly from atrial muscle cells to ventricular muscle cells
D. Atrial and ventricular muscle contracts simultaneously in systole
E. The contracting ventricles shorten from apex to base

117. Isometric (static) exercise differs from isotonic (dynamic) exercise in that it causes a greater
increase in
A. Venous return
B. Pressure in the veins draining the exercising muscle
C. Muscle blood flow
D. Mean arterial pressure
E. Cardiac work for the same increase in cardiac output

118. The net loss of fluid from capillaries in the legs is increased by
A. Arteriolar dilation
B. Change from the recumbent to the standing position
C. Lymphatic obstruction
D. Leg exercise
E. Plasma albumin depletion

119. When measuring blood pressure by the auscultatory method


A. The sounds that are heard are generated in the heart
B. The cuff pressure at which the first sounds are heard indicate systolic pressure
303
Cardiovascular System Physiology
C. The cuff pressure at which the loudest sounds are heard indicate diastolic pressure
D. Systolic pressure estimations tend to be lower than those made by the palpatory method
E. Wider cuffs are required for larger arms

120. The absolute refractory period in the ventricles


A. Is the period when the ventricles are completely unexcitable
B. Corresponds to the period of ventricular depolarization
C. Corresponds approximately to the period of ventricular contraction
D. Is shorter than the corresponding period in atrial muscle
E. Decreases during sympathetic stimulation of the heart

121. Vascular resistance


A. Increases by 50 per cent when the vascular radius is halved
B. Is related to the thickness of the wall of the vessel
C. Is related to the vessel’s length
D. Is affected by blood viscosity
E. Is greater in the capillary bed than in the arteriolar bed

122. Sympathetic drive to the heart is increased


A. In exercise
B. In excitement
C. In hypotension
D. When parasympathetic drive is decreased
E. During a vasovagal attack

123. In an adult subject standing quietly at rest, venous pressure in the


A. Foot is approximately equal to arterial pressure at heart level
B. Thorax decreases when the subject inhales
C. Hand is subatmospheric when the hand is raised above the head
D. Venous sinuses of the skull are subatmospheric
E. Superior vena cava is an index of cardiac filling pressure

124. Hyperemia in skeletal muscle during exercise is normally associated with


A. Release of sympathetic vasoconstrictor tone in the exercising muscles
B. Capillary dilation due to relaxation of capillary smooth muscle
C. A fall in arterial pressure
D. Reflex vasoconstriction in other vascular beds
E. An increase in cardiac output

125. Sinoatrial node cells are


A. Found in both atria
B. Innervated by the vagus
C. Unable to generate impulses when completely denervated
D. Connected to the AV node by fine bundles of Purkinje tissue
E. Able to generate impulses because their membrane potential is unstable

For the following four questions


A 60-year-old woman has a resting heart rate of 70 beats per minute, arterial pressure of 130/85 mm Hg,
and normal body temperature. Use the pressure-volume diagram of her left ventricle below to answer
the following four questions
304
Cardiovascular System Physiology

126. What is her cardiac output in milliliters per minute?


A. 2000 B. 3000 C. 4000 D. 6000 E. 7000

127. When does the first heart sound occur in the ventricular pressure–volume relationship?
A. At point B
B. Between point A and point B D. Between point C and point D
C. Between point B and point C E. Between point D and point A

128. When does the fourth heart sound occur in the ventricular pressure–volume relationship?
A. At point D
B. Between point A and point B D. Between point C and point D
C. Between point B and point C E. Between point D and point A

129. What is her ventricular ejection fraction?


A. 33% B. 50% C. 60% D. 67% E. 80%

130. Which statement about cardiac muscle is most accurate?


A. The T-tubules of cardiac muscle can store much less calcium than the T-tubules in skeletal muscle
B. The strength and contraction of cardiac muscle depends on the amount of calcium surrounding cardiac
myocytes
C. In cardiac muscle, the initiation of the action potential causes an immediate opening of slow calcium
channels
D. Cardiac muscle repolarization is caused by opening of sodium channels
E. Mucopolysaccharides inside the T-tubules bind chloride ions

131. A 30-year-old man has an ejection fraction of 0.25 and an end-systolic volume of 150 milliliters.
What is his end-diastolic volume?
A. 50 mL B. 100 mL C. 125 mL D. 200 mL E. 250 mL

132. In a resting adult, the typical ventricular ejection fraction has what value?
A. 20% B. 30% C. 40% D. 60% E. 80%

133. In which phase of the ventricular muscle action potential is the potassium permeability the highest?
A. 0 B. 1 C. 2 D. 3 E. 4
305
Cardiovascular System Physiology
134. A 60-year-old man’s ECG shows that he has an R-R interval of 1.5 seconds at rest. Which statement
best explains his condition?
A. He has fever
B. He has a normal heart rate
C. He has decreased parasympathetic stimulation of the S-A node
D. He is a trained athlete at rest
E. He has normal polarization of the S-A node

135. Which of the following is most likely to cause the heart to go into spastic contraction?
A. Increased body temperature D. Excess extracellular fluid potassium ions
B. Increased sympathetic activity E. Excess extracellular fluid calcium ions
C. Decreased extracellular fluid potassium ions

136. What happens at the end of ventricular isovolumic relaxation?


A. The A-V valves close C. The aortic valve closes E. The pulmonary valve closes
B. The aortic valve opens D. The mitral valve opens

137. Which event is associated with the first heart sound?


A. Closing of the aortic valve
B. Inrushing of blood into the ventricles during diastole
C. Beginning of diastole
D. Opening of the A-V valves
E. Closing of the A-V valves

138. Which condition will result in a dilated, flaccid heart?


A. Excess calcium ions in the blood
B. Excess potassium ions in the blood
C. Excess sodium ions in the blood
D. Increased sympathetic stimulation
E. Increased norepinephrine concentration in the blood

139. Which phase of the cardiac cycle follows immediately after the beginning of the QRS wave?
A. Isovolumic relaxation C. Atrial systole E. Isovolumic contraction
B. Ventricular ejection D. Diastasis

140. Which of the following structures will have the slowest rate of conduction of the cardiac action
potential?
A. Atrial muscle D. Purkinje fibers
B. Anterior internodal pathway E. Ventricular muscle
C. A-V bundle fibers

141. Sympathetic stimulation of the heart does which of the following?


A. Releases acetylcholine at the sympathetic endings
B. Decreases sinus nodal discharge rate
C. Decreases excitability of the heart
D. Releases norepinephrine at the sympathetic endings
E. Decreases cardiac contractility

142. Which condition at the A-V node will cause a decrease in heart rate?
A. Increased sodium permeability B. Decreased acetylcholine levels
306
Cardiovascular System Physiology
C. Increased norepinephrine levels E. Increased calcium permeability
D. Increased potassium permeability

143. Which statement best explains how sympathetic stimulation affects the heart?
A. The permeability of the S-A node to sodium decreases
B. The permeability of the A-V node to sodium decreases
C. The permeability of the S-A node to potassium increases
D. There is an increased rate of upward drift of the resting membrane potential of the S-A node
E. The permeability of the cardiac muscle to calcium Decreases

144. What is the membrane potential (threshold level) at which the S-A node discharges?
A. −40 millivolt C. −65 millivolt E. −105 millivolt
B. −55 millivolt D. −85 millivolt

145. Which condition at the S-A node will cause heart rate to decrease?
A. Increased norepinephrine level D. Increased potassium permeability
B. Increased sodium permeability E. Decreased acetylcholine level
C. Increased calcium permeability

146. In which phase of the ventricular muscle action potential is the sodium permeability the highest?
A. 0 B. 1 C. 2 D. 3 E. 4

147. If the Purkinje fibers, situated distal to the A-V junction, become the pacemaker of the heart, what
is the expected heart rate?
A. 30/min B. 50/min C. 60/min D. 70/min E. 80/min

148. A patient had an ECG at the local emergency department. The attending physician stated that the
patient had an A-V nodal rhythm. What is the likely heart rate?
A. 30/min B. 50/min C. 65/min D. 75/min E. 85/min

149. Which condition at the A-V node will cause a decrease in heart rate?
A. Increased sodium permeability D. Increased potassium permeability
B. Decreased acetylcholine level E. Increased calcium permeability
C. Increased norepinephrine level

150. When recording lead aVL on an ECG, which is the positive electrode?
A. Left arm C. Right leg E. Right arm + left leg
B. Left leg D. Left arm + left leg

151. When recording lead II on an ECG, the right arm is the negative electrode and the positive electrode
is the
A. Left arm C. Right leg E. Right arm + left leg
B. Left leg D. Left arm + left leg

152. Sympathetic stimulation of the heart normally causes which condition?


A. Acetylcholine release at the sympathetic endings
B. Decreased heart rate
C. Decreased rate of conduction of the cardiac impulse
D. Decreased force of contraction of the atria
E. Increased force of contraction of the ventricles
307
Cardiovascular System Physiology

For the following two questions


A 70-year-old woman had an ECG at her annual checkup. Use her lead II recording below to answer the
following two questions.

153. What is her heart rate in beats per minute?


A. 70 B. 78 C. 84 D. 94 E. 104

154. According to Einthoven’s law, if the QRS voltage in lead III is 0.4 millivolt, what is the QRS voltage in
lead I?
A. 0.05 millivolt C. 1.05 millivolts E. 2.05 millivolts
B. 0.50 millivolt D. 1.25 millivolts

155. What is the normal QT interval?


A. 0.03 second C. 0.16 second E. 0.35 second
B. 0.13 second D. 0.20 second

156. When recording lead II on an ECG, the negative electrode is the


A. Right arm C. Right leg E. Right arm + left leg
B. Left leg D. Left arm + left leg

157. When recording lead I on an ECG, the right arm is the negative electrode and the positive electrode
is the
A. Left arm C. Right leg E. Right arm + left leg
B. Left leg D. Left arm + left leg

158. A 65-year-old man had an ECG at a local emergency department after a biking accident. His weight
was 80 kilograms (176 pounds) and his aortic blood pressure was 160/90 mm Hg. The QRS voltage was
0.5 millivolt in lead I and 1.5 millivolts in lead III. What is the QRS voltage in lead II?
A. 0.5 millivolt C. 1.5 millivolts E. 2.5 millivolts
B. 1.0 millivolt D. 2.0 millivolts

159. A ventricular depolarization wave, when traveling −60 degrees in the frontal plane, will cause a
large negative deflection in which lead?
308
Cardiovascular System Physiology
A. aVR B. aVL C. Lead II D. Lead III E. aVF

For the following question


A 60-year-old woman had an ECG recorded at a local emergency department after an automobile
accident. Her weight was 70 kilograms (154 pounds), and her aortic blood pressure was 140/80 mm Hg.
Use this information and the figure below to answer the following three questions.

160. What is the mean electrical axis calculated from standard leads I, II, and III shown in the woman’s
ECG?
A. −90 degrees C. −12 degrees E. +170 degrees
B. −50 degrees D. +100 degrees

161. A ventricular depolarization wave, when traveling 60 degrees in the frontal plane, will cause a large
positive deflection in which of the following leads?
A. aVR B. aVL C. Lead I D. Lead II E. aVF

162. Mr. Smith had an ECG at a local hospital, but his records were lost. The ECG technician remembered
that the QRS deflection was large and positive in lead II and 0 in aVL. What is his mean electrical axis in
the frontal plane?
A. 90 degrees C. 0 degree E. −90 degrees
B. 60 degrees D. −60 degrees

163. A 70-year-old woman came to a hospital emergency department because she was experiencing
chest pain. Based on the ECG shown above, what is the likely diagnosis?
309
Cardiovascular System Physiology

A. Acute anterior infarction in the left ventricle of the heart


B. Acute anterior infarction in the right ventricle of the heart
C. Acute posterior infarction in the left ventricle of the heart
D. Acute posterior infarction in the right ventricle of the heart
E. Right ventricular hypertrophy

164. A 55-year-old man underwent an ECG at an annual physical, and his net deflection (R wave minus Q
or S wave) in standard limb lead I was −1.2 millivolts. Standard limb lead II has a net deflection of +1.2
millivolts. What is the mean electrical axis of his QRS?
A. −30 degrees C. +60 degrees E. −120 degrees
B. +30 degrees D. +120 degrees

165. A 30-year-old man had an ECG at his physician’s office, but his records were lost. The ECG technician
remembered that the QRS deflection was large and positive in lead aVF and 0 in lead I. What is the mean
electrical axis in the frontal plane?
A. 90 degrees C. 0 degree E. −90 degrees
B. 60 degrees D. −60 degrees

166. A 60-year-old woman tires easily. Her ECG shows a QRS complex that is positive in the aVF lead and
negative in standard limb lead I. What is a likely the cardiac axis of his heart?
A. Left axis deviation C. Normal cardiac axis
B. Right axis deviation stenosis

167. An 80-year-old man had an ECG taken at his local doctor’s office, and the diagnosis was atrial
fibrillation. Which condition is likely in someone with atrial fibrillation?
A. Ventricular fibrillation, which normally accompanies atrial fibrillation
B. Strong P waves on the ECG
C. An irregular and fast rate of ventricular contraction
D. A normal atrial “a” wave
E. A smaller atrial volume than normal

168. Circus movements in the ventricle can lead to ventricular fibrillation. Which condition in the
ventricular muscle will increase the tendency for circus movements?
A. Decreased refractory period
B. Low extracellular potassium concentration
C. Increased refractory period
D. Shorter conduction pathway (decreased ventricular volume)
310
Cardiovascular System Physiology
E. Increase in parasympathetic impulses to the heart

169. A 50-year-old man has a blood pressure of 140/85 mm Hg and weighs 90.7 kilograms (200 pounds).
He reports that he is not feeling well, his ECG has no P waves, he has a heart rate of 46 beats/min, and
the QRS complexes occur regularly. What is his likely condition?
A. First-degree heart block D. Sinoatrial heart block
B. Second-degree heart block E. Sinus bradycardia
C. Third-degree heart block

170. The following ECG tracing was obtained for a 60-year old man who weighs 99.8 kilograms (220
pounds). Standard lead II is shown below. What is his diagnosis?

A. A-V nodal rhythm C. Second-degree A-V heart block E. Atrial flutter


B. First-degree A-V heart block D. Third-degree A-V heart block

For the following question


A 55-year-old man had the below ECG tracing recorded at his doctor’s office at a routine physical
examination. Use this tracing to answer the following two questions.

171. What is his ventricular heart rate in beats/min?


A. 37.5 B. 60 C. 75 D. 100 E. 150

172. A 65-year-old man had the below ECG tracing recorded at his annual physical examination. What is
the likely diagnosis?

A. Atrial paroxysmal tachycardia D. Third-degree A-V block


B. First-degree A-V block E. Atrial flutter
C. Second-degree A-V block
311
Cardiovascular System Physiology
173. A 60-year-old woman has been diagnosed with atrial fibrillation. Which statement best describes
this condition?
A. The ventricular rate of contraction is 140 beats/min
B. The P waves of the ECG are pronounced
C. Ventricular contractions occur at regular intervals
D. The QRS waves are more pronounced than normal
E. The atria are smaller than normal

For the following two questions


A man had a myocardial infarction at age 55 years. He is now 63 years old. Use the standard limb lead I
tracing on his ECG shown below to answer the following two questions.

174. What is his heart rate?


A. 40 beats/min C. 75 beats/min E. 150 beats/min
B. 50 beats/min D. 100 beats/min

175. What is his current diagnosis?


A. Sinus tachycardia D. ST segment depression
B. First-degree heart block E. Third-degree heart block
C. Second-degree heart block

176. A 60-year-old woman sees her physician for her annual physical examination. The physician ordered
an ECG, which is shown below. What is the likely diagnosis?

A. First-degree A-V block D. Atrial paroxysmal tachycardia


B. Second-degree A-V block E. Atrial fibrillation
C. Third-degree A-V block

For the following two questions


An 80-year-old man went to his family physician for his annual checkup. Use the ECG tracing shown
below to answer the following two questions.
312
Cardiovascular System Physiology

177. What is his heart rate?


A. 105 B. 95 C. 85 D. 75 E. 37

178. What is the likely diagnosis? C. Second-degree A-V block


A. Left bundle branch block D. Electrical alternans
B. First-degree A-V block E. Complete A-V block

179. A healthy 60-year-old woman with a 10-year history of hypertension stands up from a supine
position. Which set of cardiovascular changes is most likely to occur in response to standing up from a
supine position?
Sympathetic nerve activity Parasympathetic nerve activity Heart rate

A. ↑ ↑ ↑
B. ↑ ↑ ↓
C. ↑ ↓ ↓
D. ↑ ↓ ↑
E. ↓ ↓ ↓
F. ↓ ↓ ↑
G. ↓ ↑ ↑
H. ↓ ↑ ↓

180. In an experimental study, administration of a drug decreases the diameter of arterioles in the
muscle bed of an animal subject. Which set of physiological changes would be expected to occur in
response to the decrease in diameter?
Vascular conductance Capillary filtration Blood flow
A. ↑ ↑ ↑
B. ↑ ↓ ↑
C. ↑ ↓ ↓
D. ↑ ↑ ↓
E. ↓ ↓ ↓
F. ↓ ↑ ↓
G. ↓ ↑ ↑
H. ↓ ↓ ↑

181. A 60-year-old woman has experienced dizziness for the past 6 months when getting out of bed in
the morning and when standing up. Her mean arterial pressure is 130/90 mm Hg while lying down and
95/60 while sitting. Which set of physiological changes would be expected in response to moving from a
supine to an upright position?
Parasympathetic nerve Activity Plasma Renin activity Sympathetic activity
A. ↑ ↑ ↑
B. ↑ ↓ ↑
313
Cardiovascular System Physiology
C. ↑ ↓ ↓
D. ↑ ↑ ↓
E. ↓ ↓ ↓
F. ↓ ↑ ↓
G. ↓ ↑ ↑
H. ↓ ↓ ↑

182. A healthy 27-year-old female medical student runs a 5K race. Which set of physiological changes is
most likely to occur in this woman’s skeletal muscles during the race?
Arteriole diameter Vascular conductance Tissue Oxygen concentration
A. ↑ ↑ ↑
B. ↑ ↑ ↓
C. ↑ ↓ ↓
D. ↑ ↓ ↑
E. ↓ ↓ ↓
F. ↓ ↓ ↑
G. ↓ ↑ ↑
H. ↓ ↑ ↓

183. Cognitive stimuli such as reading, problem solving, and talking all result in significant increases in
cerebral blood flow. Which set of changes in cerebral tissue concentrations is the most likely explanation
for the increase in cerebral blood flow?
Carbon Dioxide pH Adenosine
A. ↑ ↑ ↑
B. ↑ ↓ ↑
C. ↑ ↓ ↓
D. ↑ ↑ ↓
E. ↓ ↓ ↓
F. ↓ ↑ ↓
G. ↓ ↑ ↑
H. ↓ ↓ ↑

184. An increase in shear stress in a blood vessel results in which change?


A. Decreased endothelin production D. Increased renin production
B. Decreased cyclic GMP production E. Decreased prostacyclin production
C. Increased nitric oxide release

185. The diameter of a precapillary arteriole is increased in a muscle vascular bed. A decrease in which of
the following would be expected?
A. Capillary filtration rate C. Capillary blood flow E. Arteriolar resistance
B. Vascular conductance D. Capillary hydrostatic pressure

186. Under control conditions, flow through a blood vessel is 100 ml/min with a pressure gradient of 50
mm Hg. What would be the approximate flow through the vessel after increasing the vessel diameter by
50%, assuming that the pressure gradient is maintained at 100 mm Hg?
A. 100 ml/min B. 150 ml/min C. 300 ml/min D. 500 ml/min E. 700 ml/min
314
Cardiovascular System Physiology

187. A 60-year-old man visits his family practitioner for an annual examination. He has a mean blood
pressure of 130 mm Hg and a heart rate of 78 beats/min. His plasma cholesterol level is in the upper 25th
percentile, and he is diagnosed as having atherosclerosis. Which set of changes would be expected in this
patient?
Pulse pressure Arterial compliance Systolic pressure
A. ↑ ↑ ↑
B. ↑ ↓ ↑
C. ↑ ↓ ↓
D. ↑ ↑ ↓
E. ↓ ↓ ↓
F. ↓ ↑ ↓
G. ↓ ↑ ↑
H. ↓ ↓ ↑

188. In control conditions, flow through a blood vessel is 100 ml/min under a pressure gradient of 50
mm Hg. What would be the approximate flow through the vessel after increasing the vessel diameter to
four times normal, assuming that the pressure gradient was maintained at 50 mm Hg?
A. 300 ml/min B. 1600 ml/min C. 1000 ml/min D. 16,000 ml/min E. 25,600 ml/min

189. An increase in which of the following would be expected to decrease blood flow in a vessel?
A. Pressure gradient across the vessel D. Viscosity of the blood
B. Radius of the vessel E. Plasma sodium concentration
C. Plasma colloid osmotic pressure

190. Assuming that vessels A to D are the same length, which one has the greatest flow?
Pressure gradient Radius Viscosity
A. 100 1 10
B. 50 2 5
C. 25 4 2
D. 10 6 1

191. A 22-year-old man enters the hospital emergency department after severing a major artery in a
motorcycle accident. It is estimated that he has lost approximately 700 milliliters of blood. His blood
pressure is 90/55 mm Hg. Which set of changes would be expected in response to hemorrhage in this
man?
Heart rate Sympathetic nerve activity Total peripheral resistance
A. ↑ ↑ ↑
B. ↑ ↓ ↑
C. ↑ ↓ ↓
D. ↑ ↑ ↓
E. ↓ ↓ ↓
F. ↓ ↑ ↓
G. ↓ ↑ ↑
H. ↓ ↓ ↑

192. A healthy 28-year-old woman stands up from a supine position. Moving from a supine to a standing
position results in a transient decrease in arterial pressure that is detected by arterial baroreceptors
315
Cardiovascular System Physiology
located in the aortic arch and carotid sinuses. Which set of cardiovascular changes is most likely to occur
in response to activation of the baroreceptors?
Mean circulatory filling pressure Strength of cardiac contraction Sympathetic nerve activity
A. ↑ ↑ ↑
B. ↑ ↓ ↑
C. ↑ ↓ ↓
D. ↑ ↑ ↓
E. ↓ ↓ ↓
F. ↓ ↑ ↓
G. ↓ ↑ ↑
H. ↓ ↓ ↑

193. An ACE inhibitor is administered to a 65-year-old man with a 20-year history of hypertension. The
drug lowered his arterial pressure and increased his plasma levels of renin and bradykinin. Which
mechanism would best explain the decrease in arterial pressure?
A. Inhibition of angiotensin I
B. Decreased conversion of angiotensinogen to angiotensin I
C. Increased plasma levels of bradykinin
D. Increased plasma levels of renin
E. Decreased formation of angiotensin II

194. A 25-year-old man enters the hospital emergency department after severing a major artery during a
farm accident. It is estimated that the patient has lost approximately 800 milliliters of blood. His mean
blood pressure is 65 mm Hg, and his heart rate is elevated as a result of activation of the chemoreceptor
reflex. Which set of changes in plasma concentration would be expected to cause the greatest activation
of the chemoreceptor reflex?
Oxygen Carbon Dioxide Hydrogen
A. ↑ ↑ ↑
B. ↑ ↓ ↑
C. ↑ ↓ ↓
D. ↑ ↑ ↓
E. ↓ ↓ ↓
F. ↓ ↑ ↓
G. ↓ ↑ ↑
H. ↓ ↓ ↑

195. Under normal physiological conditions, blood flow to the skeletal muscles is determined mainly by
which of the following?
A. Sympathetic nerves D. Metabolic needs
B. Angiotensin II E. Capillary osmotic pressure
C. Vasopressin

196. A healthy 22-year-old female medical student has an exercise stress test at a local health club. An
increase in which of the following is most likely to occur in this woman’s skeletal muscles during
exercise?
A. Vascular conductance C. Carbon dioxide concentration E. All the above
B. Blood flow D. Arteriolar diameter

197. Which of the following segments of the circulatory system has the highest velocity of blood flow?
A. Aorta B. Arteries C. Capillaries D. Venules E. Veins
316
Cardiovascular System Physiology

198. Which blood vessel has the highest vascular resistance?


Blood flow (ml/min) Pressure gradient (mm Hg)
A. 1000 100
B. 1200 60
C. 1400 20
D. 1600 80
E. 1800 40

199. A balloon catheter is advanced from the superior vena cava into the heart and inflated to increase
atrial pressure by 5 mm Hg. An increase in which of the following would be expected to occur in response
to the elevated atrial pressure?
A. Atrial natriuretic peptide C. Aldosterone
B. Angiotensin II D. Renal sympathetic nerve activity

200. Which of the following vessels has the greatest total cross-sectional area in the circulatory system?
A. Aorta B. Small arteries C. Capillaries D. Venules E. Vena cava

201. An increase in atrial pressure results in which of the following?


A. Decrease in plasma atrial natriuretic peptide C. Increase in plasma aldosterone concentration
B. Increase in plasma angiotensin II concentration D. Increase in sodium excretion

202. Autoregulation of tissue blood flow in response to an increase in arterial pressure occurs as a result
of which of the following?
A. Decrease in vascular resistance
B. Initial decrease in vascular wall tension
C. Excess delivery of nutrients such as oxygen to the tissues
D. Decrease in tissue metabolism

203. Which component of the circulatory system contains the largest percentage of the total blood
volume?
A. Arteries C. Veins E. Heart
B. Capillaries D. Pulmonary circulation

204. An increase in which of the following would be expected to occur in a person 2 weeks after an
increase in sodium intake?
A. Angiotensin II C. Potassium excretion
B. Aldosterone D. Atrial natriuretic peptide

205. A decrease in the production of which of the following would most likely result in chronic
hypertension?
A. Aldosterone B. Thromboxane C. Angiotensin II D. Nitric oxide

206. Which of the following would be expected to occur during a Cushing reaction caused by brain
ischemia?
A. Increase in parasympathetic activity C. Decrease in heart rate
B. Decrease in arterial pressure D. Increase in sympathetic activity

207. Which part of the circulation has the highest compliance?


317
Cardiovascular System Physiology
A. Capillaries C. Veins E. Small arteries
B. Large arteries D. Aorta

208. A decrease in which of the following tends to increase pulse pressure?


A. Systolic pressure C. Arterial compliance E. Plasma volume
B. Stroke volume D. Venous return

209. Which set of physiological changes would be expected to occur in a person who stands up from a
supine position?
Venous hydrostatic pressure in legs Heart rate Renal blood flow
A. ↑ ↑ ↑
B. ↑ ↑ ↓
C. ↑ ↓ ↓
D. ↓ ↓ ↓
E. ↓ ↓ ↑
F. ↓ ↑ ↑

210. An increase in which of the following tends to increase capillary filtration rate?
A. Capillary wall hydraulic conductivity D. Interstitial hydrostatic pressure
B. Arteriolar resistance E. Plasma sodium concentration
C. Plasma colloid osmotic pressure

211. What would tend to increase a person’s pulse pressure?


A. Decreased stroke volume D. Patent ductus
B. Increased arterial compliance E. Decreased venous return
C. Hemorrhage

212. What would decrease venous hydrostatic pressure in the legs?


A. Increase in right atrial pressure C. Movement of leg muscles
B. Pregnancy D. Presence of ascitic fluid in the abdomen

213. What often occurs in decompensated heart failure?


A. Increased renal loss of sodium and water
B. Decreased mean systemic filling pressure
C. Increased norepinephrine in cardiac sympathetic nerves
D. Orthopnea
E. Weight loss

214. Which condition often occurs in progressive hemorrhagic shock?


A. Vasomotor center failure D. Decreased capillary permeability
B. Increased urine output E. Increased mean systemic filling pressure
C. Tissue alkalosis

215. A 50-year-old woman received an overdose of furosemide, and her arterial pressure decreased to
70/40. Her heart rate is 120, and her respiratory rate is 30/min. What therapy would you recommend?
A. Whole blood infusion D. Infusion of a sympathomimetic drug
B. Plasma infusion E. Administration of a glucocorticoid
C. Infusion of a balanced electrolyte solution
318
Cardiovascular System Physiology
216. A 30-year-old woman comes to a local emergency department with severe vomiting. She has pale
skin, tachycardia, an arterial pressure of 70/45, and trouble walking. What therapy do you recommend to
prevent shock?
A. Infusion of packed red blood cells D. Infusion of a sympathomimetic drug
B. Administration of an antihistamine E. Administration of a glucocorticoid
C. Infusion of a balanced electrolyte solution

217. A 30-year-old man is resting, and his sympathetic output increases to maximal values. Which set of
changes would be expected in response to this increased sympathetic output?
Resistance to venous return Mean systemic filling pressure Venous return
A. ↑ ↑ ↑
B. ↑ ↓ ↑
C. ↑ ↓ ↓
D. ↑ ↑ ↓
E. ↓ ↓ ↓
F. ↓ ↑ ↓
G. ↓ ↑ ↑
H. ↓ ↓ ↑

218. What is normally associated with an increased cardiac output?


A. Increased parasympathetic stimulation D. Polycythemia
B. Atrioventricular (A-V) fistula E. Severe aortic regurgitation
C. Decreased blood volume

219. Which condition would be expected to decrease mean systemic filling pressure?
A. Norepinephrine administration D. Increased venous compliance
B. Increased blood volume E. Skeletal muscle contraction
C. Increased sympathetic stimulation

220. Which statement about resistance to venous return (RVR) is true?


A. An increase in venous resistance causes an increase in RVR
B. Increased parasympathetic stimulation causes an increase in RVR
C. An increase in RVR causes an increase in venous return
D. Sympathetic inhibition causes an increase in RVR
E. Changes in arterial resistance have a greater effect on RVR than do equal changes in venous resistance

221. In which condition would you expect a decreased resistance to venous return?
A. Anemia D. Increased sympathetic output
B. Increased venous resistance E. Obstruction of veins
C. Increased arteriolar resistance

222. What is normally associated with an increased cardiac output?


A. Increased venous compliance D. Moderate anemia
B. Cardiac tamponade E. Severe aortic stenosis
C. Surgically opening the chest

223. In which condition would you normally expect to find a decreased cardiac output?
A. Hyperthyroidism D. Anemia
B. Beriberi E. Acute myocardial infarction
C. A-V fistula
319
Cardiovascular System Physiology

224. At the onset of exercise, what normally occurs?


A. Decreased cerebral blood flow D. Decreased mean systemic filling pressure
B. Increased venous constriction E. Increased parasympathetic impulses to the
C. Decreased coronary blood flow heart

225. If a person has been exercising for 1 hour, which organ will have the smallest decrease in blood
flow?
A. Brain C. Kidneys E. Pancreas
B. Intestines D. Non-exercising skeletal muscle

226. What increases the risk of adverse cardiac events?


A. Decreased blood levels of low-density lipoprotein (LDL)
B. Decreased blood levels of high-density lipoprotein (HDL)
C. Female gender
D. Moderate hypotension
E. Decreased blood triglycerides

227. Which vasoactive agent is usually the most important controller of coronary blood flow?
A. Adenosine B. Bradykinin C. Prostaglandins D. Carbon dioxide E. Potassium ions

228. Which statement about coronary blood flow is most accurate?


A. Normal resting coronary blood flow is 500 ml/min
B. The majority of flow occurs during systole
C. During systole, the percentage decrease in subendocardial flow is greater than the percentage decrease
in epicardial flow
D. Adenosine release will normally decrease coronary flow

229. Which condition normally causes arteriolar vasodilation during exercise?


A. Decreased plasma potassium ion concentration D. Increased plasma adenosine concentration
B. Increased histamine release E. Decreased plasma osmolality
C. Decreased plasma nitric oxide concentration

230. At the onset of exercise, the mass sympathetic nervous system strongly discharges. What would
you expect to occur?
A. Increased sympathetic impulses to the heart D. Reverse stress relaxation
B. Decreased coronary blood flow E. Venous dilation
C. Decreased cerebral blood flow

231. Which of the following blood vessels is responsible for transporting the majority of venous blood
flow that leaves the ventricular heart muscle?
A. Anterior cardiac veins C. Bronchial veins E. Thebesian veins
B. Coronary sinus D. Azygos vein

232. A 70-year-old man with a weight of 100 kilograms (220 Pounds) and a blood pressure of 160/90 has
been told by his doctor that he has angina caused by myocardial ischemia. Which treatment would be
beneficial to this man?
A. Increased dietary calcium C. A beta-1 receptor stimulator E. Nitroglycerin
B. Isometric exercise D. Angiotensin II infusion
320
Cardiovascular System Physiology
233. Which event normally occurs during exercise?
A. Arteriolar dilation in non-exercising muscle D. Decreased release of epinephrine by the adrenals
B. Decreased sympathetic output E. Decreased release of norepinephrine by the
C. Venoconstriction adrenals

234. What is the most frequent cause of decreased coronary blood flow in patients with ischemic heart
disease?
A. Increased adenosine release
B. Atherosclerosis
C. Coronary artery spasm
D. Increased sympathetic tone of the coronary arteries
E. Occlusion of the coronary sinus

235. What is one of the major causes of death after myocardial infarction?
A. Increased cardiac output C. Fibrillation of the heart
B. A decrease in pulmonary interstitial volume D. Increased cardiac contractility

236. A 60-year-old man had a heart attack 2 days ago, and his blood pressure has continued to decrease.
He is now in cardiogenic shock. Which therapy would be most beneficial?
A. Placing tourniquets on all four limbs D. Administering a blood volume expander
B. Administering a sympathetic inhibitor E. Increasing dietary sodium intake
C. Administering furosemide

237. Which intervention would be appropriate therapy for a patient in cardiogenic shock?
A. Placing tourniquets on the four limbs
B. Withdrawing a moderate amount of blood from the patient
C. Administering furosemide
D. Infusing a vasoconstrictor drug

238. The fourth heart sound is associated with which mechanism?


A. In-rushing of blood into the ventricles from atrial contraction
B. Closing of the A-V valves
C. Closing of the pulmonary valve
D. Opening of the A-V valves
E. In-rushing of blood into the ventricles in the early to middle part of diastole

239. Which of the following is associated with the first heart sound?
A. Inrushing of blood into the ventricles as a result of atrial contraction
B. Closing of the A-V valves
C. Closing of the pulmonary valve
D. Opening of the A-V valves
E. Inrushing of blood into the ventricles in the early to middle part of diastole

240. Which condition often occurs in a person with progressive hemorrhagic shock?
A. Increased capillary permeability D. Increased urine output
B. Stress relaxation of veins E. Increased mean systemic filling pressure
C. Tissue alkalosis

241. In which condition will administration of a sympathomimetic drug be the therapy of choice to
prevent shock?
321
Cardiovascular System Physiology
A. Spinal cord injury C. Hemorrhagic shock
B. Shock due to excessive vomiting D. Shock caused by excess diuretics

242. Which condition often occurs in compensated hemorrhagic shock? Assume systolic pressure is 48
mm Hg.
A. Decreased heart rate
B. Stress relaxation of veins
C. Decreased ADH release
D. Decreased absorption of interstitial fluid through the capillaries
E. Central nervous system (CNS) ischemic response

243. If a patient undergoing spinal anesthesia experiences a large decrease in arterial pressure and goes
into shock, what would be the therapy of choice?
A. Plasma infusion
B. Blood infusion
C. Saline solution infusion
D. Glucocorticoid infusion
E. Infusion of a sympathomimetic drug

244. A 25-year-old man who has been in a motorcycle wreck enters the emergency department. His
clothes are very bloody, and his arterial pressure is decreased to 70/40. His heart rate is 120, and his
respiratory rate is 30/min. Which therapy would the physician recommend?
A. Infusion of blood D. Infusion of a sympathomimetic drug
B. Infusion of plasma E. Administration of a glucocorticoid
C. Infusion of a balanced electrolyte solution

245. In which type of shock does cardiac output often increase?


A. Hemorrhagic shock B. Anaphylactic shock C. Septic shock D. Neurogenic shock

246. A 20-year-old man who has been hemorrhaging as a result of a gunshot wound enters a local
emergency department. He has pale skin, tachycardia, an arterial pressure of 60/40, and trouble walking.
Unfortunately, the blood bank is out of whole blood. Which therapy would the physician recommend to
prevent shock?
A. Administration of a glucocorticoid D. Infusion of a sympathomimetic drug
B. Administration of an antihistamine E. Infusion of plasma
C. Infusion of a balanced electrolyte solution

247. A 10-year-old girl in the hospital had an intestinal obstruction, and her arterial pressure decreased
to 70/40. Her heart rate is 120, and her respiratory rate is 30/min. Which therapy would the physician
recommend?
A. Infusion of blood D. Infusion of a sympathomimetic drug
B. Infusion of plasma E. Administration of a glucocorticoid
C. Infusion of a balanced electrolyte solution

248. What often occurs during progressive shock?


A. Patchy areas of necrosis in the liver D. Decreased release of hydrolases by lysosomes
B. Decreased tendency for blood to clot E. Decreased capillary permeability
C. Increased glucose metabolism
322
Cardiovascular System Physiology
249. Release of which substance causes vasodilation and increased capillary permeability during
anaphylactic shock?
A. Histamine C. Nitric oxide E. Adenosine
B. Bradykinin D. Atrial natriuretic factor

250. The heart continues to beat even after all nerves to it are sectioned. This property is called:
A. excitability B. conductivity C. automaticity D. contractility

251. Prepotentials are normally absent from:


A. P cells in the SA node C. Purkinje fibers
B. AV nodal cells D. working myocardial cells

252. What is the effect of vagal stimulation on the membrane potential of the SA node?
A. It increases inward calcium current C. It activates hyperpolarizing potassium current
B. It increases the slope of the prepotential D. It increases intracellular cAMP

253. Hypocalcemia is associated with QT prolongation because:


A. it is invariably associated with bundle branch block
B. it increases ventricular activation time
C. it lengthens the duration of ventricular repolarization
D. it accelerates opening of potassium channels

254. The AV node does not conduct more than:


A. 180 C. 280
B. 230 D. 330 impulses / minute

255. The maximum pressure rise in the ventricle occurs during:


A. Ejection C. Protodiastole
B. Isovolumetric contraction D. Diastasis

256. The cardiac output of a 50 year old man at rest is 6 L / min; mean HR is 75 BPM. Left ventricular
end-diastolic volume (LVEDV) is 120 ml. What is the mean ejection fraction?
A. 50 % B. 66 % C. 75 % D. 35 %

257. Venous return is reduced by all of the following EXCEPT:


A. Valsalva maneuver C. intravenous bolus of frusemide
B. positive end-expiratory pressure D. deep inspiration

258. Across which site in the circulation is the pressure drop maximum?
A. Arterioles B. Venules C. Capillaries D. Aortic valve

259. Which of the following structures are not innervated?


A. Arterioles C. AV anastomoses
B. Postcapillary venules D. Precapillary sphincters

260. At any time, the greatest fraction of blood is present in the:


A. heart B. arteries C. veins D. capillaries

261. What fraction of total blood volume is present in the capillaries at any given time?
A. 5% B. 20% C. 15% D. 1%
323
Cardiovascular System Physiology

262. The term “capacitance vessels” is applied to:


A. pulmonary capillaries C. shunts
B. thoroughfare channels D. veins and venules

263. Turbulence is almost always present when Reynolds number is more than:
A. 2000 B. 2500 C. 3000

264. Thin walled capillaries do not burst when intracapillary pressure is increased because:
A. they lack smooth muscle cells
B. the blood flow rate is less
C. they have a small radius
D. capillary hematocrit is less than whole-body hematocrit

265. That capillaries can withstand high internal pressures without bursting is explained using:
A. Bernoulli’s principle C. Poiseuille Hagen law
B. Laplace’s law D. Fahraeus-Lindquist effect

266. It follows from Laplace’s law that:


A. dilated hearts must generate greater tension in order to generate a given pressure
B. thin-walled capillaries do not burst because of a smaller radius
C. deficiency of surfactant would result in lung collapse
D. intravesical pressure does not increase when bladder is filled because an increase in radius is
accompanied by an increase in wall tension

267. Quantitatively, the most important means of increasing flow to an actively metabolizing tissue is:
A. increasing cardiac output C. increasing blood pressure
B. increasing peripheral resistance D. decreasing local vascular resistance

268. Which one of the following is not a vasodilator metabolite?


A. Adenosine C. Endothelin-1 E. ADP
B. Potassium ions D. Hydrogen ions

269. What is the chemical identity of endothelium-derived relaxing factor (EDRF)?


A. Nitrous oxide B. Nitric oxide C. Potassium D. Carbon monoxide

270. Which one of the following does not have vasodilator actions?
A. NO B. CO C. Potassium D. Angiotensin III

271. Blood flow to exercising skeletal muscle is increased by all of the following EXCEPT:
A. K+ B. norepinephrine C. ↑ in PCO2 in muscle D. adenosine
E. products of muscle metabolism
F. activation of beta-adrenergic receptors in skeletal muscle
G. activation of sympathetic cholinergic vasodilator system
H. nitric oxide

272. The most potent vasoconstrictor is:


A. endothelin 1 B. angiotensin II C. norepinephrine D. vasopressin

273. Intravenous injection of norepinephrine in the intact animal leads to:


324
Cardiovascular System Physiology
A. an increase in BP & HR C. an increase in HR & decrease in BP
B. a decrease in BP & HR D. an increase in BP & decrease in HR

274. Which of the following maneuvers evokes an increase in vagal discharge to the heart?
A. IV infusion of phenylephrine C. Pressure on the eyeball
B. Carotid massage D. Irrigation of the ear canals

275. On rising from the supine position:


A. Central blood volume increases D. Discharge from arterial baroreceptors decreases
B. Heart rate decreases E. Stroke volume decreases
C. Central venous pressure decreases

276. Marey’s law states that:


A. When BP increases, HR decreases C. When BP decreases, HR decreases
B. When BP decreases, HR increases D. When BP increases, HR increases

277. An increase in whole body oxygen demand is met chiefly by:


A. Increasing cardiac output
B. Increasing oxygen content of arterial blood
C. Increasing oxygen extraction from arterial blood
D. Increasing blood pressure

278. During severe exercise, a well-trained athlete may be able to achieve a cardiac output of:
A. 15 liters B. 25 liters C. 35 liters D. 45 liters

279. Vasomotor ischemia triggers an increase in sympathetic outflow increasing BP and thereby
facilitating restoration of cerebral blood flow. This is called:
A. Bainbridge reflex C. Head’s paradoxical reflex
B. The CNS ischemic pressor response D. Marey’s reflex

280. Heart rate is slowed by:


A. Deep inspiration C. Increased intracranial tension
B. Bainbridge reflex D. Carotid massage

281. Heart rate is slowed by all of the following EXCEPT:


A. deep expiration C. anger
B. fear D. IV infusion of phenylephrine

282. During the strain phase of the Valsalva maneuver (forced expiration with the glottis open and
maintaining an expiratory pressure of 40 mm Hg for 15 seconds):
A. venous return decreases D. heart rate increases
B. cardiac output decreases E. sympathetic outflow to blood vessels ↑
C. blood pressure decreases F. TPR gradually increases

283. The mechanism that regulates cerebral blood flow during cerebral compression is the:
A. CNS ischemic response C. Bezold-Jarisch reflex
B. Cushing’s reflex D. Bainbridge reflex

284. Activity in the noradrenergic nerves to the heart causes / has:


A. Coronary vasoconstriction B. Coronary vasodilation C. No effect on blood flow
325
Cardiovascular System Physiology

285. If the noradrenergic nerves to the heart are stimulated after giving a β-blocker, then what would be
the effect on coronary blood flow?
A. Coronary vasodilation C. No change
B. Coronary vasoconstriction D. Unpredictable

286. Of the following vascular beds, autoregulation of tissue blood flow is least prominent in the:
A. skin B. heart C. brain D. kidneys

287. When determining BP with a sphygmomanometer, a spuriously high value (of either SBP or DBP)
may be recorded when:
A. there is an auscultatory gap C. the person is obese
B. the cuff is smaller than preferable D. the arm is not placed at the level of the heart

288. on changing from the upright to the supine position:


A. Baroreceptor activity decreases
B. Leg vein pressure is reduced
C. The blood volume in the pulmonary circulation falls
D. Stroke volume increases
E. Renin activity increases

289. In the fetal circulation before birth, all the following are true EXCEPT:
A. The PO2 is higher in the ductus venosus than in the ductus arteriosus
B. Blood can go from the right atrium to the aorta without passing through the left atrium and ventricle
C. The PO2 in the aortic arch is higher than in the descending aorta
D. Blood flowing through the foramen ovale comes principally from the superior vena cava
E. Blood passes through the ductus arteriosus because of the high pulmonary vascular resistance

290. Pulse pressure increases with an increase in:


A. Stroke volume D. Systemic vascular resistance
B. Left ventricular end-diastolic volume E. Blood viscosity
C. Arterial partial pressure of oxygen

291. Acute untreated hemorrhagic shock in a patient will lead to:


A. An increase in physiological dead-space
B. An increase in the arterio-venous PCO2 difference
C. A fall in the pulmonary vascular volume
D. An increase in antidiuretic hormone secretion
E. An increase in plasma bicarbonate concentration

292. An increase in aldosterone secretion follows:


A. A sodium chloride load D. Trauma
B. A rise in blood volume C. An increase in production of angiotensin II
C. An increase in oral potassium absorption

293. stimulation of alpha-adrenergic receptors will cause:


A. Vasoconstriction of the coronary arteries
B. Increased tone in the bladder neck muscle D. Lipolysis
C. Increased platelet aggregation E. Bronchodilation
326
Cardiovascular System Physiology
294. resistance to laminar flow in a vessel is:
A. Proportional to wall thickness
B. Inversely proportional to the fourth power of the radius
C. Proportional to length
D. Independent of hematocrit
E. Proportional to the pressure drop

295. Autoregulatory mechanisms used in hypovolaemia include:


A. an increase in precapillary sphincter tone D. stimulation of the juxtaglomerular apparatus
B. an increase in capillary hydrostatic pressure E. An increase in angiotensin II
C. a decrease in baroreceptor activity

296. Myocardial contractility is increased by all the following EXCEPT:


A. Catecholamines
B. An increase in heart rate
C. An increase in fiber length
D. An increase in parasympathetic nervous system activity
E. Calcium ions

297. The carotid sinuses:


A. Have stretch receptors in their walls
B. Give afferent impulses via the glossopharyngeal nerve
C. Stimulate the respiratory center
D. Contain chemoreceptors
E. Stimulate the vasomotor center

298. capillary permeability is increased by:


A. Bradykinin B. Adrenaline C. Calcium D. Vasopressin E. Histamine

299. skeletal muscle blood flow:


A. Increases with noradrenaline D. Increases with rhythmic contraction
B. Receives 50% of the cardiac output at rest E. Increases with adrenaline
C. May cease during isometric contraction

300. Sinus arrhythmia:


A. Produces a lengthening of the P-R interval
B. Produces a lengthening of the R-R interval
C. Is maximal with breath holding
D. Is more marked during exercise
E. Is more marked in 70 year olds than in 20 year olds

301. The absolute refractory period for cardiac muscle is:


A. As long as the entire action potential
B. The period when no further action potential can be stimulated
C. Twice the length of the S-T interval
D. As long as the mechanical contraction
E. Shorter for pacemaker tissue than for normal cardiac muscle

302. In a normal resting subject, a bradycardia would be expected following:


A. An increase in carotid sinus pressure B. An increase in right atrial pressure
327
Cardiovascular System Physiology
C. Application of pressure to the eyeball E. Inspiration
D. The release of a Valsalva maneuver

303. In a healthy adult human heart the, all the following are true EXCEPT:
A. Left ventricular end-systolic volume is approximately 30 ml
B. First heart sound coincides with the onset of ventricular systole
C. Stroke volume is approximately 70 ml
D. Left ventricular end-diastolic pressure is about 50 mmhg
E. Second heart sound is caused by closure of the aortic and pulmonary valves

304. Changing position from standing to supine:


A. Increases stroke volume D. Decreases leg vein pressure
B. Increases baroreceptor activity E. Decreases the heart rate
C. Increases the pulmonary blood volume

305. Vagal stimulation produces:


A. A fall in heart rate D. Slowing of A-V conduction
B. An increase in atrial contractility E. A fall in stroke volume
C. An increase in ventricular contractility

306. In the healthy heart, an increase in stroke volume is seen with an increase in:
A. Left ventricular end-diastolic volume C. Heart rate
B. Aortic systolic pressure D. Left ventricular end-systolic pressure

307. The a-wave of the central venous pressure waveform:


A. Is caused by atrial contraction
B. Is not seen in atrial fibrillation
C. Is caused by atrial filling during ventricular contraction
D. Decreases with inspiration
E. Is followed by the v-wave

308. In the electrocardiogram, the:


A. P-R interval is equivalent to the A-V nodal conduction time
B. T-wave is equivalent to ventricular repolarization
C. Q-T interval is equivalent to the duration of ventricular contraction
D. U-wave represents sinoatrial node repolarization
E. Duration of a normal P-wave is 0.2 seconds

309. In the ECG:


A. P wave indicates arterial repolarization
B. QRS complex indicates ventricular depolarization
C. The normal QRS complex should be less than 0.12 seconds
D. T wave indicates ventricular repolarization
E. Increased PR interval occurs in second degree heart block

310. Regarding the total cerebral blood flow:


A. It accounts for about 15% of the total cardiac output
B. It is increased during intense mental activity
C. It shows significant increase if there is an increase in carbon dioxide concentration in the arterial blood
D. It shows significant decrease if the mean systemic blood pressure is reduced from 140 to 60 mmHg
328
Cardiovascular System Physiology
E. It is regulated by the cervical sympathetic nervous system

311. Vagal stimulation causes:


A. Delayed A-V conduction D. Increased ventricular contractility
B. A fall in heart rate E. Decreased stroke volume
C. Increased atrial contractility

312. What is the primary ionic basis of the prepotential in the SA node?
A. Ca influx through transient T Ca channels
B. Inwardly directed long-lasting Ca current
C. Outward Na current
D. Potassium efflux through leak channels

313. Normally, the impulse that excites the left ventricular myocardium originates in the:
A. SA node B. Purkinje system C. left bundle branch D. ventricle

314. Conduction speed is slowest in the:


A. SA node C. bundle of His E. Ventricular myocardium
B. atrial pathways D. Purkinje system

315. Activation of beta-adrenergic receptors in the heart is normally associated with which of the
following?
A. Decrease in the slope of phase 4 depolarization in SA nodal cells
B. Decrease in conduction speed through AV node
C. Inhibition of Ca induced Ca release following depolarization in ventricular myocytes
D. Accelerated sequestration of Ca in the sarcoplasmic reticulum by the Ca-ATPase
E. Reduction in the rate of rise in ventricular pressure during isovolumic contraction

316. Cardiac muscle cannot be tetanized because of:


A. Accommodation
B. Its slow rate of repolarization
C. Calcium influx during phase II
D. Voltage inactivation of Na channels at membrane potentials < 80 mV

317. The propagation of repolarization from the ventricular epicardium to endocardium is represented
by the
A. QRS complex B. QT interval C. T wave D. TP period

318. T wave inversion occurs when ventricular repolarization occurs from:


A. Endocardium to epicardium C. Apex to base of the heart
B. Epicardium to endocardium D. Base to apex of the heart

319. Which of the following is the shortest event in a cardiac cycle?


A. QRS interval B. ST segment C. ST interval D. RR interval

320. Electrical activity in which region of the heart does not result in deflections on the surface
electrocardiogram?
A. Atria D. Free wall of the right ventricle
B. Bundle of His E. Muscular portion of the ventricular septum
C. Free wall of the left ventricle
329
Cardiovascular System Physiology

321. In sinus rhythm, the last portion of the ventricle to depolarize is:
A. Interventricular septum from left to right
B. Anteroseptal region of the myocardium
C. Most of the myocardium from endocardium to epicardium
D. Posterobasal portion of left ventricle and the pulmonary conus

322. Stimulation of sympathetic nerves to the heart decreases:


A. Heart rate C. Speed of conduction
B. Force of cardiac contraction D. Refractory period

323. In which of the following leads are you most likely to observe ST segment elevation when there is
an acute and extensive infarction of the anterior and lateral wall of the heart?
A. Leads I, II and III C. Leads I, aVL, and V1-V6
B. Leads aVR, aVL and aVF D. Leads II, III and aVF

324. Clinical examination of a 45 year old man reveals splitting of the second heart sound as A2 followed
by P2 during deep inspiration, and the split was not apparent during expiration. S1 is normal in intensity,
and there is no cardiac murmur. BP is 130/80 mm Hg and pulse is 80 bpm and regular. Which of the
following is the most likely cause of this pattern of splitting of the second heart sound?
A. Aortic regurgitation C. Physiologic splitting of S2
B. Left bundle branch block D. Pulmonic stenosis

325. Normally, which of the following events in the cardiac cycle occurs at some point between S1 and
the following S2?
A. Onset of ventricular diastole D. The ‘a’ wave of the JVP
B. Atrial systole E. The ‘y’ descent in the JVP
C. Rapid ventricular filling

326. In postnatal life, steady state outputs of the right and left ventricle are matched in vivo by:
A. The Frank-Starling mechanism
B. Sympathetic influences on the SA node
C. Vagal influences on the SA node
D. Varying the afterload for each ventricle
E. Matching the tension generated by each ventricle

327. The mean systemic arterial pressure and the mean pulmonary artery pressure are respectively 90
and 15 mm Hg. What is the ratio of systemic and pulmonary vascular resistances?
A. Data inadequate B. 1 C. 6 D. 10

328. Which of the following is usually associated with turbulence in blood flow?
A. Reynolds number less than 2000 C. Decrease in density of blood
B. Decrease in blood flow velocity D. Increase in diameter of blood vessel

329. Select all correct answers. Which of the following statements is/are correct?
A. Pulse pressure is directly proportional to stroke volume
B. Pulse pressure is inversely proportional to compliance of large arteries
C. Reflected arterial pulse waves normally serve to increase coronary perfusion during diastole

330. Venous return is transiently increased during:


330
Cardiovascular System Physiology
A. Strain phase of Valsalva maneuver C. Intravenous bolus of frusemide
B. Positive end-expiratory pressure D. Deep inspiration

331. Which of the following is not a vasodilator metabolite?


A. Adenosine C. Endothelin-1 E. ADP
B. Potassium ions D. Hydrogen ions

332. Which of the following has a direct vasodilator effect on smooth muscle in arterioles in the
presence of endothelial dysfunction?
A. Acetylcholine C. Nitric oxide E. Thromboxane A2
B. Angiotensin II D. Norepinephrine

333. Regarding arterial baroreflex mechanism:


1. Baroreceptors are free nerve endings responsive to stretch
2. Herring’s nerve terminates in the nucleus tractus solitarius
3. An increase in discharge rate in afferents from the arterial baroreceptors reflexly excites vagal outflow to
the heart and inhibits sympathetic outflow to resistance vessels
4. The effect of a decrease in BP is a decrease indischarge frequency of arterial baroreceptor afferents

334. Intravenous injection of norepinephrine to a normotensive healthy adult human leads to:
A. An increase in BP & HR C. An increase in HR & decrease in BP
B. A decrease in BP & HR D. An increase in BP & decrease in HR

335. Loss of 500 ml of blood over 30 minutes will lead to:


A. An increase in HR, fall in BP C. A prominent increase in HR and BP
B. A fall in BP and HR D. A slight increase in HR

336. The ‘last ditch stand’ in defense of a falling blood pressure is the:
A. Arterial baroreflex mechanism C. CNS ischemic pressor response
B. Arterial chemoreflex mechanism D. Bainbridge reflex

337. In a healthy normotensive individual at rest, heart rate is typically increased by:
A. Deep expiration C. Anger
B. Fear D. IV infusion of phenylephrine

338. The acute effect of bilateral clamping of the carotid arteries proximal to the carotid sinuses is a/an:
A. Increase in heart rate and mean arterial pressure
B. Decrease in heart rate and mean arterial pressure
C. Decrease in heart rate and an increase in mean arterial pressure
D. Increase in heart rate and a decrease in mean arterial pressure

339. The acute effect of clamping internal carotid arteries proximal to the carotid sinuses in a dog is
most likely:
A. An increase in discharge rate in afferent fibers from the carotid sinus
B. A decrease in discharge rate of neurons in the rostral ventrolateral medulla
C. An increase in sympathetic outflow to the heart and resistance vessels
D. An increase in cardiac vagal outflow
331
Cardiovascular System Physiology
340. In hemorrhaged dogs with marked hypotension (mean arterial pressure < 50 mm Hg), denervation
of arterial chemoreceptors would:
A. Increase BP since chemoreceptors reduce sympathetic outflow
B. Produce no change in BP since chemoreceptors do not influence sympathetic outflow
C. Result in a further fall in BP since the arterial chemoreflex is sympathoexcitatory
D. Depend on whether arterial baroreceptors are reset or not

341. Normally, in an adult at rest, total blood flow to the brain is about:
A. 250 ml/min B. 500 ml/min C. 750 ml/min D. 1200 ml/min

342. Normally, the brain is perfused with what fraction of resting cardiac output?
A. 5% B. 10% C. 15% D. 20%

343. Which of the following is least likely to aggravate insult in an injured brain?
A. Hypercapnia B. Hypoxia C. Hypotension D. Hypothermia

344. Stimulation of sympathetic nerves to which of the following tissues invariably reduces blood flow to
that vascular bed?
A. Skin B. Heart C. Brain

345. An increase in discharge of noradrenergic nerves to the heart causes / has:


A. Coronary vasoconstriction B. Coronary vasodilation C. No effect on blood flow

346. Normally, in postnatal life, the left ventricle is more vulnerable to ischemia and infarction
compared to the right ventricle because:
A. diastolic pressure is comparable in both ventricles
B. Pulmonary vascular resistance is greater than systemic vascular resistance
C. The left ventricle pumps much more blood than the right ventricle
D. Left ventricular subendocardial perfusion is limited to ventricular diastole
E. Flow through the right ventricle is largely passive

347. The most frequent cause of diminished coronary blood flow is:
A. Mitral stenosis C. Atherosclerosis E. Increased venous return
B. Exercise D. Mass sympathetic discharge

348. Cardiac output is equal to:


A. Stroke volume (SV) X venous return (VR)
B. End diastolic volume (EDV) ‒ end systolic volume (ESV)
C. Stroke volume (SV) X heart rate (HR)
D. 300 ml / min
E. Cardiac index

349. Excitation of baroreceptors in arteries reflexly causes:


A. Arterial pressure to decrease D. Vasovagal syncope
B. Peripheral vasoconstriction E. Prolonged P-R interval
C. Increased cardiac output

350. Decreased oxygen availability to tissues causes:


A. Vascular muscle contraction C. Local vasodilation
B. Decrease release of adenosin D. Increase synthesis of ATP.
332
Cardiovascular System Physiology
E. Decrease in respiratory rate

351. During the cardiac cycle, the maximal right ventricular systolic pressure (mmHg) is:
A. 5 B. 8 C. 25 D. 80 E. 120

352. Which one of the following is the best index of preload?


A. Blood volume D. Left ventricular end-diastolic volume
B. Central venous pressure E. Left ventricular end-diastolic pressure
C. Pulmonary capillary wedge pressure

353. Which one of the following is the best index of afterload?


A. Left ventricular end-diastolic pressure D. Total peripheral resistance
B. Left ventricular mean systolic pressure E. Mean arterial blood pressure
C. Pulmonary capillary wedge pressure

354. Which of the following is consistent with the ECG tracing shown below?

A. Bradycardia C. Second-degree heart block E. Tachycardia


B. First-degree heart block D. Third-degree heart block

355. The mean electrical axis during the ventricular depolarization recorded in the three leads shown
below would be closest to

A. 30 degrees B. 90 degrees C. 150 degrees D. 210 degrees E. 270 degrees

356. Stroke volume is increased by


A. A decrease in venous compliance D. An increase in heart rate
B. An increase in afterload E. A decrease in coronary blood flow
C. A decrease in contractility

357. An increased preload would most likely be caused by an increase in


A. Arteriolar tone C. Myocardial contractility E. Capillary permeability
B. Venous tone D. Heart rate

358. Propagation of the action potential through the heart is fastest in the
A. SA node C. Av node E. Ventricular muscle
B. Atrial muscle D. Purkinje fibers

359. Closure of the aortic valve occurs at the onset of which phase of the cardiac cycle?
333
Cardiovascular System Physiology
A. Isovolumetric contraction C. Protodiastole E. Rapid filling
B. Rapid ejection D. Isovolumetric relaxation

360. If the QRS complex is positive in leads II and aVF and negative in lead III, the mean electrical axis
(MEA) is between
A. −30° and 0° C. +30° and +60° E. +90° and +120°
B. 0° and +30° D. +60° and +90°

361. Normal splitting of the second heart sound (S2) into two components is increased during inspiration
because
A. The closing of the aortic valve is delayed
B. The opening of the mitral valve is delayed
C. The closing of the pulmonic valve is delayed
D. The stroke volume of the left ventricle is increased
E. The heart rate is decreased

362. In a resting, healthy man, the ejection fraction is approximately?


A. 0.1 B. 0.2 C. 0.3 D. 0.6 E. 0.9

363. Increasing vagal stimulation of the heart will cause an increase in


A. Heart rate C. Ventricular contractility E. Cardiac output
B. Pr interval D. Ejection fraction

364. During exercise, there is an increase in a person’s


A. Stroke volume D. Pulmonary arterial resistance
B. Diastolic pressure E. Total peripheral resistance
C. Venous compliance

365. Phase-4 depolarization of SA nodal cells is caused by


A. An increase in the flow of sodium into the cell
B. A decrease in the flow of potassium out of the cell
C. An increase in the activity of the Na/Ca exchanger
D. A decrease in the flow of chloride out of the cell
E. A decrease in the activity of the Na-K pump

366. Blood pressure increases and heart rate decreases in response to


A. Exercise D. Increased intracranial pressure
B. Increased body temperature E. Hemorrhage
C. Exposure to high altitude

367. During exercise, cardiac output is augmented by


A. Sympathetic stimulation of resistance vessels D. Decreased mean systemic arterial pressure
B. Dilation of venous vessels E. Increased ventricular contractility
C. Decreased end-diastolic volume

368. Stroke volume can be decreased by


A. Increasing ventricular contractility D. Decreasing total peripheral resistance
B. Increasing heart rate E. Decreasing systemic blood pressure
C. Increasing central venous pressure
334
Cardiovascular System Physiology
369. The electrocardiogram is most effective in detecting a decrease in
A. Ventricular contractility C. Total peripheral resistance E. Coronary blood flow
B. Mean blood pressure D. Ejection fraction

370. Stroke volume can be increased by


A. Decreasing ventricular compliance D. Increasing heart rate
B. Increasing venous compliance E. Decreasing atrial contractility
C. Decreasing total peripheral resistance

371. Which one of the following correctly describes an event that normally occurs during the PR
interval?
A. The ventricle is contracting
B. The cardiac action potential passes through the AV node
C. There is no change in the voltage tracing on the ECG
D. The mitral and aortic valves are both closed
E. The second heart sound is heard

The phases of the ventricular muscle action potential are represented by the lettered points on the
diagram

372. At which point on the above ventricular action potential is membrane potential most dependent on
calcium permeability?
A. Point A B. Point B C. Point C D. Point D E. Point E

For the following two questions


Use the following ECG to answer the next two questions

373. During which interval on the above ECG does the aortic valve close?
A. A B. B C. C D. D E. E

374. During which interval on the ECG does the bundle of His depolarize?
A. A B. B C. C D. D E. E

375. The diagnosis of a first-degree heart block is made if


A. The PR interval of the ECG is increased
B. The P wave of the ECG is never followed by a QRS complex
C. The P wave of the ECG is sometimes followed by a QRS complex
D. The T wave of the ECG is inverted
335
Cardiovascular System Physiology
E. The ST segment of the ECG is elevated

For the following three questions


Use the following pressure-volume loop to answer the questions

376. The mitral valve opens at point


A. A B. B C. C D. D E. E

377. The second heart sound begins at point


A. A B. B C. C D. D E. E

378. Systole begins at point


A. A B. B C. C D. D E. E

For the following two questions


Use the following diagram to answer the questions

379. Ventricular filling begins at point


A. A B. B C. C D. D E. E

380. Closing of the mitral valve begins at point


A. A B. B C. C D. D E. E

For the following two questions


Use the following diagram of three Starling curves to answer the questions
336
Cardiovascular System Physiology

381. A mild hemorrhage will cause stroke volume to shift from point X to point
A. A B. B C. C D. D E. E

382. An increase in afterload and venous compliance can cause stroke volume to change from the point
marked X to point
A. A B. B C. C D. D E. E

383. The upstroke of the SA nodal action potential is produced by opening a channel that is
A. Primarily permeable to Na+ D. Primarily permeable to Cl−
2+
B. Primarily permeable to Ca E. Equally permeable to Na+ and K+
+
C. Primarily permeable to K

384. The channel responsible for the initiation of phase-4 depolarization in SA nodal cells
A. Is primarily permeable to Na+ D. Is primarily permeable to K+
B. Is opened by membrane depolarization E. Is closed by norepinephrine
C. Is opened by vagal nerve stimulation

385. Sympathetic stimulation of the heart results in


A. An increase in the activity of the SR calcium pump
B. An increase in the duration of systole
C. An increase in the duration of diastole
D. A decrease in the affinity of troponin for calcium
E. A decrease in the concentration of Ca2+ during systole

386. An exercise stress test to rule out ischemic heart disease is positive if
A. The systolic blood pressure rises D. A diastolic murmur is heard
B. The ST segment of the ECG is depressed E. The diastolic blood pressure decreases
C. The heart rate fails to increase

387. After a mild hemorrhage, compensatory responses initiated by the baroreceptor reflex keeps blood
pressure at or close to its normal value. Which one of the following values is less after compensation for
the hemorrhage than it was before the hemorrhage?
A. Venous compliance C. Ventricular contractility E. Coronary blood flow
B. Heart rate D. Total peripheral resistance

388. The constriction of a blood vessel to one-half of its resting diameter would increase its resistance to
blood flow by a factor of
A. 2 B. 4 C. 8 D. 12 E. 16

389. During aerobic exercise, blood flow remains relatively constant within
337
Cardiovascular System Physiology
A. The skin C. The brain E. The kidneys
B. The heart D. The skeletal muscles

390. Which of the following conditions causes pulse pressure to increase?


A. Tachycardia C. Hemorrhage E. Heart failure
B. Hypertension D. Aortic stenosis

391. Sudden standing evokes the baroreceptor reflex. Which one of the following will be greater after a
person suddenly stands up than it was before the person stood?
A. The end-diastolic volume C. The venous return E. The ejection fraction
B. The renal blood flow D. The pulse pressure

392. Central venous pressure is increased by


A. Decreasing blood volume D. Decreasing heart rate
B. Increasing venous compliance E. Decreasing plasma aldosterone concentration
C. Increasing total peripheral resistance

393. Capillary permeability is lowest within the


A. Kidneys B. Spleen C. Liver D. Brain E. Skin

394. Blood flow through an organ would be increased by decreasing


A. The diameter of the arterial vessels D. The diameter of the venous vessels
B. The number of open arterial vessels E. The hematocrit
C. The arterial pressure

395. A reduction in carotid sinus pressure would cause a decrease in


A. Heart rate C. Total peripheral resistance E. Cardiac output
B. Myocardial contractility D. Venous compliance

396. Which one of the following organs has the highest arteriovenous O2 difference under normal resting
conditions?
A. Brain C. Skeletal muscle E. Stomach
B. Heart D. Kidney

397. The percentage of the total cardiac output distributed to any single organ is most dependent on
A. The contractile state of the heart
B. The magnitude of mean blood pressure
C. The magnitude of diastolic pressure
D. The ratio of an organ’s vascular resistance to total peripheral resistance (TPR)
E. The magnitude of cardiac output

398. At which of the following sites does the blood flow lose the greatest amount of energy?
A. Mitral valve B. Large arteries C. Arterioles D. Capillaries E. Venules

399. Which one of the following decreases during aerobic exercise?


A. Circulating blood volume C. Skin temperature E. Mean blood pressure
B. Heart rate D. Cerebral blood flow

400. Which one of the following characteristics is most similar in the systemic and pulmonary
circulations?
338
Cardiovascular System Physiology
A. Stroke work C. Afterload E. Blood volume
B. Preload D. Peak systolic pressure

401. Which one of the following statements correctly describes the ductus arteriosus?
A. It prevents the flow of blood into the lungs of the fetus
B. It delivers oxygenated blood from the placenta to the left ventricle
C. It allows blood to flow from the aorta to the pulmonary artery in the fetus
D. Its resistance to blood flow decreases soon after birth
E. It is located in the septum between the left and right atrium

402. The greatest percentage of blood volume is found in the


A. Heart D. Capillaries
B. Aorta E. Venules and veins
C. Distributing arteries and arterioles

403. Correct statements about the increase in pulmonary blood flow during vigorous exercise include
which of the following?
A. The percentage of increase in flow is greater in the bases of the lungs than in the apices
B. The increase in flow is caused by a greater-than-fivefold increase in pulmonary arterial pressure
C. The increase in pulmonary blood flow is less than the increase in systemic blood flow
D. The increase in pulmonary blood flow is accommodated by dilation of pulmonary arterioles and
capillaries
E. The increase in pulmonary blood flow is caused by sympathetic nerve stimulation of the pulmonary
vasculature

404. Turbulence is more likely to occur in a blood vessel if


A. The velocity of blood within the vessel increases
B. The viscosity of blood within the vessel increases
C. The diameter of the vessel decreases
D. The density of the blood decreases
E. The length of the vessel increases

405. Which of the following will be lower than normal in a patient with an abnormally high intracranial
pressure?
A. Ventricular contractility C. Mean blood pressure E. Total peripheral resistance
B. Heart rate D. Stroke volume

406. After an episode of exercise training, the trained individual will have a
A. Decreased density of mitochondria in the trained muscles
B. Increased resting heart rate
C. Decreased maximum oxygen consumption
D. Increased stroke volume
E. Decreased extraction of oxygen by exercising muscles

407. Which one of the following increases during aerobic exercise?


A. Diastolic blood pressure D. Blood flow to the kidney
B. Cerebral vascular resistance E. Circulating blood volume
C. Mixed venous oxygen tension

408. Pulse pressure increases when


339
Cardiovascular System Physiology
A. Heart rate increases D. Aortic stenosis develops
B. Stroke volume decreases E. Mean arterial pressure increases
C. Aortic compliance increases

409. The distribution of blood among the various organs of the body is regulated by regulating the
resistance of the
A. Arteries C. Precapillary sphincters E. Veins
B. Arterioles D. Postcapillary venules

410. Which one of the following signs is observed in a patient who has lost a significant amount of
blood?
A. Respiratory acidosis C. Polyuria E. Low hematocrit
B. Dry skin D. Bradycardia

411. A patient is diagnosed with anaphylactic shock rather than hypovolumic shock because
A. Cardiac output is higher than normal
B. Ventricular contractility is greater than normal
C. Total peripheral resistance is greater than normal
D. Serum creatinine is elevated
E. Heart rate is greater than normal

412. Which one of the following will increase if massaging the neck stretches the carotid sinus
baroreceptor?
A. Total peripheral resistance D. Ventricular contractility
B. Right atrial pressure E. Vagal nerve activity
C. Venous tone

For the following three questions


Use the diagram below to answer the following three questions. The point marked control represents the
state of the cardiovascular system in the resting state.

413. An increase in total peripheral resistance and contractility is represented by a shift from the resting
state to point
A. A B. B C. C D. D E. E

414. Which one of the following would be consistent with a shift from the resting state (control) to point
B?
A. The person stood up suddenly D. The person’s afterload was increased
B. The person began exercising E. The person was given a positive inotropic drug
C. The person was given a transfusion
340
Cardiovascular System Physiology

415. Which one of the following would be consistent with a shift from the resting state (control) to point
E?
A. The person stood up suddenly D. The person’s afterload was increased
B. The person began exercising E. The person was given a positive inotropic drug
C. The person was given a transfusion

416. S.A. node is the pacemaker of heart because of


A. Location in the right atrium C. Natural leakiness to Na+
B. Neural control D. Natural leakiness to k+

417. Absolute refractory period in the heart


A. Corresponds to the duration of relaxation C. Shorter than refractory period in skeletal
B. Lasts till half of cardiac contraction muscle
D. Lasts till cardiac contraction

418. First heart sound occurs during the period of


A. Isometric relaxation C. Isovolumetric contraction
B. Isotonic relaxation D. Isovolumetric relaxation

419. Which of these vessels does not have sympathetic control


A. Cerebral B. Splanchnic C. Cardiac D. Cutaneous

420. Mary's law denotes relationship between heart and


A. Contractility and conductivity C. Rate and BP
B. Rate and contraction D. Contraction and BP

421. Which of the following conducting systems has the slowest conducting velocity
A. SAN B. Atrial muscle C. Purkinje fibers D. AVN

422. In heart, within physiological limits the force of contraction is directly proportional to the
A. Pacemaker activity C. Initial length of the cardiac muscle
B. A-V nodal delay D. Respiratory rate

423. The dicrotic notch on aortic pressure curve is caused by


A. Closure of mitral valve C. Closure of atrial valve
B. Closure of tricuspid valve D. Closure of pulmonary valve

424. The PR interval of ECG corresponds to


A. Ventricular repolarization
B. Ventricular repolarization
C. Atrial repolarization and conduction through AV node
D. Repolarization of AV node and bundle of His

425. Increased vagal tone causes


A. Hypertension C. Bradycardia
B. Tachycardia D. Increase in cardiac output

426. Which of the following is not increased during exercise


A. Stroke volume B. Total peripheral resistance
341
Cardiovascular System Physiology
C. Systolic BP D. Heart rate

427. When a pheochromocytoma suddenly discharges a large amount of epinephrine into the circulation
the patients heart rate would be expected to
A. Increase because epinephrine has a direct chronotropic effect on the heart
B. Increase because of increased parasympathetic discharge to the heart
C. Decrease because the increase in blood pressure stimulates the carotid and aortic baroreceptors
D. Decrease because of increased tonic parasympathetic discharge to heart

428. The 's' wave in ECG is below isoelectric line because of


A. Repolarization of ventricles
B. Change in direction of the impulse when the base of the ventricles are getting depolarized
C. Depolarization of apex of heart
D. Repolarization of apex of heart

429. The 'T' wave in ECG is above the isoelectric line because of
A. Depolarization of ventricles
B. Depolarization of bundle of his
C. Change in the direction of repolarization from the wave of depolarization of the ventricles
D. Repolarization of Purkinje fibers

430. The following are true about cardiac contraction EXCEPT:


A. The P wave initiates the atrial contraction
B. Atrial contribution to ventricular filling is most effective at fast heart rate
C. P wave is generated by atrial contraction
D. Fourth heart sound occurs during atrial contraction
E. Atrial natriuretic peptide (ANP) is released when the atria are distended by increases in blood volume

431. True statements about ECG include the following EXCEPT:


A. The P-R interval corresponds to the duration of atrial systole
B. The T-wave ends at the time of aortic valve closure
C. The ST segment represents repolarization of the ventricles
D. The QRS complex represents atrial repolarization
E. P-wave represents depolarization of the atrium

432. The following are true about micro-circulation EXCEPT:


A. Arterioles have no muscle
B. Capillaries have walls made up of a single layer of cells
C. Capillaries have no innervation
D. The capillaries contain 5% of the total blood volume at any one time
E. The capillaries contain gaps measuring 5-9um wide

433. Oxygen blood supply to the heart depends on:


A. Blood acidity C. Blood viscosity E. Hemoglobin concentration
B. Sympathetic tone D. Heart rate

434. The effect of noradrenaline on the heart includes:


A. Tachycardia
B. Increased duration of the cardiac action potential
C. Decreased potassium conductance of the membranes of pacemaker cells
342
Cardiovascular System Physiology
D. Increased strength of cardiac contraction
E. Increased concentration of free calcium in the sarcoplasm

435. The following are true about potassium EXCEPT:


A. Hypokalemia decreases the time of cardiac repolarization
B. Hyperkalemia decreases cardiac contraction
C. Hyperkalemia relaxes vascular smooth muscle
D. Hyperkalemia is seen in decreased aldosterone secretion
E. Fruits such as banana are high in potassium

436. The following are true about aldosterone EXCEPT:


A. It is secreted by the adrenal medulla
B. Its secretion is stimulated by decreased blood volume
C. It stimulates active reabsorption of sodium in the distal renal tubules
D. It causes increased secretion of potassium by the distal renal tubules
E. Abnormal secretion occurs in Conn's syndrome

437. Which of the following statements about the heart is false?


A. Contraction is initiated by a nerve impulse
B. Impulses are conducted from one muscle cell to the next
C. The heart contains a number of cells with an unstable membrane potential
D. The heart contains a number of cells with a stable membrane potential
E. The ventricles are unexcitable for most of the contraction period

438. The sinoatrial node is the pacemaker for the heart because the SA node
A. Is the most richly enervated structure in the heart
B. Is the only structure in the heart capable of generating action potentials
C. Has the highest rate of automatic discharge
D. Has the most stable transmembrane potential
E. Is the cardiac cell least sensitive to catecholamines

439. An independence of the P waves and the QRS complexes of the ECG indicates
A. An early repolarization of ventricular fibers
B. A failure of the AV node to conduct
C. A depression of the sinoatrial node
D. Slowing of conduction at the atrioventricular node
E. A conduction block in the left bundle branch

440. If the end-diastolic ventricular volumes are increased (within physiologic limits)
A. The stroke volume would be increased
B. Cardiac output would be decreased
C. Venous pressure would be decreased
D. The force of cardiac contraction would be decreased
E. The output of the left ventricle would exceed the output of the left ventricle

441. One of the major factors that controls the force of heart contraction is the
A. Initial length (preload) of cardiac muscle fibers
B. Number of intercalated disks in the cardiac muscle fibers
C. Number of gap junctions between cardiac muscle fibers
D. Degree of depolarization in the SA node
343
Cardiovascular System Physiology
E. Length of the His bundle

442. Positive inotropism refers to


A. Decreased stroke volume C. Repolarization of the SA node
B. Movement of blood elements related to a D. Increased contractility of the heart
change in chemical gradient E. Decreased end-diastolic volume

443. Increased pressure within the carotid sinus causes all of the following EXCEPT
A. A decrease in sympathetic tone to arterioles D. Vasodilatation of arterioles
B. A decrease in aortic pressure E. Atrial tachycardia
C. Reflex bradycardia

444. The effect of parasympathetic nervous stimulation on the heart is


A. Slowing of the heart D. Increased force of contraction
B. Increased activity of the SA node E. All of the above
C. Increased activity of the AV node

445. The P wave of a normal EKG is associated with


A. The dicrotic notch C. Atrial contraction E. None of the above
B. An increase in atrial pressure D. Ventricular contraction

446. Which of the following mechanisms is most important for maintaining an increased blood flow to
skeletal muscle during exercise?
A. An increase in aortic pressure
B. An increase in a-mediated adrenergic impulses
C. An increase in b-mediated adrenergic impulses
D. A vasoconstriction in the splanchnic and renal areas
E. A local vasodilatation secondary to the effect of local metabolites

447. Which of the following is most important to blood flow regulation at the local level?
A. Vessel-tissue transmural pressure difference D. Circulating neurotransmitters
B. Metabolic activity of the organ or tissue E. Cardiac output
C. Local neurotransmitters

448. Which of the following events is represented on the EKG?

A. SA node depolarization D. Depolarization of Bachmann's Bundle


B. AV node depolarization E. Atrial muscle depolarization
C. His Bundle depolarization

449. A decrease in heart rate, with stroke volume and peripheral resistance held constant, will cause a
decrease in all of the following, EXCEPT
A. Arterial diastolic pressure C. Cardiac output E. Mean arterial pressure
B. Arterial systolic pressure D. Arterial pulse pressure

450. An EKG is recorded with displays no P waves, but a normal appearing QRS and T wave. The
pacemaker most likely is located in the
A. SA node C. His bundle E. ventricular muscle
B. AV node D. Purkinje system
344
Cardiovascular System Physiology
451. Which of the following best describes conditions consistent with increased contractility?
A. Increased cardiac output for a given end-systolic volume
B. Increased cardiac output for a given end-diastolic volume
C. Decreased cardiac output for a given end-systolic volume
D. Decreased cardiac output for a given end-diastolic volume
E. The ventricles are unexcitable for most of the contraction period

452. The ventricles are depolarized or depolarizing during all of the following EKG waves EXCEPT
A. QRS complex B. QT interval C. ST segment D. PR interval

453. The tendency to form edema will be increased by


A. Arteriolar constriction C. Increased plasma protein concentration
B. Increased venous pressure D. Dehydration

454. Myocardial contractility is best correlated with the intracellular concentration of


A. Na+ B. K+ C. Ca++ D. Cl- E. Mg++

455. Cardiac output of the right heart is what percentage of the left heart?
A. 25% B. 50% C. 75% D. 100% E. 125%

456. The greatest pressure drop in the circulation occurs across the arterioles because
A. They have the greatest surface area
B. They have the greatest cross-sectional area
C. The velocity of blood flow through them is highest
D. The velocity of blood flow through them is lowest
E. They have the greatest resistance

457. Pulse pressure is


A. The highest pressure measured in the arteries D. Determined by the stroke volume
B. The lowest pressure measured in the arteries E. Decreased when arterial resistance decreases
C. The time averaged pressure in the arteries

458. During rapid acceleration, as a fighter pilot might experience, the pooling of blood in the
extremities will
A. Increase cardiac output C. Decrease arterial blood pressure
B. Increase venous return D. Decrease venous blood volume

459. The physiological function of the relatively slow conduction through the AV node is to allow
sufficient time for
A. Run-off of blood from the aorta to the arterioles
B. Venous return to the atria
C. Filing of the ventricles
D. Contraction of the ventricles
E. Repolarization of the ventricles

460. The anatomical basis of conduction in the heart is:


A. Nerve fibers C. Adherens junctions E. Desmosomal junctions
B. Intercalated discs D. Gap junctions

461. Atheromas first develop in the arterial:


345
Cardiovascular System Physiology
A. Endothelium C. T. media E. Elastic laminae
B. T. intima D. T. adventitia

462. Conduction velocity of the cardiac impulse is slowest in:


A. Atrial myocardial fibers C. Purkinje fibers E. His bundle fibers
B. Av nodal fibers D. Ventricular myocardial fibers

463. The principal determinants of mean arterial pressure are


A. Cardiac output and peripheral resistance D. Peripheral resistance and arterial volume
B. Arterial and venous volume E. Cardiac output and venous volume
C. Cardiac output and arterial volume

464. The increase in arterial pulse pressure usually observed in an elderly person with high blood
pressure (hypertension) is produced mainly by
A. An increased stroke volume D. An increased vagal activity
B. An increased heart rate E. A decreased arterial elasticity
C. Decreased cardiac output

465. The circulatory variable that is maintained relatively constant by the baroreceptor reflex is
A. Heart rate C. Peripheral resistance E. Mean arterial pressure
B. Stroke volume D. Velocity of blood flow

466. Cardiac output (in liters per minute) divided by the heart rate (in beats per minute) equals
A. Cardiac index C. Mean arterial pressure E. Blood velocity
B. Cardiac efficiency D. Stroke volume

467. The segment of the vascular bed responsible for local regulation of blood flow in most tissues is the
A. Distributing arteries C. Capillaries E. Arterioles
B. Large veins D. Venules

468. The most important function of the Starling mechanism in the heart is
A. To decrease fluid loss from the cardiac capillaries
B. To ensure that the ventricles operate at an optimum length
C. To couple the efficiency of muscle contraction to the heart rate
D. To match the output of one ventricle to that of the other
E. To ensure that right and left atrial pressures are equal

469. The initial depolarization phase of an action potential in cardiac ventricular muscle is caused by?
A. Increased Na+ permeability C. Decreased Ca++ permeability
+
B. Increased K permeability D. Inhibition of the Na+/K+ pump

470. The plateau phase of the action potential of cardiac ventricular muscle is maintained by a:
A. Balance between the rates of K+ efflux and Na+ influx across the cell membrane
B. Balance between the rates of K+ efflux and Ca++ influx across the cell membrane
C. Voltage dependent increase in K+ permeability
D. Voltage dependent increase in Na+ permeability

471. Which of the following changes in the ionic permeabilities of the membrane is associated with the
repolarization phase of the action potential of ventricular muscle?
346
Cardiovascular System Physiology
A. Increased Na+ permeability
B. Increased Ca++ permeability
C. Increased K+ permeability and decreased Ca++ permeability
D. Increased Na+ permeability and increased Ca++ permeability

472. Repolarization of the ventricular muscle cell at the end of an action potential occurs when the cell
membrane becomes:
A. More permeable to Na+ and more permeable to Ca++
B. Less permeable to Ca++ and more permeable to K+
C. More permeable to Ca++ and less permeable to K+
D. More permeable to Na+ and less permeable to K+

473. On a normal electrocardiograph a wave for repolarization of the atria is not seen. This is because:
A. The leads are not placed in a position where they can pick up atrial repolarization
B. Normally the atria do not repolarize to any significant extent
C. The wave for atrial repolarization is masked by the wave for ventricular depolarization
D. The wave is too small to be detected by external recording electrodes

474. The wave of depolarization is passing through the atrioventricular node at which phase of the
electrocardiogram?
A. During the P wave C. Between the QRS complex and the T wave
B. Between the P wave and the QRS complex D. Between the T wave and the succeeding P wave

475. Depolarization of the sinoatrial node of the heart occurs at which phase of the electrocardiogram?
A. During the P wave C. Between the QRS complex and the T wave
B. Between the P wave and the QRS complex D. Between the T wave and the succeeding P wave

476. At which phase of the electrocardiogram are the ventricular muscle cells in the plateau phase of
their action potentials?
A. During the P wave C. Between the QRS complex and the T wave
B. Between the P wave and the QRS complex D. Between the T wave and the succeeding P wave

477. Which of these parts of the electrocardiogram is caused by ventricular repolarization?


A. P wave C. The RS part of the QRS complex
B. The QR part of the QRS complex D. The T wave

478. The conduction delay caused by the atrioventricular node:


A. Ensures that the atria contract prior to ventricular systole
B. Ensures that the ventricles contract prior to atrial systole
C. Ensures that tetanic contraction of cardiac muscle is impossible
D. Is shortened by parasympathetic stimulation

479. If cardiac output remains constant, an increase in total peripheral resistance will have which of the
effects?
A. Increase mean arterial blood pressure C. Decrease end systolic blood volume
B. Decrease mean arterial blood pressure D. Decrease end diastolic blood volume

480. The cardiovascular center in the medulla of the brain stem is involved in the regulation of systemic
blood pressure by performing which of the following functions?
347
Cardiovascular System Physiology
A. Receiving afferent impulses from the carotid sinus
B. Sending impulses via sympathetic nerves to the heart
C. Sending impulses via parasympathetic nerves to the heart
D. The cardiovascular center performs all of the above functions

481. Which of the following decreases during exercise?


A. Stroke volume of the heart C. Heart rate
B. Venous return to the heart D. Peripheral resistance

482. Exercise causes all of the following EXCEPT:


A. Increased pulmonary blood flow C. Decreased blood flow to the kidneys
B. Increased peripheral resistance D. Increased coronary blood flow

483. The pulse pressure is:


A. The average pressure exerted throughout the cardiac cycle
B. The difference between the systolic and diastolic pressure
C. The maximum pressure exerted in the arteries
D. The change in pressure felt in an artery caused by closing of the aortic valve

484. Given that a person has a blood pressure of 126/78 mm Hg, that person’s mean arterial pressure is:
A. 48 mm Hg C. 102 mm Hg
B. 94 mm Hg D. 120 mm Hg

485. When measuring a person’s blood pressure no sound is heard when the pressure in the cuff is
greater than systolic blood pressure. This is because:
A. The air pressure within the cuff muffles the sound
B. No blood is flowing through the artery
C. Blood flow in the artery is laminar (none turbulent)
D. Arterial blood flow only occurs during diastole

486. The cause of the increase in systolic pressure with age is:
A. An increase in the force of contraction of the aging heart
B. Decreased compliance of systemic arteries
C. Weakening of the aortic valve
D. Decreased muscle mass which increases peripheral resistance

487. The large systemic arteries have which of the following functions?
A. Acting as a volume reservoir for blood
B. Acting as a pressure reservoir for blood
C. Regulating the volume of blood flowing to tissues
D. Exchanging nutrients and wastes with tissues

488. Systemic arterioles have which of the following functions?


A. Acting as a volume reservoir for blood C. Control the volume of blood flowing to tissues
B. Acting as a pressure reservoir for blood D. Exchange nutrients and wastes with tissues

489. Which of the following events will relax the smooth muscle of systemic arterioles?
A. Increased blood PCO2 C. Increased pH of blood (reduced acidity)
B. Increased blood PO2 D. Increased blood pressure within the arteriole
348
Cardiovascular System Physiology
490. Peripheral resistance is determined primarily by:
A. Stroke volume C. Capillary diameter
B. Venous capacity D. Arteriolar tone

491. The greatest pressure drop in the circulation occurs as blood passes through which of the following
vessels?
A. Arteries C. Capillaries
B. Arterioles D. Veins

492. In which of these blood vessels is the velocity of blood flow the lowest? (In this case blood flow
means how far a red blood cell will travel along the vessel in one second).
A. Arteries C. Capillaries
B. Arterioles D. Veins

493. Under normal resting conditions over 60% of the blood is found within the:
A. Systemic capillaries C. Chambers of the heart
B. Systemic veins D. Pulmonary circulation

494. Veins have each of these properties EXCEPT:


A. Contain most of the blood
B. Have the greatest resistance to flow of all vessels
C. Have valves which allow blood flow only towards the heart
D. Have smooth muscle in their wall which contracts in response to sympathetic stimulation

495. The volume of blood within the veins is primarily regulated by:
A. Activity of the sympathetic nervous system
B. Activity of the parasympathetic nervous system
C. Metabolic products which accumulate in blood
D. None of the above; their volume does not vary significantly

496. During which phase of the cardiac cycle are the atrioventricular, aortic semilunar and pulmonary
semilunar valves all open?
A. Ventricular filling phase C. Ventricular ejection phase
B. Isovolumetric ventricular relaxation phase D. These valves are never all open at the same time

497. The tricuspid valve:


A. Lies between the left atrium and left ventricle C. Is closed throughout ventricular systole
B. Closes when the second heart sound is heard D. Is opened by contraction of the papillary muscle

498. Closing of the atrioventricular valves during the cardiac cycle is caused by:
A. Contraction of the papillary muscles
B. Activity of the parasympathetic nervous system
C. The blood pressure in the ventricle falling below the blood pressure in the aorta
D. The blood pressure in the ventricles becoming greater than the blood pressure in the atria

499. The atrioventricular valves open because:


A. The papillary muscles contract
B. The sympathetic nervous system fires
C. Pressure in the atria is greater than pressure in the ventricles
D. Pressure in the ventricles is greater than pressure in the atria
349
Cardiovascular System Physiology

500. Consider the following phases of the cardiac cycle:


A. ventricular filling
B. isovolumetric ventricular contraction
C. ventricular ejection
D. isovolumetric ventricular relaxation
The aortic and pulmonary semilunar valves are open during which of the following phases?
A. A & B C. C only
B. B & C D. B, C, & D

501. The first heart sound is caused by which of the following events?
A. Closing of the atrioventricular valves
B. Opening of the aortic and pulmonary valves
C. Closing of the aortic and pulmonary valves
D. A shock wave passing through blood in the aorta and pulmonary artery

502. The second heart sound is caused by:


A. Closing of the atrioventricular valves C. Ejection of blood from the ventricles
B. Closing of the aortic & pulmonary valves D. Opening of the atrioventricular valves

503. Ejection of blood into the aorta will only occur under which of the following circumstances?
A. When ventricular pressure exceeds the aortic pressure
B. When ventricular pressure is increasing most rapidly
C. While the atrioventricular valve is open
D. Throughout all of ventricular systole

504. In which phase of the cardiac cycle is the blood pressure of the atrium greater than the blood
pressure of its corresponding ventricle?
A. Ventricular filling phase
B. Ventricular ejection phase
C. Isovolumetric ventricular relaxation phase
D. Ventricular pressures are always greater than atrial pressures

505. During the ventricular filling phase of the cardiac cycle, which of these statements correctly
describes the pressures in the various compartments?
A. Aortic > ventricular > atrial C. Aortic > atrial > ventricular
B. Atrial > ventricular > aortic D. Atrial > aortic > ventricular

506. The blood pressure in the aorta is greater than that in the pulmonary artery because the:
A. Blood flow through the systemic circulation is greater than the pulmonary circulation
B. Resistance of the systemic circulation is greater than the pulmonary circulation
C. Volume of blood within the systemic circulation is greater than the pulmonary circulation
D. Blood entering the systemic circulation is oxygenated

507. During the ventricular filling phase of the cardiac cycle:


A. The aortic and pulmonary valves are open
B. Pressure in the left ventricle is greater than pressure in the left atrium
C. Arterial blood pressure is falling
D. The electrocardiogram is between the QRS complex and the t wave
350
Cardiovascular System Physiology
508. During the ventricular ejection phase of the cardiac cycle:
A. Aortic pressure is greater than ventricular pressure
B. The atrioventricular valves are closed
C. The electrocardiogram is between the P wave and the QRS complex
D. Blood flow in the coronary circulation is at its maximal rate

509. In which of the following phases of the cardiac cycle does the ventricular pressure rise most
rapidly?
A. Ventricular filling phase C. Ventricular ejection phase
B. Isovolumetric ventricular contraction phase D. Isovolumetric ventricular relaxation phase

510. Which of the following will decrease cardiac output?


A. Sympathetic stimulation of the heart C. Increased venous return to the heart
B. Parasympathetic stimulation of the heart D. Increased plasma concentration of epinephrine

511. Cardiac muscle develops its greatest force of contraction?


A. At its length at the end diastolic volume
B. At a length greater than at its length at the end diastolic volume
C. At a length less than at its length at the end diastolic volume
D. Its force of contraction is independent of its length

512. Which of the following manipulations would increase the stroke volume of the heart?
A. Increased arterial blood pressure
B. Increased end diastolic volume
C. Hemorrhage of 500 ml of blood
D. The Valsalva maneuver (increasing pressure in the lungs while preventing expiration)

513. An increase in plasma epinephrine concentration increases the heart rate. This response is the
result of the action of epinephrine on which of the following cells?
A. Sinoatrial node cells C. Atrial muscle cells
B. Atrioventricular cells D. Ventricular muscle cells

514. Increased activity of the sympathetic nervous system increases the stroke volume of the heart. This
response is the result of the action of the sympathetic nervous system on which of the following cells?
A. Sinoatrial node cells C. Atrial muscle cells
B. Atrioventricular cells D. Ventricular muscle cells

515. The decrease in heart rate which occurs in response to parasympathetic nervous stimulation of the
heart is related to:
A. Reduced cytosolic Ca++ concentration
B. Increased cytosolic cyclic AMP concentration
C. Reduced rate of spontaneous depolarization of the sinoatrial node
D. Increased phosphorylation of myosin

516. Which of the following actions will slow the heart?


A. Cutting all the nerves leading to the heart C. Parasympathetic nervous stimulation
B. Sympathetic nervous stimulation D. An increase in plasma epinephrine concentration

517. Which of the following structures contains oxygenated blood?


351
Cardiovascular System Physiology
A. Right atrium C. Pulmonary artery
B. Left ventricle D. All of the above

518. In the cardiovascular system valves are NOT found between:


A. Between the vena cava (largest veins) and the atrium
B. Between the atrium and ventricle of the heart
C. Between the ventricle and the aorta
D. In the veins

519. Which of the following is the correct sequence of blood flow in the cardiovascular system?
A. Pulmonary artery, right atrium, left ventricle, aorta
B. Systemic vein, right atrium, right ventricle, pulmonary artery
C. Systemic vein, left atrium, left ventricle, pulmonary artery
D. Pulmonary vein, pulmonary artery, right atrium, right ventricle

520. Which of the following is the correct sequence of blood flow through the cardiovascular system?
A. Pulmonary vein, left atrium, left ventricle, aorta
B. Systemic vein, right ventricle, right atrium, pulmonary artery
C. Systemic vein, left atrium, left ventricle, pulmonary artery
D. Pulmonary vein, left ventricle, left atrium, aorta

521. The valve between the left atrium and the left ventricle is called the:
A. Semilunar valve C. Mitral valve
B. Sinoatrial valve D. Tricuspid valve

522. The wall of the left ventricle is thicker than the wall of the right ventricle because the left ventricle:
A. Pumps a greater volume of blood per minute
B. Pumps blood against a higher resistance
C. Pumps oxygenated blood
D. Has a higher density of Purkinje fibers

523. Closing of the atrioventricular valves is caused by:


A. Activity of the sympathetic nervous system
B. Activity of the conducting system of the heart
C. Contraction of the papillary muscles
D. Pressure in the ventricle exceeding that in the atrium

524. The atrioventricular valves open because:


A. The papillary muscles contract
B. The sympathetic nervous system fires
C. Pressure in the atria is greater than the pressure in the ventricles
D. Pressure in the ventricles is greater than pressure in the atria

525. The function of the aortic valve is to prevent backflow of blood into the:
A. Right atrium during ventricular diastole C. Right ventricle during ventricular systole
B. Left ventricle during ventricular diastole D. Left atrium during ventricular systole

526. Which of the following features is characteristic of cardiac muscle but not of skeletal muscle?
A. Its force of contraction is under the influence of the sympathetic nervous system
B. Contraction is triggered by an action potential passing across the cell surface
352
Cardiovascular System Physiology
C. Ca++ binds to troponin during the excitation contraction sequence
D. It appears striated under the electron microscope

527. Which of these statements about cardiac muscle is NOT true?


A. Cardiac muscle cells are electrically linked by intercalated discs
B. Purkinje fibers are modified cardiac muscle cells
C. Summation of cardiac contractions can occur
D. The refractory period of heart muscle is longer than that of skeletal muscle

528. Which of the following properties is characteristic of heart muscle but not of smooth muscle?
A. Innervated by the somatic nervous system
B. Consists of multinucleate cells
C. Has a striated appearance under the microscope
D. Ca++ binds to calmodulin during muscle contraction

529. Summation of contractions can occur in skeletal muscle but not with heart muscle because:
A. Heart muscle cells are connected by gap junctions
B. Heart muscle contraction time lasts longer than a skeletal muscle twitch
C. The refractory period of heart muscle is almost as long as its contraction time
D. Heart muscle sarcoplasmic reticulum has a more active Ca++ pump

530. Adjacent ventricular muscle cells are anchored together by:


A. Purkinje fibers C. Intercalated disks
B. Sarcomeres D. Dense bodies

531. The part of the heart with the fastest rate of spontaneous rhythmicity is:
A. Atrial muscle C. Atrioventricular node
B. Purkinje fibers D. Sinoatrial node

532. The conduction of the wave of depolarization through the heart could proceed by which of the
following routes?
A. Atrioventricular node, sinoatrial node, bundle of His, Purkinje fibers
B. Sinoatrial node, atrial muscle fibers, atrioventricular node, bundle of His
C. Sinoatrial node, right or left bundle branches, bundle of His, atrioventricular node
D. Sinoatrial node, Purkinje fibers, atrioventricular node, bundle of His

533. The sinoatrial node is the pacemaker for the heart because it:
A. Is located close to where the blood enters the heart
B. Has the fastest rate of spontaneous depolarization of any heart tissue
C. Is innervated by sympathetic nerves
D. Is the only nervous tissue in the heart

534. Which part of the conducting system of the heart has the slowest conduction rate?
A. Sinoatrial node C. Right and left bundle branches
B. Purkinje fibers D. Atrioventricular node

535. The right and left bundle branches of the interventricular septum consist of:
A. Sympathetic nervous tissue C. Somatic nervous tissue
B. Parasympathetic nervous tissue D. Modified cardiac muscle cells
353
Cardiovascular System Physiology
536. The conduction delay caused by the atrioventricular node:
A. Ensures that the atria contract prior to ventricular systole
B. Ensures that the ventricles contract prior to atrial systole
C. Ensures that tetanic contraction of cardiac muscle is impossible
D. Is shortened by parasympathetic stimulation

537. Which of these heart conditions would be the most likely to result in rapid death of your patient?
A. Ectopic foci of contraction C. Atrial fibrillation
B. Ventricular fibrillation D. All would result in rapid death

538. S.A. node is the pacemaker of heart because of


A. Location in the right atrium C. Natural leakiness to Na+
B. Neural control D. Natural leakiness to k+

539. Absolute refractory period in the heart


A. Corresponds to the duration of relaxation C. Shorter than refractory period in skeletal muscle
B. Lasts till half of cardiac contraction D. Lasts till cardiac contraction

540. First heart sound occurs during the period of


A. Isometric relaxation C. Isovolumetric contraction
B. Isotonic relaxation D. Isovolumetric relaxation

541. Which of these vessels does not have sympathetic control


A. Cerebral C. Cardiac
B. Splanchnic D. Cutaneous

542. Mary's law denotes relationship between heart and


A. Contractility and conductivity C. Rate and BP
B. Rate and contraction D. Contraction and BP

543. Which of the following conducting systems has the slowest conducting velocity
A. SAN C. Purkinje fibers
B. Atrial muscle D. AVN

544. In heart, within physiological limits the force of contraction is directly proportional to the
A. Pacemaker activity C. Initial length of the cardiac muscle
B. A-V nodal delay D. Respiratory rate

545. The dicrotic notch on aortic pressure curve is caused by


A. Closure of mitral valve C. Closure of atrial valve
B. Closure of tricuspid valve D. Closure of pulmonary valve

546. The PR interval of ECG corresponds to


A. Ventricular repolarization
B. Ventricular repolarization
C. Atrial repolarization and conduction through AV node
D. Repolarization of AV node and bundle of His

547. Increased vagal tone causes


A. Hypertension B. Tachycardia
354
Cardiovascular System Physiology
C. Bradycardia D. Increase in cardiac output

548. Which of the following is not increased during exercise


A. Stroke volume C. Systolic BP
B. Total peripheral resistance D. Heart rate

549. When a pheochromocytoma suddenly discharges a large amount of epinephrine into the circulation
the patients heart rate would be expected to
A. Increase because epinephrine has a direct chronotropic effect on the heart
B. Increase because of increased parasympathetic discharge to the heart
C. Decrease because the increase in blood pressure stimulates the carotid and aortic baroreceptors
D. Decrease because of increased tonic parasympathetic discharge to heart

550. The 's' wave in ECG is below isoelectric line because of


A. Repolarization of ventricles
B. Change in direction of the impulse when the base of the ventricles are getting depolarised
C. Depolarization of apex of heart
D. Repolarization of apex of heart

551. The 'T' wave in ECG is above the isoelectric line because of
A. Depolarization of ventricles
B. Depolarization of bundle of His
C. Change in the direction of repolarization from the wave of depolarization of the ventricles
D. Repolarization of Purkinje fibers

552. The blood vessels most responsible for regulation of blood pressure are:
A. Veins C. Venules E. Capillaries
B. Arterioles D. Arteries

553. Blood pressure can be determined from:


A. Heart rate x stroke volume
B. Heart rate x peripheral resistance
C. Cardiac output x stroke volume
D. Cardiac output x peripheral resistance
E. Cardiac output - stroke volume x peripheral resistance

554. Within normal limits, an increase in the end-diastolic volume of the right ventricle will cause:
A. An increase in the stroke volume of the right ventricle
B. An increase the stroke work of the ventricle
C. An increase in the mean ejection pressure of the ventricle
D. All of the above

555. Which of the following cardiac tissues has the most rapid conduction velocity?
A. The SA node D. Purkinje fibers
B. Atrial muscle E. Ventricular muscle fibers
C. The AV node

556. Ventricular repolarization in the human heart:


A. Begins in the same location and travels in the same direction as the depolarization wave
B. Is represented by the T-wave on the electrocardiogram
355
Cardiovascular System Physiology
C. Is unaltered if the ventricular activation sequence is changed (e.g., bundle branch block)
D. Occurs such that the T-wave vector has nearly the same direction as the QRS vector
E. Both B and D are correct

557. A 55-year old patient of yours wishes to begin a vigorous exercise program. As a precaution, you
assess his family history and, noting that he is at high risk for cardiac disease, perform a routine ECG. You
note in his ECG that not every P-wave is followed by a QRS and that the PR interval lengthens with each
successive beat until there is a dropped beat. The arrhythmia exhibited by this patient is:
A. First degree block C. Third degree block
B. Second degree block D. Mobitz II block

558. Ejection fraction is defined as __________. In a normal individual, the ejection fraction is typically
__________.
A. ESV / EDV; 60% C. ESV / EDV; 30%
B. SV / EDV; 30% D. SV / EDV; 60%

559. The first heart sound occurs when the __________ and __________ valves __________.
A. Mitral; Tricuspid; Close C. Mitral; Tricuspid; Open
B. Aortic; Pulmonic; Close D. Aortic; Pulmonic; Open

560. Put the following events in their proper order.


1. c-wave
2. a-wave
3. v-wave
4. P-wave
5. QRS complex
6. T-wave
A. 4, 2, 5, 1, 6, 3 B. 4, 1, 5, 2, 6, 3 C. 4, 2, 5, 3, 6, 1

561. During the isovolumetric contraction phase of the cardiac cycle, the A-V valves are __________ (C,
closed; O, open) and the aortic and pulmonic valves are __________ (C, closed; O, open).
A. O, open; O, open C. C, close; O, open
B. O, open; C, close D. C, close; C, close

562. Cardiovascular murmurs may be caused by:


A. Increased velocity of flow through a narrowed vessel
B. Significant irregularities in a vessel wall
C. Turbulent flow through a damaged cardiac valve
D. All of the above

563. Mechanisms to regulate a long-term increase in arterial pressure would cause:


A. A decrease in urine output rate and an increase in blood volume
B. No change in urine output rate
C. An increase in urine output rate and a decrease in blood volume
D. A decrease in total peripheral resistance

564. What type of blood vessel has the lowest mean blood velocity?
A. Arteries D. Venules
B. Arterioles E. Veins
C. Capillaries
356
Cardiovascular System Physiology

565. Most of the total blood volume in the body is contained in:
A. Systemic arteries C. Capillaries
B. Systemic veins D. Pulmonary circulation

566. The bicuspid valve is located between


A. Left atrium and left ventricle C. Right atrium and right ventricle
B. Left ventricle and pulmonary trunk D. Right ventricle and aorta

567. The location of the secondary pacemaker in the human heart is in the:
A. Atrioventricular node C. Left ventricle
B. Purkinje fibres D. Sinoatrial node

568. When the blood reaches the _____, this is where exchange of gases and nutrients takes place
A. Arteries C. Arterioles
B. Venules D. Capillaries

569. Diastole is when the ventricle


A. Relax and fill with blood C. Contract and eject blood into the aorta
B. Pressure increase sharply D. Reach a pressure of 120 mmHg

570. In the normal heart The majority of blood enters the left ventricle:
A. During early diastole C. After the aortic valve opens
B. During isovolumic relaxation D. As a result of atrial contraction

571. The Frank Starling law of the heart states that:


A. Cardiac output is controlled entirely by the activity of the heart
B. Blood entering the atria is pumped immediately into the ventricles
C. The heart can pump a certain amount of blood and no more
D. Heart rate controls cardiac output during exercise
E. Within physiological limits, the heart pumps all the blood that comes to it

572. Which of the following factors is most important for cardiac output control?
A. Metabolic needs of the tissues D. Venous resistance
B. Right atrial pressure E. Sympathetic nervous system
C. Mean systemic pressure

573. An increase in sympathetic stimulation of the peripheral vasculature will most likely:
A. Decrease venous resistance C. Decrease arterial blood low
B. Increase venous compliance D. Decrease arterial resistance

574. Which of the following substances causes vasoconstriction?


A. Adenosine D. Histamine
B. Angiotensin E. Hydrogen ion
C. Carbon dioxide

575. Ventricular filling is characterized by the following EXCEPT:


A. Begins after opening of atrioventricular valves
B. Give rise to a third heart sound in some healthy people
C. Ends before the start of isovolumetric contraction
357
Cardiovascular System Physiology
D depends mainly on atrial systole

576. Sinoatrial node cells is characterized by the following EXCEPT:


A. Found in the right atrial wall
B. Innervated by the vague
C. Able to generate impulse when completely denervated
D. Stable regarding their membrane potential

577. The natural rate of rhythmic discharge is greatest in which part of the heart?
A. Ventricular myocardium D. Purkinje fibers
B. Atria E. A-V node
C. Sinoatrial node

578. A decrease in the velocity of impulse conduction through the A-V node will usually cause:
A. The PR interval to increase D. Increased heart rate
B. The PR interval to decrease E. Atrial fibrillation
C. Disappearance of the T wave

579. The T wave of the normal electrocardiogram is caused by:


A. Ventricular depolarization C. Atrial repolarization
B. Ventricular repolarization D. Atrial depolarization

580. A person with a PR interval of 0.23 second indicates:


A. Atrial flutter C. Incomplete heart block
B. Nothing unusual D. Paroxysmal tachycardia

581. Blood flow through the coronary arteries is markedly attenuated during:
A. Systole C. Exercise
B. Diastole D. Isovolumic relaxation

582. Sympathetic simulation of which vessels causes the greatest increase in total peripheral resistance?
A. Veins D. Arterioles
B. Venules E. Arteries
C. Capillaries

583. Which of the following types of shock is often associated with an elevated cardiac output?
A. Septic shock D. Hemorrhagic shock
B. Neurogenic shock E. Anaphylactic shock
C. Traumatic shock

584. Bulging veins in the neck most likely result from:


A. Hemolytic anemia D. Congestive heart failure
B. Pulmonary edema E. Intermittent claudication
C. Systemic hypertension

585. When measuring blood pressure by auscultatory method the following are true EXCEPT:
A. The cuff pressure at which the first sound are heard indicates systolic pressure
B. Systolic pressure estimated tends to be higher than those made by the palpatory method
C. Wider cuffs are required for larger arm
D. The sound that are heard are generated in the heart
358
Cardiovascular System Physiology

586. Sympathetic drive to the heart increases in the following conditions EXCEPT:
A. Exercise C. Sleep
B. Hypotension D. Hemorrhage

587. The strength of contraction of left ventricular muscle increases in the following EXCEPT:
A. End diastolic ventricular filling pressure rise C. Blood calcium level rise
B. Serum potassium level rise D. Strenuous exercise is under taken

588. In electrocardiogram the following are true EXCEPT:


A. QRS complex corresponds to rapid depolarization
B. Atrial repolarization is overlapped by QRS complex
C. PR interval corresponds with atrial depolarization
D. R-R Interval normally constant during the respiratory cycle

589. Wide QRS complex in EGG recording could be seen in any of the following EXCEPT:
A. Bundle branch block C. Ventricular ectopic
B. Sinus bradycardia D. Paroxysmal ventricular tachycardia

590. The following formulae are true EXCEPT:


A. Cardiac output = stroke volume x heart rate
B. Blood pressure = cardiac output x peripheral resistance
C. Pulse pressure = systolic pressure - diastolic pressure
D. Mean arterial pressure = diastolic pressure + 3 (systolic pressure - diastolic pressure)

591. In normal EGG recording, the following are correctly paired EXCEPT:
A. P wave : atrial depolarization C. QRS complex : ventricular depolarization
B. PR segment : atrial repolarization D. T wave : ventricular repolarization

592. Regarding the QRS complex, which of the following is false?:


A. It always precedes ventricular contraction
B. It represents ventricular contraction
C. It is caused mainly by current flow through fast Na channels
D. It represents ventricular depolarization and may be prolonged in disturbances of ventricular conduction
pathways

593. The phase of the fast-response action potential that is associated with the voltage-gated calcium
current is:
A. Phase 0. C. Phase 2.
B. Phase 1. D. Phase 3 and 4.

594. Which heart sound is heared at the end of systole?:


A.S1. C. S3.
B. S2. D. S4 and S5.

595. During moderate exercise, all of the following will increase EXCEPT:
A. Heart rate and strock volume C. Mean arterial blood pressure
B. Arteriovenous oxygen difference D. Systemic vascular resistance

596. Regarding the rate of blood flow, which of the following statement is false:
359
Cardiovascular System Physiology
A. It is inversely proportional to vessel length C. It is inversely proportional to blood viscosity
B. It increases with increased pressure difference D. It is inversely proportional to vessel radius

597. Regarding the rate of blood flow, which of the following statement is false:
A. It is inversely proportional to vessel length C. It is inversely proportional to blood viscosity
B. It increases with increased pressure difference D. It is inversely proportional to vessel radius

598. Standing up quickly is expected to produce:


A. Decreased activity in the afferent division of cranial nerve IX.
B. Decreased activity of the vasomotor center
C. Increased vagal stimulation of the heart
D. Increased venous compliance

599. During moderate exercise, all of the following will increase EXCEPT:
A. Heart rate and mean arterial blood pressure C. Arteriovenous oxygen difference
B. Stroke volume D. Systemic vascular resistance

600. The velocity of impulse transmission is slowest in the:


A. A-V node C. Atria
B. Sinoatrial node D. Purkinje system

601. In the normal heart The majority of blood enters the left ventricle:
A. During early diastole C. After the aortic valve opens
B. During isovolumic relaxation D. As a result of atrial contraction

602. Blood pressure is regulated by:


A.Chemoreceptors C. Baroreceptors
B. Osmoreceptors D. All of above

603. Peripheral resistance lies mainly in the:


A. Capillaries C. Arteries
B. Arterioles D. Venules

604. Blood pressure is increased if the:


A. Peripheral resistance is decreased C. Viscosity of blood is decreased
B. Viscosity of blood is increased D. Arteriolar diameter is increased

605. Diastolic blood pressure is due to:


A. Ventricular systole C. Ventricular diastole
B. Elastic recoil of arterial wall D. None of above

606. Interatrial septum is present between:


A. Right atrium and right ventricle C. Right atrium and left atrium
B. Right ventricle and left ventricle D. Left atrium and left ventricle

607. The outer surface of the heart is enclosed by:


A. Pericardium C. Both
B. Endocardium D. None of above
360
Cardiovascular System Physiology
608. Duration of cardiac cycle is about:
A. 0.7 second C. 0.8 second
B. 0.5 second D. 0.9 second

609. Second heart sound is due to:


A. Closure of mitral and tricuspid valves C. Closure of aortic and pulmonary valves
B. Opening of aortic and pulmonary valves D. None of above

610. The stroke volume of the heart is the volume of blood that:
A. Enters each ventricle per beat C. Leaves each ventricle per beat
B. Enters both ventricles per beat D. None of above

611. Tricuspid valve is present between:


A. Right ventricle and pulmonary artery C. Right atrium and right ventricle
B. Left ventricle and aorta D. Left atrium and left ventricle

612. During sleep, the heart rate is decreased because:


A. Vagal activity is decreased C. Sympathetic activity is increased
B. Vagal activity is increased D. A and C.

613. When the baroreceptor activity is increased the heart rate is:
A. Increased C. Remains normal
B. Decreased D. None of above

614. Pulse pressure at rest is between:


A. 60-70 mmHg C. 30-60 mm Hg
B. 40-80 mm Hg D. 80-90 mm Hg

615. Cardiac activity means:


A. Electrical and mechanical activity of the heart C. Mechanical activity only
B. Electrical activity only D. None of above

616. The highest speed of conduction of cardiac excitation is in the:


A. Bundle of His C. A- V node
B. Purkinje D. None of the above

617. The duration of QRS complex in normal ECG is:


A. 0.09 second C. 0.20 second
B. 0.16 second D. None of the above

618. Cardiac muscle under normal conditions:


A. Has a duration of action potential of 0.01 second
B. Has an unstable resting membrane potential
C. Is intrinsically less permeable to sodium than the skeletal muscle
D. None of above

619. The last part of the heart to be activated is:


A. Interventricular septum from left to right
B. Anteroseptal region of the ventricular myocardium
C. Major portion of the ventricular myocardium from endocardial surfaces to epicardial surfaces
361
Cardiovascular System Physiology
D. None of the above

620. The blood flow in the blood vessels is:


A. Normally laminar C. Highest in the center of the vessel
B. Silent flow D. All of the above

621. During isometric ventricular contraction:


A. Mitral and tricuspid valves open C. Interventricular pressure declines rapidly
B. Mitral and tricuspid valves close D. None of the above

622. During late diastole:


A. The mitral and tricuspid valves are opened C. The pressure in the ventricular is very high
B. The aortic and pulmonary valves are opened D. None of the above

623. The slowest velocity of the blood flow is:


A. In the aorta C. In the large veins
B. In the medium size arteries D. In the capillaries

624. In accordance with law of laplace:


A. The aorta is more vulnerable to rupture than the capillaries
B. The capillaries are more vulnerable to rupture than the aorta
C. The normal size heart is more vulnerable to rupture than the dilated heart
D. None of the above

625. A P-R interval of 0.16 seconds would be considered as:


A. Normal C. Second degree heart block
B. First degree heart block D. Third degree heart block

626. The cardiovascular segment containing the highest percentage of circulating blood volume is the:
A. Capillaries C. Arterioles
B. Heart and arteries D. Venous system

627. Compared with other segments of systemic circulation, arterioles have the greatest:
A. Pulse pressure C. Resistance component
B. Mean pressure D. Distensibility

628. Coronary blood flow has been measured at rest to be:


A. 250 ml/min C. 15% of the cardiac output
B. 500 ml/min D. 25% of the cardiac output

629. An irregular, rapid rate with normal QRS complex and no P waves suggests:
A. Sinus arrhythmia
B. Second – degree heart block
C. Paroxysmal tachycardia with a ventricular pacemaker
D. Atrial fibrillation

630. The delay between the P wave and Q wave in the normal electrocardiogram is primarily caused by:
A. A slow transmission through the AV node and junctional fibers
B. Delay at the intermodal pathways
C. Circus movement
362
Cardiovascular System Physiology
D. The slow rate of conduction in atrial heart muscle

631. A stronger than normal heart might be observed during:


A. Sympathetic stimulation C. Stokes – Adams syndrome
B. Myocardial ischemia D. Atrial fibrillation

632. The Purkinje fibers:


A. Are myelinated axons
B. Have a conduction velocity about four times that seen in heart muscle
C. Have action potentials about a tenth as long as those in heart muscle
D. All of the above

633. Which of the following statement is incorrect?


A. Blood flow velocity in the capillaries is greater than the larg veins
B. Total surface area of the capillaries is much greater than of the large veins
C. Reduced oxygen tension in the tissue tend to relax precapillary sphincters
D. Increased sympathetic nerve stimulation tends to constrict the small arterioles

634. The peripheral vasculature under the least control of the sympathetic nervous system are the:
A. Arteries C. Capillaries E. Veins
B. Arterioles D. Venules

635. Baroreceptor impulses:


A. Inhibit the vagal center
B. Increase in number with decreased carotid arterial pressure
C. Result in increased heart rate
D. Excite the sympathetic vasoconstrictor center
E. None of the above

636. When a person moves form a supine position to a standing position, which of the following
compensatory changes occurs?
A. Decreased heart rate D. Decreased cardiac output
B. Increased contractility E. Increased PR intervals
C. Decreased total peripheral

637. At which site is blood pressure the highest?


A. Aorta C. Pulmonary artery E. Renal artery
B. Central vein D. Right atrium F. Renal vein

638. A person`s electrocardiogram (ECG) has no P wave, but has a normal QRS complex and a normal T
wave. Therefore, his pacemaker is located in the:
A. Sinoatrial (SA) node D. Purkinje system
B. Atrioventricular (AV) node E. Ventricular muscle
C. Bundle of His

639. If the ejection fraction increases, there will be a decrease in:


A. Cardiac output D. Heart rate G. Systolic pressure
B. End-diastolic volume E. Pulse pressure
C. End-systolic volume F. Stroke volume
363
Cardiovascular System Physiology
640. An increase in contractility is demonstrated on a Frank-Starling relationship by:
A. Increased cardiac output for a given end-diastolic volume
B. Increased cardiac output for a given end-systolic volume
C. Decreased cardiac output for a given end-systolic volume
D. Decreased cardiac output for a given end-systolic volume

641. The tendency for blood flow to be turbulent is increased by:


A. Increased viscosity C. Partial occlusion of a blood vessel
B. Increased hematocrit D. Decreased velocity of blood flow

642. A patient experiences orthostatic hypotension (hypotension on standing) after a sympathectomy.


The explanation for this occurrence is
A. An exaggerated response of the renin-angiotensin-aldosterone system
B. A suppressed response of the renin-angiotensin-aldosterone system
C. An exaggerated response of the baroreceptor mechanism
D. A suppressed response of the baroreceptor mechanism

643. The ventricles are completely depolarized during which isoelectric portion of the electrocardiogram
(ECG)?
A. PR interval C. QT interval E. T wave
B. QRS complex D. ST segment

644. An acute decrease in arterial blood pressure elicits which of the following compensatory changes?
A. Decreased firing rate of the carotid sinus nerve D. Decreased contractility
B. Increased parasympathetic outflow to the heart E. Decreased mean systemic pressure
C. Decreased heart rate

645. " Splitting " of the second heart sound occurs because:
A. The aortic valve closes before the pulmonic valve
B. The pulmonic valve closes before the aortic valve
C. The mitral valve closes before the tricuspid valve
D. The tricuspid valve closes before the mitral valve
E. Filling of the ventricles has fast and slow components

646. During exercise, total peripheral resistance (TPR) decreases because of the effect of
A. The sympathetic nervous system on splanchnic arterioles
B. The parasympathetic nervous system on skeletal muscle arterioles
C. Local metabolites on skeletal muscle arterioles
D. Local metabolites on cerebral arterioles
E. Histamine on skeletal muscle arterioles

647. An increase in arteriolar resistance, without a change in any other component of the cardiovascular
system, will produce:
A. A decrease in total peripheral resistance (TPR) C. An increase in arterial pressure
B. An increase in capillary filtration D. A decrease in afterload

648. The greatest pressure decrease in the circulation across the arterioles because:
A. They have the greatest surface area
B. They have the greatest cross-sectional area
C. The velocity of blood flow through them is the highest
364
Cardiovascular System Physiology
D. The velocity of blood flow through them is the lowest
E. They have the greatest resistance

649. Pulse pressure is:


A. The highest pressure measured in the arteries
B. The lowest pressure measured in the arteries
C. Measured only during diastole
D. Determined by stroke volume
E. Decreased when the capacitance of the arteries decreases
F. The difference between mean arterial pressure and central venous pressure

650. In the sinoatrial (SA) node, phase 4 depolarization (pacemaker potential) is attributable to:
A. An increase in K+ conductance
B. An increase in Na+ conductance
C. A decrease in Cl- conductance
D. A decrease in Ca 2+ conductance
E. Simultaneous increases in K + and Cl -conductances

651. The low-resistance pathways between myocardial cells that allow for the spread of action
potentials are the:
A. Gap junctions D. Intercalated disks
B. T tubules E. Mitochondria
C. Sarcoplasmic reticulum (SR)

652. Myocardial contractility is best correlated with the intracellular concentration of:
A. Na+. C. Ca+2. E. Mg2+.
B. K+. D. Cl-.

653. Which of the following is an effect of histamine?


A. Decreased capillary filtration
B. Vasodilation of the arterioles
C. Vasodilation of the veins
D. Decreased capillary hydrostatic pressure
E. Interaction with the muscarinic receptors on the blood vessels

654. Carbon dioxide (CO2) regulates blood flow to which one of the following organs?
A. Heart D. Skeletal muscle at rest
B. Skin E. Skeletal muscle during exercise
C. Brain

655. Cardiac output of the right side of the heart is what percentage of the cardiac output of the left side
the heart?
A. 25%. C. 75%. E. 125%.
B. 50%. D. 100%.

656. The physiologic function of the relatively slow conduction through the atrioventricular (AV)
node is to allow sufficient time for:
A. Runoff of blood from the aorta to the C. Filling of the ventricles
arteries D. Contraction of the ventricles
B. Venous return to the atria E. Repolarization of the ventricles
365
Cardiovascular System Physiology

657. Blood flow to which organ is controlled primarily by the sympathetic nervous system rather
than by local metabolites?
A. Skin C. Brain
B. Heart D. Skeletal muscle during exercise

658. Which of the following parameters is decreased during moderate exercise:


A. Arteriovenous O2 difference D. Pulse pressure
B. Heart rate E. Total peripheral resistance (TPR)
C. Cardiac output

659. Which of the following agents or changes has a negative inotropic effect on the heart?
A. Increased heart rate C. Norepinephrine E. Cardiac glycosides
B. Sympathetic stimulation D. Acetylcholine (Ach)

660. Which agent is released or secreted after a hemorrhage and causes an increasing renal Na+
reabsorption?
A. Aldosterone D. Antidiuretic hormone (ADH)
B. Angiotensin I. E. Atrial natriuretic peptide
C. Angiotensin II.

661. Which of the following changes will cause an increase in myocardial O2 consumption?
A. Decreased aortic pressure
B. Decreased heart rate
C. Decreased contractility
D. Increased size of the heart
E. Increased influx of Na+ during the upstroke of the action potential

662. Which of the following events is specifically seen on an electrocardiogram (ECG)?


A. Sinoatrial (SA) node depolarization.
B. Atrioventricular (AV) node depolarization.
C. Bundle of His depolarization.
D. Bachmann, s bundle depolarization.
E. Atrial muscle depolarization

663. Stimulation of the high- pressure baroreceptors is associated with:


A. An increase in cardiac contractility
B. An increase in heart rate
C. An increase in the discharge rate of vagal efferent cardiac neurons
D. A decrease in systemic blood pressure
E. Stimulation of the vasopressor center

664. An increase in systemic blood pressure leads to which one of the following effects?
A. An increase in the velocity at which blood is ejected from the left ventricle
B. An increase in cardiac output
C. An increase in the residual volume of blood in the ventricle
D. A decrease in the time it takes for the left ventricular wall to develop peak tension
E. A decrease in the maximal wall tension developed in the left ventricular muscle
366
Cardiovascular System Physiology
665. The time from the upstroke of the carotid artery pulse to the incisura (dicrotic notch) is a measure
of the period of:
A. Atrial diastole D. Rapid ventricular filling
B. Ventricular ejection E. Ventricular isovolumic relaxation
C. Rapid ventricular filling

666. An electrocardiogram (ECG) reveals no P waves in any lead This would indicate that impulses from
which one of the following structures are being blocked?
A. Sinoatrial (SA) node D. Left bundle branch
B. Bundle of His E. Ventricular muscle
C. Purkinje fibers

667. The pulmonic valve normally closes after the aortic valve because the:
A. Diameter of the pulmonary artery is less than that of the aorta
B. Right ventricular contraction begins after left ventricular contraction
C. Velocity of ejection in the right ventricle is less than that in the left ventricle
D. Diastolic pressure in the pulmonary artery is less than in the aorta
e) Leaflets of the pulmonic valve are stiffer
E. Leaflets of pulmonic valve are stiffer and harder to close, compared with those of the aortic valve

668. Which one of the following mechanisms is most important for maintaining an increased blood flow
to skeletal muscle during exercise?
A. An increase in aortic pressure
B. An increase in α - adrenergic impulses
C. An increase in β- adrenergic impulses
D. Vasoconstriction in the splanchnic and renal areas
E. Vasodilation secondary to the effect of local metabolites

669. Increasing the preload of cardiac muscle will:


A. Reduce the ventricular end-diastolic pressure (VEDP)
B. Reduce the peak tension of the muscle
C. Decrease the initial velocity of shortening
D. Decrease the time it takes the muscle to reach peak tension
E. Increase the ventricular wall tension

670. Venous return is enhanced during exercise by all of the following factors EXCEPT:
A. Increased depth of respiration C. Reduced arteriolar resistance
B. Pumping action of skeletal muscles E. An erect position

671. With regard to cerebral blood flow:


A. Is dependent on the intracranial pressure D. Is increased by hypothermia
B. Is increased by hypoxia E. Is about 750ml/min in a 70 kg man
C. Is reduced by hypercapnia

672. In the heart:


A. Excitation begins in the sinoatrial node
B. Excitation of the ventricle begins at the apex and spread to the base
C. Depolarization occurs from epicardium to endocardium
D. Vagal stimulation slows the sinoatrial node
E. The bundle of His connects the atrioventricular node to the Purkinje's fibers of the ventricle
367
Cardiovascular System Physiology

673. Following acute hemorrhage, the following compensatory mechanisms occur:


A. Increased chemoreceptor discharge
B. Increased level of circulating angiotensin II
C. Vasoconstriction of renal efferent arterioles
D. Cerebral vasoconstriction
E. Increased baroreceptor stretch

674. Stimulation of the beta receptors give rise to:


A. Tachycardia D. Pupil dilatation
B. Increased myocardial contraction E. Increase in intracellular camp
C. Vasoconstriction of visceral vessels

675. Valsalva maneuvers causes:


A. Increased peripheral resistance D. Tachycardia
B. Raised in intraocular pressure E. Increased blood volume in systemic circulation
C. Drop in blood pressure

676. Vasodilatation occurs in, EXCEPT:


A. Increased lactate concentration D. Increased in potassium concentration
B. Decreased in skin temperature E. Reduced oxygen tension
C. Increased hydrogen ions concentration

677. With regard to cerebral autoregulation:


A. Cerebral blood flow is constant over a diastolic blood pressure of 60 to 140 mmHg
B. Autoregulation is lost during the acute phase of subarachnoid haemorrhage
C. It is impaired in hypercapnia
D. It is impaired in hypoxia
E. It is mediated by the sympathetic nervous system

678. The following are true about resistance to blood flow in the vessel EXCEPT:
A. It is dependent on the hematocrit
B. It depends on the thickness of the vessel wall
C. It is directly proportional to the length of the vessel
D. It is directly proportional to the pressure drop along the vessel
E. It is inversely proportional to the fourth power of the radius of the vessel

679. The following are true about cardiac contraction EXCEPT:


A. The P wave initiates the atrial contraction
B. Atrial contribution to ventricular filling is most effective at fast heart rate
C. B wave is generated by atrial contraction
D. Fourth heart sound occurs during atrial contraction
E. Atrial natriuretic peptide (ANP) is released when the atria are distended by increases in blood volume

680. True statements about ECG include all the following EXCEPT:
A. The P-R interval corresponds to the duration of atrial systole
B. The T-wave ends at the time of aortic valve closure
C. The ST segment represents repolarization of the ventricles
D. The QRS complex represents atrial repolarization
E. P-wave represents depolarization of the atrium
368
Cardiovascular System Physiology

681. The following are true about micro-circulation EXCEPT:


A. Arterioles have no muscle
B. Capillaries have walls made up of a single layer of cells
C. Capillaries have no innervation
D. The capillaries contain 5% of the total blood volume at any one time
E. The capillaries contain gaps measuring 5-9 um wide

682. The following are true about potassium EXCEPT:


A. Hypokalemia decreases the time of cardiac repolarization
B. Hyperkalemia decreases cardiac contraction
C. Hyperkalemia relaxes vascular smooth muscle
D. Hyperkalemia is seen in decreased aldosterone secretion
E. Fruits such as banana are high in potassium

683. The carotid bodies


A. Have a low blood flow per gram of tissue D. Respond to small changes in PO2
B. Contain baroreceptors E. Are located on the external carotid arteries
C. Respond to changes in pH

684. In the fetal circulation before birth, all are true EXCEPT
A. The PO2 is higher in the ductus venous than in the ductus arteriosus
B. Blood can go from the right atrium to the aorta without passing through the left atrium and ventricle
C. The PO2 in the aortic arch is higher than the descending aorta
D. Blood flowing through the foramen ovale comes principally from the superior vena cava
E. Blood passes through the ductus arteriosus because of the high pulmonary vascular resistance

685. In the central venous pressure waveform


A. The a wave occurs after ventricular systole
B. The v wave is caused by atrial contraction
C. The a wave is absent in atrial fibrillation
D. The a wave corresponds with closure of the aortic valve
E. The v wave occurs during diastole

686. Renin activity is increased by


A. An increase in circulating adrenaline D. An increase in aldosterone output
B. Hypotension E. Hypovolemia
C. Increased sodium ingestion

687. Acute antagonism of beta adrenergic receptors causes


A. Hyperglycemia D. Pupillary dilatation
B. Peripheral vasodilatation E. A reduction in cardiac output
C. Suppression of uterine contractility

688. Capillary permeability is increased by:


A. Bradykinin C. Calcium E. Histamine
B. Adrenaline D. Vasopressin

689. Dilatation of the peripheral arterial blood vessels can be caused by:
A. Thromboxane A2 B. Adenosine diphosphate
369
Cardiovascular System Physiology
C. Endothelin E. Nitric oxide
D. Prostaglandins

For each event, select the mechanism that is associated with it:
A. Increase in contractility.
B. Increase in fiber length.
C. Both.
D. Neither.
690. An increase in the maximal velocity of shortening (Vmax)
691. A shift to a new frank-starling curve
692. An increase in intracellular Ca2+.

For each condition, select the most appropriate cardiovascular response:


A. Increased ventricular end-diastolic pressure (VEDP)
B. Increased aortic diastolic pressure.
C. Both.
D. Neither.
693. Myocardial tissue damage
694. Increased arteriolar resistance
695. Decreased ventricular contractility
696. Increased heart rate
697. Increased central venous pressure

Match the cardiac event with the interval of the cardiac cycle in which it occurs:
A. Atrial contraction.
B. Isovolumic contraction.
C. Rapid ventricular ejection.
D. Reduced ventricular ejection.
E. Isovolumic relaxation.
698. Second heart sound (S2)
699. Achievement of maximal ventricular volume
700. Closure of the atrioventricular (AV) valves
701. Opening of the aortic valve

Match each cardiovascular adjustment with the factor that is responsible for it:
A. Functional hyperemia.
B. Histamine.
C. Hypertension.
D. CO2 tension (PCO2).
E. Capillary pressure.
702. Regulation of capillary filtration rate
703. Elevation of arterial diastolic pressure
704. Metabolic regulation of blood flow
705. Regulation of cerebral blood flow
706. Increase in microvascular permeability

Match each numbered phenomenon with the appropriate phase of the ventricular action potential
shown in the figure 1.
(A) Phase 0.
(B) Phase 1.
370
Cardiovascular System Physiology
(C) Phase 2.
(D) Phase 3.
(E) Phase 4.

707. Phase of the ventricular action potential in which the membrane potential is close to the K+
equilibrium potential
708. Phase of the ventricular action potential that has the highest conductance to Ca++.
709. Phase of the ventricular action potential that coincides with diastole

Match each function with appropriate receptor:


(A) α Receptors.
(B) β1 Receptors.
(C) β2 Receptors.
(D) Muscarinic receptors.
710. Mediate constriction of arteriolar smooth muscle
711. Mediate slowing of the heart

Match each occurrence with the appropriate phase of the cardiac cycle:
(A) Atrial systole.
(B) Isovolumetric ventricular contraction.
(C) Rapid ventricular ejection.
(D) Reduced ventricular ejection.
(E) Isovolumetric ventricular relaxation.
(F) Rapid ventricular filling.
(g) Reduced ventricular filling (diastasis).
712. Aortic pressure is highest
713. Ventricular volume is lowest
714. The mitral valve opens

715. A 50-year-old woman had an echocardiogram. The results indicated a thickened right ventricle.
Other data indicated that the patient had severely decreased arterial oxygen content and equal systolic
pressures in both cardiac ventricles. What condition is present?
A. Interventricular septal defect D. Pulmonary valve regurgitation
B. Tetralogy of Fallot E. Patent ductus arteriosus
C. Pulmonary valve stenosis

716. Dilatation of the peripheral arterial blood vessels can be caused by:
A. Thromboxane A2 C. Endothelin
B. Adenosine diphosphate D. Prostaglandins
371
Cardiovascular System Physiology
E. Nitric oxide

717. Smooth Muscle is


A. Voluntary and Spindle Shaped C. Involuntary and Spindle Shaped
B. Voluntary and Striated D. Involuntary and Striated

718. Skeletal Muscle is


A. Voluntary and Spindle Shaped C. Involuntary and Spindle Shaped
B. Voluntary and Striated D. Involuntary and Striated

719. Cardiac Muscle is


A. Voluntary and Spindle Shaped C. Involuntary and Spindle Shaped
B. Voluntary and Striated D. Involuntary and Striated

720. Which type of muscle cell is multinucleated?


A. Cardiac C. Skeletal
B. Smooth D. All of the Above

721. Of the following, which is the most significant factor affecting peripheral resistance?
A. Blood viscosity C. Blood vessel radius
B. Blood vessel length D. Blood pH

722. Baroreceptors project to the:


A. Vasomotor, Cardioinhibitory, and Cardioacceleratory centers
B. Cardioinhibitory and Cardioacceleratory centers only
C. Vasomotor center only

723. Nitric Oxide is a:


A. Powerful vasoconstrictor C. Weak vasoconstrictor
B. Powerful vasodilator D. Weak vasodilator

724. Consider the following events and then put them in correct chronological order.
1. Sustained blood pressure depression
2. Release of aldosterone
3. Conversion of angiotensin I to angiotensin II
4. Release of renin
5. Cleavage of angiotensinogen
A. 1,2,3,4,5 B. 1,4,2,5,3 C. 1,4,5,3,2 D. 1,2,5,3,4

725. If MAP becomes >160mmHg, then which of the following may occur?
A. Cerebral edema C. Renal atelectasis
B. Cerebral polythemia D. Renal perfusion

726. The most common type of shock is:


A. Hypovolemic C. Neurogenic
B. Anaphylactic D. Septic

727. The primary baroreceptors are found in the:


A. Carotid sinus C. Atrial bifurcation
B. Aortic arch D. A and b
372
Cardiovascular System Physiology
E. B and c

728. Chemoreceptor activation will result in:


A. Increased heart rate and respiratory rate
B. Decrease heart rate and respiratory rate
C. Increased heart rate and decreased respiratory rate
D. Decreased heart rate and increased respiratory rate

729. A person with primary hypertension would be expected to have a plasma aldosterone level that is:
A. Greater than normal B. Less than normal C. Normal

730. An ACE inhibitor would be expected to cause a:


A. Decrease in systemic BP D. All of the above
B. Decrease in plasma angiotensin II E. 2 of the above
C. Decrease in plasma aldosterone

731. An increase in tissue lactic acid will cause the arterioles feeding that tissue to:
A. Constrict D. Increase their resistance
B. Dilate E. 2 of the above
C. Contract their vascular smooth muscle

732. Epinephrine causes the blood vessels serving skeletal musculature to:
A. Constrict
B. Dilate
C. Epinephrine has NO EFFECT on these blood vessels

733. Vascular smooth muscle responds to stretch by:


A. Contracting C. Initiating mitosis E. Secreting renin
B. Relaxing D. Initiating meiosis

734. The intrinsic autoregulation of blood pressure is:


A. Metabolic
B. Myogenic
C. Not suited to respond to long term changes in BP
D. All of the above
E. None of the above

735. If MAP falls below ________mmHg then fainting can occur.


A. 80 B. 100 C. 40 D. 60

736. If a segment of pulmonary tissue has a low O2 concentration, then the arterioles serving that
segment will:
A. Dilate C. Release nitroprusside
B. Constrict D. Release ACE

737. The thickest layer of the wall of veins is the:


A. Tunica media D. Tunica interna
B. Tunica externa E. Internal elastic lamina
C. Subendothelial connective tissue
373
Cardiovascular System Physiology
738. The basic tissue types found in large blood vessels include:
A. Muscle C. Nerve E. All of the above
B. Connective Tissue D. Epithelium

739. When compared to arteries, veins generally:


A. Are thinner walled D. Have thicker endothelium
B. Have more muscle in the tunica media E. Are more elastic
C. Carry faster moving blood

740. The blood vessels that play the most important role in the regulation of blood flow to a tissue and
blood pressure are the:
A. Arterioles C. Capillaries E. Arteries
B. Veins D. Venules

741. As blood travels from the aorta to the capillaries:


A. Pressure Increases D. Velocity Increases
B. Viscosity Increases E. Flow Increases
C. Resistance Increases

742. A patient with a hypothalamic tumor has hypersecretion of ADH. Which of the following BP
readings would be most likely for this patient?
A. 95/65 B. 115/80 C. 120/60 D. 165/100

743. The difference between the systolic and the diastolic pressures is known as the:
A. Blood Pressure C. Mean Arterial Pressure
B. Pulse Pressure D. End-ventricular Pressure

744. The tunica interna:


A. Includes a layer of dense connective tissue D. Includes a layer of simple squamous epithelium
B. Contains some small capillaries E. None of the above
C. Contains some striated smooth muscle fibers

745. Which statement is true of arteries?


A. All arteries carry oxygenated blood away from the heart
B. All arteries carry blood low in carbon dioxide away from the heart
C. Some elastic arteries release significant amounts of renin
D. All arteries lack valves
E. More than one of the above is correct

746. Peripheral resistance:


A. Decreases with increasing blood vessel length
B. Decreases if blood vessel diameter is reduced by 50%
C. Decreases in cases of polycythemia
D. Is not a major factor in the determination of mean arterial pressure
E. 2 of the above are correct
F. None of the above correct

747. As blood flows from the aorta to the capillaries of the gastrocnemius, its velocity of flow will:
A. Increase B. Decrease C. Stay the same
374
Cardiovascular System Physiology
748. Which of the following is true?
A. Arteries typically have thinner walls than veins
B. Veins typically have a much wider tunica media than arteries
C. Veins typically have much more elastin than conducting arteries
D. You typically would find fewer valves in arteries than in veins

749. Increasing blood vessel length will cause peripheral resistance to:
A. Increase B. Decrease C. Stay the same

750. As you go from arteries to capillaries, the fluid pressure exerted by blood on the vessel walls:
A. Increases B. Decreases C. Stays the same

751. If MAP increased by 10 mmHg, how much must the diastolic pressure have changed?
A. -10mmHg
B. +10mmHg
C. 0 mmHg
D. Cannot be determined from the information given

752. As heart rate decreases, diastolic pressure will:


A. Increase B. Decrease C. Stay the same

753. The volume of blood moving through a given area in a given time is the:
A. Blood velocity C. Blood pressure
B. Blood flow D. Blood resistance

754. If MAP=100mmHg, and SBP=120, then what must DBP be?


A. 80mmHg C. 100mmHg
B. 90mmHg D. 120mmHg

755. The typical diameter of a continuous capillary is:


A. 8 um C. 4 cm
B. 16 mm D. 24 km
375
Cardiovascular System Physiology

1 C 21 C 41 A 61 C 81 C 101 A 121

A, B,
2 B 22 C 42 B 62 D 82 A 102 C 122
C
A, B,
3 D 23 D 43 C 63 A 83 E 103 A 123
D, E
4 A 24 A 44 B 64 A 84 B 104 E 124 D, E

5 B 25 B 45 B 65 C 85 E 105 C 125 B, E

6 A 26 B 46 C 66 A 86 D 106 E 126 E

7 C 27 D 47 A 67 D 87 E 107 A 127 A

8 D 28 B 48 D 68 C 88 E 108 E 128 B

9 A 29 C 48 B 69 B 89 C 109 C 129 D
B, D,
10 B 30 C 50 E 70 C 90 E 110 130 B
E
11 B 31 D 51 B 71 B 91 A 111 A, C 131 D
A, C,
12 C 32 A 52 C 72 E 92 D 112 132 D
E
13 C 33 D 52 A 73 D 93 D 113 C, E 133 D

14 D 34 D 54 B 74 B 94 A 114 C, E 134 D
A, B,
15 B 35 B 55 C 75 A 95 A 115 135 E
D, E
16 D 36 D 56 A 76 D 96 E 116 E 136 D
B, D,
17 B 37 A 57 C 77 C 97 D 117 137 E
E
A, B,
18 C 38 A 58 C 78 B 98 D 118 138 B
D, E
19 B 39 D 59 D 79 C 99 C 119 B, E 139 E
A, B,
20 C 40 D 60 D 80 D 100 C 120 140 C
C, E
376
Cardiovascular System Physiology
141 D 161 D 181 G 201 D 221 A 241 A 261 A

142 D 162 B 182 B 202 C 222 D 242 E 262 D

143 D 163 A 183 B 203 C 223 E 243 E 263 C

144 A 164 D 184 C 204 D 224 B 244 A 264 C

145 D 165 A 185 E 205 D 225 A 245 C 265 B

146 A 166 B 186 D 206 D 226 B 246 E 266 All

147 A 167 C 187 B 207 C 227 A 247 B 267 D

148 B 168 A 188 E 208 C 228 C 248 A 268 C

149 D 169 D 189 D 209 B 229 D 249 A 269 B

150 A 170 B 190 D 210 A 230 A 250 C 270 D

151 B 171 E 191 A 211 A 231 B 251 C, D 271 B

152 E 172 C 192 A 212 C 232 E 252 C 272 A

153 A 173 A 193 E 213 D 233 C 253 C 273 D

154 B 174 E 194 G 214 A 234 B 254 B 274 All


C, D,
155 E 175 A 195 D 215 C 235 C 255 B 275
E
156 A 176 E 196 E 216 C 236 D 256 B 276 A, B

157 A 177 E 197 A 217 A 237 D 257 D 277 A

158 D 178 E 198 A 218 B 238 A 258 A 278 C

159 D 179 D 199 A 219 D 239 B 259 D 279 B

160 B 180 E 200 C 220 A 240 A 260 C 280 D


377
Cardiovascular System Physiology
281 C 301 B, E 321 D 341 C 361 C 381 A 401 A
A, C,
282 All 302 322 D 342 C 362 D 382 C 402 E
D
283 B 303 D 323 C 343 D 363 B 383 A 403 D

284 B 304 All 324 C 344 C 364 A 384 A 404 A


A,
285 B 305 325 A 345 B 365 A 385 A 405 B
D, E
286 A 306 A 326 A 346 D 366 D 386 B 406 D

287 All 307 A, B 327 C 347 C 367 E 387 A 407 B


A,
288 B, D 308 328 D 348 C 368 B 388 E 408 E
B, C
B, C,
289 D 309 329 All 349 A 369 E 389 C 409 B
D
290 A, B 310 A, C 330 D 350 C 370 C 390 B 410 E
A, C, A,
291 311 331 C 351 C 371 B 391 E 411 A
D B, E
C,
292 312 A 332 C 352 D 372 D 392 D 412 E
D, E
A,B,
293 313 A 333 All 353 B 373 E 393 D 413 A
C
294 B, C 314 A 334 D 354 B 374 B 394 E 414 E

295 All 315 D 335 D 355 A 375 A 395 D 415 C

296 D 316 B 336 C 356 A 376 A 396 B 416 C


A,
297 317 C 337 C 357 B 377 B 397 D 417 D
B, E
298 A, E 318 A 338 A 358 D 378 E 398 C 418 C
C,
299 319 A 339 C 359 D 379 D 399 A 419 A
D, E
300 B 320 B 340 C 360 B 380 A 400 B 420 C

You might also like